Skip to main content

Full text of "The Teacher's Hand-Book of Algebra ; containing methods, solutions and exercises"

See other formats


■i-GEBRA     : 

TEACHER'S  HAND-BOOK 


REVISED  imm 


S-5i 


idLian 


THE  LIBRARY 

UNIVERSITY  OF 
WESTERN  ONTARIO 


THE  J.  D.   BARNETT 
TEXT-BOOK  COLLECTION 


Toronto. 
Sxamiaa- 


with  the 

ead  of  in 

I  chapter, 
ninations 
indix. 
horough- 
of  each 

tcial,  and 

.  compre- 

V  for  Bx' 
an 


A  Manual  of  Spelling  ani  Dictation.     Price  30c. 
Prominent  Features. 
The  book  is  divided  into  five  parts  as  follows: 
Part  I.  Contains  the  lujrds  in  comma <t  use  in  daily  life,  to- 

't 
is 


University  of  Western  Ontario 
LIBRARY 

LONDON      -      CANADA 

VJT\oo\       .S\iL 


CI 


ass 


L>1CTAT10N     l^ESSbO.Nii  — All  llie  ICSbOlls    ilic  suiiauic     lui 

tation  Lessons  on  the  slate  or  in  dictation  book. 

Reniews. — These  will  be  found  throughout  the  book. 


MASON'S  GRADUATED  SERIES  OF  KXCUSIJ  GKA5IMAR8. 


]>X«son'j!i  Outlines   of  Kiiclisli 
Gri'aiiiiii  fii'. 

By  C.  P.  Mason,  B.  A.,  F.  C.  P.,  Fellow  of  University  ColleRe  London, 
uthorized  fof  use  of  Schools  in  Ontario.    For  the  use  of  junior  classes. 

45  Cents. 


Au 
Price, 


Miason'ssj  Slioi-ter  Eiierlisli  Graiiiinar. 

With  copious  and  carefully  graded  exercises,  243  pagfis- 


Price, 


60  Cents. 


PLEASE 


LIBRARIES 
THE  UNIVERSITY  OF  WESTERN  ONTARIO 


LONDON  CANADA 


LS-M135 


r 


/ 


>'/ 


/ 


/ 


Digitized  by  the  Internet  Archive 

in  2009  with  funding  from 

Ontario  Council  of  University  Libraries 


http://www.archive.org/details/teachershandalgeOOmcle 


lU.  J    6aci;c's  glatbcmatical  .Scries. 

THE    TEACFJER'S 

HaND-BoOI  of   ALGEBRA; 


CONTAIXISG 


METHODS,  SOLUTIONS  AND  EXERCISES 

ILLUSTRATIXG 

THE   LATEST   AIsD   BEST   TREATMENT   OF   THE   ELEMENTS 
OF  ALGEBRA. 


BY 

J.  A.  McLELLAN,  M.A.,  LL.D. 

HIGU    SCHOOL    IXSPECTOR    FOB    ONTAIilO. 


The  object  of  pure  Mathematics,  ivhich  is  a'tother  nainfifor  Algebra,  is  the  unfoUliiuj 
0/  tlm  laws  of  the  human  intelligence." — SYiiVEaTUR. 


FOURTH  EDITION -REVISED  AND  ENLARGED, 


TOBONTO : 
W.  -l  GAGE  &  COMPANY. 


1881. 


Entered  according  to  Act  of  Parliament  of  Canada  in  the  year 
1880  by  W.  J.  GagjE  &  Compamt.  in  the  otnce  ot  tne  jyiiiusior 
of  Agriculture. 


\    7/3 


PREFACE. 


This  boot— embodying  the  substance  of  Lectures  at  Teachers' 
Associations — has  been  prepared  at  the  almost  unanimous  request 
0^  the  teachers  of  Ontario,  who  have  long  felt  the  need  of  a  work 
to  supplement  the  elementary  text-books  in  common  use.  The 
following  are  some  of  its  special  features  : 

It  gives  a  large  number  of  solutions  in  illustration  of  the  best 
methods  of  algebraic  resolution  and  reduction,  some  of  which  are 
not  found  in  any  text-book. 

It  gives,  classified  under  proper  heads  and  preceded  by  type- 
solutions,  a  srreat  number  of  exercises,  many  of  them  illustrating 
methods  and  principles  which  are  unaccountably  ignored  in 
elementary  Algebras. 

It  presents  these  solutions  and  exercises  in  such  a  way  that 
the  stirdent  not  only  sees  how  Algebraic  transformations  are 
effected,  but  also  perceives  how  to  form  for  himself  as  many 
additional  examples  as  he  may  desire. 

It  shows  the  student  how  simple  principles  vnth  which  he  is 
quite  familiar,  may  be  applied  to  the  solution  of  questions  whic}> 
he  has  thought  beyond  their  reach. 

It  gives  complete  explanations  and  illustrations  of  important 
topics  which  are  strangely  omitted  or  barely  touched  upon  in  the 
ordinary  books,  such  as  the  Principle  of  Symmetry,  Theory  of 
Divisors,  Factoring,  Applications  of  Horner's  Division,  &c. 

A  few  of  the  exercises  are  chiefly  supplementary  to  those  pro- 
posed in  the  text-books,  but  the  intelligent  student  will  find  that 
even  these  examples  have  not  been  selected  in  the  usual  appar- 
ently aimless  fashion  ;  he  will  recognise  that  they  are  really 
expressions  of  certain  laws  ;  they  are  in  fact  proposed  with  a  view 


to  leau  liim  to  investicrate  these  laws  for  himself  as  soon  as  he 
has  sufficiently  advanced  in  his  course.  Nos.  8,  9,  10  and  11 
afford  instances  of  such  exercises. 

Others  of  the  questions  proposed  are  preparatory  or  interpreta- 
tion exercises.  These  might  well  have  been  omitted,  were  it  not 
that  they  are  generally  omitted  from  the  text-books  and  too  often 
neglected  by  teachers.  Practice  in  the  interpretation  of  a  new 
notation  and  in  expression  by  means  of  it,  should  always  precede 
its  use  as  a  symbolism  itself  subject  to  operations,  Nos.  23  to 
36  of  Ex,  iii,,  and  nearly  the  whole  of  Ex,  xv.  may  serve  for 
instances. 

Bv  far  the  greater  number  of  the  exercises  are  intended  for 
practice  in  the  methods  exhibited  in  the  solved  examples.  As 
many  as  possible  of  these  have  been  selected  for  their  intrinsic 
value.  They  have  been  gathered  from  the  works  of  the  great 
masters  of  analysis,  and  the  student  who  proceeds  to  the  higher 
branches  of  mathematics  will  meet  again  with  these  examples 
and  exercises,  and  he  will  find  his  progress  aided  by  his  familiar- 
ity with  them,  and  will  not  have  to  interrupt  his  advanced 
studies  to  learn  processes  properly  belonging  to  elementary 
Algebra.  In  making  this  selection,  it  has  been  found  that  the 
most  widely  useful  transformations  ar<^,  at  the  same  time,  those 
that  best  exhibit  the  methods  of  reduction  here  explained,  so  that 
they  have  thus  a  double  advantage,  A  great  part  of  the  exercises 
have,  of  necessity,  been  prepared  specially  for  this  work. 

Ai-ticles  and  exerci-^es  havp  been  prepared  on  the  theory  of 
substitutions,  on  Elimination,  &c,,  but  it  has  finally  been  decided 
to  hold  these  over  for  P*-.  ii.,  which  will  probably  appear  if  the 
present  work  be  favorably  received. 


CONTENTS. 


Chaptrr  T. — Sdbstitution,  Hoeneb's  Diyision.  Sec. 

rxait. 

Sect.  1. — Numerical  and  Literal   Substitution 1 

Sect.  2. — Fundamemal  Formulas  and  their  Applications 10 

Sect.  3. — Horner's  Methods  of  Alultiplication  and  Division,  and  their 

Applications    21 

Chapteb  n. — Prixciplk  of  Stmjietp.t,  &c. 

Sect.  1. — The  Principle  of  Symmetry  and  its  Applications 33 

Sect.  2. — The  Theory  of  Divisors  and  its  Applications 39 

Chapter  III. — Factoring. 

Sect.  1. — Direct  Application  of  the  Fundamental  Formulas   62 

Sect.  2. — Extended  Application  of  the  Formulas 71 

Sect.  3.— Factoring  by  Parts 79 

Sect.  4. — Application  of  the  Theory  of  Divisors 83 

Sect.  5. — Factoring  a  Polynome  by  Trial  Divisors 30 

Chapter  IV. — Measures  and  Multiples,  &o. 

Sect.  1. — Division,  Measures  and  Multiples  101 

Sect.  2. — Fractious 109 

Sect.  3.— Katios    122 

Sect.  4. — Complete  Squares,  <&c 130 

Chapter  V. 
Simple  Equations  of  0::e  Unknown  Quantity 138 

Preliudnai-y  Equations.  Kesolution  by  Factors.  Fractional  Equa- 
tions. Application  of  Piatios.  Equations  involving  Surds, 
Higher  Equations,  &c. 

Chapter  VI. 
Simultaneous  Equations 170 

Equations  of  Two  Unknown  Quantities.  Systems  of  Equations. 
Application  of  Symmeti7.  Equations  of  Three  Unknowns. 
Systems  of  Equations. 

Chaptkr  Vii. 
Examination  Papers 207 


CHAPTER  I. 
Section  I. — Substitution. 


Exercise  i. 

1.  If  a  =  1,   J  =  2,  c  =  3,  d  =  i,  x  =  9,  ?/  =  8,  find  the 
value  of  the  followiug  expressions  : — 

l_/l_(l_lZ^.)}. 
a-  {x—y)  —  {b  —  c){d  —  a)  —  {y  —  b){x  +  o). 
a;-y^y-{y  —  a)\d  +  c(b-c)Ci. 

{x  +  d){y  +  b  -i-c)  +  {x  —  d){a  —  b  —  d)  +  (y  +  d)(a  —  x  ~  d^. 
{d-x)^  +  {c-^v^* 

la-b){c^-b''x)-{c-d){b^-a'-x)  +  {d-b-c){d^-^^\ 
d  —  a        d  +  c  _  c)d  +  b 
d  +  a        d — c  d  —  b 

2.  If  a  =  3,  6  =  -  4,  c  =  -  9,  and  2s  =  a  +  h  +  c,  find  the 
value  of  the  following  expressions  : — 

s{s  —  a){s  —  b){s—c). 
«2  +  (s-a)2  +  (.s-6)2  +  (s_c)2. 
g9_(^s-a){s-b)-{s-h){s-c)-{s-c){s-a). 
2{s-a)[s-b){s  —  c)-\-a{s-b){s-c)^b{s-c){s-a)  +  c{s-  a){s-b). 

3.  If  a  =  2,  6  =  —  8,  c  =  1,  X  =  4^,  find  the  value  of  the 
following  expressions  : — 

a2_i2      ^2+^2      (a-6)2      (a-by 
c^^TP'    a3^63'    (a +  6)3'    (a4:6p"' 
a2  +  r/&  +  62     a2_ft3      a;  |2x-3   _  3a;-l  )a;-l 
^2^^+P'    ^F:r^,2'    21     3  '~T~i~2~' 

{a  +  b)\{a+by--c^\^ 
462c3-(a2-62-c2)2' 
oa(&-c)-hfig  (c-a)jH5^(a-6) 
(a^'6)(i-cy(c-a) 


2  SUBSTITUTION. 

4.  If  a  =  G,  &  =  5,  c  ==  -  4,  fZ  =  -  3,  finti  tlie  value  of  th* 
following  expressions : — 

y(62  +ac)+  ^(c^—2ac).  yi  b^  +ac+  v/(c2— 2ac)  [. 
a2-^/(b'^-^ac)      c+^{d^  +  c^) 

5.  If  a;  =  3,  i/  =  4,  «  =  0,  find  the  value  of : — 

{3«-v/(a:2+2/2)}2{2a;+i/(a:2+7/2+z)}. 

(a;3  _2/3)^^3/|3a;3  +  3(8a;3  +  3a;j/+2/2)y}. 
G.  Calculate  the  values  of  {x-Vy+^)^-^x^+y^-Vz^)  ^^^^ 

X'JZ 

(a)  <»=1,  ?/  =  2,  2;  =  3. 
\h)  a;  =  2,  2/  =  3,  2  =  4. 
(c)  a;=3,  2/  =  4,  «  =  5. 
(t^)  a;=10,  2/  =  ll,  2=12. 

7.  Given  a;  =  3,  2/ =  4,  z=  —5,  calculate  the  values  of 

yx+y+x)^ -d{x+ij+z)  (xy+yz+zx). 
x^{y+z)+y^{z+x)+z^{x+y)  +  2xyz. 

(5«-42)2+9C4x-;s)3-(13a:-5s)2. 

{3x  +  4.y-i-5z)^  +  {4:x  +  dy  +  12z)^  -(ox+Bij  +  lBz)9, 

8.  If  s  =  a  +  ft  +  c,  find  the  Value  of 

(2s  -  rt )  2  +  (2s  -  i)  2  -  (2,s  +  r)  2 ,  given 
(1)  a  =  3,  6  =  4,  c  =  5,  (2)  a  =  21,  i  =  20,  c  =  29, 

(3)  a  =  119,  b  =  120,  c  =  TG9,     (4)  a  =  S,  b=  -4,  c  =  5, 
(5)  «=5,  fe  =  12,  c=-13. 

9.  If  rt  =  l,  6  =  3,  c  =  5,  rf  =  7,  <'  =  9,/=ll,  prove  that 

«/>      6c      cd      de      ef       2   \a       f 

^+±  +  ±+±=111-1 

a  be      bed      ed«      def      4    \ah      e^ 
nhed      hcde      cdef       6  \a6c      defj 


SUJ^STITIITION. 


^2  4- 1,2  .^c-  -  (lb  -hc-ca~h^  +r^  +rf3  -hr-cd-  db  = 

c2  _|_(/2  -j-fS  _  cd  —  dfi  —  CC  =  f/2  -f  c~  +/2  -  rftJ  -  ^■— /^. 

10.  li  a  =  l,  b  =  2,  c  =  S,  d  =  4,  e  =  5,f=Q,  </  =  ?,  prove  that 

ci+b  +  c  =  ^cd,  a-\-b  +  c  +  d  =  ^de, 

a  +  b-\-c  +  d  +  e=^ief,  i'  +  l^+c+'^-\-o  +  f=  ifg, 

ab{a  +  b)  ab[a-{-b) 

a2+i3+,'' J.,/2  +  ,2  +;-2  =    M^_'+l). 

«3+i3^c3+f/3  =  (rt  +  Z>  +  r  +  ^)2, 
«3+/;»+c3+r/34-e3  =  (fl  +  i  +  c  +  ^  +  6-)2. 

a^-\-b^4-c^  +  d^+c^-\-p  =  {a+b+c-\-d-j-i+f}^ 

6c(64-c) 
a4+/,4  4.c4+rZ4  =  deid+e){cde-l) 

«4  +  /,4+o4+,/4+,4  =  !^+^)Mi:i); 
«4+/,4+,4+,/4+,4+/4=  .K/-b'/)J^/Z±), 

bc{b+c) 

6-2+^2=^2^   (.3+,,/3+f,S=y3 

11.  Assume  any  numerical  values  for  x,  y,  and  s,  ana  calculate 
the  values  of  the  following  expressions  : — 

(a;»  -  10ic3 -t-5a;)- +  (5x4  -  10a;2 +  1)3  -  (;*;3  + 1  )^ 
(a;+l)3_2(a;+5)3-(a;  +  9)3  +  2(a;+ll)3  +  (a;+12)3-fx  +  ]G)3. 

(«2-2/3)2+(2a:^)2_(x3+y3)2 

(«3_3x</2)2  +  (3a:3r/-2/3)3_(a;2^.,,2,3. 

{Zx^-\-4.xij+y^)^  +  {Ax^+1xyY-[ox"  +  \xy-¥y^Y. 

(K-i/)3  +  ((/-2)3  +  (2_a;)3-3(jc-?/)  {y-z)  {s-x). 
Art.  I.     If  it  =  any  number,  as,   for  example,  3,    then  x^ 
(which  =  x.x)  =  3x,  a;3  (which  =  x.x^)  =  Bx^,  a;*  (which  =  ic.a;3)  = 
3i:3,  &c.     Oi-  3  =  a;,  3.^  =  x3,  3^:3  =a;4^  Sx^^x%  &c.     Hence  jn'ob- 


4 


SUBSTITUTION. 


6  .-.  x-^-'2x-d  =  15-9  =  6. 


lems  like  the  following  may  be  solved  like  ordinary  aritlimeticaJ 
problems  in  "  Eeduction  Descending." 

Examples. 

1.  Find  the  value  oi  x^  —  2x—9  when  x  —  5, 

x"-2x-Q 
5 

5x 
-2x 

8x 
5 

15  Exjylanation. 

-9  x^  =  5x, 

x^  —  2x  =  Sx=:lC,  and 
=  6. 

2.  Find  the  value  of  x^—x^  —  Ax^  —  3.t  —  5  when  x  =  3. 

x*-x^-4:X^-3x-5 
3 

jOj  Sx^ 

rj 2x^ 

3 

7)3  6x^ 

-4x^ 

rg  2a;2 

3  .-.  a;4_a;3_4^2_3a;-.'>  =  4 

—  ifa;  =  3, 

Ps  6r 

—  3a; 

/«  3a5 


3 


P*  9 

—  5 

u 4. 


SUBSTITUTION. 

Expl-aiuition. 

:.   x^-x^  =  2x^  =  6x^, 
.:    x*  —  x^  —  4x^  =  2a;^  =  6x, 
:.   x*-x^-4:X^-Sx  =  2x=--Q, 
:.    x^  —  x^  —  4.x^  —dx  —  5  =  4:. 

8.  Find  the  value  oi  2x'i  +  12;t;3-f6a;3 -12.r+10. 

Using  coefficients  only,  we  have 

2  +  12  +  G-12  +  10 


Pi  - -10 

-fl2 

'i  +  2 

-  5 

P2    -10 

+   6 

r,  -    4 

-  5 

Ps  20 

-12  ^ 

r,  8 

-5 

P4  -40 

+10 

r^  -30 

.-.  tlie  quantity  =  —  30  if  a;  =  -  5. 
Art.  II.  If  the  coefficients,  and  also  the  values  of  x  are  small 
numbers,  much  of  the  above  may  be  done  mentally,  and  the  work 
■will  then  be  very  compact.  Thus,  performing  mentally  the  mul- 
tiplications and  additions  (or  subtractions)  of  the*  coefficients, 
and  merely  recording  the  partial  reductions  rj,r^,r^,  and  the 
result  r^,  the  last  example  would  appear  as  follows  : — 


SUBSTITUTION, 


-5  )  2     +12     -f6     -12     +10 

2 

-4 

8 

-80 

Art.  III.  lii  tlie  above  examples,  the  coefficients  ave  "brought 
down"  and  written  below  the  products ^^,  ^?2,  jfg,  jp^,  and  are 
added  or  subtracted,  as  the  case  may  require,  to  get  the  partial 
reductions  7-j,  r^,  r^,  and  the  result  ^-^^  Instead  of  thus  "  bring- 
ing down  "  the  coefficients,  we  may  "  carry  up  "  the  products  ^^, 
i>2f  Ps'  Pi^  writing  them  beneath  their  corresponding  coefficients, 
and  thus  get  7\,  r^,  r^,  r^  in  a  third  (horizontal)  line.  Arranged 
in  this  way  Ex.  2  will  appear 

11      -1      -4      -3      -5 
8  +3+6     +6      +9 


11     +2     +2     +3;       4; 
and  Ex.  3  will  appear 

2     +12       +6     -12     +10 
-10     -10     +20     -40 


2       +2       -4       +8;    -30 
Comparing  these  arraugementrf  with  those  first  given  (Ex.  2 
and  3),  it  will  be  seen  that  they  are  hgure  for  figure  the  same, 
except  that  tlie  multiplier  is  not  repeated. 

Art.  IV.  When  there  are  several  figures  in  the  value  of  x, 
they  may  be  arranged  in  a  column,  and  each  figure  used  sepa- 
rately, as  in  common  multiplication.  Where  only  approximate 
values  are  required,  "  contracted  multiplication  "  may  be  used. 
4.  Find  the  value  of  8a;5 -160a;4  +  344a;3+700a;2 -1910^;+ 
1200,  given  a;  =51. 

3      -160     +344     +700     -1910     +1200 

1  3         -7        -13  37         -23 

.      50  150     -350     -650         1850      -1150 


•7        -13       +37 
result  is  27. 


-23 


+27 


SUBSTITUTION. 


5.  Given  a;  =  1-183.  find  the  value  of  64x*  —  144a:+45  correct  to 
three  decimal  places. 


1 
1 

8 
8 

64 

0 
Gt 
6-4 
6^12 
.      ^192 

0 

75-712 
7-5712 
6-0570 
•2271 

-144 

89-5673 

8-9567 

7-1654 

•2687 

+45 

-38-0419 

-3-8042 

-  8.0434 

-•1141 

64, 

75-712, 

89-5673, 

-38-0419, 

-•0036 

•3156* -15792,  for  x 


.'.   result  is  —-004. 

Exercise  ii. 
Find  the  value  of 

a;4- 11x3- 11x2 -ISx+ll,  forx=12. 
x4  +  50x3  -  16x2  -  16x-61,  for  x=:  -17. 
2x4+249x3-125x2  +  100,  for  x= -125. 
2x3-473x2 -234x- 711,  for  x  =  200. 
6.  x»-3x2-8,  for  x  =  4. 

6.  x6  -  615x5  -  3127X-*  +525x3-2090x2 
=  521. 

7.  2x-' +401x4 -199x3 +  399x2 -602X+211,  for  x=- 201. 

8.  1000x4 -81x,  forx=-l. 

9.  99x4  +  117x3 -257x2 -325x- 60,  forx=lf. 

10.  5x'^  +  497x4  +  200x3  +  1 96x2 -218x- 2000,  forx=-99. 

11.  5x5 -620x4-1030x3  +  1045x2 -4120X+9000,  fora;  =  205. 
Calculate,  correct  to  three  places  of  decimals,- — 

12.  x3  +  3x2-13x-38  forx  =  3^58448,  forx=  -3-77931,  and 
forx=  -2-80512. 

13.  2/4- 147/2 +J/+ 38  for  j/  =  313131,  for  i/=  - 1-84813,  and 
for  y=  -3-28319. 

Exercise  iii. 
What  do  the  following  expressions  become  (1)  when  x  =  a,  (2) 
when  X  =  -  a  ? 

1.  X4   -4rtx3  +  6rt2i(;2  —  4r/3a;+a4. 

2.  i/'(x2-«X  +  rt2).  3.    ■|/(x2  +  2flX  +  fl2). 
4.    (x2+flX  +  a2)3_(a;2_ax  +  fl2)3. 

If  x  =  2/  =  3  =  rt,  hnd  tue  value  of  the  following  expressions: 


H  SUBSTITUTION. 

5.  (x-y)  (y-z)  {z-x). 

6.  (x+y)^  (y-^rz  —  a)  (x+z-a). 

8.  -^  +  Jl-  +  _^_. 
y+z       x+z       x+y 

Find  the  value  of 

r.      X     ,     X     1  abc 

9.  —  +  — when  a;  = 

a  b  a-j-h 

10.    - +  I +  — ^ ,  when  x-  A  (a-6+c)» 

a(b  —  x)         b{c  —  x)         a{x  —  c)  a 

11.  ^+    J^,  when:«=  ci^{b_-al^ 

a  b  —  a  b(b  +  a) 

■    12.   (a  +  x)  {b+x)-a{b  +  c)+x^,  when  a;  =  —. 

b 

13.  bx-\-cy-\-az,  when  x  =  b-\-c  —  a,  y  =  c  +  a  —  b,  z  =  a-^-b  -c, 

14.  <^3±^  -  __«__,  when  x=  -a. 

a(l  +  6)— Z^x         a  — 2ia; 

,^      /a;+rt\  ^      a;+2rt+^      1  w;        \ 

15.  — —      —   — ,  when  ic=i(i  — a). 

\x+bl         x  —  a  —  'lb 

16.  (p-q)  {x+2r)  +  {r-x)  (p+q),  when  a;  = '-1?^^. 

17.  a^{b-c)+b^{c  -a)  +  c^{a-  b),  v/hen  a-b  =  0. 

18.  (a+b  +  c)  {bc  +  ca+ab)-{a  +  b)  (b+c)  {c  +  a),  when  a=  -b. 

19.  (a+6  +  c)3-(«3+53_{.c3),  whena  +  6  =  0. 

20.  {x+y+z)'^-{x+y)'>'-{y+z)'^-{z+x)'^+x'^+y'^+rA,y7hm 
x+y-rz  =  0. 

21.  a3(c-62)  +  53(^^(_(^^3)_|.t.3(5_rtaj^a/;c(a6c-l),  when6-a> 

~22.    a^  i''l±^\  \b^  l^lll+^'l  \  when  a.^+6^=0. 

23.  Express  in  words  the  fact  that 

{a-by=a^-2ab+b". 
■  24.  Express  algebraically  the  fact  "that  the  sum  of  two  quan- 
tities multiplied  by  their  difference  is  equal  to  the  difference  of 
the  squares  of  the  numbers." 


SUBSTITUnOM.  9 

25.  The  area  of  the  walls  of  a  room  is  equal  to  the  height  mul- 
tiplied by  twice  the  sum  of  the  lengtli  and  breadth :  what  are  the 
areas  of  the  walls  in  the  following  cases  : 

(1)  length  /,  height  h,  breadth  h. 

(2)  height  x,  length  b  feet  more  than  the-height,  and  breadth 
h  feet  less  than  the  height. 

26.  Express  in  wards  the  statement  that 

{x-\-a)  {x-\-h)=x^-\-{a-{-h)x-\r(ih. 

27.  Express  in  symbols  the  statement  that  "  the  square  of  the 
sum  of  two  quantities  exceeds  the  sum  of  their  squares  by  twice 
their  product." 

28.  Express  in  words  the  algebraic  statement, 

{x-\-y)^=x^  ^-y^  ■^^x]){x+y). 

29.  Express  algebraically  the  fact  that  "the  cube  of  the  differ- 
ence of  two  quantities  is  equal  to  the  difference  of  the  cubes  of 
the  quantities  diminished  by  three  times  the  product  of  the 
quantities  multiplied  by  their  difference." 

30.  If  the  sum  of  Ihe  cubes  of  two  quantities  be  divided  by 
the  sum  of  the  quantities,  the  quotient  is  equal  to  the  square  of 
their  difference  increased  by  their  product ;  express  this  algebrai- 
cally. 

31.  Express  in  words  the  following  algebraic  statement ; 

—=  {x+yr-xy. 

x-y 

32.  The  square  on  the  diagonal  of  a  cube  is  equal  to  three 
times  the  square  on  the  edge ;  express  this  in  symbols,  using 
I  for  length  of  the  edge,  and  d  for  length  of  the  diagonal. 

83.  Express  in  symbols  that  "  the  length  of  the  edge  oi  the 
greatest  cube  that  can  be  cut  from  a  sphere  is  equal  to  the  square 
root  of  one-third  the  square  of  the  diameter." 

34.  Express  in  symbols  that  any  "rectangle  is  half  the  rectan- 
gle contained  by  the  diagonals  of  the  squares  upon  two  adjacent 
sides."  [The  square  on  the  diagonal  of  a  square  is  double  the 
square  on  a  side.] 

85.  The  area  of  a  circle  is  equal  to  ic  multiplied  into  the  square 


10  8UBSTITUTION. 

of  the  radius ;  express  this  in  symbols.     Also  express  in  symbols 
the  area  of  the  ring  between  two  concentric  circles. 

36,  The  volume  of  a  cylinder  is  equal  to  product  of  i'u-  height 
into  the  area  of  the  base,  that  of  a  cone  is  one-third  of  this,  and 
that  of  a  sphere  is  two-thirds  of  the  volume  of  the  circumscribing 
cylinder ;  express  these  facts  in  symbols,  using  h  for  the  height 
of  the  cylinder,  and  r  for  the  radius  of  its  base. 

Exercise  iv. 
Perform  the  additions  in  the  following  cases  : 

1.  (b -a)x-\-{c  —  b)y,  and  {a  +  h']x+{h-\-c)y. 

2.  nx—by,  {a  —  b)x—{a-\-b)y,  and  {a-^b)x~{h  —a)y. 

3.  {y—z)a^-\-(z--x)ab-\-{x-y)b^,  and  {x-y)a^ —  {z—y)ah —  {x 

4.  ax-\-hy-\-cz,  bx-\-cy-{-az,  Sbnd ex -\- ay -\-bz. 

5.  {a+b)x^+{b+c)y^+{a+c)z^,  {h  +  c)x^ +{n  +  c)y^ +  (n  +  b)z*, 
{a+c)x^  +  {a  +  b)y^  +  (b+c)i^,  and- (a  +  b  +  c)  (tcS+t/S-fgS). 

6.  x(a-b)^ -{-y{b-cy+z{c-a)»,  y{a-b)^ +z(b -c)^-i-x{c- 
z)2,  stnd  z{a-b)2+x(b-c)2+y{c-u)~\     ' 

7.  {a-b)x^-{-(b-c)y^+{c-a)z^,{b-c)x^+{c-a)y^  +  {a-b)z-^, 
and  [c-a)x^-{-{a-b)y^+(b-c)z^. 

8.  {a  +  b)x  +  (b+c)y  -(c  +  a}z,  {b  +  c)z  +  (c  +  a)x-{a  +  b)y,  and 
(a  +  c)y+{a  +  b)z  —  {b  +  c^x. 

9.  a^-3ab-:^*b^,  2b'^ -^b^+c^,  ab-ifb^  +  b^,  and  2nb-ib*. 

10.  aa;"-36x",  -dax'^+lbx",  and  -Sbx^  +  lOax". 

11.  What  will  (ax  —  by  +  cz)-\-(bx-\-cy—i(z)~(cx-\-ay-^-iyz)  be- 
come when  x  —  y  -  z  =  l  ? 


SeCTU)N    II. FuNDAMKNTAL,    FoERTULAS    AND    THEIK  APPMCATION. 

4.  By  Multiplication  we  get 

(x  +  r)   (x  +  s)  =  x'^  +  {r  +  s)  x  +  ri^ A. 

(x  +  r){x-i-s)(x-\-t)  =  x^  +  {r  +  s  +  t)x'^  +  {rs  +•  st  +  tr)x  -f  rsf B. 

From  A  we  immediately  get 
(x±yY=x''±2xy^y- [1] 


FUNDAMENTAL    FORMULAS.  1  1 

{x  +  y  +  z)'^=x^  +  2x>/  +  2xz  +  y^  +  %iz  j  ga  ["2] 

(2<0^  =  2a2  +  2  £a& [3] 

(a:+7/)  (u;  — y)  =a;2— y3 [4] 

From  B  we  derive 

(x  +  y)3=.f-^±3x2^  +  3.r//2±?/3  [5] 

=  x-^ ±y'' ±^x])  {x±y) [6] 

+  6-V2/2 [7j 

=   ^3^y3+,3    +3    (;,+_,^)    i^y^^^    (^^^■J      [-Qj 

=  . f3+ 2/3+^3  _,.3  ^  y  _^^  _,.   ,j   (^.^  _|_  y^  +  ^^.j — 3x^2...  [9] 

(2:.()3r=va3  ^  S^^s^^  62a6c  [10] 

[The  symbol  £  means  the  sum  of  all,  such  terms  as] 

Formula  [1] . — Examples. 

1.  Wo  have  at  once  {x  -YyY  +  {x  —yY  =2(^-2  +  t/2),  aud 
(x-YyY—{x  —  yY=^xy. 

2.  (a  +  6  +  c  +  tZ) 2  +  (a  —  b  —  c  +  dY  may  be  vrritten 

{(a  +  d)  +  {b  +  c)}2   +   {(a  +  d)~  (6  +  c)}2,  which  (Ex.  1)  = 
2{{a  +  dy-±{b  +  cy-'}  ;  similarly 

l^a  —  h-\  c  —  dY-\-  {a  +  b  —c  —  dY  =  {{a  —  d)  —  (b  -  c)}^  -f 
{{a  —  d)  +  {b  —  c)}^  =  2{{a  —  dY  +  ib—cY}; 

.-.  {a+b  +  c+  dY  +  (a  —  b  —  c-i-  dY  +  {a  —  b  +  c  —  d)^^ 
{a  +  b  -c-  dY  =  '2{{a  +  dY  +  (b  +c)2-f-(a-d)2  +  (6-c)3}  = 
(again  by  Ex.  1)  i{a2+b2+c''i  +  d2). 

3.  SimpUfy  {aA-b-'rcY-2{a  +  b-{-c}c  +  c^  ; 

This  is  the  square  of  a  binomial  of  which  the  first  term  is 
(a+b+c)  and  the  second —  c;  the  given  quantity    .'.  = 


12  FUNDAMENTAL    FORMULAS. 

4.   Simplify  (a  +  b)^  -  '2{a^  +  b-i )  {a-^h)^  +2{a^  +  h^). 

By  Ex.  1.  2(«.4  +  i4)  =  (rt2+i2)2  +  (a2-i2)2  ;  .-.  giyen  quan- 
tity =  (w+Z/j*  -  2(^<2  _^  i3)  (rt  +  i)2  4.  (,^2  +  /,2)2  _^  (rt2  _  i2)2  = 
{(a  +  6j2_(,i2+i2jp  +  (,,2_/,2j3=rt4_(_2a2i2+i4^(rt2   +/,2)2. 

Exercise  v. 

1.  (a;+3?/2)2  +  (a;_3?/2)3,     (i„2  ^3i2j2  _  (l^s -3^,2)2. 

2.  Siiew  that  (??ia;+'2^^)^  +  (»^  — w'i/)^  =  (wi2-t-w2)  ^2:2+2/2). 

4.  Simplify  irt  +  3i)2-f  2(rt  +  3?;)  {a-b)  +  {a-h)^]  {a-b)^. 

5.  "  (a;  +  3)2  + (:t-+ 4)3- (a:  +  5)3,  and  (|a;2 -22/2)3 - 
(i2/2 +2x2)2. 

6.  Simplify  («.  +  &  +  c)2  +  (6  +  c)2 -2(Z/  +  c)  (n  +  i  +  c) 

7.  Shew  that  {ux^byY -\-{cx+dyY -\-{ay  -  bxY-\-{cy  -dx)^  = 

(a2+i2+c2+^2)   (a;3+2^2). 

8.  Simplify  (a;- 3?/2)2  +  (3a;2-2/)3-2(3a;2-2/)  (x-3^2). 

9.  "  (a;2+a;y-2/2)2-(a;2_a;^/_?/8)2^and(l  +  2a;+4a:2)3 
+  (l-2a;  +  4.T2)s. 

10.  If  rt  +  6=  -fc,  shew  that  (2a-6)2  +  (26-c)2+(2c-«)2  + 
2(2a - b)  {'lb-c)  +  2{2h - c)  (2c -a)  +2(2c-a)  (2a - ?>)  =  tV^- 

11.  Simplify2(«-6)2-(a-2i)2;   (a2+4«?,+i2)2  _  (^2+^2)2, 

12.  "         (r/  +  />)2-(i+c)2  +  (c  +  t/)2_(,^+,,,)2. 

13.  "  (ia;-7/)3+(i_,y_^)2+(l^_a;)2+2(ia;-7/)  (i2-3t^ 
+  2{ly-z){lz-x)  +  2{ix-y)(^y-z). 

14.  Prove  that  (a;- i/)2  +  (2/-«)2+(«_a;)2  =  2(a;-2/)  (^-i/)  + 
2iy-x){z-x)  +  2{z-y)iz-x). 

15.  Simplify  (l  +  a;)4-2(l+a;3)  (l+x)3  +  2(l +a:4). 

'     16.         "        (a;+?/+;s)2-(.«  +  ?/-z)2-(?/+z-a;)2-(2+a;-?/)2. 
17.         "        (x-22/+3z)2  +  (30-27/)2+2(a;-2i/+3z)(2^-3z).- 

IQ,  u         (,(2+Z,2_c2)2_|.(c2_Zy2)2+2(i2_c2)(a2+^3_c2). 

19.  "         (x+yy  +  (x-y)*-^r-y)^x  +  y)*. 


FUNDAMENTAL    FORMULAS.  18 

21.  Sbewthat(3fl-i)3+(35-c)2-f(3c-«)2-2(6-0a)(3i-c) 
+  2(3i-c)(8c-rt)-2(a-3c)(3a-6)-4(«  +  /;  +  c)2=0. 

22.  If  z2  =  2a;?/,  prove  that  (2x2  _2/2)2  +  (z2  _2?/2)2+(a;2  -222j2 
-2(2a;3-2/2)^22_2y2)_|.2(a;3-2z2)(22_22/2)_ 

2(a:3  -222)  (2a;2  -y^)  =  (x+t/)*. 

23.  Simplify    {1+x+x^  +x^y  +  {l-x-x^  +x^)^  + 
{l-x+x--x^)"-\-{l-\-x-x^-x^y. 

24.  Simplify   {ox-{-by)*-2{a^x^  -\-  b^y^)  {ax  +  by)^  + 
^a^x*  +  b^y^). 

FORSIULAS    [2]    AND    [3]  . EXAMPLES. 

1.  (1-2.C  + 3x3)2  =  i_4a,-+6a;2 

-f4x2_19a;3 

-\-V,x* 


=  1  -  4a;+lUx-^  -  12x3-^-90;*. 

2.  (ab  +  bc  +  ni)^  =^a^h^  +  2ab^c  +  2a^bc-^b^c^~{-  2abc^+c^a^  = 
a^b^+b^c'+c^a^  +2abc{a  +  b  +  c). 

3.  {{x  +  y)^+x^+y^}^^{x  +  ij)*  +  2[x  +  y]2{x^-^y^)-{-x^  +  2x^ 
y^+y^  =  {x  +  y)^  +  {x+y)^{{xi-y)^  +  (x-y)^}  -^x^ +  2x^ij^ +  y^ 
=  2x+2/)''  +  fa:2-2/2j2+a;4+2x2;/2+7/4  =  2{(x+y)4+x4  +  2/'*}. 

4.  (x2+X;/  +  ?/2)2=x4  +  2x3;/   +  2x2^2   +  .^2j^2   -|- 2x^3    +  yi  = 

{x+y)^X'+x"y^+y^{x+tj)^. 

5.  In  Ex.  3,  substitute  b-  c  for  x,  c  -a  for  y,  and  consequently 
6  — a  {ov  x+y,  then  since  (i  — c«)2  =  («  — 6)2,  Ex.  8  gives 

{(rt-6)2  +  (/>-c)2+(c-a)2}2=2{(«-i)4  +  (/>-c)*+(c -«)*}. 

6.  Making  the  same  substitutions  in  Ex.  4,  we  have 
(a2+?/2^c2  ^ah-bc  -cay  =  {a  —b)^{b  -  c)^ +  {b-cy{c- a)^  + 
(c-a)2(a  — i)2,  or,  multiplying  both  sides  by  4, 

|(a_i)2_j.(6_c)2   4-  (c-rt)2}2=4(rt_i)2(i_c)2  +  4(i-c)2  X 

(c-a)2+4(c-a)2(a-6)2.  and  .-.  from  Ex.  5,  (a-Zy)4  +  (6_c)4  + 
(c-a)4=2(a- 6)2(6- c)2 +2(6 -c)«(c-a)2  +  2(c-a)>(a-i)3. 


14  FUNDAMKNTAI.    FOKMULAS. 

Exercise  vi. 

5.  Shew  that  («2  +62  +c2)  (a;2  +  ,y3+22)_((^a;  _}.  ^^Z^.  .)3_ 
(a^' -  &.c)  2 -}- (ex- -  <72)  2  +  (6;  -  c?/)  2 . 

6.  Prove  that  (a +  /^)x+(i  + c)2/  +  (c  +  rt)2  multii:)lied  hy  (a  —  b)'x 
-i-{h  —  c)y  +  (c  —  a)z,  is  equal  to  the  dilference  of  the  squares  of 

two  triuomials. 

7.  Shew  that  {n-h)  (a-c) -}- {b-c)  {b-a)  +  (c-a)  (c-h) - 
H(«-Z>)2  +  (&-c)2  +  (c-«)2}  =  0. 

8.  Simplify  {a-{b-c)}^ +  {b-{c-  a) ] »  f  {c -  (« - 6) } » . 

9.  Shew  that  {a^  +  b^ -x^Y^ +{aj-{-bf~x-^)^ +  2{aa^ +bb^)^ 

=  (a2  +  rt2_;j;3)2+(^,3_j.i2_,^.2)3.^2(rtO  +  ai6J2. 

10.  Prove  that  {{a -b){b-  c)  +  (/;  -  c)  (c -  «)  +  (c  -  ^0  («  -  ^^)}  '  = 
(a-6)2(6_c)2  +  (6-c)2  (c--a)2  +  (c-a)2  (a-i)2. 

11.  Square  '2a  —  ibx  —  ^cx  +  2dx. 

12.  If  a;  +  ?/  +  2  =  0,  shew  that  x^  +  //'*  +  s^  =  (x^-yZ^i^ 

(//2-z2)2_(-(23_;^2)3. 

13.  Prove  that  a- {b  +  c)^+  b^{c  +  u)~-[-c~  {a  +  b)'  +  2abc(a  -\-b  +  c) 
=  2{ab  +  bc  +  ca)'. 

Art.  V.  To  apply  foriDula  [4]  to  obtain  the  product  of  two 
factors  which  differ  only  in  the  signs  of  aome  of  their  terms  : — 
group  togetlier  all  the  terms  whose  signs  are  the  same  in  one  fac- 
tor as  they  are  in  the  other,  and  then  form  into  a  second  group 
all  the  other  terms. 

Examples. 

1.  Multiply  a  +  Z>  — c-f  (Z  by  rt  — 6  — c  — tZ ;  here  the  first  group  is 
a  — c,  the  second  i+f^  ;    •'•  we  have 

l(a  ^c)  +  {b  +  d)}  {{a-c)-(h  +  d)}={a-c)^-{b  +  d)\ 


FUxNDAMENTAX.    FORMULAS.  15 

2.  (1  J-  8a;-ff^a;3  +x^)  (1  -  So:  +  Bx^  -  x^.)  =  {(1  +  S.r^)  -i 
i3x-hx"^)}  {(l  +  3a;2)  -  (:3a-  +  a;3)}  =  (l  +  3a:3)3  -  (3ic+a;3)3  =  1 - 

3.  Find  tliecontinuedproductof  r/-f /'-}-c,  b+c  —  a,  c+a  —  b  and 
fZ  +  ^  —  <^. 

The  first  pair  of  factors  gives  {{b  +  c)-{-ii}  {{^>+f^)  —  ci}  ={6-|-r-)' 
-a*  =  63  4-2&c  +  c3-a3. 

The  second  pair  gives  {a  —  {b  —  c)}  {«  +  (&  —  '-')}  =a3  — i3_|_2^c 
--c3  ;  the  only  term  whose  sign  is  the  same  in  both  these  results 
is  26c  ;  hence,  grouping  the  other  terms,  we  have 

{2bc  +  {b^-{-:^-a2)}{2bc^{b^  +  c2  -  a^)}  = 

(26s)2-(62+c2-a3)2=2a262+262r34-2c3a2_^,4_J4_c4. 

4.  Prove  (a^-^ab  +  by -a^b^  =  {a^  +  ab)^  +  {ah  +  b^)^. 

The  expression  ={n^+h^)  {a^  +2ab+b^)  =  (a^  +b~)  (a -^by  = 
a^ia+by  -{-b^{a  +  by  =  {a2  +ab)^  -\r(ab  +  b^)2. 

Exercise  vii. 

1.  (a2  +  2ab  +  b2)  {a^-2ab+b'). 

2.  (ix-2  -x,j  +  yi)(ix^+y2+x7j). 

5.  {a^-ab  +  2b^)  {a^  +  '(b+-lb^) ;  (x*-\-M:y)  (x'^-4xi/). 

4.  {{x  +  y)x-y{x-y)}  {{x-y)  x-y{y-x)\. 

5.  Simplify:  (x+3)  (a;-3)  +  (.r  +  4)  (a;-4)-(a;  +  5)  {x~5). 

6.  "  (H-a;)4  +  (l-a;)4-2(l-a;2)3. 

7.  (x3+j/3)2  -  (2a:?/)2  -  (a;2_2/3)2. 

8.  (2r72_362+4c3)  (2a3  +  3J3_4c2). 

9.  (2a  +  i-  3c)  (i  +  3c-2«) ;  {2a—b-Bc)  (b—Sc—2a). 

10.  (x'^-\-y^)  (x2+7/3)  (a;+2/)  (x-7y). 

11.  (x^+xy  +  y^)  (x^-xy+y^)  {x'^-x^y--'-\-y'>^).        ' 

12.  (a  +  /;-a/>— 1)  (a-f /j  +  a/y+1). 

13.  Prove  (a2-f/;2  4.^2)(//j_|_^.2  _  ^.a^^^a +a2  _/;2)  (^2  _l/,2_  ^s) 
=  4/;'*o4  when  a^  =  />4_j_,.4, 

14.  (a;2+y2_6a.^)    (a.2+^2  +  6^.,^). 

1/5.  (a-4- 2x3 +  8x2— 2x4-1)  {x'^+2x^-^Sx^-+2x-^l). 


16  '      FUNDAMENTAL    FORMULAS. 

16.  Multiply  (2x—y)a^  —  (a;-f-?/)«a;  +x^  by  (•2x-y)c''^-^ 
{x+y)  ax  -x^. 

Prove  the  following : 

17.  (a^  +b^  +c^  +ab  +  bc+ca)2  -  (ah  +  bc  +  ca)3  =  (a+  5  +c)^ 
x{aZ  +  h^-i-c^). 

18.  (a^-Jrh^  +c^  +ab  +  bc+ca)^  -{a^  +ah  +  ca-bc)^  = 
{{a  +  b)[b  +  c}}'-  +  {(b+c)  (c  +  a)}2. 

19.  4(ai+cd)2-(a3+/^3-c2-r/2)3  = 
'i+b+c-d)  (a  +  b-c  +  d)  (c+d  +  a  —  b)  (c  +  d  —  n  +  b). 

20.  Find  the  product  oi  x^  +i/^  +z^  —  2xy  +  '2xz  —  2yz  and  x^-^ 
r^+z^-2xy-2xz-{-27jz. 

21.  (a;2  +^3+^:^3/2)  (x3  -a;.yi/2+y3)  (a;*  -^z*). 

22.  (l-6a  +  9rt2)  (L  +  2a  +  3«2). 

2'3.    {(»r+7i)  +  (jO  +  (/)}  (?n-7+7)-n). 

24.  Obtain  the  product  of  l-{-x+x^,  x^+x  —  1,  x^—x  +  1,  and 
l+x  —  x^. 

25.  («-/>3)2  (a-fi2)2  (a2+&4)s  (a^  4.^,8)8. 

26    Shew  that  {x^  +  xy  4-  y^)''  (x^  -  xy  ^  2/')'  -  (x^J/^)^  = 

Formula  A. — Examples. 

1.  Multiply  x'— a;+5_by  a;^  — «— 7  :  here  the  common  term  is 
x^  ~x,  the  other  terms  +5,  and— 7,  hence  the  product  =  (x^  —x)^ 
4-(_7  +  5)  (xj  -a:)H-(-7x5)  =  (a;^-a;)2-2(a;2  -  x)-  '6o  =  x^  - 
2x^-x^  +  2x-3o. 

2.  (x  —  a)  {x—3a)  (a;+4a)  {x+6a) :  taking  the  first  and  third 
factors  together,  and  the  second  and  fourth,  we  have  the  product 
=  {x^+3ax  -  Aa^){x^  +  3ax-18a^)  =  (x-+Bax)^  -  (da'-^  +  lSa^) 
X(a;2  +  3aa;)-72a4  =  &c. 

Exercise  viii. 

■    1,  {x''+2x+3)  {x''  +  2x-'k);  {x-y  +  3z)  (x-y  +  5z). 

2.  (a;+l)  {x+5)  (a;+2)  (a;+4) ;    (x^+a-b)  {x^+2b-a). 

3.  (a»-3)(a2_l)(a3  +  5)  (a3  +  7);    {x^  +  x-^  +  l){x*+x^ -^). 

4.  {{x+yy -ixy)}  {{x+y)^ +oxy}. 


rUNDAMKNTAL    POEMULAa.  17 

C.   (nx+y  +  S)  (nx+y  +  l). 

7.  (x+a-y)  (x-i-a  +  ST/). 

8.  (a;2«  +a;"  -  a)  (a;^"  +x"  -  b). 

11.  Multiply  together x- 2  + 1/ 2,  a;-2  +  T/8,  a;-2-i/2,  ana 
x-2- v/3. 

12.  (x+rt  +  ?))  {x+b-c)  (x-a  +  b)  (x+b  +  c). 

13.  (a+i+c)  (a  +  b-\-d)  +  {a+c  +  d}  (b+c+d)  -  (a+b+c+cr)^. 

14.  Prove  that 

{2a  +  2b~c){2b  +  2c-a)-\-{2c  +  2a-b){2a+2,b-o)  +  {'2b+2c-a) 
(2c  +  2a  -b)  =  9{ab+bc+ca). 

Formulas  [5]  and  [6] . — Examples. 

1.  We  get  at  once 

{x-hy)^  +  (x-7jy  =  2x{x^+3y'>). 
{x  +  y)^-(x-y)3  =  2y{Sx^+y^). 

2.  Simplify  {a  +  b+c)^ -S{a  +  b+cyc-\-S(a-{-b  +  ry^ -c^. 
This  plainly  comes  under  formula  [5] ,  the  first  term  being  n-\-h 

+c,  the  second  —c;  hence  the  expression  is  {(a  +  i/+c)— c}»  = 
{a+b)\ 

3.  Shew  that  {x-  +xy+y-)^ -^{xy  -x-  —y^)^ - 
6a;y(x*  +x-y'^  +y*)  =  6x^y^. 

This  comes  under  formula  [G] ,  the  first  term  being 
{x^+xy-\-y'^),  and  the  second-  {x-—xy+y^) ;  we  have  therefore 
{{x''+xy+y^)-ix"--xy+y^)}^=^{2xy)^  =  8x^yK 

'  Exercise  ix. 
Simplify 

1.  (l-a;2)3+(H-a;2)3,     (x^ +xy^)^ -(x'' -xy^)^. 

2.  (a  +  26)3-(rt-6)3,     (3a-6)s-(3a-2i)3. 


18  FUNDAMSNTAL    FORMt'LAS. 

3.  {x+y-z)^-{-Z(x-^y-z)'Z+z^-^-^x-^y~-z)z''. 

4.  (a-i)3  +  (rt  +  i)^+6«(«2_Z>=i). 

5.  {x-yY+{x+yy+Z(x-yY  {x+y)-^y-x)  (x^y)^. 

6.  {1+x+x')^  -{l-x+X'Y  -Qoil+x''  -^-x^). 

7.  (a_fe_c)3+(6+c)3  +  3(6+c)2(a_i_c)  +  3(a-Z»-c)2(64-5). 

8.  (3a;- 47/  -i-  oz)^  -(02  -  4v/)3  4-  3(5z  -  4^/)^  (3:c  -  ^y -r  bz)~ 
3(3x--47/  +  52)2(5s-4v/). 

9.  (l+x+a;2)3+3(l-a;3)(2-fx'-)  +  (l-a;)3. 

10.  Shew  that  a(a-26)3  -i(i -  2a)3  =  (a_i)  (^+6)3. 

11.  Shewthata3(a3_2i3)3_,.Z,3(2fl3_i3)3=(a3^/,3)/a34.i3)3 

12.  (x2+x^+7/-)3  +  8(x2+2/-)  (a:4+:^*'+2/*)+(x3-a;2/+^2)3. 

13.  Shew  that  aS^^s  +  263)3  4.  ^3(2a3  +  t3)3  a.  (3a-Z/-')3  =, 
fG+7a3i3_i.z,6)2_ 

14.  SimpHfy  {ax-h^yy+a^y^ -r-^^x^  —^(i'hxy{ax+hy). 

15.  What  will  a3^^3_|_c3  _3a6c  become  when  a-\-h  +  c  =  0  ? 

16.  Find  the  value  of  x*^  -y^''  -\-z^  -^-Zx-y^z'^  when  x-  —y-  +z'^ 
=  0. 

Formulas  [7] ,  [8]  and  [9] . — Examples. 

1.  Simplify  (2a;-8?/)3  +  (47/-  5x)^ +{^x~ y)^ - 
%2x-'3ij)  {4.y-5x)  {3x-y). 

By  [8]  this  is  seen  to  be  {(2x-  3«/)  +  (4y  —  5x)-\-i'3x-y)}^  = 
(0)3=0. 

2.  Prove  that(a— i)3  +  (i-6)3  +  (c-fi)3  =  3(a-i)  (b-c)  (c-a). 

In  [8]    substitute  a  — i  for  x,  b  —  c  for  y,  and  c  — a  for  2;  for 
these  values  x-\-y+z  =  Q,  and  the  identity  appears  at  once. 

3.  Prove  {a-^b  +  c)^  —{b  -\-  c  —  aY  —  {a-\-c—b)^  —{a-^b  —  c)'^  = 
24.abc. 

In  [7]  leta;  =  6+c  — a,  y  =  c-\-a-b,  z  =  a+b—c,  and  therefore  « 
4-//  =  2c,  '//-|-2  =  2a,  z+x='lb,  and  this  identity  at  once  appears. 


FtTNDAMKNTAXi    FORMULAS.  19 

Exercise  x. 

1.  Cube  the  following:  l—x-\-x-,  a  —  b  —  c,  1  —  '2x-\-r,x-  —4z^. 

2.  Simplify  (a-=  +2z-l)^  +  {9x-l)(x"  +  2a;-2)  — 
8.   *Prove  that  (x-\-y){y+z)(z+x)  +x?/z  =  (x  +  y-i-z)(xj/  +  i/z+zx) 

4.  Prove   that     {ax  —  by)"^  -f-  a^y'^'  —  h^x^  -\-^ahxy  {ax  ~  by)  = 
{a^  -  h^)  (z^ +y^). 

5.  Simplify  {x-27jy+{y-2zy^  +  {z-2a;)^+S{x-y-2z)x 
(y—z-2x)  {z—x  —  2y)  +  {x+y+z)^. 

6.  SimpHfy  (2a;-'  -  Sy^ +4z-)-  +  {27j-^  -  Sz=  +  ix^)^  + 
{2z-  -3x--\-iy-)^. 

7.  Simplify  {2ax-by)3+{2by-cz)^  +  {2rz—axy-\- 
B{2ax-\-by  —  cz)  {2by  +  rz  —  ax)  {2cz+ax  —  by) . 

8.  Prove  {x^ -^Qx-y  —  y^)^  +  {'^xy{x  +  y)]^  =  {{x-  y)'^ -\-^x^y] 
X  [x^+xy+y-]^. 

9.  Prove  ^x^+y^+z^) -{x->rV+z)^  ^{'^x-^  4y-^  z)  {x  ~  y)^  + 
(iy  +  iz+x)  {y-z)-+{iz-\-4:X+y)  {z  —  x)~. 

10.  Ti  x-{-y-\-z  =  0,  shew  that  a:3-(-?/3_|_23  _ga.^2_ 

11.  Ifa;  =  2!/  +  Sz  shew  that  sc^  —  St/^  —  27z^  —  lSxyz  =  0. 

12.  Shew  that  {x""  ^xy  +  y"^)^ +  {x'^  ~xy  +  y-)^+^z^ - 
Cz"-  {x^-'rx^-y''+y^)  =  0,i{x^+y^+z^'=0.       ' 

13.  Prove  that  8{a-\-b  +  r)^  -  {a^  b)^  -  (6  +c)3  _  (c+a)^  = 
S{2a  +  b+c)  {a  +  2b+c)  {a-[-b+2c). 

Prove  the  following  : 

14     {ax  —  hy)^  -\-b^y^  =a^i:^  ■\-?jahxy{by  —  ax). 


•Note  that  the  right-hand  member  is  formed  from  the  left-hand  one  by  changing 
additions  into  multiplications,  and  multiplications  into  additions;  hence  in  (x+y+ 
t).{x.y+y.z+z.x;  the  si<;ns  -f-  and  .  maybe  inteichauged  throughout  without  alter- 
ing the  value  of  the  expressioo. 


20  FUNDAMENTAL    FORMULAS. 

15.  a^+b^-tc^-3abc  =  ^{{a-b)^  +  {b-c)^+(c-a)^}  x 
(a+b  +  c). 

16.  (a  +  b  +  c)  {(a  +  b-c)  [b +c-a)  +  {b  +  c-a)  {c  +  a-b)  + 
{c  +  a-b)  {u  +  b  —  c)}  =  {a.  +  b  —  c)  {b  +  c-a)  (c+a  —  b)  +  8abc. 

17.  a^  +  b^+c3+2labc={a  +  b  +  c)^  -3{a{b-cy+b{c-a)^  + 
e{a-b)^}. 

18.  {a+b+lc){a-b)2  +  {b  +  c+-la){b-c)^+{c  +  a+7b){c-a)^ 

'=2{a  +  b  +  c)^  -5iabc. 

19.  (a+b  +  c)  {(2a-i)  (2b-c)  +  (2b-c)  (2c-a)  +  (2c-a)x 
(2a-b)}  =  (2a-b)  (2b-c)  (2c-a)  +  (2a+b-c)  (2b  +  c-a)x 
(2c  +  a-b). 

20.  li  x'^{ij  +  z)  =  a^,y^(z+z)  =  b^,  z^(x  +  y)  =  c^,  ariidixyz  =  abc, 
shew  that  a^ +b^ +c^ +2abc=(x+y)  (y-\-z)  [z+x] 

Expansion  of  Binomials. 

We  have  from  formula  [5] 

(a+b)^  =a^  +3a'^b  +  3ab^  +b^  ;  multiplying  by  a  +  b  we  get 
(a+-h)^  =  a^  +  '^i^b  +  Qa^b''' +Aab^  +  h^  ;  multiplying  this  by 
a+bviQ  get 

(a+b)^=a^  +  5a^b  +  lOa'ib^  +  \Q>a^b^  +  bab"^  +  bf^ . 

From  these  examples  we  derive  the  following  law  for  the  form-  . 
ation   of  the  terms  in  the  expansion   of  a+b  to  any  requked 
power  : — 

(1).  ThQinclex  of  a,  in  the^?-sf  term,  is  that  of  the  given  power, 
and  decreases  by  unity  in  each  succeeding  term ;  the  index  of  6 
begins  with  unity  in  the  second  term  and  increases  by  unity  in 
each  succeeding  term. 

(2).  The  coefficient  of  the  first  term  is  unity,  and  the  coefficient 
of  anv  other  term  is  found  by  multiplying  the  coefficient  of  the 
immediately  preceding  term  by  the  index  of  a  in  that  term,  and 
dividing  the  product  by  the  number  of  that  preceding  term.  It 
will  be  observed  that  the  coefficients  equally  distant  f^om  the 
extremes  of  the  expansion,  are  equal. 


MULTIPLICATION    AND    DIVISION,  21 

Exercise  xi, 

1.  Expand  (j;+ 3/) «,    {x  +  y)',    {x+y)\   (x+t/)". 

2.  "What  will  be  the  law  of  siifus  if  —  y  be  •written  for  y  in  fl)  '? 

3.  Expand  (a- 6)',    (rt-2MS    (26-a)4. 

4.  Expand  (!+?«)«,    (7«  +  l)^   (2ot+1)6. 

5.  What  is  the  coellicient  of  the  4th  term  in  {a—b)^°  ? 

6.  Expand  (x3_y)4^    (a-262)s,    (a^-^h^)^. 

7.  In  the  expansion  of  (a  —  b)^^,  the  third  term  is  66^^  "/y^,  find 
the  5th  and  6th  terms. 

8.  Shew  that  {x+y)'^—x'^—y'^  =  oxfj{x-{-ij)(z^+xy+y^). 

9.  From  (8)  shew  that  2{{a  -  b)^  +  {b  -  c)^  +  (c  -  a)"}  =^_ 
5{a-b)  (b-c)  (c-a)  {(a- 6)3  +  (6-c)3 +  (c- a)2}. 


Section  III. — Horner's  Methods  of  Multipucation  and 
Division. 


Examples. 
1.  Find  the    product   of    kx^-{-lx^+mx+n    and   ax^+bx  +  c. 
Write  the  multiplier  in  a  column  to  the  left  of  the  multipHcand, 
placing  each  term  in  the  same  horizontal  line  with  the  partial 
product  it  gives  : 

kx^  +Ix^  +mx         -|-w  ;...Q 


-^hx 
+  r 


akx''        +alx^         -\-amx^      -{-anx^  p^ 

-\-hkx*        -i-blx^         +hmx^  -\-bnx     p^ 

-{-ckx^        -\-clx^    •\-cmx+cn p^ 


akx^  -f  {al-\-bk)x*'  +  (aw  +  bl+ck)x'^  +  (««  +  bm  +  cl)x-  + 
{bn-\-cm)x-\-cn P. 

Art.  VI.  The  above  example  ha^  been  given  in  full,  the  pow- 
ers of  X  being  inserted  ;  in  the  following  example  detached  coeffi- 
cients are  used.  It  is  evident  that  if  the  coefficient  of  the  first 
term  of  the  multipher  be  unity,  the  coefficients  of  the  multiplicand 
will  be  the  pame  a?  th^se  of  the -first  partial  product,  and  may  be 
used  for  them,  thus  saving  the  repetition  of  a  line. 


22 


MULTIPLICATION    AND    DIVISION. 


2.  Multiply  3x4  -2x^-2x  +  3  by  a;3  +3.-?;  -  2. 


+3 
—  2 


-2     +0      -2 

+9     -G     +0 

-6     +4 


+  3 


■6     +9 

0     +4 


I  Sx^  +  lx^  -  12.c4  +  2.^3  _  3,,.3  +  13a; _  6. 
3.  Find  the  product  of  (x-3)  (a;  +  4)   (x-2)  (a; -5). 


+  4 

1 

-3 

+4 

-12 

4-24 

-2 

1 

^1 
-2 

-12 

-    2 

-5 

1 

-1 
-5 

-14 
+   5 

+  24 
+  70 

-120 

X4 

-  6x3 

-9x3 

+  94x 

-120. 

4,  Multiply  x3  -  4x2  +  2x  -  3  by  2x^-3 


1 

-4 

+  2 

-3 

2 

0 
0 
3 

2 

-8 
0 

+  4 
0 
0 

-6 
0         0 

')         0 
-3    +12 

0 
-6   +   9 

2x«- 

-8x-' 

+  4x'* 

-9x3+12a;2- 

-6x  +9 

[x^  X  X"*'  =X*] 


I  lu  this  example  the  missing  terms  of  the  multiplier  are  supplied 
iby  zeros ;  but  instead  of  writing  the  zei'os  as  in  the  example,  we 
|may,  as  in  ordinary  arithmetical  muitiplicatioa,  "  skip  a  line  " 
for  every  missing  term. 

5.  Multiply  x*  -2x3+1  \v  x*  -x3  +  3. 


1 
-1 
+  3 


1    +0    -2   +0+1  ,  ^       4_    ,, 

_1   _o   +2   -0    -1  1-^    ^^*   ~^  J 

+3   +0   -6   +0   +3 


dx^ 


+  6a-*      -7a;- 


+  3 


MUT.TIPLICATION    AND    DIVISION. 


29 


e.  Fin(3th«valueof(a;+2)(a-f8)(ar-f4j^a;  +  o)-9(x  +  2)(x+3} 
X  (.^  +  4)  +  3(j,  +2)(.r-h  3)  +  77(a;-i-  2)  -  85. 

+  5 
-9 


+  4 


-♦-3 


+  2  i 


1      -4 

+  4 

-IG 
+   3 

+77 

1      +0. 
+3 

-13 
+  0 

1     +3 

+  2 

-13 
+  6 

4-38 
-26 

+  78 
-85 

9 


7.  Find  the  coefficient  of  aj*  in  the  product  of  x  —ax^  +  bx^^ 
cx-T-d  and  x^-\-}jx-\-q. 

1     —a     +6      -c 
—  ap 

+  ?     ■ 


+  d 


Exercise  xii. 
Find  the  product  of 

1.  (l+a;  +  a;2+a;3+x4)(l-a+a;3_a:7+a.8_^.i24-;i-i3). 

2.  (l+xs)(l -a;5  +x^){l+x+x^  +x^ -{-x^). 

3.  (x-5)  (x+Q)  ix-l)  {x+S);  (2x^-x^-\.l)  {x^-x+2.) 

4.  {x=  +  5x3  _  iGa;  - 1)  {x^  -  5x^  -  16x  +  1). 

6.   (6x«-2=+2x*-2a;3+2a;2  +  19a;+6)  (3a;3+4a;+l). 
Obtain  the  coefficients  of  x^  and  lower  powers  in 

6.  (l  +  ^yx-ix^  +  ^^X^-^^^X^)  (1  -  ),X 

7.  Multiply  tlx'  -x^+2x-A  by  3a;5-2a;2 -x-l. 


13.3  _    1  ^.3  _     r.    -,.4\ 


524  MULTIPLICATION    AND    DIVtSION. 

Simplify  the  following  : 

8.  (x+l)  (a;+2)  (a;+3)  +  3(a;+ 1)  (a;  +  2)  -  10(x+l)-f9. 

9.  x{x+l)  {x+2)  {x+B)-dx{x+l)  {x+2)-'2x{x  +  l)  +  2x, 

10.  x{x-l){x-2){x-S)+dx{x-l){x-2)-2x{x-l)-2x. 

11.  (x-l)  {x+l)  {x+S)  {x  +  5)-14:(x~l)  (x+l)-rl. 

12.  Given  that  the  sum  of  the  four  following  factors  is  —  1,  find 
(1)  the  product  of  the  first  pair;  (2)  the  product  of  the  second 
pair  ;  and  (3)  the  product  of  the  sum  of  the  first  pair  by  the  «im 
of  the  second  pair. 

+x^      +a;i* 

13.  Given  that  the  sum  of  the  three  following  factors  is  equal 
to  —1,  find  their  product. 

(1)  x     +z'^    +x^     +a;'3 

(2)  x'^    +x^    +a;io    +x^^ 

(3)  a:4    +x^    +X''      +x^. 

Art.  VII.  "Were  it  required  to  divide  the  product  P  in  the 
first  of  the  above  examples  by  ax'^  +  hx+c,  it  is  evident  that  could 
we  find  and  subtract  from  P  the  partial  products  p^,  p^,  (or  what 
would  give  the  same  result,  could  we  add  them  with  the  sign  of 
each  term  changed),  there  would  remam  the  partial  product^,, 
which,  divided  by  the  monomial  ax^,  would  give  the  quotient  Q. 
This  is  what  Horner's  method  does,  the  change  of  sign  being 
secured  by  changing  the  signs  of  b  and  c,  which  are  factors  in 
each  term  oij)^,  p^,  respecUvely. 


(1) 

X     -hx^ 

(2) 

x^    +a;8 

(3) 

x^   +.r* 

(4) 

X«     +X'' 

ITDLTIPLICATION    AND    DIVISION. 


25 


Oi   ^ 


+ 


I   I 


4- 


+ 


+  1 1 


+ 

+"^ 

+ 

H 


+ 


+ 


+ 


I     I 


2 

m 

.2 

>» 

CO 

rS 

2 

- 

rw 

0 

<D 

0 

a 

© 

^ 

CO 

fcjD 

-♦J 

*cn 

0 

0 

P 

rO 

:^ 

a 

5 

tp 

'>- 

tp 

> 
"Ejd 

■^ 

S-i 

a 
qa 

0 

-f3 

0 

a 
0 

o 

o 
_'g 

3 
'? 

"-3 
1 

G 
0 

a 

^ 

0 
-4^ 

a3 

CO 

-+3 

a 

© 
'53 

> 

CO 

0 

f^ 

■© 

^ 

.2 

a" 

o 

a 

j:< 

^ 

_> 

-73 

<D 

"3 

^ 
-§ 

o 

.2 

?^ 

•g 

.a 

?c 

"0 

a 

_a 

1=1 

o 

% 

2 

'03 
>> 

0 

0 
CO 

Pi 

'3 

a 

0 
0 

a 

to 

-1^ 

« 

&i 

fl 

r^ 

a 

.2 

CD 

s 

a 

-12 

0 

2 

CQ 

0 

CO 

g) 

1 

-J-3 

% 

3 

0 

to 

p 

■rH 

a 
0 
0 

O) 

w 

a 

03 

0 

a 

1 

c» 

© 

-1-3 

a 

X 

^ 

^ 

CO 

rO 

(-; 

a 

0 

CO 

'> 

1 , 

o 

-1-3 

i=! 
0 

■40 
CO 

to 

?c 

-1-3 

fcC 

1 

a 

-1-3 

.2 
0 

0 
a 

o 

^ 
>< 

c 

rs 

a 
p 

CD 

CD 

0 
© 

tn 

1p 

.5^ 

a 

'> 

^ 

£ 

-4-3 

.9 

c3 

a 

c 

a 

s 

1-1 

1 

OQ 

'3^ 

0 

'  ' 

o 
to 

1-5 

n 

m 
■0 

-4J 

"5 

2 
'-1-3 

a 

0 

2 

a 
s 

-t3 
CO 

© 

c3 

"o 

0 
0 

2 

to 

0 
to 

"^3 

<fl 

3 

O 

CO 

to 

'S 

"^ 

0 
CO 

CO 

-(-3 

'Si) 

.3 
0 

oq" 
-1-3 

0 

pO 

CD 

■w 

0" 

"Eb 

'> 

0 

a 
0 

CQ 

0 

© 

-ts" 

"> 

2 

1 

0 

0 

Si 

<S 

-1^ 

a 

CO 

0 

"5 
eg 

CO 

5 

2 

tl-l 
0 

a 

a 

a) 

0 

CO 
c3 
^: 
m 
cS 

to 

a 

c3 

.2 

© 
0 
0 

-1-3 
ca 

© 

-1^ 

0 

0 

S 

0 

-1-3 

CO 

1 

>> 

OQ 

a 

s 

a 

a 

0 

i=H 

© 

a 

CO         • 
0)    ^ 

2 

0 
0 
-5 

CO 

2 

2 

a 

0 

a 
0 

a> 

CO 

1 

=0 

CO 
<D 

.a  s 

'S  'S 
0  0 

0 
CQ 

5  J 

1     en 

26  MTTLTIPLICATTON    AND    DIVISION. 

2.  Divide  Bx^  +lx^  -  12a;*  -+-2a;»  -  Bx^  +  13a:-  6  by  a;'  4- 3a?  -  2. 

j  3     +7     -12  .+  2   -8  +13   -6      ,    o^^.^^^i 
_3  _9     ^.   6   -0  -f6   -   9  \^    ^      -^  f 


+  2 


-4+0-4   +6 


3x4-2x34.  0   -2z+3 


Compare  this  example  with  the  second  example  of  Horner'a 
Multiplication,  performing  a  step  in  multiplication,  then  the  cor- 
responding step  in  division  ;  then  another  step  in  multipHcation 
and  the  second  (corresponding)  step  in  division,  and  so  on. 

3.  Divide  a; •  ~  3a;«  +  Ax^  +  ISx^  _  7V4  12  by  xJ  -  Bx^  +  3a:- 1. 


+  3 
-3 
+  1 


1     _3     +0     -4  +18  +0  -7  +12 

+  3     +0     -9  -86  -27 

-3      -0  +9  +38  +27  [a;7-a;3=x4]. 

+  1  +0  -3  -12  -9 


I  xi   +0     -3a;2-12a;-   9;       6a;2+  8a;  +3 
The  quotient  is  therefore  x^  —ox'^  -  12a;— 9,  and  the  remainder 
6a;2+8a;  +  3. 

4.  Divide  a;»  - 3a;'  -Ca;"'  +2a;4  +  5a;3  +  4a;2  +  1  by  x^  +  2a;- 1. 
The  zero  coefiicient  in  the  divisor  may  be  inserted,  or  it  may  be 
omitted  and  allowance  made  for  it  in  the  2a;  — line.  See  examples 
4  and  6  in  multiplication. 


—  2 
+  1 

1 

_3  +0  -5  +2  +6  +4  +0  +1 
„2  +6  +4  -4  -6  +2 

1  -3  -2  +2  +3  ^1 

1 

_3  _.2  +2+3-1;  0  +5  +0 

[x^  -r  a;3  =x^] .    The  quotient  is  therefore  x''  ~Bx* 
+  3a;— 1,  and  the  remainder  5x. 


2a;3-4.2a;2 


5.  Divide  lOa;" -lla;^-3a;4  +  20a;3  +  10a;3  +  2  by  bx^-Bx^A- 
2x-2. 


MXILTIPLICTl  ION    AND    D^V^SION. 


27 


Arranging  as  in 

,   10    - 
+3 
-2 

the  ordinary  method,  we  Jiave 

-11    -8  -f20      -t-10   -1-0  -f2 
6-3-6      +12 
-4  +  2      +4-8 

+  4       -2-4+8 

5 

2 

-1-2  +  4 

24-12-flO 

24x3-..12x+10 


5a;3_3a;3+2x-2 
We  first  draw  a  -vertical  line  with  as  many  vertical  columns  to 
the  right  as  are  lesg  by  unity  than  the  number  of  terms  in  the 
divisor.  This  will  mark  the  jjoint  at  which  the  remainder  begins 
to  be  formed.  We  then  divide  5  into  10,  and  thus  obtain  the 
first  coeflBcient  of  the  dividend.  We  next  multiply  the  remaining 
terms  of  the  divisor  by  the  2  thus  obtained.  Adding  the  second 
vertical  column  and  dividing  by  5,  we  obtain  —  1  ;  we  multiply 
by  the  —  1,  add  the  nest  column  and  divide  the  sum  by  5,  and  so 
on  for  the  others. 

This  method  is  not,  however,  always  convenient.  If  the  first 
term-  of  the  dividend  be  not  divisible  by  the  first  term  of  the  divi- 
sor, the  woric  would  be  embarrassed  with  fractions.  Yv'^e  may 
then  proceed  as  in  the  following  examples  : 

6.  Divide  z^-dx^+x^+Bx''-x  + 3  by  2a;3+a:3 -8x  +  l. 

Let  2x  =  y,  or  x  =  — . 
^  2 

Substitute  ~^  for  x  in  the  dividend  and  divisor,  and  we  have 


¥1 

2^ 


24 


+ 


y 


+  1^_ 
-1-     23 


+  3  - 


%3 

23    '      23  2     '    '"    ■      23      '    23 

_  y5  — 2x3?/4+227/3  +  23x8j/2-24?/+25x3 


+  — ^  +  1 


2^ 


y3+y8_2x3y+22 
p  ~~~ 

_  y^  -  6<At+  4</3  +  24y3  - 16?/+96 


■y3 +2/2 -6.7  +  4 A. 


28 


MULTIPLICATION    AND    DIVISION. 


Dividing  y*  -G?/^  +  4?/3  +24?/3  - 16?/+96  by  yS  4.^9  _  67/+4  by 
the  ordinary  metliod,  and  the  quotient  by  2^  we  have 


y^-7y+17 
23 


J_     39^^2  -  1147/  -  28 
p-    "73^^/2_62/-+.4  • 


Substituting  for  y  its  value  2a;,  and  simplifying  we  get 


2 


7.7;         17 


1       39x2 -57a; -7 
8'    2a;3+a;2_3a.+i* 


5. 


By  comparing  the  dividend  of  ^  with  the  original  question,  we 
find  that  we  have  multiplied  the  successive  coefficients  of  the  divi- 
dend by  2",  2^,  22,  &c.,  and,  omitting  the  first  term,  we  have 
multiplied  the  successive  coefficients  of  the  divisor  by  the  same 
numbers.  Dividing  then  by  Horner's  division  we  get  the  coeffi- 
cients-1,  -7,  17,  and  for  coefficients  of  remainder,  -39,  114, 
and  28.  The  first  of  these  divided  by  2,  22,  2^  are  the  coeffi- 
cients of  a;2  &c. ;  and,  -39,  &c.,  are  divided  by  1,  2,  23.  Hence 
the  work  will  stand  as  follows  : — 


a.5_3a;4+a;: 
12         4 

5+  3x2-     a;+   3-2a;3+x2_3s5+l 
8           16     32               12       4 

-1 

+  6 
-4 

1    -6     +4 

-1      +7 

+  6 

+  24     -   16+96 
-17 

-  42     +102 

-  4     +  28-68 

1-6+4 

1-7   +17 

a;2 
*  Quotient  =  -p   — 

-39     +114+28 
Ix         17         1     39x2- 

114.7;    28 
^^      4 

x^ 
2 

^          ^     2x3 +x 

7x         17         1        39x2 
4           8    ~    8'     2x3  + 

2-3X  +  1 

-57x-7 
x2  -  3x+"l* 

*It  v/iU,  in  general,  be  as  convenient  to  multiply  the  dividend  by  such  a.  nnia- 
her  as  will  make  its  first  term  exactly  divisible  by  the  ftrst  term  of  the  divisor,  and 
afterwards  divide  the  quotient  by  this  multiplier. 


MULTIPLICATION    AND    DIVISION, 

7.  Divide  5x'+2  by  3x^-2x+B. 


29 


5x^ 
1 

0    0 
3    9 

0 

27 

0    +2 
81    243 

+  2 

-9 

5 

0    0 

10  +20 

-45 

0 
-50 
-90 

0  +  486 
-280 
+225  +1260 

5 

+  10  -25- 

-140 

-  55  +1746 

+  2  H-  8a;2-2a;+3 


Coeffs.  of  Quotient  = 1 — 

3    ^  32        ys  34 


-2   +9 


1     65 --"^M 


Quoncnt= 4.  —    - 

3^9 


'J.^)X 


27 
Exercise  xiii. 


140 


84     3-2+3 

55a; -582 


81    3a;2-2a;+3" 


1.  Divide  Gx-^  +  5x^-llx^-Cix-+10x-2  by  2a:2+3a;-l. 

2.  (5x«  +  Gx^  +  l)r(x2  +  2^  +  l). 

8.   {a^-Ga  +  rj)-r{a^-%>-{-l). 

4.  (x-5  -4a;3^2  _  8a;2^3  _  17^.^4  _  i2f/S)-^(a;3  _2xv_3//2). 

5.  (a6  -  Sa'^x^+du^x^-x'')  ^  {a^  -'Sa^x  +  Sax^  -x^}. 
Divide 

6.  4a;4+3x2-3a;  +  lbya;2-2x  +  3. 

7.  lOx^+oa;'^ -90x3 -44x2  + lOx'+l  byx2-9. 

8.  x^  —x^y-\-x^  11^  —x^  11^ ■\-xy^  —y^  \>y  x^  —y^. 

9.  Multiply  x^  —  -ix^'i  +  6x2r(2  -4x^3  +  a*  by  x^  +  2xa  +  a-, 
and  divide  the  product  by  x^  —  2x3(/  +  2xa3  —  «*. 

Divide 

10.  x'>-ax4  +  /-x3-fex2+rtx-l  by  x-1. 

11.  6x''+7x4+7x3+6x2  +  6x+5  by  2x2+x  +  l. 

12.  60(x-'+^4)  +  91x//(x3_2/2)  by  12x2  -  13x(/  +  5y». 


so 


JVITTL.TIPIJCATTr'N    ANT>    DIVTSTON. 


13.  Gx'>  -iSlz''  -hldx-*^  +  81x^  -8-ix-  +8ex-  iSi  by*- 80. 

14.  Crx"^  -x^  +2x'^  -2x^  +2x^  +  19x  +  Q  by  Bx^+4x  +  l. 

15.  a(«+25)3-6(2a  +  6)3  by  (a-b)^. 

16.  {x+i/)^^-B{x+y)H  +  3(x-fy)z'--^z-'hy{x+y)^ -\- 
2(x+  !,)z+z"-. 

17.  10xi"  +  10a;«  +  10a;»-200  by  x' 4-:o=' -.c  +  1. 
J  8.  6?/?x^  4-  {lm-{-cm)x^  +cnx^  -\-abx+ac  by  hx+c. 

19.  Multiply  1+ 2^a;-18.'c3  by  1  -  L3a;3  4.  a^c^   and  divide  the 
product  by  l+yrc  — Sx^. 

Find  the  remainders  in  the  following  cases : 
'20.   (x'- H- 3:c2  +  4x  +  5)  -:-  (x -  2). 

21.  (x^-3x'-^+x-d)^{x-l). 

22.  (:c'i-f4a;3-f6a;  +  8)-(a;-h2) 

23.  ^27a:i-v/4)-^(Sa;-2^). 

24.  {3x^  +ox'^  -dx^-hlx^  -ox-\-S)-^{x"  -2x). 

25.  (5x4 +  90a;3 -1-80x3 -100a;+500)^(a;+ 17). 


Art.  viii.     The  following  are  examples  of  an  important  use  of 

Horner's  Division  : 

1.  Arrange  x^  —Gx^-\-lx—5  in  powers  of  a; -2, 

II  -6  7—5 

2  2         —8         —2 


1 

-4 

-1; 

-7 

2 

2 

-4 

1 

-2; 

—  5 

2 

-1-3 

1: 

0 

Hence,  a;3-6x"+7x-5  =  (a;—2)3-5(3;-2)-7,  or  as  it  is  gen- 
erally expressed,  x^  -  Gx'^  -f-7x— 5  =  ?/5  —  5y  —  7  if  y  =  «— 2. 


MULTIPLICATION    AND    DIVISION. 


81 


S.  Express  x*-hl2x-+-ilx-^  +  66x+28  in  powers  of.  x-^^6. 


-3 

1 

12 
-3 

47 
-27 

-GO 

2S 
-18 

-3 

1 

9 
-3 

20 
-18 

6i 
-G 

10 

-8 

1 

6 
-3 

2- 
-9 

0 

-3 

1 

3; 
-3 

—7 

1; 

0. 

Hence  x'^  +  12z^+^lx^  +6Gx+28  =  y/4  -  7?/-  +10  if  y=x+'6. 

After  a  few  solutions  have  been  written  out  in  full,  as  in  the 
above  examples,  the  writing  may  be  lessened  by  omitting  the 
Hnes  opposite  the  increments  (—2  in  Ex.  1,  and  3  in  Ex.  2),  the 
multiplication  and  addition  being  performed  mentally.  The  last 
example  written  in  this  way  would  appear  as  foUows : 

j  1  12  47         G6         28     . 

-3  11  9  20           6       (10) 

1  6  2          (0) 

1  3  (-7) 

1  (0) 

Exercise  xiv. 


1  Exp 

2. 

3. 

4. 

5. 

6. 

7. 

8 

y. 


ress  x^  —  5a;-  +3x  — 8  in  powers  of  x  — 1. 
x^-\-dx''+Gx+9  "  x+1. 

x^  -8a;3  +24x2  - 32a;+97  in  powers  of  x-  2. 


a;4  +  12a;3  +  5a;2_7 
3.c5-a;3  +  4a;2  +  5.T-8 
a;4-7a;3  +  llx2-7a;+10 
a;3_2x--4.c+9 
a;»-9a;27/+6x/y2-82/» 
x^  —  5x^1/  +  5x1/^  —  y* 


x+2. 

x-2. 

x-1^. 

x— §. 

x-2y. 

x-y. 


12 

SYMMETRY. 

If). 

'•       8x^-{-11x^y+10xjj^+8?j^ 

<« 

2a; +  y. 

11. 

-       x3-^x^+^x-^^4 

(( 

i^-TV 

12. 

"       a;*  +  8a;3-15a;-10 

<t 

x+2. 

CHAPTEE  H 


Section  I. — The  Principle  of  Symmetry. 


Art.  ix.  An  expression  is  said  to  be  symmetrical  with  respect 
to  two  of  its  letters  when  these  can  be  interchanged  without 
altering  the  exj)ression  : 

Thus  if  in  a^  +  a^x  +  ax^  +  x^,  we  write  x  for  a,  and  a  for  x,  we 
get  x^+x'^a+xa^+a^,  which  is  identical  with  the  given  expres- 
sion. So,  in  x^  +b^x-l-ha-]-a-x.  if  Vfe  interchange  a  and  5,  there 
results  x^  +  a -a; +  ai  + i^a;  which  is  identical  with  the  given  ex- 
pression ;  but  it  will  be  seen  that  the  expression  is  not  symmetrica] 
with  respect  to  x  and  b,  or  x  and  a. 

An  expression  is  symmetrical  with  respect  to  three  of  its  letters 
a,  b,  c,  when  a  can  be  changed  into  b,  b  into  c,  and  c  into  a,  without 
altering  the  expression. 

Thus  a^  +i3  4-c3  — 3abc  remains  unaltered  by  changing  a  into  b, 
b  into  c,  and  c  into  a,  and  is  therefore  symmetrical  with  respect 
to  these  letters.  So,  a^b-{-b^a-\-a^c+c^a+b2c-{-bc^,  and  (a-b)^ 
^  (6_c)3  -j-  (c  —  a)3,  are  each  symmetrical  with  respect  to  a,  b,  c. 

Again  (x-a)  (a  —  b)^  +  {x—b)  (b-c)^  +  (x-c)  (e  -  a)^  is  sym- 
metrical with  respect  to  a,  b,  c,  but  not  with  respect  to  x  and  any 
of  the  other  letters. 

Generally,  an  expression  is  symmetrical  with  respect  to  any 
number  of  its  letters  a,  b,  c,    .   .    .    h,  k,  when  a  can  be  changed 

into  b,  b  into  c,  c  into  d h  into  k,  and  k  into  a,  without 

altering  the  expression. 


SYMMETRY.  88 

A  symmetric  function  of  several  letters  is  frequently  represented 
by  writing  each  type-term  once,  preceded  by  the  letter  2  ;  thus  for 
a-|-6+e+  .  .  .  -\-l  we  write  2a,  and  for  ah-\-ac-\-ad-\-  .... 
•\-br-\-hd-\-  .  .  .  (?*.  e.  the  sum  of  the  products  of  every  pair  of 
the  letters  considered)  we  write  'Lab. 

Exercise  xv. 
Write  the  following  in  full : 

1.  2«26,  2(«  — ^)^,  2«(/'— c),  2«'^(x  — r),  y,aH^c,  Z{a  +  b) 
X{c—a){c—b),  2  [(a  +  c)--b^},  and  y,a{h-[-c)^,  each  for  a,  b,  c. 

2.  :SaAe,  y,a^b,  2a'bc,  2{a  — ^),  and  i;a2(a-i),  each  with 
respect  to  a,  6,  c,  d. 

Shew  that  the  following  are  symmetrical : 
8.  (x  +  a)  {((+b)  {h-{-x)-\-abx,  with  respect  to  a  and  b. 

4.  (rt+i)^  +  (rt  — 6)3  with  respect  to  a  and  b,  and  also  with 
respect  to  a  aud  — />. 

5.  (rt&— xy)^  —  ("+'j— 03  — y)  {«6(a;  +  y/)— a:iy(a  +  /-')}  with  respect 
to  a  aud  A,  aud  also  with  respect  to  x  aud  y. 

6.  a'-{b—c)  —  h^{(i  —  c)  —  c^{b —  a)  with  respect  to  a,  6,  c. 

7.  {ac-]-bd)-  -\-{hc  —  ady  with  respect  to  a^  and  b-,  and  also 
with  respect  to  c^  aid  r/-. 

8.  x^  +^*^+3x//(a;2+x//  +  //2)  with  respect  to  x  and  ?/, 

9.  {x^—y^+'Bxy{2x-\-y)}^  +  {y^-x^+3xy{2y-^x)}^  with  res- 
pect to  X  and  y. 

10.  a(a-}-26)3-4-/>(6  +  2a)3  with  respect  to  a  and  6,  and  also 
with  respect  to  a  and  —  b. 

n.  ab[{{a  +  c){b  +  c)  +  2c{a-hh)}^  -  (a-cy  (h-c)^]  with 
respect  to  a,  b,  c. 

12.  a^h"^  +b'^c^  +c'^a^  +2abc{a  +  b+c)  with  respect  to  ab,  be,  ca 
With  respect  to  what  letters  are  the  following  symmetrical  ? 
18.  xyz  +  5xy+2{x''+y'^). 
14.  2{a^x'^  +h^y3)-2ab{xy+yz+zx). 
1.5.  {P-k^)--\-iii^{  f+h\^->-(9j-h-2g-'y. 


B4  SYMMETRY. 

16.  (x+y)  (x—z)  {y  —  z)  —  xyz. 

17.  a^b'^-\-h^c^-{-G'^a'^-'labc{a  +  h-c). 

18.  aj6-//«+2«-3(x-2-2/'-')(;?/2-z2)   (2-^40-*). 

19.  (a.-H/,)3+(a  +  r;)2  +  (7;-c)4. 

20.  (a  +  i)4  +  ((»-c)4  +  (i+c)4  +  (fl  +  r)4. 

21.  (a4-^>)4  +  (a-c)4  +  (/;+c)4  +  (,<4-c)4  +  (c-     /,)4. 

Select  the  type-terms  in  : 

22.  «3+2ai  +  />2+2k  +  c2+2m 

23.  a(&2_c3)4-i(c3-a3)-fc(a2-5-')-}-(f^-f/>)  (^  +  ^')  (^  +  «)- 

24.  «(64-c)2+Z^(c-ha)3-fc(a-fi)2-12aic. 
Write  down  the  type-terms  in  : 

25.  (.x-+?/)5,  (aj _y)^(a5+Z/)'' --^^ -?/■'• 

26.  {x^yy  +  {x-y)\  (x+yY  -{x-yy . 

27.  (a;+?/+2)4,    {x—y-zy. 

28.  (a+6+c+t?)4,  (aO+&2_^c-'+ci2)2, 

29.  (a-fZ*)3a.(/;_|_o)3  +  (c-fa)3. 

Art.  X.  In  reducing  an  algebraic  expression  from  one  form 
to  another,  advantage  may  be  taken  of  the  principle  of  symmetry : 
for,  it  will  be  necessary  to  calculate  only  the  type-terms,  and  the 
others  may  be  written  down  from  these. 

Examples. 

1,  Find  the  expansion  of  {a-\-h-\-c+d-\-e-\-&o.Y 

This  expression  is  symmetrical  with  respect  to  a,  b,  c,  &e. ; 
hence  the  expansion  also  must  be  symmetrical,  and  as  it  is  a  pro- 
duct of  ^(X'o  factors,  it  can  contain  only  the  squares  a^,  b^,  c^,  &c., 
and  the  products  in  pairs,  ab,  ac,  ad  ...  ,  be,  bd,  &c.  ;  so  that 
a 2  and  nb  are  type-terms. 

Now  (a  +  b)-  =a-  -\-2ab-{-b^  ;  and  the  addition  of  terms  involv- 
ing a,  b,  c,  &.G.,  will  not  alter  the  terms  a^  -f  2«6,  but  will  merely 
give  additional  terms  of  the  same  type.  Hence  from  symmetry 
we  get 


SYJIMETKY.  ^5 

(a  +  ^-f  ,:4-i+f+&c.)-  =a-  +2fr6+2ac+2rtrf  +  2flt'+ 

+  ^2    +2i:"+2W+2k'  + 

a.c2    +2a/+2t;e-|- 

+  (/2   -\-2de  + 

+^2    +....,. 
This  may  be  compactly  written 

(2(t)2  =  £0.24-22^6. 

2.  Expaucl  (u  +  h)^. 

This  has  been  foimd  by  actual  multiiilicatioii — see  formula  [5] 
—but  we  may  also  proceed  as  follows  : 

(1)  The  expression  is  of  three  dimensions,  and  is  symmetrical 
with  respect  to  a  and  b. 

(2)  The  type-terms  are  a^,  a-b. 

Hence  («  +  6)3  =«3_|_^3  ^n[a~b-{-b-a),  where  n  is  numerical. 

To  find  the  value  of  n,  puta  =  ft  =  l,  and  we  have  (1+1)3  = 
l  +  l  +  ?i(l  +  l);  .-.  7i  =  3. 

b.  Expand  {x+y+z)^. 

This  is  of  three   dimensions,  and  is  symmetrical  with  respect 
to  a:,  y,  z.     AVe  have 

(x+i/+2)3  =  {{:x+,j)-¥z]  3  =  (:c+v/)a  +&C. 

=  x^'\-^x^y+k(i.,  Avhich   are  type-terms,  the  only  other  possible 
type-term  being  xyz. 

Now,  since  the  expression  contains  'dx'-y,  it  must  also  contain 
3x-2,  that  is,  it  must  contain  3x-(?/4-z)-     Hence 

(x  +  y^-z)-^  =  x^-^dx^-[y+z) 
+2/3  +  32/2(z+a;) 
+z^  +  dz^'{y\-x) 

+     n{xyz),  where  n  is  numerical,  and 
may  be  found  by  putting  x  =  y  =  z=l  in  the  last  result,  giving 

(1  +  ] +1)3  =  1  +  1 +  1.-1-8(1 +  1)4-3(1  +  1) +  3(1 +!)+»; 
.•.     n  =  G. 


36  SYMMETRY. 

4.  Similavly  we  may  shew  that 

{a  +  b-[-c+d)3=  a^-^3a-{b+c  +  d)  +  6bcd 
+  b^-\-'3b^{c  +  d+a:)  +  Gcda 
+  c^  +  Bc^{d+a+b)-[-6dab 
-\-  d^  +Bd^{a  +  b-\-c)  +  Gabc. 

5.  Expand  (a  +  b  +  c-^Szc.)^. 
The  type-terms  are  a^,  a^b,  abc. 

Expanding  (a  +  b  +  c)^,  we  get  a^  +  oa^b-\-6abc-{-&0» 
Hence  by  symmetry  we  have 

6.  Simplify  {a-{-h-2cy+{h  +  c-2a)^+{c  +  a-2b)^. 

This  expression  is  symmetrical,  involving  terms  of  the  types 
a^  and  ab.  Now  a^  occurs  with  1  as  a  coGfficient  in  the  first 
square,  with  4  as  a  coefficient  in  the  second  square,  and  with  1  as 
a  coefficient  in  the  third  square,  and  hence  Ga^  is  one  type-term 
of  the  result :  ab  occurs  with  2  as  a  coefficient  in  the  first  square, 
with  —4  as  a  coefficient  in  the  second  square,  and  with  —4  as  a 
coefficient  in  the  thiid  square,  and  hence  —  bah  is  the  second 
type-term  in  the  result:  hence  the  total  result  is  G  (a2-|-i2_^c8 
•—ab  —  bc—ca). 

7.  Simplify  {x-{-i/+z)^-^{x-y -z)^  +  {y~z-x)^ -\-{z  —  x-y)^. 
This  is  symmetrical  with  respect  to  x,  y,  z;  and  the  type-tenns 

are  x^,  Bx^y,  Gi'yz  : 

(1)  x^  occm-s  in  each  of  the  first  two  cubes,  and  —x^  in  each 
of  the  second  two  cubes,  :.  there  are  no  terms  of  the  type  x^  in 
the  result, 

(2)  Sx^y  occurs  in  the  ,^rsf  and  ihird  cubes,  and  —Sx^y  in  the 
second  and  fourth,  .'.  there  are  no  terms  of  this  type  in  the 
result. 

(3)  G.vyz  occurs  in  each  of  the  four  cubes,  /.  24x^2  is  the  total 
result. 

8.  Fi-ove  (a^  +  b^+c^+d^)  (rv^ +x^ -\-y^+z-)- 
{alV^^^bx-\-cy+dz)-  =  {'ix  -6w.')2 -f-(ay  — [cu,')2  +  {az  —  dwy -\- 
{by-cx)^+{bz-dx)^+{cz-dy)^. 


SYMMETKY.  87 

The  left  hand  member  (considered  as  given)  is  symmetrical 
with  respect  to  the  pairs  of  letters,  a  and  iv,  b  and  x,  c  and  y, 
d  and  t,  that  is,  any  two  pairs  may  be  interclianged  witliout 
affecting  the  expression.  As  the  expression  is  only  of  the  second 
degree  in  these  pairs,  no  term  can  involve  three  pairs  as  factors  : 
hence  the  type-terms  may  be  obtained  by  considering  all  the 
terms  involving  a,  b,  w,  x;  these  are  a^ic^,  a-x-,b^w',  h^x-, 
—  a^w^,  —b^x"^,  —  2rtiiia;,  and  are  the  terms  oi  {ax  — hw)^  -which,  is 
consequently  a  type-term.  From  (ax—bw)'^  we  derive  the  five 
other  terms  of  the  second  member  by  merely  changing  the 
letters. 

9.  Prove  that 
(a;2_y.)3  +  (^2_2;g)3  ^(z^-^x>jy-?.(x^-yz)  (y^-zx)  (z^--xy)  is 
a  complete  square. 

The  expression  will  remain  symmetrical  if  (x^—yz)  (y^—zx) 
(z^ —a;?/),  instead  of  being  multiplied  by  -3,  be  subtracted  fi-om 
each  of  the  preceding  terms,  thus  giving 

(x^  -yz)  {{x^-yzY-  {y^  -  xz)  {z'^—xy)) 
4-  (?/2  -  2x)  { (j/2  -  za;)  2  -  (z3  -  xy)  {x^     yz) } 
■^[z--xy)   {{z'^—xyY-{x--iiz)  (y^ -zx)} 
=  (x^  —yz)x{x^-{-y'^-^z^  —  2xyz) 
-f-ifec. 

+  &C. 

=  (a;  3  -|-?/  3  _j_^  3  _  pjxjiz)  (a;  3  +.V  ^  +s  ^  —  Sxyz) . 
Exercise  xvi. 
Simplify  the  following : 

1.  (a+64-c)2  +  (a+6-c)3-f(5+c-«)2+(c-f«-?;)3 

2.  (a-6-c)2  +  (i-a,-c)2  +  (c-a-6)2. 

3.  (a-t-Z^H-c-d)2  +  (i+c+(Z-a)2-{-(c-|-(Z+rt-i)2-|- 
{d+a-\-b-cY. 

4.  {a+b+cy  - a{b-^c -a)  -h{a-\-c -b) -c{a-\-b - c). 

5.  {x-\-y-\-z+ny  -\r{x-y - z-[-nf  +{jc-y-{-z  -n)- -\- 
{x-'ry—z  —  n)^. 

6.  (a+6+c)3  +  (a+fe-c)3-f(6+c-a)3-f(c+a-6)3. 


38  SYMMETRY. 

7.  {x -  2y  -  82) 2  +  (7/  -  2^ -  dx) 3 H- (2  -  2x  -  877)2. 

8.  (??ia+?i/*-l-rc)3  —  (ma-\-nb  —  rc)'^  —  (jii+j-c  —  ??/a)3  — 
(rc-f- WW  —  «^)  ^ . 

9.  «{6-}-r)(i2-fca-r/2)4.&(c+a)(c24-a2-62)_[_ 

c(«  +  6)((<2_^i2_c;-'). 

10.  (fl/y  +  6c+crt)2  _  2abc{a-}-h+c). 
Prove  the  following  : 

11.  {ax+h]/+cz}- -{-{bx-\-cy-\-az)^  +{cx-{-ay-{-hz)^  + 
{ax-'rcy-\-bz)^  ^{cx-\-by-\-az)^  -{-{hx-\-ay-{-cz)^ 

=  2(a2  +  (;2-|-c2j(a;2-{,2/2-|-22)-[-i(ai+ic+ca)(.ryH- ?/z +;?a:;. 

12.  {a  +  h+c)^+{b+c-a)^  +  (c+a-by-\-{a-\-b-c)* 

18.   (^a  +  b  +  c)^  =  Za'^  +  ilaH  +  eia2b^  +  V2Za2bc. 

14.  (£a)4=s«^  +  4Sa3ft  +  62rt2/;2  4-i2sa2ic  +  242rt6c^. 

15.  (a2+/j2+c3)3  +  2(a&+6c  +  ca)3-3(«2  +  i2+c;2)x 

(^,?;4.ic  +  C«)2  =  (a34.i3_|_c3_.3aic)2. 

IG.   (a-/.)2(6-c)2  +  (i-f)2(c-a)2  +  (c-a)2(a-&)2  = 
(a2+/>2^.e2_a6-r/c-6c)2. 

17.  (2rt-?;-c)-'(2&-c-«)2  +  (26-c-a)2(2c-a-6)2  + 
(2c-a-&)2(2a-fc-c)2  =  9(«2+fc2  +  c2_afe_6c-oa)2 

18.  (rtr2  +  2?9rs+Ps2){aa;2  +  2te//  +  fv/2)_ 
-( a?-a;  +  b {;nj  4  sa;) -j-csij}^  =  {ac-b^) {ry  - sx) 3 . 

19.  {u^  +ab  +  b''){c^  +  cd-\-d'^)  =  {aG  +  ad  +  MY~  -\- 
{ac-\-ad+bd){bc-a(l)-\-{bc-ad)^. 

20.  Sbew  that  there  are  two  ways  in  wliich  the  given  product 
in  the  last  example  can  be  expressed  in  the  form  p-  +p(j  +  g-,  and 
two  ways  in  which  it  can  he  expressed  in  the  form  p^  -pg+g^- 

21.  6{iv^ ^x^+ij^  +z^)^  ={w  +  x)'^  +{w-x)^  +  {iv-\-yy  + 
{w-y)^  +  {w  +  z)^  +  (w-z)^  +  {x+yy-j-{x-yy  +  {x+zy  + 

ix-z)^  +  {y+zV-\-{y-zy' 

22.  |{(«  +  />+6')5  +  (a-i-c)6  +  (5-c-a)6+(c-a-i)'>}  = 
x\\a-\-b  +  c)^-]-{a-b-cy  +  {b-c-a)^-t{c-a-by}x 
ij(a+6+c)2  +  (a-5-c)2-|-(i-c-a)2  +  ((;-«-i)2}. 


THEORY    OF    Dr%'ISORS.  89 

Section  II. — Theory  of  Divipors. 


Any  expression  which  cau  be  reduced  to  the  form  /7y"-f-^'e""^  + 
«c"""*+  •  .  •  .  +  ■  •  ■  +hx  +  k,  in  which  n  is  a  positive 
integer  and  a,  h,  c,  .  .  .  .  h,  k  are  independent  of  x,  is  called 
a  PoLTNOME  in  x  of  degree  n. 

The  expressions /(a;)",  F{x)",  pix)"",  are  used  as  general  syniL  . ; 
for  polynomes  ;  the  index  n.  m.  indicates  the  degree  of  the  poly- 
nome. 

Theorem  I,  If  the  polynome/(x)"  be  divided  hyx—a,  the 
remainder  wiU  be/(rt)". 

Cor.   1.    f{xY  —  f{aY  is  always  exactly  divisible  by  x  —  a. 

(Particular  case:  k"  — a"is  always  exactly  divisible  bya*  — «). 

Cor.  2.  If /(a)"  =  0, /(a;)"  is  exactly  divisible  by  x  —  a,  i.e.,f{x)" 
is  an  algebraic  multiple  of  x  —  a. 

Cor.  3.  if  the  polynome  /(a;)"  on  division  by  the  polynome 
^{x)'"  leave  a  remainder  independent  of  x,  such  remainder  will  be 
the  value  of /(a;)"  when  ^{x)'"  =  0. 

Examples.— Theorem  1.  . 

1.  Find  the  remainder  when  x^  —'lx^-\-13x^  —  IGx^  -\-  Ox  — 12  is 
divided  by  a;  — 5. 

The  remainder  wiU  be  the  value  of  the  given  polynome  when  5 
is  substituted  for  x.     (See  Art.  III.). 


1 

-7 
5 

+  13 
■    10 

16     +9 
15     -5 

-12 
20 

1 

-2 

3 

-1   '     4; 

8 

Hence  the  remainder  is  8. 

2.  Find  the  remainder  when  (x  —  a)^  +  (x  —  h)^  -j-  (a-\-h)3  is 
divided  by  ar+a. 

For  X  substitute  -  a,  then  {-2a)^ +(-n-h)^-\.  {a  +  />)  3  =  _  Sa^. 

8.  Find  the  remainder  Vfhen  x^+a^  +  l>^  +  {x-{-a){x+b){a-{'b) 
is  divided  by  x-i-a-\-b. 


40  THEORY    OF    DTVISOKS. 

For  X  substitute  —(a  +  l)  and  ^ve  get 
-(a  +  b)^+a^  +  h^+ab{a  +  b)  =  -2ah{a+bl     See  Formula  [6]. 

4.  Find  the  remainder  when  {x^  +  2ax  —  2a^)^{x^—2ax—^a'^) 
+  B2{x  -  «)4(a;+a)4  is  divided  by  x^  -  2«3.  ' 

x^  —  2a^  may  be  struck  out  wherever  it  appears. 

Tliis  reduces  the  dividend  to 

(2ax)^(-2ax)+B2{x-a)^[x  +  a)^  =  -IGoAx^ -^d2(x^  -a^)*. 

In  this  subafcitute  2a^  for  x^  and  it  becomes 
-64««  +  32rt.8  =  _32a8, 
which  is  the  required  remainder. 

Exercise  xvii. 

1.  Find  the  remainder  when  Sx'^-{-(iOx^ +  o4x^  —  60x-\-58  is 
divided  by  a; +  19. 

2.  Find  the  remainder  when  px^  —  dqx--{-'drx  —  s  is  divided  by 
z  —  a. 

3.  What  number  added  to  4;c'  4- 34aj4  +  5Sx^-h21x^  -  123a;-  41 
wiU  give  a  sum  exactly  divisible  by  2x-+13  ? 

4.  What   number   taken    from    lOaj^'^  -  20a;8 -lOa;^  - -SOa;*- 
8*9a;24-20  will  leave  a  remainder  exactly  divisible  by  10a;-  — 11  ? 

•  Find  the  remainders  from  the  following  divisions  : 
g.  (a;4-l)5_a;5  ^^^.  +  1,  aud  (a;+rt+3)3  -  (a;+«+l)3  ^  x+2. 

6.  x"+y''^  x-y;  x^''-{-y-'^  ^  x+y;  x^"+'^+y'^''^-^^  ^  x-\-y. 

7.  (a;  +  l)3+a;3  +  (a;-l)3-f-a;-2. 

8.  (a;-a)3(a;+rt)3  +  (a;2-262)3  -^a;3+.';^ 

9.  {x^  +ax+a^\{x^  -c(x+a^)-{x^  -'dax-^2a^)(x^  -\-Bf(x  +  2a-') 
-- x"- -\-2n^ . 

10.  (9a3+6rtZ»  +  4A3)(9a3-6a/;+4Z/2)(81r/4_30rt3i3  +  IGi*) -i- 
f3a-26)2. 

11.  a'{x-a)^-\'h-{x-b)^  -^x  —  a-b. 

12.  {ax  +  by)^-{-a^y^-\-b^'x^  —  8abxy{ax  +  by)  -r-  (a-i-b){x4-y). 
18.  x'^  +  ci'^ -{•  b'^  —  oabx -i- x  —  a  +  b;    also -4- a;  +  a— 6   aiso* 

x~  a  —  b. 


THEORY    OF    fciVlSuSS.  41 

14.  Any  polynome  divided  by  x  —  1  gives  for  reruaindev  the 
sam  of  the  coefficients  of  the  terms. 

Examples. — Cor.  1. 

1.  x'^-i-y'^  is  exactly  divisible  by  x+y. 

In  "  x^  -  a^  is  exacth'  divisible  by  a;  — «,"  substitute  ~y  for  a. 

2.  m.r^  —  px^  ^qx-{-}n  -r p +q  is  exactly  divisible  by  a;+l. 
This  may  be  written 

{)>  x"  - px- -\-(jx}  —  {;«(  — 1)3-  p(_i)2_|.^^_l)i.  ig  exactly  divi- 
sible by  a;- (-1). 

3.  {x- -{-Qxii-{-hj^Y -\-{x" -\-~xy-\-Ay^)'^   is  exactly  divisible  by 
(;>-+2?/)3.     For  (x3+G.r?/-f  47/3)5 -(_a;2_2a;y -42/2)5  ig  exactly 
divisible  by  {x'^+C)xy-\-Ay^)  —  {—x^  —  '2.xy—^y^),  which  is 
2(a;2-f  4a;iy+47/3)  =  2(a;+2?/)2. 

Exercise  xviii. 
Prove  that  the  following  are  cases  of  exact  division  : 

1.  a;'2n+i_f.j/2n+i  H-  a;  -f-y  ;  x-"  - y^n  ^  a:-f  ?/, 

2.  a-134.,/13  -^a;4_j_^/4  ;  a;3o^y3o  _i.a.(;+yC.  also -=- a!i«>4-2/"' 
also  -^-  X'  +y^. 

8.   (fla;+6?/)5  +  (te+fn/)-' -^  (a  +  i)(a;  +  ?/). 

4.  («a;  +  i?/-fc.;)3  — (6x+f?/H-fl2)3  h-  (a  — i).«  +  (i— c)^-i-(c  — a)2. 

5.  (2?/-cK)''-(2a;-?/)«-r3(7/-a;). 

6.  (2i/-a;)2'»+i+(2a;-?/)2"+i--i/+a;. 

7.  {my  —  nxY  —  {mx  —  ny)^  -^  (m+n)  (y  —  x). 

8.  (a:-|-2/)«  +  (a:-i/)«^2fx2+7/2). 

9.  (a;2-fx^+y2)3  +  (a;3_a;^  +  2/2)3H-2(a;3  +  //2}. 

10.  (,  +  i)9_(a-i)9  -^2/>(3^/ 2+^/2), 

11.  (•j'^  +  5ft.i-+i2)7_|_(a;2_Z,a,-+i2)T  ^2{x+b)». 

12.  (a  +  6)*«+2+(«-6)4«+2-=-  2(a3  +  i2). 

13.  {x^  +  Sxy{x-rj)-  y^\^  +  {x^  -Qxy{x-y)-y^}^-^2{x-yY 

14.  3x-3-.5a;2  +  4x-2-r-x-l. 


42  THEORY    OP    DmSOBS. 

15.  Any  polynome  in  x  is  divisible  by  «— 1  when  the  stun  of 
the  coefficients  of  the  terms  is  zero. 

16,  Any  polynome  in  x  is  divisible  by  ic+l,  when  the  sum 
of  the  coefficients  of  the  even  powers  of  x  is  equal  to  the  sum  of 
the  ('oefficients  of  the  odd  powers.  (The  constant  term  is  in- 
cluded among  the  coefficients  of  the  even  powers). 

Examples. — Cor.  2. 

1.  Show  that  a{a-\-1h)^  —  h{1a-\-hy  is  exactly  divisible  by  a +6 . 
By  Cor.  2,  the  substitution  of  —  b  for  a  must  cause  the  polynome 
to  vanish. 

Substituting ;  a{a-  2a)  3  -f  a{2a  -  a)  ^  =  -  a* +a*  =  0. 

2.  Show  that  (ab  —  xij)^  —  {a  +  b  —  x-y){ab(x-ry)  —  xy{a-\-h)}  u 
exactly  divisible  by  {z  —  a){y—a),  also  by  {x  —  b){y-b). 

For  X  substitute  a  and  the  expression  becomes 

{ab-ayy  -  {h  -  ij){ab[a  +  y)  —  ay{a-{-b)}  =» 
aHb-y)^-{b-y){a^{b-y)}=0. 

The  expression  is,  therefore,  exactly  divisible  by  ic  — a.  But  it 
is  symmetrical  with  respect  to  x  and  y,  hence  it  is  divisible  by 
y  —  a,  and  a,s  x  —  a  and?/  — «  are  independent  factors,  the  expree> 
sion  is  exactly  divisible  by  {x  —  a){y  —  a).  Again,  the  given 
expression  is  symmetrical  with  respect  to  a  and  b,  hence,  making 
the  interchange  of  a  and  b,  the  expression  is  seen  to  be  divisible 
hy(x-b){y-h). 

3.  Show  that  6{a'^-\-b^+c^)  -  Bia^+h^ +c^){a»  +  h^  +  c»)  is 
exactly  divisible  by  a+i+c. 

For  a  substitute  —  (6+c)  and  the  result  which  would  be  the 
remainder  were  the  division  actually  performed,  must  vanish. 
^6{-  {b-{-c)^  +b-  +c^}  -  5{-{b  +  c)^  +0^  +cS}  {{b-\-c)^  +  b^  +  c^} 
=  6{  -(6+c)5  +  6^  +c^}-j-dObc{b+c){b-^  +bc-\-c^).  See  [1]  and  [6]  . 

The  expansion  being  of  the  5th  degree,  and  symmetrical  in  b 
and  c,  it  will  be  sufficient  to  show  that  the  coefficients  ofb^,  b*Cf 
63c>  vanish,  she  coefficientr,  of  b^c^,  be*,  c^  being  the  coefficients 


THEORY    OF   DI^^S0R3.  48 

of  the  former  terms  in  reverse  order.     Calculating  the  coefficients 
of  these  type-terms  we  get 

6{ -564c -1063c8-...}+30(64c +  263^2  +  ...), 
which  evidently  vanishes.     Hence  the  truth  of  the  proposition. 
4.  Ifa  +6  +  c  =  0,  ^(a5  +b''  +c-=)  =  ^[a-  +b^  +c3)-^(a3  +  63  +c^). 

In  the  last  example  it  has  heeu  proved  that  the  difference  of  the 
quantities  here  declared  to  he  equal,  is  a  multiple  of  a  +  6-fc,  i.e., 
in  this  case,  a  multiple  of  zero.  Hence  under  the  given  condition 
they  are  equal. 

Exercise  xix. 

Prove  that  the  following  are  cases  of  exact  division  : 

1.  (ax  —  by)^  +  {bx-ay)^  —  {u^+b^){x'^-y''^)-7-a,b,x,  y,  a+\ 
x-y. 

2.  ax^  -  (rt2  +6)a;2  +62  -f-  ax-b.     (Substitute  ax  for  b.) 

c    J  {ax+bTj)^-{a-b){x+z){ax+by)  +  {a-b)^xz-^x+y. 
\  {ax-by)^  -{a  +  b){x+z){ax-by)-^{a  +  b)^.iz  -^  x  +  'y. 

4.  da^x^'-iax^ —10axy-^Ba^xy  +  2x^y-{-oy^  -r-2ax  —  y, 

6.  l-''2a*x—16-'32a^x--^4:-8a^x^  +  -Qax'^  —  x^  -i-  'Gax—2x^. 

6.  x^ +x^y'+X'y-^y^ -i- x^-ry. 

7.  {c-d)a^-\-G{bc-bd)a  +  d{b^c-b2d)  ^  a  +  3b. 

8.  3r{x-^y)^+y{^\x-y)'^  -^x-y. 

9.  a{a  +  2h)^-b{b  +  2a)^  -i-a-h,  also  H-  a+b. 

10.  a^-^2a^b  +  a^b^+a2x^-2abx^+b^x^  -i-  {a-h){x-ha). 

11.  a(ft-c)3  +  6(c-a)3+c(a-6)3  ^  {a-b),  (b-c),  (c-a). 

12.  a3{b-c)-\-h3{c-a)  +  c^{a-b)  -J-  (a-Z>),  (b-c),  (c-a). 

13.  a4(6-c)  +  64(c-a)  +  c4(a-6)  -4-  (a-b),  (b-c),  (c-a). 

14.  (a-6)2(c-J)2  +  (6-c)2(ci-a)3_(,Z_6)s^a^.)s  ^  ^a-b), 
{b-c),  {c-d),  {d-a). 

15.  {(a-i)2+(6-c)--^  +  (c-a)2}{(a-5)2^2  +  (&-c)2a2  + 
(c_a)263|_|(a_6jS^.4-(6-c)2a  +  (<:-a)2Z>}2    -^   (a-i),    (b-c), 
(e-a). 

16.  (a;+!/)(!/+~')(2+a;)  +a;.V2  -r  a.-  + J/+2. 


44  TBT'.OKY    OF    DIVISORS. 

17.  ab{a'  —  b^)+oc(b-  -c")-^ca{c^  ~a^)  -^a  +  b-^e. 

18.  {ab-bc-ca)l-an^-h^c^-c^(i^  -^a-k-b-c. 

19.  (rt  +  26)34.(26-3c)3-(8c-a)3+rt.3 4.863  _27c» + 
?+26-3c. 

20.  aH^+b^c^+c^a^  -Za^h^c^-~ab^bc-^ca. 

ExAilPLES. CORS.    3    AKD  2. 

1.  Find  the  value  oiAx^  -\-2x^  -  ox'^  +  23.C+6  when  2^2  =  3a5  ^  4, 
Since  2a;2  —  3a;H-4  =  0,  we  have  simply  to  find  the  remainder  on 

division  by  2a;2  — 3x+4,  and  if  it  is  independent  of  sc,  it  is  the 
yaiue  sought,  Cor.  3. 

14  0  9  -5       28  6 

3  ■  6  9  15-3 

~4  -8  -12   -20  4 

~2~|  "2  3  5  -    1;       0  10 

Hence  the  required  value  is  10. 

2.  What  value  of  c  will  make  x^  —  5x"  +7x  —  c  exactly  divisible 
by  a;- 2. 

If  2  be  substituted  for  x,  the  remainder  must  vanish.  Cor,  2. 

j  1.    -5         7      -c 

2  2     -6    2 

!l     -3         l;2-c 
Hence  2  — c=0,  or  c  =  2. 

3.  What  value  of  c  will  make  6x^  —  5x^+cx^-20x^+19z-5 
vanish  when  '2.x^=dx—l  ? 

By  Cor.  3,  the  remainder  must  vanish  when  the  given  poly- 
nome  is  divide  by  2a;2  —  Sx+l.  We  may  divide  at  once  and  find, 
if  possible,  a  value  of  c  that  will  make  both  terms  of  the  remainder 
vanish,  or  we  may  first  express  cx^  in  lower  terms  in  x,  and 
then  divide  and  find  the  requii-ed  value  of  c  from  the  remainder. 

1st.  Method,  (see  page  28), 

g      _10  4c       -160  304  -160 


3 
-2 


18         24     12C+36      36c -420 

-12  -16       -8c-24     -24cH-280 


8  4c-f-12  12c -140;    28c- 140      -24c+120 


THEOKY    OF    DIVISORS.  45 

Hence  28f;  =  140  and  2  It- =  120.     Both  of  these  are  satisfied  by 

2nd  Method,     x^  =  ix{3x-l)  =  ^x^- ^x=i{3x-l)-ix  = 
2\x-^  —  ix—  lix  —  i  ;  .-.  cx^  =  l^cx  —  ic. 

Substituting  for  cx^  in  the  given  polynome  it  becomes 

6.^5_5a;4-20x3  +  (13c  +  19)x-|c-5. 
Divide  and  ajjply  Cor.  3. 

6       -10           0  -160  28c +  304  -24c- 160 

3                  18         24  36  -420 

—2     -12  -16  -   24  280 

"6            8         12  -140;  28c -140  -2Tc  +  120 

"We  thus  obtain  the  same  remainder  as  by  the  former  method, 
and  consequently  the  same  result.  A  comparison  of  the  two 
methods  shews  that  they  are  but  slightly  different  in  form,  but 
the  second  method  shows  rather  more  clearly  that  c  need  not  be 
introduced  into  the  dividend  at  all,  but  the  proper  multiples  of  it 
found  by  the  preliminary  reduction  can  be  added  to  or  taken 
from  the  numerical  remainder,  and  the  "true  remainder"  be 
thus  found,  and  c  determined  from  it. 

Exercise  xx. 
Find  the  value  of 

1.  X*  -Bx^  +  ix^  —  Bx+4:,  given  x^  =x  —  l. 

2.  a:^-2a;4-4a;3+13a;^-lla;-10,  given  (a;-l)2  =  2. 

3.  2x-^- 7*4 4-12x3 -11x2  + 2a; -5  given  (x-l)3  +  2=:0. 

4.  3x6  +  llx=  +  10x3+7x2+2x  +  3  given  x'  + Sx^ -2x-t-5  =  0. 

5.  6x^  +9x«  -  IGx*  - 5x3  -  12x3  -  6x  +  60  given  8xA  +x-4  =  0. 
What  values  of  c  will  make  the  following  polynomea  vanish 

under  the  given  conditions. 

6.  x4  +  13x3  +  26x2+52x+ 8c,  given  x  +  11-0. 

7.  x4 -2x3—9x2 +2cx- 14,  given  3j-+7  =  0. 

8.  X*  — 4x3— x2  +  lGx  + 6c,  given  x2  =x+ 6. 

9.  2x4 -  10x2 +4cx+6,  given  x3 +  3  =  3x. 

10.  2x4+x3-7cx2  +  llx+10,  given  2x  =  5. 


4:6  THEORY    OF    DIVISORS. 

11.  4a-4+ra-2  +  110ic-105,  given  2a- 2  -   5a;4-15  =  0. 

12.  3x^-~16x'^  +  cx3-5x^-lUx-{-200,  given  x^  =  Sx-A. 

13.  What  values  of  ;j  and  g  vfiW  ma.'ke  x'^-\-2x^  —lOx^  —2^^+^l 
vanish,  given  x^  =  3{x  —  l)  ? 

14.  What  values  oi p  and  q  will  make  a^  ^  _  g^  1  0  _^  10^ k  _  15^16 
-}-29a*  —pa^+g  vanish,  given  (a^  —2)2  =«"  —  3  '? 

Theorem  II.  If  the  polyuome/(x)"  vanish  on  substituting 
for  a;  each  of  the  n  (different)  values  a-y,  a^,  a^  .     .     .     a„ 

f{xy  =  A{x-ay){x-a2){x-('^)      ....      {x-a„) 
in  which  A  is  independent  of  x  and  consequently  '6  the  coefScient 
of  af  in /(a;)". 

Cor.  If  /(a;)"  and  ^(a;)'"  both  vanish  for  the  same  ni  different 
values  of  a;, /(a;)"  is  algebraically  divisible  by  f  (a:)™. 

Examples. 

1.  x^-i-ax^+bx  +  c  will  vanish  if  2,  or  3,  or  —4  be  substituted 
lor  X,  determine  a,  b,  c. 

The  coefficient  of  the  highest  power  of  a;  is  1  ; 

.-.    x^  +ax2  +  bx  +  c=  {x-2){x-3){x  +  4:)  =x^  -a;2  -  14a;4-24. 

.-.    a=  -1;  b=  -14:  c  =  24. 

2.  x^+hx^-\-cx-{-d  will  vanish  if  —3  or  2,  or  5  be  substituted 
for  a;,*detennine  its  value  if  3  be  substituted  for  x. 

The  given  polynome  =(x-^-3)(x  —  2){x  —  5); 

.:  the  required  value  is  (3  +  3)(3-2)(3- 5)  =  -12. 

3.  aa;3  4. 3/;^2_|_3ca;+c?  will  vanish  if  for  a;  be  substituted  —3, 
or  1-,  or  1^,  but  it  becomes  45  if  for  x  there  be  substituted  3  ; 
determine  the  values  of  a,  b,  c,  d. 

The  coefficient  of  the  highest  power  of  a;  is  a; 

.-.    ax^  +  3b.v^-\-3cx  +  d=^a{x+2,){x-\){x  —  H) 
:.    a(3  +  3)(3-i)(3-li)  =  45;      .-.    a  =  2. 
:.    'lx^  +  3hx^  +  3cx+d  =  <2,{x+3){x-\){x-\\) 
.-.   6  =  f,  c-=  -'6k,  d=  H 


THEORY    OF    DIVISORS.  47 

4.  If  x^  +px^-{-qx+r  vanish  for  x  —  a  or  b,  or  c,  determine  p,  q, 
and  rin  terms  of  a,  b,  c. 

x^  +px^  +qx-[-r  =  (x  -  a){x  -  b){x  -  c) 

=  x^  ~{a  +  h-\-  c)x^  +  {ah  +  bc+ca)x  —  abc 
.'■  p=  —{a-^b-\-c)      or  —  2rt. 
q=       ab  +  bc+ca  or       Sr//; 
»•  =  —  abc  or  —  2  abc. 

0.  If  x^+jJX'  -i-']x-\-r  vanish  for  x  =  a,  or  b,  or  c,  determine  the 
polynome  that  will  vanish  for  x  =  b  +  c,  or  c+a,  ©r  a  +  h. 

Since  x^-\-px^  +qx+r  vanishes  for  x  =  a  or  b  or  c, 

a; 3  _2)a-2_|_,^^._^  -^j^  vanish  for  a;=  —  a  or  —6  or  —  c, 

and  — ^i-— a+^+c; 
But  the  required  polynome  will  vanish  for 

x=  —p  —a,  or  —p—b,  or  —p  —  c; 
that  is,  for  x+p=   -a,  or  —6,  or  —c. 
Hence  it  is         {x-^-j')^  —p{x+p)^+q{x+p)—r  = 

x^-\r'lpx^  +  {p^-\-q)x-\-pq  —r. 
The  following  is  the  calculation  in  the  last  reduction.     (See 
page  31). 

\         -p  q  -r 


1  0  q ;     pg- 

1  p;  p^  +  q 

1;  2p 

1  ■ 


6.  In  any  triangle,  the  square  of  the  area  expressed  in  terms  of 
the  lengths  of  the  sides,  is  a  polynome  of  four  dimensions ;  and 
the  area  of  the  triangle,  the  \engths  of  whose  sides  are  3,  4,  and 
5,  respectively,  is  6.  Find  the  polynome  expressing  the  square 
of  the  area. 

Let  a,  b,  and  c  be  the  lengths  of  the  sides,  and  A  the  required 
polynome. 

1st.  The  area  vanishes  if  any  two  of  the  sides  become  together 
equal  to  the  third  side,  hence  ii  a-\-b  =  c,  A  =  0,  and  consequently 
A  13  divisible  hy  a-{-b  —  r.  Similarly  it  is  divisible  by  b-{-c-a 
and  bv  r  +  a  —  b. 


48  THEORY    OF    DI^'lbORS. 

2nfl.  The   area  vani'^he'!    if  the    three   sides   vanish   tn^rether, 
hence  if  a-\-h-{-r  =  0,  .4  =  0,  aud  consequently  A  is   divisible  by 

We  have  thns  found  four  linear  factors,  but  A  is  of  only  four 
dimensions. 

.-.  A  =  vi(a  +  b  +  c){h  -^-c  - a){c+a-b){a+h -  c), 
in  which  m  is  a  numerical  constant. 
But  63  or  3G  =  m(3  +  l  +  5)(i  +  5-3)(6  +  3-4)(3  +  4-5) 

=f51Gm  ;  .-.  m  =  ^^. 
(The  above  includes  all  the  ways  in  which  the  area  of  a  triangle 
can  vanish,  for  the  vanishing  of  only  one  side  involves  the  equal- 
ity of  the  other  two,  or  if  «  =  0,  6  =  c,  and  .-.  a-^b  =  c,  which  is 
included  in  1st.  ;  if  two  sides  vanish  simultaneously,  the  three 
must  vanish). 

EXAJIPLES    ON    THE    CoROLLARY. 

7.  Prove  that  (z+l)^ ^  -x^  ^  -1x -  1  is  divisible  by 

Factoring  the  latter  expression  we  find  it  vanishes  for  ar  =  0,  or 
—  1  or  —  ^.  Substituting  these  values  in  the  former  polynome, 
it  also  vanishes.  But  these  are  different  values  of  x,  hence  the 
truth  of  the  proposition. 

8.  (x-\-y  4-z)^  -x^  -  y^  —z^  is  divisible  by 

{x-\-y  +  z)^  -  x^  —  y^—z^'. 

The  latter  expression  vanishes  il  j;=  —y,  so  also  does  the  former. 

By  symmetry  they  both  vanish  if  ^/=r  —  z  and  ii  z=  —x.  Hence 
they  are  both  divisible  by  {x-{- y){i/  +z'(z+x).  But  this  expres- 
Bion  is  of  three  dimensions,  as  also  is  the  latter  of  the  given  poly- 
nomes,  hence  it  is  a  divisor  of  the  former. 

9.  Prove  that  {{<(  +  by -}-{c  +  dy}{a-b){c-d)  + 

{{b+c)''  +  {a  +  dy}{b-c)[a  -  d)  +  {{b  +  d)''  +  {ci-ay}{b-d){c-a) 
is  algebraically  divisible  by  {a  —  b){c  —  d){b  —  c){a-d){b  —  d)(c  —  a) 
y^^a  +  b+c  +  d),  and  find  the  quotient. 

Let  a  =  b  and  the  former  polynome  reduces  to 
{(^a+cy+{a  +  dy}{a-c){a-d)-}-\{a-\-dy-{-{c+ay}{a-d){c-a) 


THEORY    OF    DIVISORS.  49 

which  vanishes,  the  second  comiilex  term  differing  from  the  first 
only  in  the  sign  of  one  factor,  having  (c  —  a)  instead  of  («  -  c). 

Hence  the  former  polynome  is  divisible  by  a  —  b,  and  by  sym- 
metry it  is  also  divisible  by  a  —  c,  by  a  —  d,  by  b  —  c,hyb-d,hyc  —  d. 

Again,  (a  +  b)^ +(c-\-d)^  is  divisible  by  {a-\-b)-}-(c -rd);  for,  on 
putting  a  +  6=  —  (c -f-c/),  it  becomes  {—{c-\-d)}''-{-{c  +  d)^  which 
=  0. 

Similarly  the  other  terms  of  the  former  of  the  given  polnomes 
are  each  divisible  by  a-\-h-\-c-[-d,  and  consequently  the  whole  is 
so  divisible. 

Now  all  these  factors  are  different  from  each  other,  hence  the 
former  of  the  given  polynomes  is  divisible  by  the  product  of  these 
factors,  i.e.,  by  the  latter  of  the  given  polynomes. 

Both  of  these  polynomes  are  of  seven  dimensions,  hence  their 
quotient  must  be  a  number,  the  same  for  all  values  of  a,  b,  c,  d. 

Putrt  =  2,  b  =  l,  c  =  0,  d=-l,  and  divide.  The  quotient  will 
be  found  to  be  —  5. 

...    {(a  +  by+{c-\-dy}{a-h)(c-d)  +  {{b  +  cy+(a  +  dY}  X 
(b-c){a-d)+  {{b+d)^  +{c  +  a)^}{b-d){c -  a)  =  -5{a-b){c-d) 
x{b-c){a-d){b-d){c-a){a-\-b+c  +  d). 

'  N.B. — It  is  not  always  necessary  to  find  the  factors 
of  the  divisor,  as  the  following  examples  show. 

10.  Prove  that  x2+x+l  is  a  factor  o^  x^'^+r,^ +  1. 

.-^2  4-a;+l  will  be  a  factor  oi  x^'^+x''  +1  provided 

a;i4+a;7  +  l=:0  if  x2+a;+l  =  U. 

Ifa;2+a:  +  l        =0 

.-.  x^+x-+x        =0 

.-.  x^+x^--\-x+l-=l 

.-.    X3  =1 

,  .  ^*=landa;i2  =  l 

x''  =x  and  x^^  =  x^ 
..x^'^+x'+l      =x^+x  +  l  =  0 
..  ic2+a;+l  is  a  factor  of  ic^^+x^yfl. 


60  THEORY    Ol-^    DIVISOKS. 

Art.  XII.     Two  other  methods  of  proving  this  proposition 

are  worthy  of  Botice. 

1st.  x^+x+1  will  be  a  factor  of  x^^+x''  +1  provided  it  is  a 
factor  of  {(.-t;i4+a;7  +  l)  +  a  multiple  of  (x^+x  +  1)}. 

a;^^+.<;' +1  differs  by  a  multiple  of  x^  i-x+l  from 
x^^+x''^{x^+x+l)+x^{x^+x-^l)+x'+x'^{x^+x  +  l)  + 
x{x^+x+l)  +  l 

=  x^^{x^-{-x  +  l)+x^{x^-^x+l)+x^(x^-^x+l)-ix\x---rx-rl)  + 
(x^+x+l) 

=  {x'  3  ^^-i  +a;6  _|.,^3_j_i)(^3  +a;  + 1). 

Hence  x^  +x+l  is  a  factor  of  x^ ^ -\-x'^  + 1. 

,-,    -,     x^^+x'+l         a;3i-l      x-1 

2nd.    — „ —-   =   -  . — —  .  -^ — -   = 

x^-\-x-t1  X'  —1       .c-^  — 1 

{x^^-l){{x^^-l)-x{x^^-l)] 

{x'  -  l)(a;3  -  1)  - 

(a;2  1_l)(a;>5_l)     ^     x{x^^-l){x^*-l) 
{x'-l){x^-l)  {x^-l){x'-l) 

But  we  see  at  once  that  on  reduction  both  of  these  fractions 
give  an  integral  quotient,  hence  {x'^'^-^x~ -i-1) -i-x^ +x  +  l  gives 
an  integral  quotient. 

11.  x^+x+1  is  a  factor  of  (x  +  iy  -x''  -1. 
If  a;-+ic+l  =  0,  (a^+l)^ -ic^— 1  will  vanish  also,  for  in  such 
case  a;+l=  —x^. 

...  ^r^j^xy  ^x'  -\  =  {-x'^y  -x-"  -X^-x^^-x''  -\, 
■which  by  the  last  example  vanishes  if  x^  +x+ 1  =  0  ; 
.-.  je-+^  +  l  is  a  factor  of  {x-\-\y  —x"^     1. 

For  X  substitute  —  and  multiply  by  y^  and  \f  respectively, 
V 
and  this  example  becomes 

a;2_j_a;,,y_j_^2  is  a  factor  of  {x-\-yy  —x''  —y''. 


THROUY    OF    DIVISORS.  51 

Exercise  xxi. 
Determine  the  values  of  a,  b,  c,  d,  e,  in  the  lollowing  cases : — 

1.  z^  +  Sbx"  +  'dcx-\-d  YiimHhes  lor  x  =  2,  or  3,  or  4. 

2.  x^+cx^+dz  +  e  "         "    a;=l|or  -3  or  4^. 

3.  z'^+bx'-\-cx+24:  "         "    z  =  2ov-d. 

4:.  ax^-{-bx'^+cx+dO.        "         "    x-=3or-5or2. 
C).  nx^+cx^ -dOx  +  e.        "         "    «=  1^  or  -4,  or  2^. 

6.  81x'i  +  6rx2+4f?a;+«     "         "    a;=  ij  or  -3}  or  1^. 

7.  ax'i'+bx^-hcx'  -81      "         "    ,«  =  |  or  f  or  3. 

8.  ax'^+cx^+dx+e  "        "    ,f  =  2  or  1^   or   -1   and  be- 
comes 14  for  a;=l. 

9.  ax^-{-cx-\-d  vanishes  for  x=i.\,  or  23-,  and  becomes  49  for 
x  =  3,  determine  its  value  for  a;  =  —  3. 

Given  that  x^  -  px^  +qx  —  r  vanishes  for  x  =  a,  or  b,  or  c,  deter- 
termine  the  polynome  that  vanishes  for 

10.  x  =  c.-\-l,  ori  +  1,  or  c  +  1. 

11.  a;  =  a  — 1,  or  6  — 1,  or  c  — 1. 

.1  1  1 

12.  x=l  -—,  or  1-—,  or  1-—. 

a  0  c 

13.  x  =  (tb,  or  be,  or  ca. 

14.  x  =  a^,  or  b^,  or  c^. 

f  r  ^ 

15.  x  =  a{b  +  c),oi-  b(c  +  a),  of  c(a-{-b).  ^a(J)+c)=q 1. 

I  ^*  i 

16.  x=  or or     - —  \ — '—=- l.,i- 

c  a  0  [    c  c  ) 

Prove  that  the  follovfing  are  cases  of  exact  division : 

17.  {X  --  1)  1  3  _a;G  _L(a;2  _3.  J.  1)2  ^  ^s  _2x^  +2x- 1. 

18.  {x-l)^^-x^-\-{x'^-x+l)^-^x^  —  2x^  +  2x-l. 

19.  (a;-2)io(2a;— 5)i»-«io  +  2iO(a;3-4.r+5)5^ 
a.3_0a;2  +  i3a._io. 

20.  (x2+4:X+3y^-x'^-x''-5x-S^x^+6x^+8x-\-B. 

21.  (9a;-4)2i(a;-l)2i-.a;3i-(9a;2-14a;+4)3i-=-(a;-l)x 
(9a;-4)(9x2_l4a;+4). 

22.  {6(a;-l)}i3_(2a;3  +  3a;-4)>3_[.(2a;3_3a;-f2)i3_^ 
{2;c^  +Bx-i){2x^  -8x  +  2){x--l). 


52 


THEORY    OF    DIVISOKS. 


23.  {2{z+l)(x-2)]-^-'  -\-(x^  -Sx+3)^'  -idx^  -5x-iy^ 
ix-]-l){x-2){x^  -Sx-\-3){3x^  -5x-l). 

24.  {6{x-  !)}!«- (2a;3  +8a;-4) i  ^  -  (2a;=— 3a;+2)i  ^  + 
2(2x2+3a;-4)8(2a;3-8a;  +  2)4^(a;-l)(2a:2+3a;-4)(2a;2-3a;  +  2) 
.     25.   {2(x+l)(a; -2^,[2o_fa;2_3a.+3)3o_(3a.3_5a:_i)2o_^ 
2(a;--3a;  +  3)9(3a;3-5a;-l)ii^(.T4-l)(x-2)(a;3-3ic+3)  x 
(3a;2-5a;-i). 

26.  l+a;4+a;8  -^  l-frc'4-a;3. 

27.  a;io+a;^?/^+^yio  -^  a;3-|->'^^•//  +  ?/^• 
28.   l-^x^+x^+x^+x'^^  -j-  l  +  x+a;2+ic3+a;*. 

29.  l+x^+x^-\-x^^  +  x^^  H-  l  +  a:+a:3+a;3+a;4. 

30.  a:i^+a!i°^^+a;^?yi*'4-//'^  -^  x^+x-y+zy'^-\-y^. 

31.  a;!' +a;-i+a;3+x--J-l  -=- a;4+^--+.c3+x+l. 

32.  l+x+x--{-x^+x^+x^+x^^  ^ 

1  +a;+a;2  +a;3  +a;4  +a;*  +x6. 

Find  the  quotient  of  the  Ibllowing  divisions  in  which  B  denotes 
the  product 

{b-c){c-a){a-b){a-d){b-d){c-d)  ; 

33.  {h^c^  +a^d2){b-c){a~d)-i-{c2a^  +b^d^){c-a){b-d)  + 
{a^b''+c^d'^){a-b){c-d)  ^  D. 

34.  (bc+ad){b^  -c^){a^  -d^)  +  {ca  +  bd){c^  -a^){b^—d^)  + 
{ah+cd){a^-b^){c2-d^)  -r-  D. 

35.  {b  +  c){a  +  d)(b^-c--^){a^-d^)-{-the  two  similar  terms  h-  D. 

36.  {b^+c^){a2+d^){b-c)(a-d)+  "  "  -^  i). 

37.  {6c(6  +  c)3+a£i(a  +  tZ)3}(6-c)(a-f/)4-  "  -^  D. 

38.  {6c(i  +  c)  +  a%^+rf)}(&3_c2)(a2_,/2,^.  »  _^  j». 

39.  {6c(63+c3)+af/(a3+cZ3)l(ft_c)(a-(i)+  "  -=- D. 

40.  (6+c-a-rf)4(6-c)(a-(;)+  "  -^  D. 

41.  The  sum  of  the  fractions  ]-,  |,  i, ^,  increased  by  the 

sum  of  their  products  two  by  two,  increased  by  the  sum  of  their 

products  three  by  three, increased  by  their  product  is 

equal  to  n. 


THEORY   OF    DIVTS0E3.  53 

42.  In  any  trapezium  the  square  of  the  area  expressed  in  terms 
of  the  lengths  of  the  parallel  sides  and  the  diagonals,  is  a  poly- 
nome  of  four  dimensions,  determine  that  polynome. 

43.  In  any  quadrilateral  inscribed  in  a  circle,  the  square  of  the 
area  expressed  in  terms  of  the  lengths  of  the  sides,  is  a  polynome 
of  four  dimensions,  find  that  polynome. 

Theorem  III.  If  the  polynome /(a;)"  vanish  for  more  than 
n  different  values  of  x,  it  vanishes  identically,  the  coefficient  of 
every  term  being  zero. 

Cor.  If  a  rational  integral  expression  of  n  dimensions  be  divi- 
sible by  more  than  n  linear  factors,  the  expression  is  identically 
zero. 

Examples. 

{z-a){x  —  h)  {x—b){x-c)  {x-c)(:>-—a) 

^'  {c^a){c^^b)  +  (a-b){a-c)  "^  (b'^){b':^)  -^  =  0,  if  a, 
b,  and  c  are  unequal ;  for  this  is  a  polynome  of  two  dimensions  in 
X,  but  it  vanishes  for  x  =  a,  and,  therefore,  by  symmetry  for  x  =  b, 
and  for  a;  =  c,  that  is,  for  three  different  values  of  x,  hence  it 
vanishes  identically. 

2.   {(a  +  5)2-f(c+c/)2}(rt-ft)(c-J)  +  {(6-f6)3  +  (5  +  c03} 
{b-c){a-d)  +  {{c  +  a)^  +  {b  +  dy}{c-a){b-d}  =  0. 

Substitute  b  for  a  and  the  expression  becomes 
{(6+c)2+(6  +  rf)2}(6-c)(6-rf)  +  {(c  +  &)3  +  (5+^/)2t(c_6)(6_rf) 
which  vanishes,  hence  the  given  expression  is  divisible  by  a~b, 
and  consequently  by  symmetry  it  is  divisible  by  (a—b),  (b  -  c), 
(c  —  d),  {a-c),  (b  —  d),  and  (a-d),  But  the  given  expression  is  of 
only /o?tr  dimensions,  while  it  appears  to  have  six  linear  factors, 
hence  it  vanishes  identically. 

Exercise  xxii. 
Verify  the  following : 

X^r-Z^    ,    (x--b--)(y^-b2)(z^-b^)     (X^'-C^){y2_c!i)(z2_c2) 

z=x^+y-+z^-b^-c^. 


54  THEORY    OF    T>TVTS''Pg, 

1  1  1 


{x-\-<(){a-b){ci-c)       [z+b){b-c){b-a)      {x+c){c-a){c-b) 
1 


(x  +  a){x-\-b){x+cy 
5.  bc{b^—c-)  +  ca{c"  —  a-)-{-nb(n^  —  b^>  = 

{a  +  b  +  c){a^{b-c)^b'^c-a)-\-c^[a-b)}. 

,,  a-\-x  a+y/ ^  a+z  ^     a 

x{.c-y)[x—z)   "*"   y{v-x){y-z)         z{z-x){z-y)    ~   xtjz 

"^2  (5  _  cy+F 2  (c  -  a )  +  c'^  («  -  ^) 

(«,3  +  ?,3)(,C+,p)(^3+J2). 

(rt -i)(6  — c)(c  — rt) 

10,  {-x+y+z){x  —  y-\-z){x+y—z)-{-x(x  —  y+z){x-hy-z)  + 
y{x  +  y-z){-x  +  y+z)+z{-x+y+z){x-y-\-z)  =  Axyz. 

(^3-^2)3  4-(&3-c2)3^(g3_^3)3 

(a+l;)(6+c)(c+ttj 
(a-6)3  +  (Z'-c)3+(c-a)3. 

2(x//  +  7/2+2a;)2. 

Theorem  IV.  If  the  polynomes  J\x)".  (pix)"^  {n  not  less  than 
w)  are  equal  for  more  than  n  different  values  of  x,  they  are  equal 
for  all  values,  and  the  coefficients  of  equal  powers  of  x  in  each 
are  equal  to  one  another. 


THEORY    OF   DIVISORS.  55 

(This  is  called  the  Principle  of  Indeterminate  Coefficients.  The 
full  use  of  it  cannot  l)e  exhibited  till  the'student  is  able  to  work 
simultaneous  equations.) 

Examples. 

{a-bXa-c){a-d)    +   [b-<i){b-cXb- d)    + 

':^ rfs 

{c-a){c-L){c~d)    +    {d-a){d-b){d-c)    "    ^• 

Assume 


ix-a){z-b){x-c){x-d)    " 
^i  B  C  D 

m  which  A,  B,  C,  B  are  independent  of  x. 

Multiply  by  {z-a){z-h){x-c){x-d). 

:.  x^  =  {A  ■\-B+  C'+ Z))j;3  +terms  in  lower  powers  of  x. 

Now  this  equahty  holds  for  more  than  three  values  of  x,  hold- 
ing  in  fact  for  all  finite  values  of  x. 
:.    A^B-^C+D  =  Q 

Again  multiply  both  sif  es  of  (a-)  by  x^ci 

{x-b)ix-c){x-d)  -    ^  l^^rr  +  -^~;r  +  -^-TJ^-^-'*)' 

Put  x  =  a 


m 


a- 

=    A. 


{a-b)(a-c){a-d) 

^'  ^^"""'"-^    (.-.)(/-c)(6-J)    =  B.  *o. 
Adding 

(<^-'^)i"-<^)(«-^)   "^   (i-a)(6-c)(7>-rf)    +   {c-a){c-b)(c-d) 

(a-b){a-c)      "*■     (6-c)(Z;-a)     +   ~(7^a)(V=6)" 
=  (a-r6  +  c)2. 


66  THEORY    OF    DIVISOES. 

Assume  x^—i)x^-^qx—r  =  (^x  —  aj(x  —  b)[x  —  c).  (<^). 

.".  x^+j)x^+gx+r—ix+a)[x+b]{x  +  c).  ((3). 

x^+px^  +  gx^-\-rx  A       ,      B      ,       C 

-  3  —^ =x+^p+ \ J  +    ■ (y). 

x-'  -px^+qx  —  r  ^       x  —  a        x-b       x  —  c         ^' 

Multij)ly  by  x^  —px'^+qx  —  r  and  equate  the  coefficients  of  the 

terms  in  x-.     {In  multiplying  the  fractions  in  the  right-hand 

member  of  {q'),  use  the  factor  side  of  («).} 

q  =  q-2p-+A  +  B-{-C 

:.   A  +  B  +  C=2p^. 

Multiply  both  members  of  (y)  by  ic  —  a 

x(x  +  a)(x+b)(x-i-c)  (        ^  B  C   )  , 

—  7      7  -/         r^~    =  A-\-]x-\-2p+   J  + (x  -  a). 


Put  x  =  a, 

2a^{a  +  h){a  +  c) 
[a  —  h){a—  c) 
By  symmetry 


=  A. 


2fe2(64-r)(6  +  a)  _  ^c^{c  +  a){c+b) 

^b-c){b-a)     -^^-^^     {c-<(){c     6)      -^' 
a^a+b){a  +  c)  _^  bJ{b-}-c){b+a)  ^  c^(c+a){c-^b) 
{a  —  b){a  —  c)  \b  —  c)(b~a)  {c  —  a){c—b) 

=  ^(A+B  +  C)=p^. 
=  (a  +  b  +  c)^. 
3.  Extract  the  square  root  of  1+x-j-x^  -}-x^  -\-x^  -\-&c. 
Assume  the  square  root  to  be  l+ax+^x^-\-cx^-\-(lx'^-{-&c. 
.-.  l-^-x-^x^  +x^-\-x^-{-&c.  =  {l  +  ax-\-bx^-{-cx^  +dx^+  &c.y 
=  l-\-2ax  +  {a^  +'Ib)x^-\-2{ab+c)x^  +  ^2d-{-2ac  +  b^)x^  +  &c. 
.♦.  2a  =  1  .•.  «  =  ^ 

2h^a^  =  l  .:  6  =  i(l-i)    =1- 

2{c+ab)  =  l  ...  c  =  i-(^x|)=A 

2d  +  2ac-^b^  =  l      .-.  d^h{l-r%-^\}  -  x%V 
.-.  ^/{1+x+x^ -{-S^G.)  =  l-\-lx  +  ^lx-  +f\x^  +  j\\x^  +  &C. 
(Note. — As  it  is  frequently  necessary  to  determine  the  coeffi- 
cient of  a  particular  power  of  x,  a  few  ipreliminary  exercises  are 
given  on  this  subject.) 


THEORY    OF    DIVISORS.  57 

Exercise   xxiii. 

Determine  the  coefficient  of 

1.  x"^  m  {l-\-ax^^-^(l  +  bxy+{l-cxy. 

2.  x^  in  {l+x-\-2x-  +^x^)il-x+3x-  +x^  -5x^\ 

3.  X*  in  (l+.c+2.>-3+3a;3+4a;*+&c)(l-a;+x3-a;3+a;*-&c). 

4.  jc3  in  ^(a;-6)(a;-c)(a;  — rf)  +  i:>'(a;-a)(a;— c)(a;— d)  + 
C{x  —  a)(x  —  b){x  —  d)  +  D(x  —  a)(x  —  b)(x  —  c). 

5.  a;*  in  (l-aa;)3(l  +  a5:)«. 

6.  a;4  in  (H-«a;)3(l -ix-)^. 

7.  In  the  product 

(l+ax+bx'^+cx^+&c){l-ax+bx^-cx^-}-&<i.) 
prove  that  the  coeffi.cients  of  the  odd  i)owers  of  x  must  be  all 
zeros. 

Determine  the  value  of  the  following  expressions : 

,.1  1 


{a-b}{a-c){a-d)    ^   (b-a){h-c){b-d) 

1 1 

{c-a){c-b){c-d)  "^  {d-a){d-b){d-c)' 

q  ^  I  * 

\a-b){a-c){a-d)    ^    {b-a){b-c){b-d)    "^   *"" 

It)-    1 Tw w jr  -h  three  similar  terms. 

(a  —  o)[a—c){a  —  d) 

11.    ^ +        "  " 

(a  —  b){a—  c)  {a  —  d) 

l-^-    7 TTT \i 1\    +  three  similar  terms. 

(a  —  b){a  —  c){a  —  a) 

13. 4-        ««  " 

{a-b)(a-c){a-(I)    ^ 

a{a+b){a+c) 

{a-b){a-c)  ^      '^'^ 

jg^    a3(a+fe)(«+c)  _^        ,^ 

(a  — 6(a  — c) 


•       (a_i)(a-c)  "^ 


68  THEOKY    OF    DIVISOKS. 

17     a{a+b){a+c){a+d)  --14. 

'• '  •    7 TVT TT T\  +  three  similar  terms. 

{a  —  b){a—c){a  —  d) 

cr-(a  +  b)(a  +  c){a+d)  _^ 

^^-      {a-b){a-c){a-d)~^ 

(a— 6)(a  — c)(«.  — (Z) 

20.  , ^rv>        .  4-two  similar  terms. 

[a—o)[a—c)    ' 

[For  numerator  use  x^.-{-2px^-{-{i}^-^q)x  +  (pq  -  r).} 

{2a+b\2a  +  c) 

21.  -7 TTT T"  -f  two  similar  terms. 

(a  —  b){a  —  c) 

[For  numerator  use  x^  —  2j9a;-  +  (/>'  i-?)*;  —  (f  9 —■'■)•] 

a(&  +  c)  .      , 

22.    PT/ "\  +  two  similar  terms. 

{a  —  b){a—c) 

[For  numerator  use  x{x+p).'] 

b+c+d  ^^  .    „ 

23.  -, TTT r? 3^  +  three  similar  terms. 

{a  —  b){a  —  c){a —a) 

a^(bc  +  cd+db) 

^^-    {a-b){a-c){a-d)^ 

bc-i^-cd  +  db 

OK ,J 4_       <«  "  «« 

-^^^    (a-5)(a-c)(a-^)  ^ 

Extract  the  square-root  of  (to  4  terms)  : 

26.  1+x.  I    27.  1-a;.  |     28.  l  +  2.x-}-3a;* +4x3-1.  &c. 

29.  1  -  4a;  +  10a;3  -  20a;3  +  35a;*  -  56a;'  +  84a;« . 

30.  Extract  the  cube-root  of  1+x.     (To  4  terms). 

Art.  XI.  1.  Find  the  condition  that  pz^+2qz-\-r  and  p'x^ 
+  '2q'x+r'  shall  have  a  common  factor. 

Multii^ly  the  polynomes  hj  p'  and  p  respectively,  and  take  the 
difference  of  the  products,  also  by  r'  and  r  respectively,  and 
divide  the  difference  of  the  products  by  z. 

p'px^  +  2p  'qx  4-_p  V  I         ;;)•  'x^  +  2qr'x + rj- ' 

pp'x-  +  2pq'x+pr'  I  p'rx^-\-2q'rx+r'r 

2(  pq '  —p 'q)x+{pr' ~p '/•)         !        {pr'-p'r)x  +  2{qr'  -r'q). 
Multiply  the  former  of  these  remainders  by  (pr'-p'r)  and  the 
latter  by  2{pq'—p'q),  and  the  difference  of  the  products  is 
{pr'  —p'r)^  —A.{pq'  -  p'q){qr'  —  r'q). 


TBEORY    OF    DIVISORS. 


59 


But  if  the  given  polynomes  have  a  hnear  factor  this  remainder 
must  vanish,  or 

(pr'  —p'r)^  =A(pq'—p'q){qr'  —  r'q). 
U  the  given  polynomes  have  a  quadratic  factor,  the  linear  re- 
mainders must  vanish  identically,  or  (Th.  III.) 

pq'—p'q  —  O,  pr'—p'r  =  0,  and  qr'—r'q  =  0, 

p  q  r 

or,  _   =    -i-   =    — 
p'  q'  r' 

2.  Find  the  condition  that  px^  +  Sqx"^ +  3rx+s  shall  have  a 
square  factor. 

Assume  the  square  factor  to  be  (x  —  a)^.  On  division,  the 
remainder  must  bo  zero  for  every  finite  value  of  x,  and  conse- 
quently (Th.  III.)  the  co-efficient  of  each  term  of  the  remainder 
must  be  zero.     Divide  by  (x  —  a)^,  neglecting  the  first  remainder. 


a 

P 

3q 
pa 

3r 
pa-  +3qa 

» 

a 

P 

pa  +  3q 
pa. 

pa^  +  3qa+3r  ; 
2pa^  +3qa 

R 

V 

2va4-3o : 

3{ija^4-2aa  +  r) 

:.  pa^-\-2qa-{-r  =  0: 

.".   px^  +  2qx+r  is  divisible  by  x—a  (Th.  I.  Cor.  2), 
or,  px^  +3qx^  -^3rx+s  axid  jJx^ +2qx+r  have  a  common  divi- 
sor.    Multiply  the  latter  polynome  by  x  and  subtract  the  product 
from  the  former,  and  the  proposition  reduces  to 

If  px^-{-3qx^ +3rx-'rs  have  a  square  factor,  ;ja;^-t-2(7a;-}-r  and 
ya;2_|.2ra;+s -will  have  the  square-root  of  that  factor  for  a  com- 
mon divisor. 

.3.  li  px^-\-3qx'^  +  3rx-{-s  yanishfor  x  =  a,  or  b,  or  c,  find  in 
terms  of  x,  p,  q,  r  the  value  of 


x  —  a        X  —  I)        x  —  c 
Beduce  to  a  common  denominator  and  add  the  numerators 
Bx^-2{a  +  h  +  c)x  +  (nb-{-hc  +  ca) 
{x  —  a){x  —  b)(x  —  c) 


60  THEORY    OF    DTVISOKS. 

Multiply  both  numerator  and  denominator  hjp  and  reduce  by 
Tb.  n.,  and  Ex.  4  of  Th.  11. 

8 (pa; 2  ^2qx-T-r) 
pz^+3qx'^  -i-'drx-i-s 
r^m+i        ^.m+1        ajTO+i        S(23x"^+^  +  2(/x'"+^-}-rx'''-+^) 
'  '  X  —  a        X  ~  b       X  ~  c  px^  +3ja;3  -fSric+s 

4.  If  2)x^  +  'dqx^  -{-Brx+s  vanish  for  x  =  a,  or  b,  or  c,  express  in 
terms  ofp,  g,  r,  s,  the  following,  a  + 6 +  c,  a^+b^+c^,  a.3-hi^+c' 

,      a"'  +  6™+(;'". 

Divide  a;^+^  by  x  —  a. 

1 

a         r?3  a^  a"*         aw^+l 


1  a         flS  a3 d'"  ;        a"«+i 

Similarly  divide  a:'"+^  by  x—b  and  also  by  x~c. 
add  together  the  quotients 

;^w+l  2;'"'+l  x'"^'^^ 

i 7  +  =Sx'^  +  ia  +  b+c)x'^~^  +  (a-  +  b^  ^.  c2W»-> 

x~  a        X—  h        X  - c  ^  '  ^  ' 

Hence,  by  the  last  example,  the  required  expressions  are  the 
coefficients  taken  in  order,  beginning  with  the  second,  of  the 
terms  in  the  quotient  of  3(|;a;"^+3  -]-2qx'"'+^+rx'''+^)  ^  {px^+Sqx^ 
+  Srx-{-s).     These  may  now  be  found  by  Horner's  Division. 

0.  Writing  Sj    for   a  +  5+c,  Sg  for  a^-\-b^-{-c^,   &c.,   express 
(a— 6)*  +  (6  — c)4  +  (c  — f/)*  in  terms  of  s^,  s^,  .s-g,  s^. 
By  actual  expansion 

{x--ay+{x-b)'^  +  (x-cy  = 
SxA-A{a-{-b  +  c)x^-^G{a^+b^-i-c2)x^-4.{a^+b^  ^c^i)x+ 
a^  +  b^  +  c'^  =  Sx^-4s^x^-i-6s^x^~4:S^x  +  s^. 
Put  z  =  a,    —b,    =  c  in  succession. 

(a_6)4  ^(c_a)4    =3a4_4sja34-6s2rt3-4s3a+S4 

(a_&)4_(.(5_c)4  =r3ft4-4s^63  +  6s2/'2  -4.<?3/)  +  S4 

(i  — c)^  +(c-a)4    =3c4  — 4sic3^_  esgcS  —  4.';3r+S4 
...  2{(a-6)4  +  i6-r)4  +(c-a)4}=3s4-4sic^3  +  6s„^-4.93.9,+3s.t 
.-.       (a-6)4  +  (/;-c)4+(c-a)4    =s,S4-4.s-i.S3  +  3s' 
in  which  s^  is  written  for  3  or  1  +  1  +  1,  i.e.,  a'^  +  ^^+c". 


TBTEORY    OF    DIVISORS. 


61 


Exercise  xxiii.    (a). 

1.  Determine  the  condition  necessary  in  order  that  x^  +'px-\-q 
and  x^-'cp'x  +  q  may  have  a  common  divisor. 

2.  The  expression   x^-^Za'^x^ ^'dbx^ +cx^-\-odx'+oe^x-\-P 
will  be  a  complete  cube  if 

e  d  c  —  rt® 

ah  oa* 

8.  Prove  that  ax^  +  hx^c  ?aid  a-^bx^  +  cx^  will  have  a  common 
quadi-atic  factor  if 

fc2c2  =  (c2  _a2_}.J,2)((.2_c[2+a^>). 

4.  Prove  that  aa-^-\-hx"+c  and  a+bx^  +  cx^  will  have  a  com- 
mon qaadiatic  factor  if 

a262  =  (((2  _  c2)(fl2  _c3  ^ic). 

6.  Prove  that  ax*  +  bx^+cx+d  and  a+hx-^cx^+dx*  willhaA-e 
a  common  quadratic  factor  if 

[a-{-d)     ^  =  {b-c){bd-ac). 

6.  x^+px-  +qx+r  will  be  divisible  by  x-+ax  +  b  if 
a^-22m^-\-[p-+q)a+r-pq  =  0,  and  b^ -qb^ +  rpb-r^  =  0. 

7.  x*+px-^q  will  be  divisible  by  a;-  4-ax-^b  if 

a^-^qa^=2J^  and  {b^- +  q){h^  -  q^  =  p^^ . 

8.  Determine  the  condition  necessary  in  order  that  a;*  +  'i;t»3'^ 
+Gya;2+4ra;  +  «  m?.y  have  a  square  factor. 

'U.  x^-\-4:px^-\-Qqx'^ +4:rx+t  vanish  for  x  =  a,   or  6,  or  c,  or  c?. 
find  in  terms  of  x,  p,  q,  r,  t,  the  value  of 

X"             x"             aj"  x" 

+  1  H }-- 


*  a;  —  a        a;  —  6        x  —c        x  —  d' 

10.  2  a,    S«-,    2a^,    S«*,    2;«^,    2a®- 

11.  ^(a-h)^,    2(a-i)*- 

12.  Determine  the  valuesof  the  expressions  in  Ex.  9,  10, 11,  for 
the  polynome  a:*  —  lix^  -j-x  —  38, 


62  FACTORING, 

CHAPTEE  III. 
Section  I. — Direct  Application  of  the  Fundamental  Formulas 


Formulas  [1]  and  [2].  (x±y)^  :=x^±2x!/-^y^,  &c. 
Art.  XII.  From  this  it  appears  that  a  trinomial  of  which  the 
extremes  are  squares,  is  itself  a  square  if  four  times  the  product 
of  the  extremes  is  equal  to  the  square  of  the  mean,  and  that  to 
factor  such  a  trinomial,  we  have  simply  to  connect  the  square 
root  of  each  of  the  squares  by  the  sign  of  the  other  term,  and 
write  the  result  twice  as  a  factor. 

Examples. 
1.  4a;4-80ic27y3_|_400?/4  =  (2a;3~20?/2)(2a;3-20?/3) 

8.  (a—b)^  +  (b-~c)^-h2{a--b){b-c).     This  equals  (a- 6 4-^- c) 
X  (a  —  b  +  b  —  c)  =  {a  —  c){a  —  c). 
4.  x^  +y^  +z^ -\-2xy  —  2xz ~2yz. 

Here  the  three  squares  and  the  three  double  products  suggest 
that  the  expression  is  the  square  of  a  linear  trinomial  in  x,  y,  z. 

An  inspection  of  the  signs  of  the  double  products  enables  us 
to  determine  the  signs  which  are  to  connect  x,  y,  z:  we  see  that 

1st.     The  signs  of  x  and  y  must  be  alike. 

2nd.     The  signs  of  x  and  z  must  be  different. 

3rd.     The  signs  of  y  and  z  must  be  different.     Hence  we  have 
x-\-y  —  z,  or  —x—y-{-z=  —  (x+y  —  z),  and  the  factors  are 
{x+y-z)(x-hy-z). 

Exercise  xxiv. 

1.  9h/3_|_12w?,  +  4;  c2'»_2t;™  +  l. 

2.  ^«-2v/-'523+z«  ;  iex^y^  +  lQxy^-{-4y^. 

3.  9a2634-12a6c+4o^;  36a;>2-24a;i/3-|-4?/* 


PACTORINa.  63 


5.  (rt  +  6)2+c2-2c(a+6);  9x« -^x^y^+r^x^y^. 
G.z^'+(x-yr--Ozfx-y);-r-]      +-        -2- 


b  I  \  a 

8.    (a;2-x^)2_2(a;2-xy)(a;^/-2/2)  +  (a;?/-,y2^2. 

10.  (3;>;-4!/)2  +  (2a;-3;/)2-2(3x-4?/)(2x-37/). 

11.  (x^  -xy+y^)^  +  (x^  +zy+y^y  +  2{x^+z^l/^-\-y^). 

12.  (oa;2  +  2x?/  +  7i/3)2 4.(4a;3  +6//2)3  -  2(4a,-2  +67/3)  x 
;5a-2-L-2a;i/ -1-72/2). 

18-  ItJ  +(ir)  -2ly)     • 

14.  a2-h^3+c3-2<r6-26c+2rtc. 

15.  a* +  fc4+t4-'2«26«-2a2c3  4-262^3. 

16.  (a-6)S  +  (/>-c)2  +  (6--«)2  +  2(a-i)(&-c))-2(a-6)(c-a) 
+  2(6-c)(a-c). 

17.  4a4-12a26  +  953+16«2c  +  16c2-24?yc. 

Formula    [4].     x^ —y^  =  (x+y)(x—y). 

Art.  XIII.  In  this  case  we  have  merely  to  take  the  square- 
root  of  each  of  the  squares,  and  connect  the  results  with  the  sign 
4-  for  one  of  the  factors,  and  with  the  sign  —  for  the  other. 

Examples. 

1.  {a  +  hy-{c-\-d)K 

This={{a+b)+(e+d)}{(a+b)-(c-{.d)} 
=  [a+b  +  c-i-d){a-{-b  —  c—d). 

2.  Factor  (z^+^ry+y^)^ -(x^ -xy-hy^)^. 

Here  we  have 

{{x^  +  5xy+y^)  +  {x^  -xy  +  y^-)}{{x^'  +  5xy-[-y^)-(-'^     xy^y-')\ 
=  2{x-  +  -2xy+y-){Gxy}  =  12xy(x+2j)K 

3.  a^-b^  -c-+2hc. 

This  =  a2-(i-cj3  =  (a  +  h  -c)[a—b  +  c). 


64 


FACTORING. 


This  =  4rt-//2-(rt2+/,2_^2)2 

=  {'lab  +a^  -h^  -  C-'  ,(-lab  -  a-^  -I-  -{-r-2). 
The  former  of  these  factors  =  {a  +  b)'^  —  c^  =  {a  +  b-\-c)(a-^b—i:)i 
and  the  iatter  =  c-  —(a  —  b)^)  =  (c+a  —  b){c  —  a+b). 
.'.  the  given  expression 

=  {a-{-b  +  c)[a  +  b  —  c)(c-{-a—b){c--a-\-b'). 

Exercise  xxv. 


1.  49a3-462. 

2.  9«3_ii3. 

3.  81a4-16/)4. 

4.  lOOx- -  3G.y2. 

5.  5n2b-2i)hx-y^. 

6.  9a;«-18i/'i. 

7  9   /-S  _  1 

8.       4.ij^-^x-z^. 


9.  81a4_l, 

10.  a-i--1654. 

11.  a>6_ii6. 

12.  a2-/j2+2&c-c>. 

13.  (rt  +  2i)■•^-(3a;-4J/)^ 

14.  (.-K-' +^3  j3_  4^-2^2/ 

15.  (x-t-7/j^— 4;i2^ 

16.  (3x  +  u)2-(5;rH.?)» 


17.  4a;37/2  _(a;2 4.^/2  _;j2)2, 

18.  ix'^+xij  —  y'Y  —  ix'^—xtj—y-)^. 

19.  (x2_y2+22)2_4^223. 

20.  (a+&+c+c^)2-(a-6+c-rf)''». 

21.  (2+3x+4a;3)3_(2_3x+4x-')'. 

22.  {ci-'-{-b''+4.aby'-{a'^->rb^y^. 

23.  (fl.2  -Zy2 4.c2  _fZ3^2  _  (2ac— 2M)3. 

24.  (x3-^3_23)3_4^223. 

25.  (a6_a3i3 0-^0)3  _(,f6_5a3i3_^fcG)3. 

26.  ai2_?,i2-|..Ga363_6/^9a3  +  869a3_Sa«i«. 

27.  (x2+_y3+23  -a;^/— 2/2  — 2:x)2— (x?/  +  ?/2  +  2X)*. 

28.  (a;-  +^2  ^gS  _  2ccy  +  2xz  -  2?/^)  -((/-f  z)3. 


2r3a2_«4 


./■4 


29.  2a362+262c-2+2c3a 

80.  a;4+i/4+z4-2iv;2*/2-27/3s3-2£2a;2 


FACTORING.  65 

Fos^ruLA  A.     {x~r){x-'rs)=X'-{-(r+s]z-rri. 

SXAMP::,ES 

1.  x''-9x-^20  =  (x-'-5\{x^^4:). 

2.  {x~y)2-^-x-?j-110^(x-y  +  n)(x-y-10). 

3.  (o^-ob  +  b^  |2  +  6b{a2-(,b-{-h2)-  4024.  n^-  ^ 
^(rt2  _  rti4_i3)  +  (2rt  +  3i)  [  {(^2  -ah-\-b2)  -  (2a -Sb]}. 

4.  (a;3_5a;)2-6(x2-ca;)-40  =  (x3  -ox4-4)(a;2  _  J^-IO). 

5.  (ffx4-%  +  c)2  —  {vi  —n){a.r-\-by-{-c)—vtn 
=  (fla;  +  %  +  c  —  ]ii){ax  +  %  +  c4-  //,j. 

Art.  XTV.  It  will  be  seen  that  the  first  (or  covimon)  tei-m  of 
the  required  factors,  is  obtained  by  extracting  the  square  root  of 
the  first  term  of  the  given  expression,  and  that  the  other  teims 
are  determined  by  observiuG;  two  conditions  : 

(1)  Their  product  must  equal  the  third  term  of  the  given  ex- 
pression. 

(2)  Their  sum  (ah/ebraic)  miiltiplied  into  the  common  term 
already  found,  must  equal  the  middle  term  of  the  given  expres- 
sion. Hence,  to  make  a  systematic  search  for  integral  factors  of 
an  expression  of  the  form  x^  +  bx±:  c,  we  may  proceed  as  follows  : 

1st.    Write  down  every  pair  of  factors  whose  product  is  c. 

2Dd.  If  the  sign  before  c  is  +,  select  the  pair  of  factors  whose 
suvi  is  b,  and  write  both  factors  x+,  if  the  sign  before  bis  -\-  ;  x  —  , 
if  the  sign  before  6  is  ~ . 

3rd.  But  if  the  sign  before  c  is  — ,  select  the  pair  of  factors 
whose  difference  is  b,  apd  write  before  the  laryer  factor  x-'r  ox  x— , 
and  before  the  other  factor  as—  or  x-\-,  according  as  the  sign  be- 
fore 6  is  +  or  — . 

Examples. 
1.  a;2  +9a;^20.     The  factors  of  20  in  pairs  are  1  and  20,  2  and 
10,  4  and  5.     The  sign  before  20  is  +,  hence  select  the  factors 
whose  sum  is  9.     These  are  4  and  5.     The  sign  before  9  is  +, 
hence  the  required  factors  are  (a;-f  4)(x4-5). 


(56  FACTORING. 

2.  a;2  -  8.r-f  12.  Paii-s  of  factors  of  12  are  1  and  12,  2  and  6, 
3  and  4.  Sign  before  12  is  +,  therefore  take  pair  whose  sum  is 
8.  These  are  2  and  6.  Sign  before  8  is  — ,  hence  the  factors 
are  (x  —  2)(x—G). 

3.  a;2_21a;— 100.  Pairs  of  factors  of  100  are  1  and  100, 
2  and  50,  4  and  25,  5  and  20,  10  and  10.  Sign  before  100  is  -, 
therefore  take  the  pair  whose  difference  is  21.  These  are  4  and 
25.  The  sign  before  21  is  — ,  therefore  x—  goes  before  25,  the 
larger  factor,  and  the  factors  are  (a;-f  4)(a3  — 25.) 

4.  aj^-f  12a;-108.  Pairs  of  factors  of  108  are  1  and  108, 
2  and  54,  3  and  36,  4  and  27,  6  and  18,  9  and  12.  Sign  before 
108  is  —  ,  therefore  take  the  pair  whose  difference  is  12.  These 
are  6  and  18.  Sign  before  12  is  +,  therefore  x+  goes  before  18, 
the  larger  factor,  and  a;—  before  6,  the  other  factor;  hence  the 
factors  are  (a;  — 6)(a;-|-18). 

Note. — It  will  be  found  convenient  to  write  the  factors  in  two 
columns,  separated  by  a  short  space.  Taking  Ex.  2  above,  pro- 
ceed thus  :  Since  the  sign  of  the  third  term  is  + ,  write  the  sign 
of  the  second  term  (in  this  case  — )  above  both  columns. 

1  12 

{x-2)  (x-6) 

Ex.  3  above.  Smce  the  sign  of  the  third  term  is  — ,  write  the 
sign  of  the  2nd  term  (in  this  case  — )  above  the  column  of  larger 
factors,  and  the  other  sign  of  the  pair  + ,  above  the  other  column. 

+ 

1  100 

2  50 
fx+4)          {x-25) 

6.  a;2-81r4-64. 

Here  we  have  the  factors 

1,         64 
a'-2,  a;-82 

4,         16 
and  since  the  last  term  has  the  sign  +,  and  the  middle  term  has 
the  sign  — ,  we  write  —  over  both  columns. 


FACTOEIJJO,  grr 

6.  x"-^l2x-Gi. 

-  + 

1,  64 

2,  32 
.r-4,  a;  +  10. 

Here,  since  the  last  term  has  the  sign  — ,  we  write  the  sign 
1^  +  )  of  the  middle  term,  over  the  column  ot  larger  factors,  and 
the  sign  —  over  the  other  column. 

7.  x*-10x-2-144. 

Here  we  have  the  pairs  of  factors  : 
+  - 

1,  141 
'A,  72 
4,       yo 

a;  +  8,  a;- 18. 
And  since  the  sign  of  the  third  term  is  — ,  we  write  the  sign  oi 
the  second  term   (in  this  case  — )   above  the   column  of  lai-yer 
factors,    and   the   other   sign  (of  the  pair  +)   above  the  other 
column. 

Exercise  xxvi. 

1.  X2-5.C- 14;  a;-'-9a;+14;  x^+lx+12. 

2.  x2-8x+lo;  x^-ldz+Sk;  x- -7x-G0. 

3.  4a;2-2a;-20;  9x2 -150a;4-600. 

4.  ^x2+Mx-3(i;25z'+i0x+lo;  9a;« -27^3  4.20. 

5.  ^\x^ +  Hx-\-12;  16a;4-4a;2-20. 

6.  x'i-(a3  +  Z/3)a;2 +6/2^3  ;  4(^._}.v/)a  _4(^._|.^)_fj9, 

7.  (x^  +2/2 j2  - (a2-fe2)(a,-2  +^-^)-aHK 
8".   {u  +  h)^-2c{a+b)-dc^. 

9.  (x4-?/)2  +  2(x2  +y2)(^x  +  ;j)  +  {^^  -  y'^y- 

10.  (a  +  6)2-4a/y(a  +  6)-(«2_62j2. 

11.  {x:-+x>j+y^Y+x^-y^-5xii-'2ij^-2x^. 

12.  «2_2rt(6_c)-3(6-c)2. 


68  FACTORING. 

13.  (x'^+y^)2+2a^{x°-{-y'')  +  a^-b*. 

U.  {x^  -lOxy  -4:(x^  -10x)—0Q. 

15.  (x2  - Ux  +  40)2  -  25(^2  _  14a:+40)  - 150. 

16.  {x^  -xi/  +  >j^)'^+2xy[x-'-x>ji-ij^)-Qx'i/K 

17.  04-823  +  2;  «4-2.c2-3;   dx^  +  Qx'i'y^ -lOy*. 

18.  c-'"  +  t'"-2;  a;«-a:3  — 2;  x'"'-2x"'y'' -8y^\ 

19.  x-"'—{a  —  b}x"'y"~uhy-''. 

Art.  XV.  Trinomials  of  the  form  ax^  +wa;  +  c'  (n  not  a  squa  e) 
may  sometimes  be  easily  factored  fi'om  the  iollovving  coiiBiileia- 
tions  : — 

The  product  of  two  binomials  consists  of 

1st.  The  product  of  the  Jirst  terms. 

2nd.         "  "         second     " 

3rd.  The  sum  {alysbraic)  of  the  products  of  the  terms  taken  dia- 
gonally. 

These  three  conditions  guiile  us  in  the  converse  process  of 
resolving  a  trinomial  into  its  binomial  factors. 

Examples. 

1.  Kesolve  6.e'^-10;c//  +  6;/2. 

Here  the  factors  of  the  first  term  are  x  and  6a:,  or  1x  and  8x- ; 
those  of  the  third  term  are  y  and  Qy,  or  2y  and  'dy.  These 
pairs  of  factors  may  be  arranged 

(1)  (2)  (3)  (4) 

X  2x  y  2y 

Gx  8x  6y  By 

Now,  we  may  take  (1)  with  (3)  or  (4),  or  (2)  with  (3)  or  (4) ; 
but  none  of  these  combjnatious  will  satisfy  the  third  C(nlition. 
If,  however,  in  (4)  we  interchange  the  coeflicients  2  and  3,  then 
(2)  and  (4)  give 

2x         3y.     and 

Sx         2y,     where  we  can  combine  the  "  diagonal" 
products  to  make  13,  and  the  factors  are 


FACTOBING.  69 

2a;  —  By,  and 
Sx  -  2y. 
The  coefficients  of  (2),  instead  of  those  of  (4),  ruaght  have  been 
interchanged,  giving  the  same  result. 

2.  6x^-l5zi/+(Jy'^. 

Here,  comparing  (2)  and  (3),  Ex.  1,  we  see  that  their  diagonal 
products  may  he  combined  to  give  15,  and  the  factors  are 
2x—y,  and  Bx—6y. 

3.  6a;--2U.c?/+6//2. 

Here,  again  referring  to  Ex.  1,  we  see  at  once  that  it  is  usele^.- 
to  try  both  (2)  and  (4),  since  the  diagonal  products  cannot  be 
combined  in  any  way  to  give  a  higher  result  than  ISxy.  Birt  com- 
paring (1)  and  (4),  we  obtain  by  interchanging  the  coefficients 
in  (4)  x—By,  and 

6x—2y,  wliich  satisfy  the  third  condition. 
Or,  we  might  interchange  the  coefficients  of  (3),  and  take  the 
resulting  terms  with  (2),  getting  2x—6y,  and 

Sx—  y. 

4.  Gx^+SSxy-GyK 

Here  the  large  coefficient  of  the  middle  term  snows  at  once 
that  we  must  take  (1)  and  (8)  together.  Interchanging  the  co- 
efficients of  (1)  we  have 

Gx—  y,    and 
a  +  (jy.    The  same  result  will  be  obtained  by  inter- 
changing the  cociiicients  of  (3). 

Exercise  xxvii. 


1.  6x^-Blxy  +  6y'. 

2.  6x2-1-9x2/ -6?/ 2. 

3.  56x^  -  7 6xy  + 20}/^. 

4.  56x2 -36x?/- 20^2, 

5.  56x2-1121x^+20^2. 

6.  6Gx2-68x?/  +  20i/2. 

7.  66x2 -558xJ/- 207/3. 

8.  56x3 +3ex/y- 207/2. 


11.  6x2  -  16x//  — 6?/2. 

12.  6x2-f5x(/-6?/2. 

13.  66x2+562x//-f20?/2. 

14.  56x2-122x^-1-202/2. 

15.  56x2-102x!/-20?/2. 

16.  56x2 -229xi^-f- 207/3. 

17.  56x3-94.r//-h20?/2. 

18.  56x2 -276x//- 207/3. 


9.  56x2-67xJ/-}-20?/2.  j     19.  36x2— 33xi/-15?/2. 

10.  56x2  +  3x(/-20i/2.  I     20.  72x^ -19xy-iOy^. 


70  FACTOKINO. 

Art.  XVI.  More  f/merally,  trinomials  of  the  form  nx^-\-hx-}-f 
{a  not  a  square)  may  be  resolved  by  Formula  A,  thus 

Multiplying  by  a  we  get  a^x^  -\-bax+ac.  Writing  z  for  ax  this 
becomes  z^  +  is-J-ac.  Factor  this  trinomial,  restore  the  value  of 
z  and  divide  the  result  by  a. 

Examples. 

1.  6x"  +5x -  4.  Multiplying  by  6,  we  get  (6.r) 2  +  o(Qx)  -  24  or 
2  ■  -f. 52  _  24.  Factoring,  we  get  {z  -  3){z  +  8),  hence  the  required 
factors  are  ^{6x-3)(dx+8)  =  {2x—l){3x+4:). 

2.  6a;3  -  13:cy  +  6//3.  ^  Factoring  z"  -  ISs^  +  SGy^  we  get  {2 -Ay) 
[z  —  dy),  hence  the  required  factors  are  ^(6u;  — 4?/j(6.B  — %)  = 
{3x-2y){2x--S7j). 

3.  33-14x-40aj3.     Factoring  1320 -14^-23  we  get 
(30-2)(44+z),  hence  the  required  factors  are  Jg(30-  40*)  x 
lu+i0x)  =  {3-ix){ll+lQx). 

NoTB. — The  factors  may  conveniently  be  arranged  in  two  col- 
umns, each  with  its  appropriate  sign  above  it. 

+ 

Ex.  1,  above  1  24 

2  12 

^(6a; -3)(6a;+8)  =  (2a:~  l)(Sx+4). 

Ex.  2,  above  1         3(> 

2  18 

3  12 

|(6x-4)(6a;-9)  =  (3x-2)(2ar-3). 

[Another  method  of  factoring  trinomials  of  the  form  ax'^  +  bx-^o 
is  as  follows  : 

Multiply  by  4a,  thus  obtaining  4:a^x^  +4:abx+  iac.  Add  h'^  -b*, 
jvhich  will  not  change  the  value,  Aa^x'^  +  Aahx  +  b"  —b^+iac  ;  by 
[1]  this  may  be  written  {2.ax-\-b)--^{b^  —  Aac).  Factor  this  by 
[4]  and  divide  the  result  by  4a. 


FACTORING.  71 

Ex.     Factor  562-' +  137a; -27885.      Multiply    by    4x56    or 
2x112,  1122a;^  +  2. 187. 112a;— 624G240.     Add   1373 -1372,  then 
1122a;3  +  2.137.n2x+1372  -  (1372  +624G240)  =  (112a;+137)3  - 
6265009  =  {(112a;  -f  137)  +2503}  {(1 12a;  +  137)  -  2503}  = 
(112a;+2640){112a;-2366). 

We  multiplied  by  4  x  56,  we  must,  therefore,  now  divide  by  that 
number.     Doing  so,  we  obtain  as  factors  (7a;  +  165)'(Sa;— IGO).} 


Exercise  xxviii. 


1.  10x2+a;-21 

2.  10.,-2  -  29a;  -  21. 

3.  10a;2+29a;-21. 
^.  6a;2-37a;+55. 

5.  12a3_5a_2. 

6.  12a;3-37a;+21. 
V.  12a;2  +  37a;  +  21. 

b.  15aG  +  13rt362-2064. 


9.  12a;-^-a;-l 

10.  9a;37/3-3x?/i'-62/«. 

11.  4a;3  +  8a;/y  +  3?/3. 

12.  U^x^-lhz^-dx*. 

13.  6a;4-a;3?/3-35.//4. 

14.  2a;'i+a;3-4o. 

15.  4a;4-37a;=2/2  +  9?/4s 

16.  4(a;  +  2)4  -37a;2(a;+2)3  -t-9a;* 

17.  6(2x  +  37/)3+5(6a;2  4-5a;^-6y2)_6(3a;-2(/)3. 

18.  6(2a;+37/)4  +  5(6.c3-f  oa;.y-6?/2)3  -G(3a;-2y)*, 

19.  Q{x^-\-xij-\-y^Y+Vd[x^+x-y-+y'^)-dSo{x''-z>i-\-y^)^. 

20.  21(a;3  +  2xf/+2!/3)3_6(a;2-2a;?/-l-2?/3)3_5(a;4+4?/4). 

Section  II. — Extended  Application  of  the  Formulas. 

Art.  XVII.  The  methods  of  factoring  just  explained  may  be 
appHed  to  find  the  rational  factors,  where  such  exist,  of  quadratic 
multinomials. 

Examples. 
1.  Eesolvel2a;3-a;y-20?/2+8x+4l7/-20. 
In  the  first  place  we  find  the  factors  of  the  first  three  terms, 
vrhich  are 

4x+5y,  and 
8a; -4r/. 

Now,  to  find  the  remaininy  terms  of  the  required  factors,  we 
must  observe  the  following  conditions : 


79; 


FACTORING. 


1st.     Their  product  must  =  —  20. 

2nd.  The  sum  [algebraic)  of  the  products  obtained  ty  multi- 
plying them  diagoually  into  the  ?/'s,  must  =Aly- 

3rd.  The  sum  of  the  products  obtained  by  multiplying  them 
diagonally  into  the  ic's,  must  =Sx. 

We  see  at  once  that  —4  with  the  first  pair  already  found,  and 
+  5  with  the  second  pair,  satisfy  the  required  conditions,  and  .'. 
the  factors  are 

Ax+^y  -4,  aud 

3a;-4?/+5. 

2.  p2+275r-2(7'-*4-77r— 3r2    -{--pq. 
Here  the  factors  oi p^  +  V9.  —^q^,  aro 

p+2q,  and 
2^—q. 
Now  find  two  factors  which  will  give  -  3?--,  and  wliich  mnlti- 
plied  diagonally  into  the  ps   and  ^-'s  respectively,  will  give  2pr, 
and  7qr  ;  these  aro  tound  to  be  —  r  taken  with  the  Jirst  pair,  and 
+8r  taken  with  the  second  pair.     Hence  the  required  factors  are 
p  +  '2(j — r,  knd 
p  —  q-f-  3r. 

Art.  XVIII.  But  the  following  examples  illustrate  a  surer 
method. 

3.  x--\-xy-2.y^-\-2xz  +  lyz-Sz^. 

Keject  1st  the  terms  involving  », 
2nd.         "  "  y, 

3rd.         "  "  X. 

and  factor  the  expression  that  remains  in  each  case. 
1st.         x^+xy-2y^    =  {x~y){x+2y). 
2nd.        x^+2xz-2z^    =  {x+dz)(x-z). 
3rd.   -2tj-'+7yz-Sz^    =  {~y+Sz){2y -z). 

Arrange  these  three  pair  of  factors  in  two  sets  of  three  factors 
each,  by  so  selecting  one  factor  from  each  pair  that  two  of  each 
set  of  three  may  have  the  same  coefficient  of  x,  two  may  have  the 


rACTOKiNCj.  73 

HiTiie  coeScient  of  y,  and  two  the  same  coefficient  of  s  (coejficimi 
including  sign).     In  this  example  there  are 

X—   y,   aj  +  Pz,    —   y  +  ?z, 
and  x-\-'2y,   x-    z,        2r,-   z. 
From  tlie  first  set  select  the  common  terms  (including  signs) 
and  form  therewith  a  trinomial,  x—y  +  cz. 

Repeat  with  the  second  S3C,  and  we  get  x+2.y  —  z. 

:.x^+xy-2y^  -f-  2xz+lyz-dz^  =  {x-y+B;.}{x+2y-z). 
4.  Bx-  -8xy-By^  -iriiOx+27. 

1st.         dx^-Sxy-3?/^    ={Sx  +  y){x—dy). 
2nd.       8a,-3 +30x4-27      ={Qx+3){x+9). 
8rd.   -3^2  -fii7      =(2/+3)(-32/  +  9). 

.•.  the  factors  are  ('Sx+y-\-d){x  —  By+0). 

6.  6rt3_7a6-b2rtc-2063  +  64ftc-48r3. 

1st.  6a3-7a&-20i3    =  (2rf-5&)(3a+45). 

2nd.         6a2-u  2ac-48c3    =  (2/;(-f  6c-)(3a-8c). 
3rd.    -2U{y3+64ic-48c3    =(-5/>-i-t;c)(46-8c). 

.'.  the  factors  are  (2a  — 56-i-Gc)(3a  +  4i  — 8c). 
Exercise  xxix. 

1.  7x^  -xy-Gy^  -6x-20y-lQ. 

2.  20x3 -loary- 51/2 -68a; -42^-88. 

3.  3x^+a;3«/3- 47/4 +  10x3 -17^/3 -13. 

4.  20x2-20?/3+9xz/+28x+35jr. 

6.  72x- -  8^/3 -I- 55a;y+i2y— 169x4-20. 

6.  x^  —xy—  12y'^  —5x—15y. 

7.  8x2  +  18x//+9//3+2xz-s3. 

8.  Gx3-h6?/3-13xj/-8z3_2//c+Sxz. 

9.  6x*- 102/4 +  llx3//2- 25^3  j.i0y2^2Dr/2z3- 15x2 +  10x»z". 

10.  15x4  -lG;/4  -  22x2(/2  4-  loz*  + 14^232  ^  50x3;r2. 

11.  4ft3 -1563- 4^6- 21c2-3G/>c—8rtc. 

12.  rt4  +  />4+c4-2rt362_2;,3c2-2c3a3. 


74  FACTORIXG. 

Art.  XIX.  Trinoraials  of  the  form  ax*^  +  Ix"^  +r  can  always  be 
broken  up  into  real  factors. 

If  a  and  c  have  different  signs,  the  expression  may  be  factore(3 
by  Art.  XVI. 

If  a  and  c  are  of  the  same  sign,  three  cases  have  to  be  consid- 
ered :  i.  6  =  2^/(ac),  ii.  6>2v/'(ac),  iii.  6<2y(ac) 

Case  I.  6  =  2]/(flc).  This  case  falls  under  Art  XII.,  formula 
[1] .  where  examples  will  be  found. 

Case  II.  i>2y(ac).  This  case  falls  under  Art  XVI.,  where 
examples  will  be  found.  The  following  additional  examples  are 
resolved  by  the  second  method  of  that  article. 

EXAMPLSS. 

1.  4a;4  +  5a;22/2  4-1/4. 

Here  we  see  that  (f?/^)*  will  make,  with  the  first  two  terms, 
a  perfect  square,  and  we  therefore  add  to  the  given  expression 
{^y'^Y  —{%]j^Y .     The  expression  then  becomes^ 

-(2x2 +^2/^)^- Ay*- 

=  (2a;' +f2/' +  |2/2)(2x2 +1 2/' -  i2/2) 

=  (2a;2  +22/2)(2x*  +  -i7/^)  =  (a;2  +t/2  j(4a;2  +y2). 

2.  3a;4  +  6a;2+2. 

Here  multiplying  by  4x3,  and  completing  the  square  as  in 
Ex.  1,  we  have 

36a;4+72a;2  +  62  +  24-62  =  (6x3  +  6)2_12 
=  (6a;2  +  6-T/12)(6x2+6  +  |/12),  which  divided  by  4x3  give 
the  required  factors. 

Proceeding  as  in  Ex.  2  we  have,  by  multiplying  by  4a, 

ax^j^hx'^  4-c  =  {4a2a;*+4a6a;2  +63  -  h^  4-4ac}  ^-  4rt 
=  {2aa:3  4-i  +  ^(i2_4«c)}{2a^3+/,_^(i8_4«c)}-4a. 


FACTOEIN».  75 

Exercise,   xxx. 

1.  x^+7x--hl;  4a,-'5-Mlrc3+l. 

2.  a;4-f7a;2i/S+^4  ;  Sx'^+5x^y^-\-y^. 

3.  Ax*  +  Wx-+S;  S{x+)/)^  +  5z^{x  +  y)'+z^. 

4.  a;*  +  7a;2_v-'+3-}-!/4;  x'^  +  7x''y^  +  Siy^. 

6.  3x*+8x'^y^+i^\y^;  36x4+96a;2+56. 

7.  5x*4-20a;3+2;  4rt*  +  12«2  +  l. 

8.  4(x+2/)*  +  12(x+!/)'22+24;  5.r*+20a:3y2+2y4. 

9.  9x*  +  14a;2+4;  2x*-{-12x'^{y  +  zy' +  15{y-^z)^. 

10.  2a;4  +  12a;8  +  15;  7x4  +  40a;2 +45. 

11.  8a;*  +  36a:2y2+29?/*;  7ic4+20a;-r/=» -20^*. 
'12.  7(rt-6)4+16(a-6)2c3+5c*  ;  |«4  +  3«2i3+54. 

13.  3a;4+6x'.v'+2y4;  3(r^  +  /;)*  +  6(a3-63)3+2(a-i*)4. 

14.  49a4-84a3i-'+2264  ;  25m'^  +  60m^n^  +  27n^. 

15.  49(j??.  +  ?i)4-84(m^  -n2)2+22(?«-«)4. 

Case  III.     b<2i/{ac).      This  case  may  be  brought  under 
Art.  XIII.     The  following  examples  illustrate  the  process  of  re 
duction  and  resolution. 

ExAliPLES. 

1.  a;*-7a;'+l. 

We  have  to  throw  thig  into  the  form  a'  —  6'  : 
a;*-7a;2  +  l  =  (x2  +  l)2-9x3  =  (x2+l  +  3.-c)(a;2  +  l-8a:). 

2.  9x*  +  3z^y^+iy^  =  {Sx^  +  2y^)^-9x-y^ 
=  {3x^-+27j'^-3xy){dx^  +  2y^-^3xy). 

3.  x*+y^  =  {x^  +  y^y-2x^y* 

=  {x^+y'+xy,/2)(x^+y^-xyy^2). 

=  {x^+y^  +ixy){x-'  +y"  -^xy). 

6.  az*  +  bx^-^c^{^a.  x^  ^-^/c)^  -  {2  y{ac) -b}x^ 
=  {ya.  x^  +  ^/c-  y^{2y'^ -O^x}  X 

{j/a.  x^+y'c+^/{2y^-^~b)xl 


76  FACTORING. 

Art.  XX.  It  is  seen  from  these  examples  that  we  have  merely 
to  add  to  the  given  expression  what  will  make  with  the  Jirst  and 
last  terms  (arranged  as  in  Ex.  5)  a  perfect  square,  and  to  subtract 
the  same  quantity.  In  Ex.  2,  e.  y.,  the  square  root  of  9a;*  =  3a;2, 
the  square  root  of  4j/*  =  2?/ 2,  .-.  Sa;^ +2//*-^  is  the  binomial  whose 
square  is  required  ;  we  need  .".  I'lx^y^  ;  but  the  expression  con- 
tains 8x^y^  :  .*.  we  have  to  add  and  subtract  12x^y^  —  Bx^^y^  = 
<dx^y-K 

Hence  we  derive  a  practical  rule  for  factoring  such  expressions. 

(1).  Take  the  square  roots  of  the  two  extreme  terms  and  con- 
nect them  by  the  proper  sign  ;  this  gives  the  first  two  terms  of 
the  required  factors. 

(2)  Subtract  the  middle  term  of  the  given  expression  from 
twice  the  product  of  these  two  roots,  and  the  square  roots  of  the 
difference  will  be  the  third  terms  of  the  required  factors. 

6.  «*+T'ca;-//2-{-y*.  Here  i/x*=a;2,  |/y4  =  )/8,  and  the  first 
two  terms  of  the  required  factors  are  x^-\-y^  ;  twice  the  product 
of  these  is  +2x'^y^,  from  which  subtracting  the  middle  term, 
^^z^y^,  we  get  l^aj^^s.  the  square  roots  of  this  are  +lxy. 
Hence  the  factors  are  x--\-y^  +  ^^vy. 

Note  that  since  v/7/*=  4-//^,  or  --y^,  it  may  sometimes  hap- 
pen that  while  the  former  sign  will  give  irrational  factors,  the 
latter  will  give  rational  factors,  and  conversely. 

7.  x'^  —  llx^ y^-\-y^.     Here,  taking  -\-y'',  we  have 

x''-^y^-\-xys/Vd,  anda:»  +  7/2-xy/v/13. 
But  taking  —  ?/-,  we  have 

x^  -y^+Sxy,  &uA  x^  -y^  —  3xy, 
Sometimes  both  signs  will  give  rational  factors. 

8.  lQx*'  —  17x^y^-{-y*.     Here  we  have 

(^Ax''^^y2^2xi/){ix^+y^-3xy,  and  also 
{4^'^  -y^  +5xy){lx-  -y^  -  5xy). 


FACTOKING.  rjrj 

Exercise  xxxi. 

1.    x4+2^2^2+9?/4,  ^4_a.2^2_|_y4^  ^.4+^2_,^--;_|_,,4. 

3.  a;-*H-l,  x-t+%-1,  l-12//2-j-iG^-i. 

5.  y^-x'^  +  Ux^!/'^,  x^"  +4:y»,  x^  +  ix^  +  16. 

7.  ^^'"  +  04/'",  «^'"+4i/»'",  ix4  +  ^9gyi-5|x3.(/-'. 

8.  4*4-8x2  +  1,  7*2^3  _x^.4_30//4,  a;4  +  aV' 

9.  7»2^.4+,;-j^4_(2m/i4-^)x-2_,y2^  ^im  ^  2*'"--(/*'". 

10.  16x4-25*2  +  9,  4*4  _  1(5^2 +4^  13*2^2  _y_^4  _  4^*. 

11.  4*4-12^l'*2^2+g^4^  *4+6*2+25. 

12.  </4+i4  +  (a  +  6)4,   l+«44.(l+,,)4. 

13.  {x-^y)*-7z^x-i-yr--\-zK  X 

14.  (rt-(.6)4  +  7e2(,/4.i)2+c*t. 

15.  16</4  +  4(i_f)4_9,<2(/,_,.)2, 

16.  4(«  +  //)4+9(a-i)4-21(«2_/;2)3. 

17.  {x^+y^-xy}'^-7{x^-i-y^y'+{x+yy. 

18.  (r72+ai  +  />2)4  +  7(rt3_^,3)2+(a_fc^4. 

19.  lGa4  +  4a2  +  l,  *4_4i^2+i6. 

20.  x4+81y8_63a:2,/4,  l+24+2oz». 

21.  (a2  +  l)4  +  4(a2  +  l)2a2+16a4,  (^+l)4  +  2(*2  _  1)2^ 
n(a;-l)4. 

Art.  XXI.     We  cau  apply  [4] ,  Art.  XIII.,  to  factor  expres- 
sions of  the  form  ax'*'  -\-bx^  +  rbx  —  r- a.     This  may  be  written 
«(a;4  -ri) +  i*(* -'  +  >•)  =  {a{x^-r)+hx]{x^-\-r). 

ExAilPLES. 

1.  6*4  +  4*3+i2x-54.     This 

=  6(x4 _  9)  +  4a;(x2+3)  =  (x2+3){6(x2 _ 3)+4x} 
=  (*2  +  3)^G*2+4^_i8). 


78 


FACTORING. 


■4){ll(a;9  +  4)+10x} 


2.  Ila;4  +  10a;3-'i0a;-176.     This 

=  11  (.r*  -  16)  +  10a;(a;3  -  4)  =  (x"" 
=  {z^-A){Ux^+lQx+U). 

3.  40;«^  +  30:c3 +60.^-160.     This 

=  10(4ic4-16)  +  15a;(2a;2+4)  =  (2ic2+4){10(2.r3-4)-f  15.r} 
=  (2a;^+4)(20a;'-*+15a;-40). 
Note. — To  determine  r,  take  the  ratio  of  the  coefficient  of  x^ 
to  the  coefficient  of  x. 

Exercise  xxxii. 
Resolve  into  factors 


1.  x'>^  +  2x^  +  6x-9. 

2.  2a;4  +  2a;3+6x-18. 

3.  x^+3x^+12x-16. 

4.  3a;*+a;3-4a;-48. 

5.  5x^  +  4:x^  -V2x~4:5. 

6.  10x4  + 5x3+ 30a; -360. 

7.  ia;4+20x3+4a;-T^^. 

8.  25x'i-40x3+8x-l. 

9.  37ix4-30.c3+48x-96. 

10.  63x* - 39x3 +52a;- 112. 

11.  810x4 +  Va;3  +  |x-2i. 

12.  242x4  -33x2-3x-2. 


lO.  4X  -f-ipQX'    ^5"^   ^Tj^' 

14.  80x4- 32x3?/+ 64x?J-320«,'4. 

15.  24x4-12x32/+30x.(/3-1502/'4. 

16.  2x4+^s;3,^_8a;_y3_512,/'l. 

17.  1 1x4  +  10x3 -12x-15fi 

18.  40x4  +  30x3+60x-160. 

19.  13x4-12x32/+72xi/3-468s^4. 

20.  3x4  +  3x3^  +  12.r//3-482/4. 

21.  5x4 +4x3^- 12x//3 -45^4. 

22.  4x4- 14x3 //+28x//3-16y4. 

23.  x4+80x32/+16xy3_^i^y4. 

24.  2x4-x3?/+6xy3_72y4. 


Art.  XXII.     Formulas  [1]  and    [4]    may  sometimes  be  ap- 
plied to  factor  expressions  of  the  form 

ax^  -\-  bx^  -{-ex-  -\-rhx-{-r^  a. 
This  may  be  put  under  the  form 

a(x4+r2)  +  te(x2+»-)  +  rx3  =  a(x3+r)2+Z-x(x2+r)  + 
(c  — 2ar)x2,  which  can  sometimes  be  factored. 

Examples. 
1.  x44-6x3+27x2+162x+729. 
We  have  x4+729+6x(.'c2+27)+27x3. 

=  (x2+-27)2+6x(x3+27)  +  9x2-3Gx3 
=  {x24-27+3x}3-36x3,  which  gives  the  factora 
a;3  _  3.^_|_27,  and  x2+9x+27. 


FACTORING.  tjrg 

2.  a.4  4.4a;3_|_4a;0_|_20x+25.     This 

=  (x3+5)2  +  4a;(a:3+5)-6a:2 
=  (a:2  +  5)2  +  4a;(a;3  4-5>+ Jx'2  -  lOx^ 
=  {a;2  +  5+2^-a,VlO}  {a,-3  4.5-h2^;-t-aV10}. 

Exercise  xxxiii. 
Eesolve  into  factors  : 

1.  x4- 6x3  + 27x3 -162a;+729. 

2.  a;4  +  2a;3  +  3a;3+8x+16. 

3.  x'*+x3  4-3;2_|_a,  +  i, 

5.  ia;4-12x3_6a;2-12x-f4. 

6.  a:4 +  14x3 -25.1-3 -70X+25. 

7.  16x*- 24x3- 16x3 +12x+4. 

8.  x*  +  5x3-16x2  +  20x+16. 

9.  x4+6x3-llx2-12x+4. 

10.  x^+4x3//+x2(/3-+12x//3+9y<. 

11.  x4+6x3-9x2-6x  +  l. 

12.  x*+4x3/y-19x3!/2+4x?/3+y*. 

13.  4x4 +4x3//- 65x3^3 -10x?/3+25j/'t. 

14.  X*  +6x'7/  — 9x2?/3  _6x>/3+|/4, 

15.  X*  +  6x3//  +  10x2?/2+12x?/3  4-4^4. 

16.  9x4 +  18x3?/- 52x2?/-^ -12x!/3+4?/4. 

17.  11x4  +  10x3?/+39^Vt-*--2/''  +20x^/3  +  44?/*. 


Section  III. — Factoring  by  Parts. 


Art.  XXIII.  To  factor  an  expression  which  can  be  reduced 
to  the  form  a.F(x)+6/(x). 

When  the  expression  is  tluis  arranged,  any  factor  common  to 
a  and  6,  or  to  F{x)  and  f{x),  will  be  a  factor  of  the  whole  ex- 
pression. The  method  about  to  be  illustrated  will  be  found  use- 
ful in  cases  where  only  07ie  power  of  some  letter  is  found. 


80  factoring. 

,  Examples. 

1.  Factor  acx^  —  ahx  —  hc^x  +  b^c. 

Here  we  see  that  only  one  power  of  a  occurs,  and  we  therefore 
group  together  the  t^rms  involving  this  letter,  aud  those  not  in 
volving  it,  getting 

ci{fx^  —  hx)  —  hc"x-\-h^c 
=  ax{cx  —  h)  —  bc{cx  —  b)  —  {ax—  hc){cx  —  h). 

2.  Factor  m^x^ —mna'^x  —  mnx  +  n^a^. 

Here  we  observe  that  a  occurs  in  only  one  power  (a^). 
Therefore  we  have 

—   a^[mnx  —  n^)  +  m'^x^  —ninx 

=  —na^[mx — n)  +  vix{i)ix  —  n) 

=  (})ix  —  n)()nx  —  na-). 

3.  2x^+'iax  +  Shx  +  6ab. 

Here  we  observe  that  the  expression  contains  only  one  power 
of  both  a  and  6.  W^  niay,  therefore,  collect  the  coefficients  in 
either  of  the  following  ways  : 

a{4:X  +  iib)  +  [2x'^+Shx), 
or,  b{'3x  +  Ga)  +  {2x^ +Aax). 
Now  the  expressions  in  the  brackets  ought  to  have  a  common 
factor,  and  we  see  that  this  is  the  case.     Hence, 
a{Ax  +  6b)-i-{2x^+Sbx) 
=  2a(2a;+36)  +x{2x+Sb)    =  {2x+db){x+2a). 

4.  abxy  +  b^ij^-\-acx  —  c^ 
—    a(hxt/-\-cx)  +  b^y^ —c^ 

=  ax{by-\-:;)  +  {by  +  c){by  —  c)=  (by  +  c)  (ax+by  —  c). 

5.  y^  -  {2a  +  b)y''  +  {2ab-\-a^)y  -  a^b 

=  _  ^y2  _  2ay-\-a^)+7j^  -  2ay^  -\-a^y 
==-b{y^-2ay  +  a2)+y(y2-%iy  +  a2) 

^{y-b){y-a)^- 

6.  2x^y-^2bx'^  -  bx^y  +  Aabx^y  -x~y^+iaxy-  -  2abxy^~2ay^. 
=  b(2x^  —  x^y+4:ux^y~  2axy^)  +  2x^y—x^y^  +  iaxy^  —  2ay^ 

=  bx{2x^  —x^y+iaxy  ~2ay^)+y{2x^  —x-y-\-iaxy  —  2ay'-) 
=  {y-\-bx){2x^—x^y-\-4:axy  -  2ay'^). 


FACTOKINO, 


81 


And  1x^—x^y-\-^oxy  —  1a]i^ 

=  a{Ax!i - "Iij-  \+2x^—x^!/ 

=  2c,//(2^  - 1/)  -\-x-{'2x  -y)    =  {1ay+x^-){1x-  tj). 

7.  x^  +  {2a-b)x--{:2^ab-a^)x-a^b 

=  b{-x^-'lax-a^)-{-x^+'2ax-+a^x 

=  -h{x  +  aY+x{x  +  a)^    =(x-6)(x+«)3. 

8.  ]'X^-{l>-(l)x--\-{p-q)x+q 
^qix"^  —x+l)-\-})X^—px'^-{-rx 

=  q{x^  -x+\)+px{x^  -x^-l)    ={px+q){x^--x^\). 

Exercise  xxxiv. 


1.  x^y—x-z—  'y^+yz. 

2.  uhxy  -\-b' y^  ■\-iicx  —  c^. 


8.  z^z^+ax^ 

4 

6 


6.  x^ —b^x-  —a-x-\-a^b^. 

7.  x^—a^x--b^x^  +  a^b^. 

8.  8x2  +  Vlax+  Wlx  ■+-  15a6. 

9.  a~^{ac-b^)x- -if-bcx^. 
10.  a3+(ac-63j^2_i,.^3. 


2x3  —  ax  —  4/)x-+  2a6. 
x--\-'lbx-\-Zax  +  ^cib. 

11.  ^^^3  f  (flc-6'/\r2-f^/'+r(rx-|-^//. 

12.  7>.r3-(^ 4-7^-2 +  (7'  +  7)x-?. 

13.  a^-\-ab-\-1iic-^h^-\-lbc-'dc^. 

14.  a;3+(f,^ij^.2_,_^,/^i^_g^a^ 

15.  iviix^-\-{^inq  —  np)x^  —  {mr  +  nq)x-\-nr. 

16.  a;^  — (a  +  i  +  c)x*  +  {rt6  +  &c+ac)x  -abc. 

17.  x'  +  (a  —  6  —  r)x-  —  (a6 — be  +  ca)x+a6c. 

18.  x^^[a-^b  —  c)x^  —  {bc—ca  -al)x  —  ahc. 

19.  ^3^3  _y3j.2^_f,22;^_j_^3j^2  ^(,x^ yz-\-x^z  —  xyz-^-ay^z. 

20.  a^bx^-\-ab'xy  -f- acdxy-\-bcdy^  —  aefxz — btfyz. 

21.  a2_g3  _  a(^i_ciaj2_j.(.^^_^._^^(.2_ 

22.  mx^  —  7ix^y-\-rx"z  —  mxy'^-\-ny^  —ry^z, 

23.  awx^  ^(^,)i(jy — nay-\-iv(z)x  —nby^  —ncyz. 

24.  (am  —  bc)n)x--{-{a)H  —  bcn)x-^a7i  +  ncx. 

25.  a-b-c^—b-c-xy —  a^c^yz  +  c^xy'-z  —  a^b'-zx  +  b-x^yz-\  a-z-^.'y 

26.  x'  -m^x^-{n-7i-)x^  +  {m^7i  —  7n-n^x^—a{x^-{-n^—n). 

27.  l-(rt-l)x-(rt-/;  +  l)^-+(a  +  6-c)x3-(6  +  f)x4  +  cx*. 

28.  a3x3-rt2(i_c4-,/)x2y_(a6c-aW+«cJ).r?/2  +  6v/?/3. 

29.  vi^npx^  —  (n^p  —  vi-n^  —vi^pq)x^  —{n^-\-7ipq  —  in^nq)x  —  n^q. 


82 


PACTORTXa. 


-{n^q-+n-^^x)y^. 

Art.  XXIV.  Sometimes  an  expression  which  does  not  come 
directly  under  the  preceding  form,  may  be  resolved  by  first  find- 
ing the  factors  of  its  parts. 

ExAiMPLES. 

1.  abx^ -\-aby^ —a-xy  —  h'^xy. 

Here,  taking  ax  out  of  the  first  and  third  terms,  and  hy  out  of 
the  second  and  fourth  terms,  we  have 

ax{bx  —  ay)  —  by(bx  —  ay),  and  hence 
(ax  — by)  (bx  — ay). 

2.  x'^-{a  +  b)x^  +  {a2b  +  ab^)x-a-b2. 

Here,  taking  the  first  and  last  terms  together,  and  the  two 
middle  terms  together,  we  have 

(x^+ab){x^-ab)-{a-]rb)x^+ab{a+b)x 
=  (x^-tab){x^-ab)-{a  +  b)x{x^     ab] 
=  {x^  —  ab)  {x"  +ab  —  (a-\-b)x}  ={x^  —ab)[x  —  a)(x  —6), 

3.  ic3w_4a;m_|_3.     This  equals 

X3m_^m  _.  3(a.OT  _  1)  =  x"'{x-'"'  -  1)  -  3(ic'«—  1) 
=  x'n{x^+l){x'"'-l)-3{x'"-—l) 
=  {x'^-l){x"^{x'^+l)  —  S}, 

Exercise  xxxv. 


1.  a^  —ab-\-ax  —  bx. 

2.  abx^  +b^xy  —  a'^xy  -aby^. 

3.  x'^+ax^  -a^^x-a"^. 

4.  rt''^a;  +  2a2x2-r2aa;3-f-a;*. 
6.  acx^  ■\-{ad  —  bc)x  —  bd. 

6.  2&x*-5x3+a;   -1. 

7.  a^  —b^  +  (>x  -ac  —  bx  +  bc. 

8.  a^  +  {l  +  a)ab  +  b-'. 

9.  x'^+2xi/{x^—y-)—y^. 
10.  x^-y^+x'^+xy+y^. 
xl.  2b  +  {b-^-A)x-2bx^. 

12.  x3  +  3a;2-4. 

13.  p3-jo2<7-2p(/3  +  273. 


14.  a3-fa2-2. 

15.  3a264_2«62     1. 

16.  ^y»-3^  +  2. 

17.  2a3-.7.2i    _a?>2+2^/3. 

18.  i3m  +  J'2»i._2. 

19.  ySn_  %j1n^n  _  2  (/««2«  4-  gSn^ 

20.  a3_4rt52_f.3/;3. 

21.  a'2"i-3a^c"  +  2c2i. 

22.  aa;3-(«2^.j)a;2  4.^,3. 

23.  35.«2n_6a2a;"-9a4. 

24.  a2^2+2rt7jc2-a2c2-63c2. 

25.  am^ —ab^ -\-b^7n  —  m*. 

26.  |-6a2  4-27a4. 


PACTORING.  as 

27.  {x-7j)^  +  (l-x+y)(x-y)z-  zK 

28.  2Am^  -28m^n  +  Gmn^  -In^. 

29.  a;w+"4.a;'!y«+a;'"y^+?/w+w. 

80.  x*  +  2x^y-a^x^+X'7j^  -2axij^  -y^. 

Section  IV. — Application  of  the  Theory  of  Divisors, 

Art.   XXV.     By  Theorem  I.  we  prove  that 
^•"  —  a"  is  divisible  by  x—  a  always 
'xf'  —  a'^  "  "         <■'■  x-\-a  -when  n  is  even 

x''  +  «"  "         "  "  a;+a  when  ?i  is  0(/<i. 

By  actual  division  we  find,  in  the  above  cases ; — 

=    a:''-^+ar''--a-f     .     .     .     a;a''-^+rt"-i ....(1). 

X  —a  ^   ' 

aj"  — a" 

X  4-«  ^ 

x'  +«" 
«  +a 

Examples 


a;«-i_a;— iiff^      .     .       -a;a"-'+a"-^ (3). 


1.  Resolve  into  factors  x^  —y^  ;  here  x—y  is  one  factor  and  by 
(1)  the  other  is  x^+xy+y^. 

2.  Resolve  a^-^(b  —  c)^  ;  here  a  +  (b  —  c)  is  one  factor;  and  by 
(.3)  the  other  is  a^ -a(b-c)  +  ih- (-y. 

3.  Resolve  a;'"  4-1 024?/io.     This  =  (a;-)5  + {(2  7/)2}5,  one  factor 
of  which  is  a;^  +  (2?/)3,  andbv  (3)  the  other  factor  is 

=  x^^  -ix'^y^-i-iax'-y'^  -Gix^y^  +256y^. 

4.  Resolve"(a;— 22/)^  +  (2a;  — ?/)3  into  factors. 
Here  by  (3)  we  have 

(a;-27/)3  +  (2x-v)^ 

\,-2y    +2x-y      =  (^  -  2^)'  -  (^" ''  2^/^^-^  - 2/)  +  (2:.  -y)' 
;.    the  factors  are 

Bix-y){'lx^-13xij+7y^), 


S4  FACTOPaNG. 

By  (1)  we  see  that  this  =  "^^^  =  'i-'+l/'K-'-v') 
^  ^  X   —y  x  —  y 

=  {x  +  y){x^-xy  +  y-'){x''+xy  +  y^). 
6.  Eesolve  x^  ^  —  x^  ° a+x^ a-  ~ x* a^  -\-x'' a*^  —  x^a'^+x^a* 


x 


12_«12 


-a;4<,7+T3a«-a-2a9-fa;a'"-rtii.     This  = 

x  +  a 

~  x+a  ~  x  +  a 

t=  (x3  +«3)(^-4  -a;2a3  +  a4)(x-«)(^2  J^xa  +  a^)i^x^  —xa+a"^). 

Exercise  xxxvi. 

Factor  tlie  following  : — 

1.  x^-y^,  x^~l,  ;c3+8,  8a3-27a;3,  8-}-a3a;». 

2.  a;5-aio,  27a3-Gi,  a^S-is,  a;io_32^5. 
d.  Find  a  factor  wliicli,  multiplied  into 

a^+(i^h-\-a'^b^  +ab^-\-b^,  will  give  a^-h^. 

4.  By  what  factor  must  x^  —  i.x'^ y+lQxy'^  —  Qhj^  be  multiplied 
togivex'^-256//4  ? 

6.  F&cior  x'' ^x^y+x'^y^  +x^y^+x^y^+x^ y^  -\-xy'^  f  y'. 
Find  the  factors  of  the  followi-p'- : 

6.    (3//2- 2^-2)3  _  (3^3  _  2^2)3^   „8_16^4. 

8.  b{x^  —a^)+ax[x'^  —a-)  +  a'^yx  —  a), 

10.  x^—y^+2xy{x^+x-y^-+y^). 

11.  (a3_5c)3  +  8/;3c3^  a;iw_a4»i. 

12.  x3-3«x2+3a2a;-a3  +  /;3. 

13.  a;3  +8?/3  4-4a:?/(x-2  -2^//  +  -l?/2). 

14.  8x3_Ga;y(2x+3//)+27»/3. 

15.  l-23:+4.c2-8x3. 

16.  a^  -\-a^bc-\-a^b^c^ -\-a^b^c-^  +ab-^c'^-\-b'^c'^. 


FACTORING. 


85 


Art.  XXVI.  The  principles  illustrated  in  Section  II.,  chap. 
II.,  may  be  applied  to  factor  various  algebraic  expressions,  as  in 
the  following  cases  ; 

Examples. 

1.  Find  the  factors  of 

{a  +  b+c){ab  +  bc-{-ca)  —  (a  +  b){h+c){c-\-a). 
1st.  Observe  that  the  expression  is  symmetrical  with  respect 
to  a,  b,  c. 

2nd.  If  there  be  any  monomial  factor  a  must  be  one.  Put- 
ting a  =0,  the  expression  vanishes.  .•.  a  is  a  factor, 
and,  by  symmetry,  b  and  c  are  also  factors.  .-.  abc 
is  a  factor. 

8rd.  There  can  be  no  other  literal  factor,  because  the  given 
expression  is  of  only  three  dimensions,  and  ahc  is  of 
three  dimensions. 

4th.  But  there  may  be  a  numencal  factor,  m  suppose,  so  that 
"we  have 

.    {a+b-\-c){ab-\-bc-{-ca)-  {a+b)(b-\-c){c+a)  =  mabc. 
To  find  m,  put  a  =  b-c=l  in  this  equation,  and  «t  =  1. 
.'.    the  expression  =  rt&c. 

2.  Eesolve  a^{b-c)+b^{c-a)+c2{a-b). 

1st.  For  a  =  0  this  does  not  vanish.      .-.    a  is  not  a  factor, 

and  by  symmetry  neither  is  b  nor  c. 
2nd.  Try  a  binomial  factor ;  this  will  likely  be  of  the  form 
b  —  c;  put  b  —  c  =  0,  i.e.,  b  =  c  in  the  given  expression, 
and  there  results 

a2  (c  -  f )  +  c2  (c— a)  +  c2  (a  -  c),  which  =  0, 
.".    6  —  c  is  a  factor,  and  by  symmetry  c  —  a  and  a  —  b  are  fac- 
tors.    Since   the   given    expression   is  only  of  three 
dimensions,  there  can  be  no  other  literal  factor ;  but 
there  may  be  a  numerical  isictoi;  m  (say),  so  that 
^t^{b-c)  +  b^{c-a)  +  c^{a-b)  =  m{a-b){h-c){c-a). 
To  find  the  value  of  «t,  give  a,  b,  c,  in  this  equation,  any  values 
which  will  not  reduce  eitner  side  to  zero;  let«=l,  b  =  2,  c  =  0 


86  FACTORING. 

and  we  have  2  =  r??(  — 2),  or  ?»-=  —  1 :  so  that  the  given  expres- 
sion =  —  (ii  — i)(6  — c)(c  — a),  or  {a  —  b)[b  —  c)(a  —  c). 

3.  Eesolve  a3{b-i-c^)-\-bl{c  +  a^)-^c'^{a-{-b')  +  abc{abc+l). 
Here  we  see  at  once  that  there  is  no  monomial  factor : 

put  h+c^  =0,  i.e.,  h—  —c^,  and  the  expression  becomes 
a.3( -6'2+c2)-cG(c  +  rt2)+c3(a4-c4)_c3a(-c3a+l)  wiiich  =  0; 
.'.  b-{-c^  is  a  factor,  and  by  symmetry  c  +  a^  and  a+b^  art;  also 
factors  ;  and  proceeding  as  in  former|examples  we  find  m=l ;    /. 
the  expression  =  (6 +  c2)(f-fa2)(rt  +  i3), 

4.  Besolve  into  factors  the  exj)ression 

(a-i)3+{6-c)3  +  (c-a)3. 
As  before,  we  find  that  there  are  no  monomial  factors. 
Let  a  — 6  =  0,  or  a  =  &,  and  substituting  b  for  a  the  expression 

becomes  zero ;  hence 

«  — 6  is  a  factor. 

B}'  symmetry    b  —  c         " 
and         c—a         " 
Hence  the  factors  are 

m,(a—b)(b—c)(c  —  a). 

To  find  m  let  a=0,  6  =  1,  c  =  2,  and  we  hav» 

6  =  2m,  or  7?z=3. 
The  factors  are,  therefore, 

S(a-b){b-c){c-a). 

5.  Eesolve  into  factors 

a3(6-c)4-63(c-a)+c3(a-6)„ 

As  before,  we  find  that  there  are  no  monomial  factors. 
Let  a— 6  =  0,  or  a  =  b  ;  substituting  b  for  a,  the  expression  be- 
comes zero  ; 

therefore  a—b  is  a  factor. 

By  symmetry     b  —  c         " 
and     c  —  a         " 
Now  the  product  of  these  three  factors  is  of  three  dimensions, 
while  the  expression  itself  is  of  four  dimensions.     There  must, 
therefore,  be  another  factor  of  one  dimension.     It  cannot  be  a 


FACTORING.  87 

monomial  factor,  for  the  expression  has  no  such  factors.  It  can- 
not be  a  binomial  factor,  such  as  a+b,  for  then,  by  symmetry, 
b-\-c  and  c-{-a  would  also  be  factors,  which  would  give  an 
expression  of  six  dimensions.  It  cannot  be  a  trinomial  factor, 
unless  a,  h,  and  g  are  similarly  involved.  For  instance,  if  a  —  b+c 
were  a  factor,  then,  by  symmetry,  b—c-\-a  and  c  —  a-\-h  would  also 
be  factors,  and  the  dimensions  would  be  six  instead  of  four.  The 
other  factor  must,  therefore,  be  a+b  +  c.     Hence, 

a^{b  -c)  +b^{c-  a)-{-c^{a—b)  =  m(a—b)(b  —  c){c  -  a){a  +  b  +  c). 

To  find  m,  put  a  =  0,  b  =  l,  and  c  =  2,  and  we  have 
—  6  =  6«i ; 
:.    m  =   —  1. 
Hence  the  factors  are 

—  {a  —  b)(b  —  c)(c  —  a)(a-\-b+c^, 
or,         {a  —  b)(a  —  c)(b  —  c)(a-'rb  +  c). 

6.  Prove  that 

a^^b^  +  c^^S(a+b){b  +  c)(c  +  a) 
is  exactly  divisible  by  a+b  +  c,  and  find  all  the  factors. 

Jjeta  +  b+c  =  0,  or  a=  —{b+c);  substituting  this  value  for  a, 
we  have 

-{b+c)^  +  h3+c^  +  Sbc{b+c),  or 
-(6  +  c)3  +  (6+c)3  which  =  0,  and 
therefore  a+b+c  is  a  factor. 

As  before,  we  find  that  there  are  no  monomial  factors.  Since 
a+b+c,  the  factor  already  obtained,  is  of  one  dimension,  the 
other  factor  must  be  of  tico  dimensions,  and  cannot,  therefore,  be 
a  binomial ;  for  if  a+b  were  a  factor,  by  symmetry  b+c,  and  c+a 
must  also  be  factors.  The  factors  in  that  case  would  give  a 
quantity  of  four  dimensions,  while  the  expression  itself  is  only 
of  three  dimensions.  Nor  can  a~+b^+c^  be  a  factor.  For 
if  So,  the  other  factor  must  involve  a  numerical  multiple  of  the 
first  power  of  a,  and,  therefore,  on  taking  the  first  power  of  a  out 
of  terms  involving  first  and  third  powers,  we  should  have  left 
some  numerical  multi]ple  of  a^+b'+c^,  instead  of  wliich  we  get 


SS  FACTOEING. 

a2-f3(i-i_c)2,  Nor  can  af  ■}-{h-\-c)'-'  be  a  factor,  for  symmetry 
would  requii-e  two  other  factors,  viz.:  b--\-{c+a)",  &ndc^ +{a-\-b)^, 
thus  giving  a  quautifcy  of  scffw  dimensions. 

The  only  factor  admissible  is,  therefore,  {a-\-h+cy. 
Hence 

a^  +  i>-'-i9'+3{a-i-b){b+c){c-^a)  =  rn(a-\-b-^c){a  +  b+c)* 

=  vi[a-j-b-^c)^. 
To  find  m,  let  a  =  l,  6  =  0,  and  c  =  0,  and  we  have  l=m. 
Hence  the  factors  are 

{a  +  b+c){a-l-h  +  c)(a-{-b+c). 

7.  Simplify 

a{b-^c)^-i-b(a  +  c)^+c{a+b)2  —  (a+b){a-c){b-c) 
-(a-b){a-c){b  +  c)  +  {a  —  b){h-c)ya  +  c}. 

Let  a  =  0,  and  the  expression  becomes 
be '--^cb^-\- hc(b  —  c)  —  bc{b -\-c)  -  bc(b -  c),  which  equals  zero  ;  there- 
fore a  is  a  factor  ;  by  S3'mmetry  b  and  c  are  also  factors. 

The  expression  is  of  three  dimensions,  and  abc  is  of  three 
dimensions,  there  cannot  therefore  be  any  other  hteral  fs-,ctQr. 

Hence  the  expression  =mabc. 

To  find  m,  let  a=b  =  c  =  l,  and  we  have 

m=12. 
.*.  the  expression    =12abc. 
In  the  preceding  examples  the  factors  have  been  linear,  but  the 
principle  applies  equally  well  to  those  of  higher  dimensions.    (See 
Th.  ii.  Cor.)  . 

8.  Examine  whether  x^+l  is  a  factor  of  a;3"  +  2jc-"-l-3.r"-|-2. 
Let  a;"-i-l  =  0,   or  x"=—l,  and   substituting,    the   expression 

vanii-Ves,  therefore,  x"-\-l  is  a  factor. 

9.  Examine  whether  a'^  +  b^  is  a  factor  of 

2a'^+aSb+2a^b'-i-^ab^. 

Let  a^  +  b-  =0,  or  a^  =  —b-,  substituting,  we  have 
264_a6-''— 2Z-4+ai3  which  =  0,  and 
therefore     a^-j-  6- is  a  factor. 


FACTORING.  §9 

10.  Prove  that,  a^ +h^  is  a  factor  of 

JjQt  a^-\-h^=0,  or  a^  =  —  h^;    substituting,  we  have 
-a^h^ -ab^-b^-^-a-b^+ab^-i-b^,  \vliich  =  0,  aud 
therefore  a^-\-b^  is  a  factor. 

Exercise  3cxxvii. 
Eesolve  into  factors 

2.  hc{b  -  c)—ca{a  -c  )  -ah{b  -a). 

5.  I^a+b)^-{b-^c)^  +  {c-ay. 

6.  a(b-c)^+h{c-aY-\-c{a-bY. 

7.  {a->rb  +  c){ab  +  hc+ca)-ahc. 

8.  a^{c~b^)  +  b^{a-c-)  +  c^{b-a^)  +  abc{abe-l).     - 

9.  a2(6  +  c)  +  /j3(6-  +  a)  +  c2(a  +  />)4-2«6c. 

10.  {a-h){c-li){c-k)-ir{h-c){a-h){a-k)  +  {c-a){b-'i){h-li). 

11.  x^ij"  -f  x^y^+x'^z'^  +x-z'^  +  ij'^z-  +7j^z'*^  +  2x^y-z'^. 

12.  {a-by-^{b-c)^+{c-ay 

13.  aft(a+i)  +  /;c(/»4-c)+ca(cH-rt)+("^  +  ^^+'-^). 

14.  a^{c-h^)+b^{a-c^)-\-rA{b-a^)+abc{a^b2c^-l). 

15.  a;4(.y3-z2)_|_.y4(23_^2) +24(^3  _,y2), 

16.  a;4^.y4_j_24_2a;2y3_2^223_2z2^2. 

17.  {b  —  c){x  -  b)(z  —  c)  -\-(c  —  a){x  —  c){x  —  a)  +  (a  —  b)[x  —  a){x—  b). 

18.  (a+fc)3  +  (/,+c-)3  +  (c  +  «)34. 

19.  Shew  that  a^  +<i'^b-  —ah^  -  b^  has  a^  —  b  for  a  factor. 

20.  Shew  that  (x  +  ?/) "  -  a: ^  -  ?/ ^  =  7xy{x+y) {x^  +xy+y^)^. 

21.  Examine  whether  x^  —  5x-\-Q  is  a  factor  of 

a;3_9^2_{_26a;-24. 


90  FACTOEING. 

22.  Skew  that  a  — 6+c  ig  a  factor  of 

a^{b+c)-b^{c+a)  +  c^fn+h)-^ahc. 

23.  Shew  that  a'^+3b  is  a  factor  of 

and  find  tbe  other  factor  . 
24.  Find  the  factors  of  a^{b-c)+b^{c-a)+c'>^{a-h). 


Sectiosst  v.. — Factoking  a  Polynome  by  Trial  Divisoes. 


Art.  XXVII.  To  find,  if  possible,  a  rational  linear  factor  of 
the  polynome. 

Substitute  successively  for  x  every  measure  (both  positive  and 
negative)  of  the  term  k,  till  one  is  found,  say  m,  that  makes  the 
polynome  vanish,  then  x  —  m  will  be  a  factor  of  the  polynome. 

Examples. 

1.  Factor  a;3-t-9x-+16j;+4. 

The  measures  of  4  are  +1,  +2  and  +4.  Since  every  coeffi- 
cient of  the  given  polynome  is  positive,  the  positive  measures  of 
4  need  not  be  tried.  Using  the  others,  it  will  be  found  that  —2 
makes  the  polynome  vanish ;  thus 

1         9         16         4 

-2     -14     -4 


-2 


17  2;       0 

Hence  the  factors  are  {x  +  2){x^-{-7x-\-2). 

The  labour  of  substitution  may  often  be  lessened  by  arrang- 
ing the  polynome  in  ascending  powers  of  a;,  and  using  1  — 
(measure  of  k)  instead  of  the  measures  of  k.  (This  is  really 
substituting  1  -f  measure  of  k,  for  1-i-x).  Should  a  fraction 
occur  during  the  course  of  the  work,  further  trial  oi  that  measure 
of  k  will  be  needless. 


FACTORING.  gi 

Examples. 
2.  Factor  x^  -  lOa-3  -  63a;+60. 

The  measures  of  60  are  +1,  +2,   ±3,  ±4,   +5,  fee.     Neither 
4-1  nor  -  1  will  make  the  polynome  vanish.     Try  2  ;  thus 

:     60         -G3         -10         1 

1  I  30 


2  I     30  -161- 

A  fraction  occurring  we  need  go  no  further.      —  2  will  also  give 
&  fraction,  as  may  easily  be  seen.     Next  try  3  ;  thus 


60         -G3  -10         1 

20 


20         -14i 


A  fraction  again  occuring,  we  may  stop.      —  3  will  also  give  a 
fijaotion.     Next  try  4  ;  tlius 

I     60         -63  -10         1 

1  i  15  -12 


4  1  15         -12  -   5i 

Next  try  —4. 

I  60         -63  -10 

-1    :  -15 


4  1     15         -19i 
Ne^t  ix^vaeg  5  we  find  it  fails,  then  try  —  5,  thus 


-1 

60 

-63 
-12 

-10 
15 

1 
-1 

5 

12 

-15 

1; 

0 

The  remainder  vanishes  as  required  ;  the  factors  are,  therefore, 
(a;+5)(a;2-15a;+12). 

Art.  XXVIII.  When  k  has  a  large  number  of  factors,  the 
number  that  need  actually  be  tried  can  often  be  considerably 
lessened  by  the  following  means. 

Add  together  all  the  coefficients  of  x  (including  the  constant 
terra  k) ;  let  the  sum  be  called  h^. 


92  FACTORING. 

From  the  sum  of  the  coefficients  of  the  even  powers  of  x 
(including  k)  take  the  sum  of  the  coefficients  of  the  odd  powers  of 
x;  let  the  remainder  be  called  k^.  (In  the  coefficients  are  in- 
cluded the  signs  of  the  terms). 

1st.  If  k^  vanish,  x  —  1  will  he  a  factor  of  the  polynome. 

2ud.  If  k^  vanish,  x-\-l  will  be  a  factor  of  the  polynome. 

3rd.  If  both  k^  and  k,^  vanish,  x^  —1  will  be  a  factor  of  tlie 
polynome. 

4th.  If  neither  k^  nor  k.^  vanish,  (writing  p  for  "  a  positive 
measure  of  k  greater  than  1  ")  ; 

[a)  We  ijeed  not  try  the  substitution  of  p  for  a;  unless^  — 1  be 
a  measure  of  k^,  and  j9+l  a  measure  of  k^. 

(B)  Nor  need  we  try  the  substitution  of  —p  for  x  unless  /J-fl 
be  a  measure  of  /tj,  and  p  —  1    a  measure  of  k^. 

(In  trying  for  measures,  the  signs  of  k,  k^,  and  k^  may  be 
neglected. 

Examples. 

1 .  Find  the  factors  of  a;^  -  lOa;^  -  63a;+60.     (See  Ex.  2  above). 
B.exek  =  QO;  k^=     1 -10-63  +  60= -12, 
k.^=-l- 10+63+60  =  112. 

Tabulating  the  trial-measures  we  get 


12 

1, 

2, 

3, 

4, 

60 

2, 

3, 

4, 

5, 

6, 

10, 

112 

4, 

7, 

12 

3, 

4, 

6, 

60 

2, 

3, 

4, 

5, 

6, 

10, 

112 

1, 

2, 

4, 

(It  is  evident  that  12  is  the  highest  measure  of  60  we  need  try 
in  the  upper  table,  for  the  next  measure,  15,  would  give  14  as  a 
trial-measure  of  12,  and  higher  measures  of  60  would  give  higher 
trial-measures.  Similarly,  10  is  the  highest  measure  that  need 
be  tried  in  the  lower  table.) 


FACTORING. 


98 


In  the  upper  table,  8  is  the  only  measure  of  60  that  gives  a 
full  column  ;  heuce  of  the  positive  measures  of  GO  we  need  try 
only  the  substitution  of  3  for  x. 

In  the  lower  table,  2,  3,  and  5  give  full  columns,  hence  we 
must  try  the  substitutions  —2,  —3,  —5  for  x. 

On  trying  the  four  substitutions  to  which  we  are  thus  restricted 
we  find  —5  is  the  only  one  for  which  the  polynome  vanishes. 
(See  Ex.  2  above). 

2.  Find  the  factors  of  .r4+12x3-40a;2-|.67a;-120. 
A- =-120;  /.•,=1  4-12-40+07-120= -80;       » 
A-2  =  1-12 -40- 07 -120  = -238. 


80 

1, 

2, 

4, 

5, 

120 

2 

3> 

4, 

5, 

6, 

8, 

10, 

12, 

15,  &c. 

233 

7, 

80 

4, 

5, 

16, 

120 

2 

3, 

4, 

5, 

c, 

8, 

10, 

15,     20, 

21, 

'&c. 

238 

1, 

2, 

7, 

14,     21, 

The  upper  table  gives  us  6  as  a  trial-measure,  and  the  lower 
gives  us  —3  and  —15. 


Trying  these 

i  we  get 

-120 

07 

-40 

12 

1 

1 

-20 

6 

-   20 

n 

-120 

67 

-40 

12 

1 

-1 

40 

8 

-40 

35 1 

i     - 120 

67 

-40 

12 

1 

-1 

8 

—  5 

3 

-1 

15 

-      8 

5 

-   3 

1: 

0 

94 


FACTOEINO. 


Hence  z+ 15  and  x^  —  Sa;^  +  5a;  —  8  are  the  factors.     The  latter 
cannot  be  resolved,  for  our  tables  above  tell  us  we  need  try  only 
x—Q,  a;+3,  and  ic+15.     The  first  two  have  been  found  not  to  be 
-factors,  and  15  will  not  measure  8. 
4.  Factor  a;*  -  27a;2  -f  14a:+ 120. 

A- =  120;  /.-,  =  1-27+14+120  =  108 
A",  =  1-27-14  +  120=    80. 


108 

1, 

2, 

3, 

4, 

9 

120 

2, 

3, 

4, 

5, 

6, 

8, 

10, 

80 

A 

5, 

108 

v3. 

4, 

6, 

9 

120 

2, 

4, 

5, 

6. 

8, 

10, 

80 

1 

2, 

4, 

5. 

12, 


15,  &c. 
16, 


12,     15,  &c. 


The  upper  table  gives  ua  3  and  4,  the  lower  table  gives  us  -  2, 
—  3,  and  -5.     Using  these  in  order  we  get 


Hence  «  —  3  is  a  factor. 

Hence  a;  —  4  is  a  factor. 

Hence  a:+2  is  a  factor, 
and  there  remains  a;+5,  a  factor. 

Hence  the  factors  are  (x-3)(a;-4)(a;+2)(a;  +  5). 

5.  Factor  x^  -  px^-\-  {q  -  V)x^-  +px  -  q. 

k=-q;  A-,=l-/>  +  (7-l)+2>-?  =  0; 
k.,=^l+p  +  {q-l)-p-q  =  0. 
Since  both  k^  and  k^  vanish,  the  polynome  is  divisible  by  both 
a;  — 1  and  a;+l. 


120 

14 

-27 

0 

1 

1 

40 

18 

—  o 

-1 

3 

40 

18 

-  3 

-1; 

0 

1 

10 

7 

1 

4 

10 

7 

1; 

0 

-1 

-5 

-1 

2 

5 

1; 

0 

-1 


-p 
1 


-p+l 
-1 


q-1 
-P  +  l 

q-p 
+P 


P 

q-p 


1 


FACTORING.  95 

Hence  the  other  factor  is  x-  —  px-\-q. 

6.  Factor  x^  +  1ax^+{a^ +a)x--{-2n^x-^a^. 

k  =  a^  ;  /f^  =  l+2a  +  (a2+a)-f2a^-f-a3  =  («4-l)3; 

A-2=l-2a4-(a3+r/)-2a2+a3  =  a3_«2_„_i_ 

The  positive  measures  of  k  are  1,  a,  a^,  a^.  Of  these  1  may 
be  rejected  at  once,  since  neither  k^  nor  k^  vanish,  and  a^  and  a^ 
may  also  be  rejected  since  k^  or  (a-)-l)3  is  not  divisible  by*either 
a2  +  l  or  a^±l.  But  ky  is  divisible  by  a+1,  and  k^  is  divisible 
by  a  —  1  ;  thus  we  need  only  try  the  substitution  of  —  a  for  x. 
(See  4  0,  page    92) 


1 

2a 

a^+a 

2a2 

a» 

—  a 

-a^ 

-a2 

--a? 

1 

a 

a 

«2; 

0 

—  a 

0 

-a3 

10  ■     a;         0 

Hence  the  factors  are  {x-\-a)-{x^-\-a). 

7.  Factor  a;^  -[(,,  +  c)x^  +{b  +  nc)x-bc. 
k   =  —be ; 

kj  =      1  —  (a+c)  +  {b+ac)  —  bc=  1  —a+b—c  +  ac -be 
kj=  -l-(a+c)-{b-\-ac)'-bc=  -(l+a-\-b+c  +  ac+bc). 

The  factors  of  k^,  other  than  1,  are  b  and  c.  k^  is  not  divisible 
by  either  6  +  1  nor  by  c+1.  However,  A;  j  is  divisible  bye— 1, 
and  k^  is  at  the  same  time  divisible  by  c+1,  .-.  we  need  only  try 
the  substitution  of  c  for  x.     (See  4  «,  page  8G). 


c 


(b  +  ao)     —be 
—  ac  be 


I     1  —a  b ; 

Hence  the  factors  are  (x  —  c){x^  —  ax-{-b}. 


96 


rACTOEING. 


Exercise  xxxviii. 


1.  a^—9a^  +  liJa-4:. 

2.  a;^-9a;3  4-2Ga;-24. 

3.  .r3-7a;2-j-i6a;-12. 

4.  a:3_i2:,.+  i6. 

5.  x^  +  '3x^  +  ox  +  S. 

6.  a;4_)_i^.3_|_i0x2+12a;+9. 

7.  a;3-8^+2. 

8.  a;4  +  2a;3-|-9. 

9.  m^  —  Bm-n  +  Amn^  —  9,11^ 
10.  a;3_^2a;3  +  2. 


11. 


5)ii^n-\-8i)i)}^  —  An^. 


12.  63+//2(._|.76,.2^  39,-3. 

13.  ;/'4 -.4»/»3  4-3,;4. 

14.  a4_7,,3/^_j..28,(//3_iC64. 


15.  a:3-ll.T2-f39a;-45. 

16.  a;3H-5.T2 4-7x4-2. 

17.  «3_8a3_i93„_j.i95. 

18.  p^-Sp^-Gp-S. 

19.  «4  4-Grt3_3rt2_7a4_6. 

20.  rt«"-6a^"4-lla-"-6. 

21.  a4_41«2/,2_|_1664. 

22.  «4_^,2/,2_2((634-264. 

23.  p^-4p^-\-Gp-'4:. 

24.  a:-"4-4a;-''-5. 

25.  2/^ 
26 
27. 
28.   2u'"-a-"- 


,4  _  5^34.8^2  ._  8. 

„4_2„3  +  3«2_2r,4-l. 
«3+„2/,2_|_a/,2_3^3. 

■a"  4-2. 


29.  a;4- 18x3  + 113.^3  _  288x4-252. 

30.  x'^-dx^y-^-'ZOx^y^-ddxy^  ^18y*.    ' 

Art    XXTX.     To  find,  if  possible,  a  rational  iioear  factor  of 
the  pulynoiBe 

ox"  4- />x""^  +  rx"~'  -f- 4-  ^'^  +  ^• 

First  Method.     Multiply  the  polynome  by  «"~^. 

(aa:Y+b{axY-^+ac{ox)''-^  + -\-a"''h{ax)-\-a"-Vc; 

or  writing  y  for  ax, 

y^  4_  /;_,y"-i  .4.  ^r  ?/"-^  4- -\-a''-'hy -{■  a^-^k. 

Factor  this  polynoiie  by  the  method  of  the  last  article,  replace 
y  by  ax,  and  divide  the  result  by  a"~^ 

Example. 

Factor  3x4-|-5x3  -33x2  4-43x-20. 
Multiply  by  33  and  express  in  terms  of  Bx. 

(3x)4-h5(3x)3-99(8x)2  4-387(3x)-54&; 
or,    ?/4 -1-52/3- 99^2  _j_3872/_  540. 


FACTORING. 


97 


B.eveJc=  -540;  ^•,=1  +  5-90  +  387-540=  -246; 
Zo  =  1  - 5  - 99 - 387 - 5i0  =  - 1030. 

82,     123,     240. 


246 

1, 

2 

3,  G,  41, 

540 

9 

B, 

4, 

1030 

5, 

246 

3, 

6, 

41,  &c. 

540 

2, 

5, 

1030 

1, 

(Trying  by  factors  of  246 
instead  of  by  factors  of  540, 
for  couvenienct). 

The  only  factors  of  540  in  full  columns  are  4  in  the  upper 
table  and  2  in  the  lower  one  ;  hence  we  need  try  only  the  subriti- 
tutions  4  and  —2. 


1 

-540 

387 
-135 

-99 
63 

5 

-9 

1 
-1 

4 

—  135 

63 

-  9 

-1; 

0 

Hence //  — 4  is  a  factor.     The   substitution    —2  need   not  now 
be  tried,  since  we  see  tliat  135  is  not  a  multiple  of  2.     The  other 
factor  is  therefore  y^  +  O//^  -63// +  135. 
Replacing  ij  by  3a;  aud  dividing  by  27  ; 

^V(3a;-4)(27x3-f81x2-189a;+135) 
=  {Sx-A){j:^  +3x3  -7x  +  5), 
which  are  the  factors. 

Art.  XXX.  Second  Method.  Writing  »?  for  "  a  measure  of 
a,"  and  p  for  a  "  measure  of  k,  positive  or  negative  ;" 

For  X  substitute  every  value  of  p-m  till  one,  s^j  p'-^m'  be 
found  which  makes  the  polynome  vanish;  then  in'x  —  p'  will  be 
a  factor.  Should  a  fraction  be  met  with  in  the  course  of  substi- 
tution, farther  trial  of  that  value  ui p~in  will  be  useless. 

Should  k  have  more  factors  than  a,  it  will  genei'ally  be  better 
to  arrange  the  polyuome  in  ascending  powers  of  x  and  use  values 
of  m  4- JO  instead  of  p-i-iit,  making  jj  positive  and  vi  positive  or 
negative. 


98  FACTORING. 

To  reduce  the  number  of  trial-measures,  calculate  b.^  and  Jc^,  as 
directed  on  page  92,  then  1,  2,  3  hold  as  on  that  page,  but  in  4 
read  ^?  —  w  for  p  —  1  and  p-{-m  for  p  +  1. 

Examples. 

1.  Factor  36*3  + l71a,2-22x+480. 

k  =  4.S0,k^=     36+171-22+480  =  085 
^2=  -36+171+22+480  =  637. 
m  may  have  any  of  the  values  +1,  +2,  +3,  +4,  +6,  +9, 
+  12,  +18,  +86. 

In  forming  the  table  write  out  the  measures  of  ^^  ;  take  each 
measure  in  succession  and  add  to  it  each  value  of  m  separately, 
should  the  sum  measure  480,  i.e.,  k,  add  to  it  the  same  value  of 
m,  and  should  the  new  sum  measure  637,  i.e.,  Jc^,  keep  the  mea- 
sure of  480,  writing  above  it  the  value  of  m  used.  Should  the 
sum  in  either  case  not  be  a  measure,  another  value  of  vi  must  be 
tried  ;  when  all  the  values  of  m  have  been  tried,  another  measure 
of  665,  i.e.,  k^  must  be  tried  till  all  have  been  tested.  (Measures 
of  y^i  or  665  have  been  used  in  this  instance  because  they  are 
much  fewer  than  those  of  480  ;  measures  of  k^  or  637  would  have 
done  equally  well). 


m  = 

+  3, 

+  1, 

+  3 

—  2 

-3 

-9 

-3 

665 

1, 

5, 

7 

5 

7 

19 

19 

480 

4, 

6, 

10 

3 

4 

10 

16 

637 

7, 

rr 

< . 

13 

1 

1 

1 

13 

Hence  the  only  substitutions  that  need  be  tried  are 

4 '     6  "    lO'        3'        4'      lO'      16'  X ' 

Arrangement  in  ascending  powers  of  x. 

By  actual  trial,  as  below,  we  find  ^|  is  the  only  one  of  these 
giving  a  zero  remainder. 


FACTORING. 


99 


8 

480 

-   22 
360 

171 

86 

4 
1 

120 

844 
80 

6 
3 

80 

9| 
144 

10 
-2 

48 

12-2 
-320 

228 

-266 

8 
-3 

160 

-114 
360 

133; 

-230 

4 
-9 

120 

-  95^ 
-432 

10 
-3 

48 

-45-4 
-   90 

21 

-36 

16 

30 

-     7 

12; 

0 

(The  coefScients  are  written  only  once,  and  understood  for  the 
other  lines  of  substitution.) 

Hence  the  factors  are  3a;+16  and  12x-  -  7x-\-?>0. 

The  latter  factor  cannot  be  resolved,  for  16  will  not  measure 
30,  and  all  the  other  factors  left  for  trial  by  the  tables  above, 
have  been  tried  and  have  failed. 

2.  Factor  lOx^ -x^(15i/-\-4:z) -z^{4:0y^ -6yz)-^ 
a;(60!/3  +  16?/2z)_247/3z. 

Here  m=  ±1,  +2,  ±5.  or  +10.     k=  -2iyH. 

^•l  =  10 -(152/+4z)-(40y2-6yz)  + (607/3  + 167/22)  _24?/32 
=  10  -  15y  -  40y2-f-637y  3  -  2^(2  -  3v/  -  8?/2  + 1 2?/3) 
=  (5_22)(2-37/-8y/2+i22/3). 
7,-3  =  (5  +  2z)(2  +  32/-8y2_i2^3)^  as  may  easily  be  found 
by  making  the  calculation. 

We  get  at  a  glance  2z  a  factor  of  7c,  2z  —  5  a  factor  of  k^,  and 
2z+5  a  factor  of  k^  ;  hence  taking  m  =  5,  we  are  directed  to  try 


the  substitution  —  for  x. 
5 

10       -(Uy  +  iz)        -(407/2-67/2) 

2?                              4z                     —  6yz 

(607/3  +  16,/=2) 

-16?/-z 

-247/3- 

2%\ 

5        2     -By                —87/2 

127/3; 

0 

100 


FACTORING. 


Hence  5x  —  2z  is  a  factoi',  tlie  other  being 

The  latter  factor  being  homogeneous,  the  method  of  this  article 
may  be  applied  to  it. 


771=  +1  or 
111  =  1, 

±2 

,    /c  = 

2, 

12, 
1, 

/., 

=  3, 
-1 

k,  =  15. 

3 
12 
15 

1, 

1, 
3, 
5, 

3, 
4, 

5, 

3 
2 
1 

The  other  columns 
are  not  full. 

Hence  the  trial- substitutions  (arrangement  m  ascending  powers 
of  x)  are  i,  f ,  i,  =±. 


12 

-8 

-3 

2 

1 

6 

-  1 

-2 

2 

6 

-1 

-2; 

0 

2 

4 

2 

3 

2 

1; 

0 

Final  factor  is  2y+Xr. 

Hence  the  factors  ai-e  {x  —  2//)(2x  —  dij){x+2)j),  and  these,  with 
the  factor  5x  —  2z  already  found,  give  the  complete  resolution  of 
the  polyuome  proposed. 

(The  factor  ijx  —  2z,  might  easily  have  been  got  by  the  method  of 
Art.  XXIII.,  page  79,  but  the  present  solution  shows  we  are  inde- 
pendent of  that  article.  It  may  also  be  obtained  by  rearranging 
the  polynome  in  terms  of  ^). 

Exercise  xxxix 
Factor 

l.aa;3-20a:3+38a;-20;  x^  -7x^y  +  lGx>/^  -12ij^. 

2.   12x^-i-6x^yi-Z!j^  +  3y^  ;  8x^ -Ux+G. 

8.  3x^-15ax+a^x-5a^;  2x^-{-9x''y-^7xy^ -Sj/^. 

4.  254_7i3c_463c2_|.j(.3_4c4  ;  l5a3-}-47rt8i  +  13rt52_i263. 

5.  4p^  +  8p^q+lp^q^^-hSpq^  +  3q^. 

6.  150a;'i  -125x^y+d'dlx^y^  +920zy^  - 1152?/*. 

7.  36a;^— 6(9-7?/)a;3-7(9  +  14?/)x3//+3(49-40?/)a;?/2  +  180?/3. 

8.  lOx*  -x3(] 5y+42)  +  j;3(40(/2 _|_Gy2)  +x(G0(/3 -iG^^g) _ 24:y''z 


DIVISION.  101 


CHAPTEK  IV. 


Section  I. — Division.      Measures  and  Multiples. 


Art.  XXXI.  When  one  quantity  is  to  be  divided  by  another 
the  quotient  can  often  be  readily  obtained  by  resolving  the  divisor 
or  dividend,  or  both,  into  factors. 

Examples. 

1.  Divide  a2_2a64-62-c2  4-2«Z-(/3  hy  a -\h-\-c—d.  Here 
we  see  at  once  that  the  dividend  ={a  —  b)^-(c  —  d)^,  and  .•.  quo- 
tient =  a-h  —  [c-d)  =  a  —  h  —  c-\-d. 

2.  Divide  the  product  of  a^-\-ax-^x^  and  a'^-\-x^  by  a^-\-a^x^ 
+x^.  Here  a^+x^  ^{a-{- x){a^  —  ax-{-x^),  and  the  divisor  = 
{a^-]-az-\-x^){a^ —ax+x^)  :.  the  quotient  is  a -fiC. 

3.  Divide  a^+a^b+a^c  —  abc-b^c-bc^  by  a^-bc.  The  divi- 
dend isa(«2_5c)-|-6(«2  _i(.)_|_e(a3  -he)  .".the  quotient  =a+b->-c. 

4.  (a^+b^-c^  +  Sabc)~{a-{-b-c). 

Dividend  =a^-{-b^-{-Sab{a-\-b) -c^ -Bnb{a+b)-\-Sabc  =  {(X.J^b)^ 
—  c3  —  dab(a  -\-b  —  c)  which  is  exactly  divisible  by  a-f  ft  —  c ;  quotient 
=a2-[-ft2_^(.2  _(ib-{-bc-\-ca. 

5.  Di\ide  z^  —x'^y-{-x^y^  —  x^y^  +xij'^  —y^  byx^—y^. 

The  dividend  is  (Ai't.  XXV,)  evidently  (x^— ?/*')  -^  (^3+;/),  and 
this  divided  by  x^  —y^  =  {^^+y^)  -^  {x-\-y)=x'^  —xy-\-y-. 

6.  Divide  b{x^  +a^)-\-ax{x^  -  a'2)  +  a^{x+a)  by  {a-\-h){x-ira). 
Striking  the  factor  x-\-a  out  of  dividend  and  divisor  we  have 
b(x^  —  ax-\-a^)-\-ax{x  —  a)-{-a^=b[x^  —  ax  +  a^y-\-o{x^  —  ax-^-o"^) 
=  {a-^b){x^  —  a£-\-a^)  .•.  quotient  =z^  —ax-\-a^. 

7.  DWidie  apx^  •\-x^{aq  +  bp)-\-x'^{ar -\-bq+pc)+x{<ic-\-br)  ->t- cr  by 
ax-  -^bx-^" 


102  DIVISION, 

Factoring  the  dividend  (Art.  XXIII.)  we  have 

/.   the  quotient  =  the  latter  factor. 

S.  Divide  Q>x^  -  ISaa;^  +  l^a^x^  -  l^a^x-  ha^  hy  2a;'  -  ?>ax~  a?. 

This  can  be  done  by  Art.  XVII.  The  divisor  is  2a;3  -a^  -  3ax, 
and  we  see  at  once  that  Sx-  -\-5a^  must  be  two  terms  of  the  quo- 
tient. 

Multiplying  diagonally  into  the  first  two  terms  of  the  divisor, 
and  adding  the  products,  we  get  -{-la^x^  ;  but  -{-ISa^x'^  is  re- 
quired. .'.  -\-Qa-x^  is  still  required,  and  as  this  must  come  from 
the  third  term  multiphed  into  —  Sax,  that  third  term  must  be 
—  2ax  ;    .".  the  quotient  is  3x^-{-5a^  —  2ax. 

j^OTE. By  multiplying  the  terms  -  2ax,  —Sax,  diagonally  into 

the  x^'s  and  a^'s  respectively,  we  get  the  remaining  terms  of  the 
dividend  ;  it  is,  of  course,  necessary  to  test  whether  the  division 
is  exact. 

9.  Divide  2a^-a^h-12a-b''-5ab^  +  U^  hj  a^-b^-2ab. 
Here,  as  before,  one  factor  is  a'^  —  h^—2ab;    :.  tivo  terms  of 

the  other  factor  are  2a^-Ab^.  Multiplying,  as  in  the  last 
example,  we  get  -6a^b^  ;  but  -12a^b^  is  required.  .*.  —6a^b^ 
is  still  needed,  and  +3a&  is  the  third  term  of  the  required  quo- 
tient, which  is  therefore  2a^  —  4:b^-\-Bah. 

Prove  that 

10.  {l+x+x"'+  .   .   .  .   -Jr.v''-')(l-x+x^-   ....   +x''-^) 

=  l-f.x2+a;4+    ....    +x--"-^ 

l-x"       1  -f  a;" 
Product  =  Y=^'-     Y+^ 

^~''^     =  1+x^+x*-^     ....  H-a;^"-*. 


-x' 


11.  Divide  (a^-bc)^-]-Sb^c^  by  a^+he. 

=  (a2-&c)3  4-(26c)3  hy  {a^-bc)  +  2be 
^  (a2  _/,c)2  -(^2  _5c)  X  2ic+(2ic)« 
=  a^'-4.a^bc-\-lb^c^. 


DIVtSON.  103 

12.  Divide  l+2357947691a;^  by  l-lla:4- 121^3 
Dividend  =l4-(lia;)» 

=  {1  -(llx)3.f  (lla;)«}{l  +  (lla:)H 
Divisor  =  { 1  +  (11a;) 3  }  -^  (l-fllx). 
.-.  quotient  =  U-(llx-)3+(llx)«}(l+llic)„ 

Exercise  xi. 

Finrl  the  quotients  in  the  following  cases : 

1.  1  —x-\-x^  —x^  -^1—x. 

2.  l-2x^+x^  ^x^+2x^  +  l, 

3.  x^^+a^x^+a^^  ^x*-a^x-+>i^, 

4.  2;4H-4a;2i/3-32?/4-Hcc-27/. 

6.  l-4x3+12a;3-9a;*^l  +  2a;-S.T'. 

6.  {a^  -2nx-\-x-){a^+'da'^x  +  Sax--\-x^)  -^a'i  -x^. 

7.  x^—i/^+z^-\-dx>/z^x—i/  +  z. 

8.  6a4_a36  +  2«262 4-13^63  +  454  ^  2^»-3a64.4^». 

9.  4:X^  —  x^y" +Qxy^  —9i/*  -r-  2x^+Sy^—xy. 

10.  rt*+i4_c4_2rt263-^«3_J2_c2, 

11.  21a4-16rt3i-f.l6a2^3_5ai3  +  264  ^  Sa- -ab-{-b^. 

12.  2ffl3_7a2_46rt-21  H- 2a2  4-7rt  +  3. 

13.  {a^(b-c)+b^{c-a)-^c^{a-b)}  ^  a~\~b-^c. 

14.  x^  —  Sax'^-'r3a-x  —  a^-\-b^-—x—a+b. 

15.  a;*-?/4+0^-f-2a;322_2y2_i  _^a.3_^2u-^s_i. 

16.  «•*  — (a  +  c}x3  +  (6  +  ac)j;3  _;)fa;  -^  x  —  c. 

17.  x3+a;3?/  +ic?/3+?/3  _^  x+y. 

18.  x'^  —x^y+x^y^  —  x^y^-JrX^y^  -x^y^-'rxy^  —y'  -^x*+y^. 

19.  a'i+i4-C*-2fl3fe2_2(;3_l   -HVi2_/;2_c2_l. 

20.  a*  -  ai3_ac3-2y 36+264 +  26c3^3«3f_ 363c  _3c4 
■f  a  +  3c-26. 

21.  a36-6a;2-{-rj2a;-a:3  -^  (x  +  6)(a-a;). 

22.  a(6-c)34.6i^c-a)3+f(a— 6j3  -^  ^^ -a6-ar+6c. 


1*^4  MEASURES    AND    MULTIPLES. 

23.  a^b^  +  'lahc'  -  a^c^  -  b^c^  H-  ab+ac-i>o, 

24.  x^  +  y^  +  dxij-1  --^-x  +  y-l.  \ 

25.  x^-x"-^-^  x^~x+l. 

26.  a4-29a3-50a-21 -^a2_5«_7. 

27.  (2x-y)^a*  -  (x+y)2a^x^  +2{x+y)axi'  - x^  -^ 
(2x~y)a^  -c  {x-{-ij)ax-x^. 

28.  (a;3-l)a3_(a;3^^2_2)a2+(4x3+3a;+2)a-3(a4-l) 
^  (a;-l)rt3-(a:-l)a  +  3. 

Art.  XXXII.  The  Highest  Common  i?  actor  of  two  algebraic 
quantities  maVj  in  general,  be  readily  found  by  factoring.  The 
H.  C.  F.  is  often  discovered  by  taking  the  sum  or  difference  (or 
sum  a7id  difference)  of  the  given  expressions,  or  of  some  multiples 
of  them. 

Examples. 

1.  Find  the  H.  C,  F.  of  {b-c)x^-i-{2ab-2ac)x+aH-a'^o,  and 
(ab  —  ac-{-b'''—6c)x-{-a^c-\-ab'^  -a'^b-abc. 

Taking  out  the  comm.on  factor  b—c  we  get  {b  —  c){x'^  +'2ax+ab) 
and  {b~c){{a—b)x-a^+ab}  ; 
.-.  b-cis  the  H.  C.  F.  of  the  i^iven  expressions. 

2.  Find  the  H.  C.  F.  of 

l—x  +  y+z  —  xy+yz  —  zx  —  xyz,  and 
l-x—y—z+xyi-yz+zx—xyz. 

Their  difference  is  2y-]-'2z  —  '2xy  —  2zx  =  2{l  —  x){y-\-z). 
Their  sum  is  2-2x-i-2yz-2xyz  =  2{l-x){l+yz). 
.-.  theH.  C.  F.  is  (l-x). 

3.  Find  the  H.  C.  F.  of  x^+3x^  -&x^  -9x-3,  and 

a;5  _2.7;4_Ga;3+4a;2  +  lBx+6. 

The  annexed  method  of  finding  the  H.  C.  F.  depends  on  the 
principle,  that  if  a  quantity  measures  two  other  quantities,  it  will 
measure  any  multiple  of  theii-  sum  or  difference. 


MEASUKES    ANO    MULTIPLKS. 


1 
1 

+  3 
-   2 

0-8 

-6  +  4 

-9-3     (a) 

+  13  +   6     (b) 

5 

+  6   -12 

-22   -   9     (c) 

2 

1 

+  6 
-   2 

0   -16 
-6  +  4 

-18-6 
+  13  +  6 

3 

+  4 

-   6   -12 

-   5               (d) 

15 

16 

+  18   -36 
+20  -30 
-   2   -   6 

-66   -27 
-60   -25 
-6-2 

1  +  3 

+  3+1     Cf) 

25  +30 
27  +36 

-60  -110   -45 
-54   -108    -45 

-2-6 

-6-2 

(a)  X  2 


(c)x3 
(i)x5 


(.)x5 
{d)  X  9 

1  +  3  +  3  +~i  {g) 
E.  C.  F.  =  (a;+l)3. 
The  coefficients  are  written  in  two  lines,  (a)  and  {b).  They 
are  then  subtracted  so  as  to  cancel  the  first  terms,  (a)  is  next 
multipUed  by  2,  and  added  to  cancel  the  last  terms.  If  (c)  and 
{d)  had  been  the  same  their  terms  would  have  been  the  coefficients 
of  the  H..  C.  F.  Since  they  are  not,  we  proceed  with  them  as 
with  (a)  and  (6)  till  they  become  the  same.  "When  (a)  and  (6) 
do  not  contain  the  same  number  of  terms  it  is  more  convenient 
to  find  only  (c),  and  then  use  this  with  the  quantity  containing  the 
same  number  of  terms.  The  general  rule  is  to  operate  on  lines 
containing  the  same,  or  nearly  the  sams  number  of  terms. 

4.  Find  the  H.  C.  F.  of  3a;3+2a;2-14^-+8,  and 
6a;3-lla;3  4-13x-12. 

3   4-  2   -14  +  8         (rt) 
6   -11   +13   -12         (6) 

6  +  4   -28  -fl6  («) X 2 

15   -41+28         (c)  W-(«), 

(5-7)(3-4) 

H.  C.  F.  =  3a;-4.  {d) 

If  (a)  and  (i)  have  a  common  factor  its  first  term  must  measure 
8  and  6,  and  its  last  term  must  measure  8  and  12.     (f)  is  not 


106  MEASURES    AND    MULTIPLES. 

therefore,  the  H.  G.  F.  Eesolve  (c)  into  factors.  5z  —  7  is  not  a 
factor  of  (a)  and  (b).  If,  therefore,  (a)  and  (5)  have  a  common 
factor  it  is  3a;  — 4.  On  trial  Sa;  — 4  is  found  to  be  a  factor  of  (a) 
and  .-.    it  is  the  H.  C.  F.  of  (a)  and  (6). 

5.  li  X-  +px-\-q,  and  x^+j-x+s  have  a  common  factor,  prove 
that  this  factor  is 

x+  - — .     If  x— a  be  the  common  factor  then  the  remainders 
^  p-r 

on  dividing  the  given  expressions  by  x—a,  must  be  zero,  i.  e., 

a^+pa+q  =  0,  and  a^-\-ra-\-s  =  Q.   or 

s  —  o 
{p~-r)a  =  s-q,  .-.  a  =  -^,,  and 

s  —q  a— s 

x  —  a  =  x— -.    —x+    '-_ — -• 

p —r  P—^ 

6.  What  value  of  a  y^iW  m?i,'ke  a^x^+{a-\-'2)x-^l.  and 
^23.3  _j_rt2  —5^  have  a  common  measure. 

They  cannot  have  a  monomial  factor.     Neither  can  they  have 

one  of  two  dimensions  unless  (a  +  2)  vanishes,  i.e.,  unless  a=  —2, 

in  which  case  the  expressions  become  4a;"^  +  l,  and  4x"  —1,  which 

have  no   C.  F.     Hence  if  the  given  quantities  have  a  C.  F.,  it 

must  be  of  the  f@rma;+»i;  dividing  a2a;3_j.a3  _  5  by  x+m,  we 

have  for  remainder, 

5-«^  1       ..       ox  •       ■>•  , 

a^m^  +  a^ -5  =  0,  or  m^=:      ^„     ;  .-.  'rn=  — -/(t. -a-),  m  which 

|/(5  — a^)  must  be  possible  and  integral,  .-.  a^=4:,  (a^^icrives 
values  to  m  which  on  trial  fail)  and  a  =  +  2,  of  which  the  positive 
value  must  he  taken,  a.nd  .-.  2x+l  is  the  C.  F. 

7.  If  the  H.  C.  F.  of  a  and  b  be  c,  the  L.  C.  M.   of 

a6  -  66 
(a+6)(as-/j3),and(«-6)(«3^&3)is ^— • 

Let  a  =  mc,  h  =  nc,  and  .-.  a^  =  7n^e^.  b^=n^c^.     Thus 
(a  +b  )  =  c  (»«   +n  );  («-  —b  )  =  c  (m  —n  ),  and 
(a3_f.J3)  =  c3(m3+n3);  (^3 -63)  =  c3(w,3 -nS). 
,..  (^a-irb){aS-b^)=c'^{m-{-n){'m.^—n^),  and 


MEASURES    AND    MULTIPLES. 


107 


TheH.   C.   F.  of  tlie  last  expressions  is  c^(m*-n^),  .'.  the 
L.  C.  M.  =  c4(„,,6_„G)=    -_^_^ =—^2— 

8.  U  (x-a)'  measures  x^+qx-\-r,  find  the  relation  between  q 
and  r. 

Let  a;  +  7?i  be  the  other  factor,  then 
x^+gx  +  r  =  i;c-ay{x^in)^x^  +  {m  -  2a)x%+{a^  -  2a«»)a;+OT«  = 
equating  coefficients, 

771  — 2a  =  0,  a- —  2rtw  =  *7,  via^  =r 
,-.  J7t  =  2a,  and  .•.  a2_4rt3  _^^  2^3  =7-,  and 

«^  =  —  -fr,  01  « '^  =  -  07  ■»  *^<^  ^^  =  "^  °^'  ^"^  =  "4" 

;-3  g3  J.2  ^3 

.-.  -=-^,or    J-+^=0. 

Or  thus : — 
Dividing  x^  +qx+7-  by  (a;— a)'  we  find  the  remainder 
{q  +  da^)x+r-2a^ 
and  as  this  will  be  the  same  for  all  values  of  z,  we  have,  by  equat 
ing  coeffijcients, 

and         r  —  2a^=0, 
or  q3  =  -21a^ 

and  r2  =       4a^ ; 

^2         ^3 
therefore  "T"  +  07  =  0,  as  before. 

Exercise  xli. 
Fh3d  the  H.  C.  F.  of  the  followmg  : 

1.  2x*+3x^+5x^+dx-S,  Sx^  -2x^  +  10x^  -Cx+B. 

2.  x3  +  (a  +  l)a;2-f(a  +  l)x-}-a,  x"^  +  {a-l)x^ -{a-l)x  +  a. 

3.  px^-{p-\-q)x^+{p-q)x  +  q,  px^  -{p+q)x^  +(p  +  qyx-\-q. 

4.  ax^-{a-b)x^-{b-c)x~c,  2axJ^i-{-{a-{-2b)x^-\-{b  +  2c)x+o. 

5.  l-3|a:-3Ja:2  +  ia;3-x-i,  l-l^^x-'dx^  +  l^^^x^-i-x*. 

6.  ac^+bc""  +  {a  -|-  6)c"+*,  a^c-'+a'^c"  +  c-"6'  +  b'c\ 


108  MEAST3ST',^    AND    MUI/TIPLES. 

7.  a^x^  -\-a^  —2ahx^  +b^x'^  +a^h'^  -^a'^b,  and 
2a-^x^  -  Sa^xS  +3ffiC  _ 2b^x^  +  5aH^x^  - 3aH^. 

8.  (aa;4-fc?/)3-(a-&)(:c+z)(rta;  +  %)+(a-i)2a;5;,  and 
(aa;-6z/)2-(a  +  fe)(ic+2)(«a;-%)  +  («  +  5)2a;z. 

9.  «(i2-c3)  +  ?y(c3-a2)+c(a2-63)  and 

10.  ^s^+a-^  +  a'^  +  l,  and  a^t" _««»•+ «"•  —  !. 

11.  If  a;3  4.fl^2_^j^^c^  and  x^-\-a'x  +  h',  have  a  common  factor 
of  one  dimension  in  x,  it  must  be  one  'the  factors  of 

{a  —  a')x^  +  {b-b')x+c. 

12    Determine  the  H.  C.  F.  of  {a—b)'^+{b-c)^-i-{c-a)<^,  and 
(«2  -  b^)-"'  +  {b^  -c2)^  +  (c2  -  a*)^. 

13.  Find  the  H.  C.  F.  of 

2(w3_2?/2_y+2)a;3+3(?/2-l)a;2  — (2i!/3_y2_  2?/+l),  and 
8(?/3_4?/2-|-o?/-2)a;-+7(?/3_2?/  +  l)x-(3^3_5_(^2  +.^  +  l). 

14.  If  cc24-j;a;+?,  and  a--  +??7j5+37.  have  a,  common hnear  factor, 
shew  that 

{n—q)^-\-  n{m—pY  =m{m—p)(n  —  q). 

15.  Find  the  L.  C.  M.  oi x^ -Sx^  +  dx-1,  x^ -x^ -x+l, 
a;4  _  2a;3  +  2a;  - 1,  and  x*  -  2a;3  +  2x-  -  2x+l. 

16.  Find  the  L.  C.  M.  of 

a;3  +  6a;2  +  lla;+6,  x^ +  7x^ +Ux+S. 

.^3  4.8a;2  +  19a;-f  12,  and  a;3+9a;3+26a:+24. 

17.  Find  the  value  of  y  which  will  make 
2(?/2+2/)a;2  +  (ll2/-2)a;+4  and 
2(?/3_i_^2)aj3_|_(ii2/2_2^)a;3+(2/2  +  52/)a;  + 52/-1,  have  a 

common  measure. 

18.  The  product  of  the  H.  C.  F.  and  L.  C.  M.  of  two  quantities 
is  equal  to  half  the  sum  of  their  squares,  one  of  them  is 

2:^3  _  ll;c2  +  ITx  -  6  ;  find  the  other. 

19.  Ifa;+aand  x-a  are  both  measures  of  x^+px^+qx+r^ 
shew  that  pq  =  ^> 


FRACTIONS. 


109 


20.  If  x^+qz'-j-r  and  x^ -tvix+n  hare   a  oommon   measure 
(«  — a)2,  show  that  q^n^=m^r^. 

21.  If  the  H.  C.  F.  ofa;3  +px  +  qa.nd  o:^  +  mx+n,  be  a-+a,  their 
L.  C.  M.  is 

22.  If  x2+5J5  +  l,  and  x^+px^+qx  +  l,  have  a  common  factor 
of  the  form  x  +  a,  shew  that  (;;  — 1)3—5(^0  — l)-i- 1  =  0. 

23.  Jlx^+px'+q,  and  x^+mx+u,  have  a+a  for  then-  H.  C. 
F.,  shew  that  their  L.  C.  M.  is 

x^  +{m  —  a+p)x^  -\-p{m  —  a)x^  +a^  {a— p)x  +  u^  (a  -  p){m  —  a). 

24.  If  X-+PZ  +  1,  and  x^+px^ +qx-\-l,  have  x-a  for  a  com- 
mon factor,  shew  that  a= 

25.  Find  the  H.  C.  F.  of  {a^  -b^y  +  {b'  -c^y+{c^  -a^)^, 
and  a"  (6  —  c)  +  i^  (c  —  a)  +  c^  («  —  Z>). 

26.  If  a  be  the  H.  C.  F.  of  b  and  c,  &  the  H.  C.  F.  of  c  and  a, 
y  the  H.  C.  F.  of  a  and  h,  and  S  the  H.  C.  F.  of  a,  h.  and  e,  then 

the  L.  C.  M.  of  a,  6,  c,  is   -^.  ^    • 

27.  If  x-f  c  be  the  H.  C.  F.  of  x-3  +ax+.^,  and  x^  +a<x-\-h\  their 
L.  C.  M.  Will  be  x^  +  (a+a'-c)x-+{aa' -c^)x+{a-c){a' -c)c. 

28.  Shew  that  the  L.  C.  M.  of  the  quantities  in  Ex.  2  (solved 
above)  will  be  a  complete  square  ii  x  =  y''^  -\-z^  —y^z^. 

29.  Find  the  H.  C.  F.  of  a;«+2a;«+3a;4  -2a.'2  +  l,  and 

Qx''+  x' -{-llx'^ -Ix^ -"l. 


Section  II. — Feactioks. 


Art.  XXXIII.  "When  required  to  reduce  a  fraction  to  its 
lowest  terms,  we  can  often  apply  some  of  the  preceding  methods 
of  factoring  to  discover  the  H.  C.  F.  of  the  numerator  and  de- 
nominator. 


110  FEAOTiO^b. 

ExAMPLiES. 

J        ac-\-bij+ay-\-hc      ^     c{a+b)+y{a+b)  c-\-y 

a/+'2bz-h2ax^bf  ~  J\a+b)+'2x{a+b)    ^  f+2x' 

2     a^—ba^—a'-ib^  +  ab^  a^a^+b^ -ab{a+b)} 

a^—bw^  —  ab^  +  b^      ~    a{a'^  —  b'^)  —  b{a^  —  b'^) 

_    a(a-\-b){a  —  b)'  .  a 

{ci~b){a^—b^)    ^    a^^b^' 

g     x^  +x^il-\-x^y^  -{-x-y^  -\-xy'^  +y^ 
x^—x^y+x^y^—x^y^^-zy^—y^' 

Here  the  numerator  is  evidentlj  {x*^  —y^)  ^  (x-y),  and  the 

denominator  is    ^   ~y   ,     The  result  is  .".   ^^•^. 
x+y  x-y 

4     {x+7/)'^ -x^ -y^    _       5x^y  +  lQx^y^  +  Wx^y^  +  5xtj^ 

(«-h/r*-f-'«M-/^    ~    (x+y)^-x^y^  +  {x^  +y-t)^^x^y^ 

^^Uj  a;3+y3-t--2.y>/(a;+y) } 

(a;3 +^3 +0;?/)  { (3;+2/)  ^ +a;i/ +a;3 +2/2  -  a;?/} 

5xy{x+^j){x^+xy+y^)  5xy(x+y) 


5. 


2(a;2+5Cj/+v/3)2  2(a;2+a;!/+2/') 

x2-12a;+35 


a;3_ioa;2+3ia;_30 
Here  we  see  at  once  that  the  numerator  =  (a;— 51  («  — 7) ;  and 
it  is  plain  that  x  — 7  is  not  a  factor  of  the  denominator;  we  .*.  try 
x  —  5  (Horner"s  division),  and  find  the  c»uotient  to  be  x^  —5x+Q. 
x-1 


.  the  result  = 
6. 


x^  —?>x+Q 

a;4  +  2a;^+9 


J*— 4a;3  +  Sa;  — 21 
The  factors  of  the  numerator  are  at  once  seen  to  be  a;'+2a:+3, 
and  a;3  — 2x+3,  of  which  the  latter  is  one  factor  of  the  denomin- 
ator, the  other  being  (Horner's  division)  '•.''  —  'lx  —  1:  .'.  the  result 
is  ^±2Hi3 
a;3-2a;-7* 


FRACTIONS.  ill 

Exercise  xlii 
Reduce  the  following  to  their  lowest  terms  : 
x^-lx  +  6  3xj/^-lSx!/+Ux 


1.    - 


3 -2x2 -8a; -96  7//3- 17^2^6// 

x*+a-x--\-a'*^  a-2-{-a;-12 

x'^+ax^  —a^x  —  a^         x*  —  oa;^ +7a._  3* 

a;3_3a;+2  x'^+2x^+9 


x^  +  -ix'-5         x*-ix^-{-4x'^-9 

2+hx x^  +  2x^-\-\2x 

*•    2ft+(62^ja;  -  2bx^'  x\+4x 

5a''+10a*x+5a^x^  ^Ox'^+x^-l 

^-    a^x+2a^x^+2ax^  -fx* '        25^*  +  SajS^T^ITl* 

.r'  —x^y+x^y'^  — x^y^ •\-x^y^  —x'^y^  -\-xy^  —y^ 
x'  -^•x'^y+x^y^4-x*y^  +x^y^-r-x-y^  -hxy^  +  y^* 

'a  6  , 

_    Sa''x*-2ax^-l ^^    T  +  T/  ^^+  ^' 

4La^x^ —la^x^  —  'dax^-k-1  i  a     ■     o 

a^{b-c)  +  b^{c  -  «.)  +(-3  (a  -  fc) 
aic(a  —  i)(6  —  6-)(6- -  a) 

Q  («  +  &  +  c)^ 

10.  From  Ex.  4  (solved  above)  show  that 

(a-&)*+(6-c)^  +  (c-«i^    _   (r^6)3  +  (6-c)3  +  (c-«)a 
(a-6)5  +  (6-6-)*+(c-a)-    "  5(a-'6)(6-c)(c-a)       ' 

{x+yY-x^-y^ 
12.  Shew  that 


112  FKACTIONS. 

Art.  XXXIV.  In  reduciug  comT)lex  fractions  it  is  often 
convenient  to  multiDh'  both  tex-ms  of  tiie  comxJiex  fractiOB  by  the 
L.  G.  M.  of  ail  the  denominators  involved. 

Examples. 

1.  Simplify   i(^+H)-l(l-H 

Here  the  L.  C.  M.  of  all  the  denominators  involved  is  12; 
.-.  multiplying  both  terms  of  the  oompiex  fi-actiou  by  12,  and 
removing  brackets,  we  have 

ex+S-8+6x    _      12a;      _      8a; 
21  — 4a;  — 17      "    4  —  4^   ~    l—x 
„  a  —  h 


l-\-ah 


1  +  ^^ —•    Here  multiplying  both  terms  by  1  -^-ah,  W3  get 

l-{-ab 

a{l^ab)-a-^h         6(a24-l) 


Here  multiplying  both  terms  of  the  frac- 

by  4  —  a;,  the  given  fraction  becomes  at 

,  and  now  multiplying  both  terms  by  4,  we 


tion  which  follows  a;  — 1  by  4  — a;,  the  given  fraction  becomes  at 
1 


I,                    4  4 

have =    --' 

Ax—4,-\-4:  —  x  dx 

It  may  be  observed  that  when  the  fraction  is  reduced  to  tha 

form  —  -^   — >  we  may  strike  out  any  factor  common  to  the  two 
h  d 

denominators,  and  also  any  factor  common  to  the  two  numerators ; 

it  is  sometimes  more  convenient  to  do  this  than  to  multiply 

directly  by  the  L.  C.  M.  of  all  the  denominators. 


REACTIONS. 

a-b\ 
a+bJ  ^ 

113 

4.    Simplify   (— ^  +  ^-^)  ^  (^— ^^  -  ^;^^pp!  . 

Here  the  numerator  of  the  first  fraction  is  (a4-6;' 4- (a  —  i)^ 
and  tho  denominator  is  a^  -  h^  ;  the  numerator  of  second  fraction 
is  {u^-{-b")^ —  {a^ —b-}^y  and  the  denominator  is  a*—b-^;  the 
former  denominator  cancels  this  to  a--}-h''^,  wiiich,  oi'  course,  be- 
comes a  multiplier  of  the  first  nnmerator  : 

\a^+b^){{a  +  b)^  +  (a-bf\   _  (^24^x3 
•'•  ^^  "^^^'^       {a^'+b'^ ) 2  _  (^3  -  i ■■' )  2         ~       'laH-'" 
Occasionally,  vre  at  once  discover  a   common  complex  factor, 
strike  this  out,  and  simplify  the  result. 


/I  1 \ >      1 

here  the  den.  =    I  —  +  "r      —  "v 


1       1 

—  +  —  — 

1 
c 

1                        i                        i 

0  • 

/I       1        i\ 
=    — f-  -7-  -f  — 

(^- 

mon  factor  we  have 

1 

1       1\  ^      . 

— ,  and  cancelling  the  eom- 

o  c  I  ° 


J^    ,    2_        J_'  ^"-^^  mif.fciiDlying  by  abc,  this  = ^^^ 

a  ^    b     ~     c  '  bc+ca-ab 

Exercise  xliii. 
Simplify  the  following : 


1  -  ^{1-1(1 -x)} 


2.   1: 


x—v 


1+a  x+p 


114 


FRACTIOWS. 


«=-f  &3 


a-\-b  •4--T- 


x—1  ij  —  l  t  —  1 

Sxt/z  X  y       "^      & 


yz-  zx  —  xy  1  1  i 

*•  ?/    "*"    z 

_2  _2_  ^  ,^4  +  64  J,  c4 

^     «^    "^    ^^    "^    ci^    "^        a^b^c- 
a  0  c 

be  ac  at) 


/a-f6        a'-\-h"  \         la-b        o^-b^\ 
\a-b  ~^  a-  -0^  ]    "^  \a±h  ~  a^^^j 

1 


1       1 
—  + 


a  h  -j-c 

2(1 -a:)        (l-.'cV 


„/a-M3       (a-oY  /a~b\ 


FBACTIONS.  115 

~b' 


13. 


15.  /^-z:^'  +  -J-i^)  ^  f  J-^^^      Ir^n 

16.  find  the  value  of 

17.  Find  the  ralue  of  ^/{l  --,/(l  -x)} 

\l  +  bj  Vl  +  6/ 

18.  Find  value  of 

V(a+l>x)  +  -i/(a-hx)    ^^^^  ^  ^       2«g     ^ 
l/(c-f  ^a-')  — |/i«-<^.c)  i(l-fc-)' 

Art.  XXXV.  When  the  sum  of  several  fractions  is  to  be 
found,  it  is  generally  best,  instead  of  reducini?  at  once  all  the 
fractions  to  a  common  denominator,  to  take  iwo  (or  more)  of 
them  together,  and  combine  the  results. 

Examples. 
1.  Find  the  sum  of 

x+y  y  —  x  x^ —y^ 


2x—2y         iia;+%  x"+y^ 

Here  taking  the  first  two  together  we  hare 

(x±y)^±^-y)^   =    ^1±J^;  now  add  this  to  _  ^^Zl" 
2(a;2-2/2)  x"—y"  x^+y- 

and  we  get  {^^ +y^r- i^^_^J^^    ^     4^/J  ^ 


116 


FRACTIONS. 


2.  Find  the  snm  of 


1+^  4a;  837  l~x 

r^  "^  1+^3  +  i+a;4  "  r+^-* 

Here,  taking  the  first  and  the  last  together,  we  have 
(l+.c-)2-(l-a;)2    _      Ax 

l-a;3  ""    l-a;3' 

taking  this  result  with  the  second  fraction,  we  have 

/     1  1     \ 8^  . 

[l  +  x^   +  1-x^-J    -    1-x^' 
now  take  this  result  with  the  remaining  fraction  and  we  get 

''''[l-x'^    '^  1  +  x^l   -    1-x^' 
ar'"  x""  1  1 

^-    ¥:^1    -   xr^    -   ^-^   +  ;^M^'      T'"^^^S   m    pairs 
those  whose  denominators  are  alike,  we  have 
x'"-l         a:-"-l 

The  work  is  often  mr.de  easier  by  completing  the  divisions  repre- 
sented by  the  fractions. 

2ic+l  4:X+o 

4.  Find  the  sum  of  1+  0|773T\   —   oTTo'    By  dividing  num- 
erators into  denominators,  this 

3  _1 3_  1 

=  '^-^'^+  2^32  ~^~   2x  +  2   =    2^^  ~   2x+-2 
3a;-f3-a;  +  l    _    x+2_ 
2a;2-2         ~  x^-l 

X  x—9         x-{-l         x—8 

5.    t:  4-  ^  —  T   -   a  '■  we  have,  by  division 

x~2     '    x—i         x—1         x  —  Q  •' 

2  2  2  2 

1+  ^32  +1-  ^^7   -1-   ^Ti   -1+  ^36'^^ 

2  _2_         _2_  2  2  (2a; -8)  2(2a;-8) 

x-2    ^  ^^   ~   ^^   ~   a;-l   "(^2)(a;-6)  ~  (x~l)ix-l) 

f  1  1  ) 

=  (-ia;~16)  |3.a_8a;+12   ~  a;3  -8x+lj 
=  {80-2C.x)-^{x^-16x^  +  8'dx^  -  152,«-f  84). 

[denominator  =  {x^  -  8a;)2+19(x2  -  Sa;)  +84J. 


FRACTIONS.  117 

6.  Find  the  value  of 

x-\-2a         x+2h     ,  4ab 

— TT  +     — Ki  when  X  =  -—  v 
z-2a     '    X—  2b  a  +  b 

.     -D     ,.  .  .       ,  ifi  ib 

Bydivision,l+^^3^-hl+^3:^^ 

/a  b     \ 

=  2  +  4    -jj^  +  ~3;ji|  ;   but  the  quantity  in  -tbe  brackets 

(a+b)x  —  4ah 

.'.  the  value  of  the  given  expression  is  2. 
Exercise  xliv. 

Simplify  the  following  : 

1     X —  a         x^+ax+a^  x^  —  a^ 

5  x  +  a  x^—a'^ 

^     _rtM-i3 a^-Sa^b-j-Sab^-bs         a{a~b)- b{a-h) 

a^-ab  +  b'^    '^  a^  -  b^  ~~;^2..f^b+b^      • 

/I  1  2a     s 

3.    ( + f-    — , r|    V 

\a  -\- x         a  —  x         a^+x'^J    ^ 

t     1  1  2x 


\a  +  x         a—x         a^+x'^j' 

a  b  ab  ah 


a  +  b         a  —  b         ab—b^  a--\-ab' 

^     Z+2x         2  -  Qx         16x-a;3 

5.    — _    4- 

2-x  2  +  X    ^     a;2-4 

g     1  _  1  1 

•  2   i3x-2//)/  2  \3a;+2y/' 

g      x+1  a;-l  l-3a;  a;  1 

*  2^^  ~  2x+l    ~  x(l-2x)  "^  a;(4ic3  -1)   +  ^a--^^^ 

1  4  9  a;-l 


2x+2  a;+2    '^'   2(x+3)         (a:+2)(a;+3)" 


118  FRACTIONS. 

x  —  y  x+y  x^-\-y'  x^—y* 


+ 


(        1  1        1 

\x  -\-  a  X  +  b  j 

(  o-\-x  4:ax  8a^x  a  —  x 

,^     5a;-4         12s;4-2  10^+17 


9       ^    lla;-8  18 

a  <i  a2  2rt3_i3_a63 

,^     12a;  +  10a     ,     117a  +  28a; 
Sx+a      ^      9a  +  2a; 

4x-17         8a;-30         l(^j^-^3  5a!-4 

1^-    "^irr     ~    2x-7     +    ^2s"-"5'    ~    a;-l  * 

a  +  6+2c         a  +  6  +  2d 
17.    Find  the  value  of  -j^jz:^^  +   a+6_2rf 

4cc< 
vlien  a  +  h=    ^'7^' 

■I  Q  -^- _        _J_ • 

(a-5)^"              (a-?>r     _           1                     ^  1   _ 
1^-    (a_5)»-l   ~   («-&)"  +  !         (a-5)"-l  "•"  (a-6)'M-l• 
^^^___  1 1 

1+.^  1-^  2        _     2ce3 

21-    fZ^3    +   i+:c3    ~    l-a;3  x^  +  l' 


FRACTIONS. 


119 


Art.  XXXVI.  The  follo\ving  are  additional  examples  in 
which  a  knowledge  of  factoring  and  oi  the  principle  of  symmetry 
is  of  advantage. 

Examples. 


{x+z)-~^^   "^   {y+'xy^'-^'    "^   {z  +  y)^-x 


.2 


Cancelling  the  common  factor  x  —  y+z  in  the  two  terms  of  the 

first  fraction,  there  results  r- ,    hence   hy    symmetry,   the 

x-\-y-\-z 

denominators  of  the  other  two  fractions  will  be  x-\-y+z,  and  the 

numerators   will   be   y-{-z—x,    z-\-x  —  y;    .'.    sum   of  the  three 

numerators = a; +^+2,  and  the  result  =1. 

ab  be  ca 

2.    Simphfy  (,_^)(,._^)   +   (^rr]0(^T)   +   {b-c)ib-a) 

The  L.  C.  M.  of  denominators  is  evidently   (a—b){b  —  c){c  —  a). 
This  gives  for  numerator  of  first  fraction  —ab(a  —  b),  and  by  sym- 
metry the  other  numerators  are  —bc{b-c),  —ca{c  —  a). 
ab{a—h)+bc{b  —  c)-\-ca{c  —  a) 


we  have 


{a  —  b){b  —  c){c  —  a) 
(a  —  b)(b  —  c)(a  —  c) 


1. 


~  {a  —  b){b  —  c){c  —  a) 

2.  Eeduce  the  following  to  a  single  fraction  : 
a  b 


{a-h){a-c)ix-a)    "^    {b-a){b-c){x-b)    ^  (c-a){c-b){x-cy 

Here  the  L.  C. ll.ia  {a-b){b  —  c){c-a){x— a){x-b){x—c)  ;  the 
numerator  of  the  first  fraction  is 

—  a{b  —  c){x—b){x  —  c),    and    .-.    by   symmetry  that   of 
second  is  —h{c—a){x—c)(x—a),    and  that  of  third  is 

—  c{a  —  b){x~a)(x  —  b);  and  their  sum  is 

—  {a{b  —  c){x-b){x  —  c)-{-b(c-a){x  —  c){x  —  a)  + 

c(a  —  b)[x  —  a)  (x  —  b)}. 
This  vanishes  if  a  =  6,  hence  a  — &  is  a  factor,  and  .'.  by  sym- 
metry b  —  c  and  o  —  a  are  also  factors.     Zl^ow  the  product  of  these 


I'AU  FRACTIONS. 

is  of  the  tliird  de^-ee,  while  the  whole  expression  rises  only  to 
the  fourth,  hence  «-.  cannot  he  involved.  The  other  factor  must 
therefore  be  of  the  form  iix+n,  in  which  m  is  a  number. 

To  determine  n  put  x  =  0,  and  the  expression  becomes 
aficja  — 6  +  6  — c-j-c  — a}  =0;    .'.  w  =  0,  orthe  other  factor  is  vix. 
'  To  determine  w  put  «  =  0.  6=1,  c  =  —  1,  and  m  will  be  found  to 
be  1.     The  numerator  is  .-.  x{a  -  b)(b  —  c)[c  —  a),  and  the  result  is 

X 

{x  —  a)[x  —  b){x  —  c) 

o    o-       Te             a  +  h                        6+c  c-{-a 

3.   Simpliiv J. ,    _^ _^ 


[b-c){c-a)    ^   {c-a){a~b)    ^    (a-h){b-c) 
L.  C.  M.  of  denominators  is  {a  —  b){b  —  c)(c—a)  ; 

.-.  iirst  numerator  is  a-  —b-,  and  by  symmetry 
second  "  b'-<~,'And 

third  "  c2-a3  ; 

the  sum  of  these  =  0,  which  is  the  required  result. 
4.  Eeduce 

^-y  ~^    y-z  "^  z-x  "*"  {x-y){y-z){z-x) 

Here  the  numerator  becomes 

2{y-z){z-x)+2{x-y){z-x)  +  2{x-y){y-z)  + 
{x  —  y)^-i-  {y  —  2) -  +  (z  —  a;) ^ ,  which  is  evidently 

{{■^^-y)+iy-^)  +  {'^-^)}'=0. 

ff(3  +  263|3  (2a3+63)3 


ff/3  +  263|3  (2a3+6^) 


Observe  that  the  denominators  become  the  same  by  clianging 
the  sign  between  the  fractions,  and  that  the  expression  is  sym- 
metrical with  respect  to  a  and  6,  The  numerator  of  the  first 
fraction  is  a^~  + 6^963 -+-12fl^6^+8a36^,  and  by  symmetry  that 
of  the  other  is  —b'^"—Gb^a^  —  l'2b^a^—8b^ci.^.     Their  sum  is  .: 

=  (a«_6G)f^(G.|_iC+e,^3/,3_8a.3^,3|  =  (rtG_iG)(a3_^3^2 

=  («3_}./;3)^rt3  _i3)3^  and  since  the  denominator  of  the  given 
expression  is  (a^  —6^)3  .-.  the  result  is  a^+i'. 


FRACTTOVR.  121 

Exercise  xlv. 
Simplify  the  following : 

a-\-b  b-\-c  c-\-a 

1  1  1 

^-    (a-i)(«-c)   "^   {b-a){b-c)    ''"    (c-aj(r-/7)' 

a  — 6  5— >•  c  —  a  (a  —  h)(b  —  c)(c—o) 

^-    a+b   "^   /7+"c    ■*■    c+a   "*"    (a+/>)(6+o)(c  +  «)' 

flS  &2  r» 


(a+b){a-}-c){x  +  a)'^{a-{-b){b-c){x+h)     {a+c)(h  -  c){x+c) 

a;3  y2  23 

(x-y){x-z)   ^   {y-x){y  -  z)         {z-x}{z-y) 

flZ  J)3  (.3 

^-    (a  -  b)(a  -  c)    "^    (b  -  a)[h  -  c)   +   (c  -  a){c  -  h) 

"  (W(R  "(RJT^nFfe^)- 

/a;5  -  2?/3\3         /2a;3-7/3\3 

11.    ___! + ^ + 

(6+c-2rt)(c  +  a-26)    ^   (c  + a- 26) («+/»- 2-)   ^ 

1 


(ffl+6-2c)(i+c-2a) 


,„      62 -c2  c2-a2  a2-62 

12.     -— -^    +    7-— -TIT    +    /--^ 


(6  +  c)3    "^   (c+a)2    ^   (a+6)2 

13  «^  ,     ^ , 

(a-6)(a— c)(a;-fl)  (6-«;(6- cj(a;- /A 

C2      

{c  —  a\{c  —  h)[x  —  c) 


122 


RATIOS. 


14.        %+g)        J y{z+x)  z{x+y) 

(x~y){z-x)     '     (y-z){x-y)    "^    {z-x){y-z)' 


15. 


If5. 


{a-i-b)^-\-{b-c)^+{a+cy-'      ,       2  2 


0! 


(a  +  6)(^  — c)(a  +  c)  a+c  Z>-c     '     a+6 

1  1  1        . 


+    „/,         -w,.       „x     + 


ici^a;-a)(a;-6)   ^  a{^~a){x-a)   ^  b{b-a){x—b) 


Sectiox  III. — Eatios. 


Art.    XXXVII.     If  -7-  =  -^     .-.  ad  =  bc.     Now, 

-,.     .•  ,  h  d 

diviamg  ad  — he  by  ca  we  have  —   =   —....      (1). 

»•       ad  =  bchjcd       "  —    =    —....       (2). 


C  d 

d 


ad  =  hchjah        "  -^    =    4"     •      •     •     •       (S)- 


Also  :^^:^   =   each  of  the  given  fractions      .     .     .      (4). 


mh[-Y)+nd[-jj     {mb+nd)^ 


(5). 


ma-j-  Jic ^. 

mb-\-nd         mb    -j-nd         ~ "' "mb'+ml"'  ~  b  d 

A  very  im'portant  case  of  this  is  w  =  1,  m  =  + 1,  hence 

a  c  a-{-c         a  —  c 

b     ~     d     ~    b-{-d    ~    b  —  d 

a—b         c—d 
Also    —.-7    =    —,—j (6). 

For  by  (2)  and  (5) 

a  b  a—b         a-^b         a — h         c—d 

c     ~     d     ~    c  —  d    ~    c-\-d    "    a+b    ~    c-\-d 

-      --1      4--1 

Or  tl       •  ^~  =   -'  =     '^'  -    ^~^ 

'  a-i-b  a  c  ~    c+d 


RATfOS.  123 

Generally,  toprove  tbat  if  —    =   — ,  any  fraction  whose  nu- 

b  d 

merator  and  denominator  are  homogeneous  functions  of  a  and  b, 

and  are  of  the  same  degree,  will  be  equal  to  a  similar  fraction 

formed  with  c  instead  of  a  and  d  instead  of  h  : — Express  the  first 

fraction  in  terms  of  — ,  and  for  _  substitute  its  equivalent  . 

h  b  ^  d' 

and  reduce  the  result. 

By  (2)  the  fractions  may  be  formed  of  a  and  c,  and  b  and  d. 

if  —   =   — ■   =   — ,    ■ LZ_   _   —  or   —  or  — (7) 

b  d  f      vib+nd+pf  b  d  j 


'"'{t)  +  "ii)  +  At) 


ma-\-nc-{-pe 

mb+nd+pf    ~  mh-^-nd-^jij 

a 
{mb+nd->rpf)Y 


a 


vib  +  nd-^pf  b 

If^   =   ^   and    ^   =   Z. 
b  d  n  q 

ma+pc  pa-T-mc  ma         pa 

,  ~   .    =  ^  ~     ,    =   —    or  ^       or  &c       ....     (8) 

nb  +  gd  qb±_nd  no  qb  ' 

T-,      ma         pc         via  +  pc  ,      ,„, 
For  —    =   ^—   =    — -^-     by  (o) 
nb  qd  nh  +  qd 

pa  mc         pa^inc 


qb  ml  qb±_nd 

But  ^   _  -P^^  hence  the  equalitv  stated  in  (8). 
nb  qb 

If  ^   =   A  =   ^  and    !!L   =   A   =   ^, 
b  d  f  n  q  s 

ma  +  pc  +  re  pa-'-rc  +  tne  via 

— ,~     ~   r    =       ,   ■     ■  , — 7    =    &c.,    =    —    =    &c.   .   (9), 

nb±qd  +  sf  qb±sd  +  nf  no  ^  ' 

If  an  upper  sign  be  taken  in  a  numerator,  the  corresiDonding 
upper  sign  must  be  taken  in  the  denominator ;  if  a  lower  sign, 
the  corresponding  lower  sign,  otherwise  all  the  signs  are  inde- 
pendent of  each  other. 


124  t.Avios. 

Examples, 

a  c  oa  —  4:b 

1.    If  -r   =   — r,  show  that  ,^^ — ^-y   = 

-Ji  _  4 
Tlie  given  fraction  =  -»— ™—   =    »-»,■..-»«=»„    =   _ — — — - 
7-^  +  5  -        ■ 

a  c  2^(3  ^3^2^         2c3+3c3(i 

2-  I^T   =   T'^^^*^^^   3^il^463    =    3c3d^4^3' 


Dividing  the  given  fraction  by  6^  we  have 

a  G 

and  this  becomes,  on  substituting  for— its  eqaal-^. 


a  3  rt3 

2p-  +   8^ 


a2 

c3  C 

2 —   ■\-   3 


rf3   -r  ^,^2    ^   2c3_+3c3rf 

8:7?  -  ^ 


a3  +  i3  /«»  \ 

3.  If  3a  =  26,  find  the  vahie  of  ^^^3^.   This  =   (p-  +  ll  h- 

(^   _  —      [by  dividing  both  numerator  and  denominator  by 
\b'^  b  j 

a       2 
6»3] .     But  from  the  given  relation  —  =  ^r  we   have,  by  substi- 


a 

tuting  for  -r-' 


c      ^  a3  4-63  6  /«+ft\* 

4.    If -7-  =  -T-    Prove  that  -,-",",v  x   -7  =      — ,~i     • 

a  b  a-\-b 

We  have  -  =   -j  =  -_^'     Also 

,3+^3    _   ^/^  \       /^         ^J  ^  ^^.,,^d  this  multiplied 

i3      -     d'^   \63      ^        /  U3     ^        j  d^ 


C 


KATI03. 


12i 


o-   ii    ^3 — :,^..  jj;     ,—    =   --. ,  shew  that  a;  =  — -. 

Multiplying  botli  terms  of  second  fraction  by  x,  it  becoiaea 

x^-{-ax^  —  bx 

^z~^^y.2Z£i)x'    ^'^'^  ^^^^  °^  ^^  given  fractionti  = 

diffei'ence  of  numerators 


diflfereuce  of 

denomiuaiors ; 

= 

c 

c 

=  1 

-•.    X'-x-ax  —  h—x-- 

-  ax-{-b 

, 

or  'lax  = 

2b 

■    x 

h 
a 

6.    If 

a 

c 

*                        ac  +  ce-^-ea 
=  -r,  shew  that ,  ,  ,   ,,.  ,-.;   = 
/                        bd  +  dJ-{-fb 

A3+^S 

For 

ac 

Td' 

ce 

ea     ac-\-ce-\-ea 
JU     bd  +  df+fb' 

By  (7)  making  »i; 

=  7i  = 

A.lso 

a2 

C3 

«2      a.2-\-c^-\-e-' 
~p~b^'-fd^+f-- 

By  (7). 

But 

ae 

Id 

~6^ 

hence  the  required 

equality. 

Tiie  problem  is  a  particular  case  of  (9),  with  all  the  signs  + 
and  a  for  m,  b  for  n,  c  for  p^  &e. 

(If  the  fractions  yiven  equal  to  one  another  have  not  monomial 
terms,  instead  of  seeking  to  express  the  proposed  quantity  in 
terms  of  one  fraction  and  then  substituting  an  equivalent  frac- 
tion, it  is  often  better  to  assume  a  single  letter  to  represent  the 
common  value  of  the  fractions  given  equal,  and  to  work  in  terms 
of  this  assumed  letter.) 

„    J,     a  +  b  h  +  c  c-fa 

S{a  —  b)    ~    4(6  — c)     ""    5(c  — a)' 
prove  that  32a +35^4- 27c  =  0. 

Assume  each  of  the  riven  frs.ccion3  =  a;,  so  that  a-^b  =  3(a  —  bjx, 
b+c  =  i{b  —  c)x,  c-jra  =  iJ{c  —  a)£, 


126  RATIOS. 

or    — ^—   -4-    — !—   _i_    — ! —  =  a;(a  — 6-f /;— c-fr  — a)   =  0. 
o  4  5 

/.    adding  these  fractions  we  have  82a  +  356+ 27c  =  0. 

This  example  might  also  be  worked  as  a  particular  case  of  (7), 
thus 

a-\-h  h-\-c  c-\-a 

^{a'-b)    ^    4(6 -c)    ^    5(7^ 

20(a4-6)  +  15(&+c)  +  12(c+rt)    _    32a+356+27c 
""    60(a-  h)  +  60(6  -  c)  +  60(7^ a)    ~  0  '"' 

.-.    32a  +  356+27c  =  0  x  -^^-  =  o 
3(«  — 6) 

8.  If—  +   '—    =z   — J—  —   —   -I-  ^l,  prove  that 
63   ^  ,/2  fZ  i  6  f^  ./'  j 


/  g  +  c+g  \- 

U  +  f/+//     "    63+tZ3+y2 

Transposing  terms,  &c.,  we  have 


t3 


2rtC              c3               ^3 

2c«               6-3 

-^    +    ^    +    y5     - 

"df      "^    rf^ 

=   0, 


/ 

that  is,  the  sum  of  two  essentially  positive  quantities  =  0  ; 

.'.  each  of  them  must  =  0  ;  hence  we  have 

a  c  ^     e  c 

-  -  -^   =   0,  and  -^   -  -    =    0; 

O.  C  <;    ,       .       «3  f^2_|_c2^g3 


Also 


b  d  /  63  bz+d^-^p 

a-\-c-\-e  _     .     ^^  /^+c+^\-. 

/a  +  c+e\2  a^+c^+eP 


XbA-d^'fl 


b^d^fl  62+t/2-|./a 


RATIOS.  127 

Exercise  xlvi. 

_,     ,„    <7  c  a^  —ab+b^  c^  —  cd  +  d" 

1.  If  _   =    -.  prove      ^,_^,,        =    ^Sri^- 

n    Tt   a  c  «2_o3  /«-«v^  /«+c\^ 

2.  H  _  =  _,  prove  __-   =  (^-_-^j     =  (— ). 

y.  Given  the  same,  shew  that  each  of  these  fractions 

4.  If  2x  =  dy,  write  down  the  value  of 

•2x^-x^y+y^    ^^^  ^^  x'^-3x^y  +  2ij^ 
'x^>/+xy''  +  2>j^'  {^x--y-'^)^ 


If  —   =   —   =   — ^,  shew  that  _-   = 


ma  —  7}e—pe 


b  d  f  b  mh  —  nd  —  yf 

6.  From  the  same  relations  prove  that  —   =    ( -, \ 

b^  \b-md—np 

7.  If  '±^   =   Af^-±^+^%then:«3^(i_«).(i+«). 

1  —  x  a\l—x-\-x'^! 

°-  ^      ,,     ,  -//  -      .    =    a,   prove  that «  =  .-, — ^• 

^    Ti?  wa;4-a+i  mx—c—d  b  —  c 

9,  If ! — —    — — ,   prove;*;   — 

nx+a-l-c  nx  —  h  —  d 

10    If     ^~^  '^""'^  c  —  a 


ay-\-hx         iz  +  cx         cy  +  az  ax  +  by+cz 

then  each  of  these  fractions  =    ,  a  +  64-c  not  being  zero. 

x-i-y+x 

11.  -^1'+^-   =    i±i^'=    -^^±^,then8a  +  96  +  56-  =  0. 

a-b  2(6 -c)  B{c.-a} 

\/o.-\-  \/{a-x)  1        ,  ,       a  —  x  11  — a 

12.  If  ^  .; r   =    — '    shew  that =   L-ri 

-j/a  — 1/ (a— aj)  a  a  \l+a, 

a;2— wz  y^—ocz  ^  ,  ,      , 

13.  If  — T r   =    -/5 T'    and  a;,  y,  z  be  lanequaJ,  shew 

x[\-yz)        y^l-xz) 

that  each  of  these  fractions  is  equal  to  x-^y-\-z. 


«-  h-  c^ 

^   18.  If  -^ =    —, = ,  shew  that  rt2a.+62»+c2a 

x/ —yz         y- —zx         z^  —  xy  '' 


19.  If   -7^r —   =   ,-^' —    =    ~ — -,,    thenwHl  {a-h)x-\- 
a-\-h  —  c  h-\-c  —  a  r+a  —  b  ^  '      ' 


128  RATIOS. 

1A    Tr^'+-'^  +  l         y^+%1  +  1 

14.  it  -—^    =     -- — (^    r^-'    shew   that   each    of    these 

X'  —  Zx  +  o         y^  —  ly-f-o 

fractions  ={xy—l)-T  (xy  —  S). 

2ryx^-16         3(^3  _4)  4  3 

15-  If  TOxTS     =    -2^34-'    «bewthat^_^.   =   y 

4&C  iZ+SS         y-h2c 

16.  If  .V  =   ^^^  shew  that  •-_^^   +  ,;^-^  =2. 

1     /.S+Z^SX  1     //;2+,2N  1     /,2+«,2N 

25a2 +2762  +  22^2=0. 

^   18.  If  -5 =    - 

x^  —yz  y 

=  («2+62+c2)(a:  +  v/  +  2) 

19.   If    -y-"^—    = 
ib-~c)y  +  [c-a)z  =  0. 

a  C  p.  ,r,2  4_^2_!_^2\2  „4  +  ,,.44.^4 

^^-  If  T  =  T=^7  ^'^^^  (/^I'+TmT^'    ^  /74+J4-474- 

•,„  J>x4-oi/  cy  +  lz  riz-\-cx 

21.  If  r^'    =    ^^        =  ,    shew  that 

(t  —h  h  —c  c  —  a 

[a-\-h-\-c){x-{-y-\-z)  =  ax-r-hy  -^cz. 

^^    ^„  x^ —  5x^a  —  a^  +  6x'i"         x-~a  ■,      r ,-, 

22.  It  — 9—r-i — ; o  i^^^~    =      ~~"'    shew  that  each  of  these 

x^  +X''a-\-xa^  +(c^  x  +  n 

expressions  =  ■* , 

1    in-h\  1  (h-c\  1  ir-a\ 

23.  If    TT     -TT        =     —  ;T^        =     ^-uT/'    ^^tl  «'  '''  ^  ^^ 

b    \a-X-b>  o\o+r/  i(jV''+f'-' 

different,  shew  that  1  f)b  + 1 1 6  + 1 5  =  0 . 

^■  +  7/2.2  1  —  7/2 

- ,'    )     =  ^~    ,-    pioTethata-2+v2+22+2aTy3  =  l. 

2^5.  If  Jl—    =    JL-   =    ^1_,  shewthata+?;+c  =  0. 
x  —  y         ?/  -  z         2  —  « 

26.  If  _   =    — ,  prove  that  — -    -      -/^--r^-SV,-^- 
b  d  a  —  h  y  V'c)  -y{bd) 


KATiOS.  129 

27.  It"  —  =  —   =   — ,  tbeu  each  is  equivalent  to 

b  d  f  ^ 

-  -      ^  ,  hence  siiow  that 

lb-{-iuU+n/ 

a  b                          e               -. 

2z-\-^—y  ~  ^x^%j-z  ~    %j-r'lz  —  J 

-c  _  y 2 

2a  +  2i-c  ~  '"Ib  +  'Ac-a  ~    2c+2a-6* 

28.  L  —  =   — ,  prove  that    _   ^  (      ^__  . 

29.  li'  --^   =    — L   =   -,-^,  prove  that 

±{y-z,  +  -^{z-x)  +  I-{x-y)^Q. 

30.  If  ^ "^ ^^    =    — -^' -^    =       ^    ""        . .  then  vvill 

±{i-x)  -r  -^(/n-^)  +  — («-z)  =  0. 

/x  tiiy  nz 

31.  Lz  -    ^  ^  ^ ,  and  ?/  =    ^-^ —    .  —  /,  shew  that 

//  ■^ 

\/iaz-  —ai  I 

a;  =   *  -^ 

z 

32   If  ^'"1:   =    ^'"—   =    ''"-"^    =   1,  shew  that 
a3  62  c2 


33.  If  I:L    =    ^    =    -.    and  4    =   ?^   .   ^_    1. 
X  y  s  w         b^  c' 

prove  tnat  —^    +  -^  +  -^  =3    .   .    .,  v  o  • 


a  c 


S4.  Tf  —    =    -r   =    -7-    =    &c.,  then 


.  a'^-c''' 


_    a"c"^''-(a"-o"4-g")' 


130  COMPLETE    SQUARES. 

a,  a.  fl,  a„ 

65.  If  -r^    =   T^  =  ~   =     .     .     .  =    7^'    then 

»!  Oo  ^3  0^ 

^  +  B+C         .4     ,     i?     ,      C 

36.  If  7 = f-   IT   +   — ' 

abc  a  0  c 

&-n(i{A  +  B+C){a-\-h  +  c)  =  Aa  +  Bb+Cc, 

A  B       ^       G        _ 

then  will  ^;:^   +   .-^^   +  y:^   -    0- 

and  also +   +  ^ 


11  1 

«+—         b+-r  c+— 

a-h         i/k  zl  x^  y^  a-^ 

37.  If  -5^   =   7T    =    -5-'    and  -^  =   ^^    =    -^   =1» 

thenwill^ly   +   y  +   -j  =j^   +   ^  +  1? 


Section  IV.  —  Complete  Squares,  &c. 


1.  What  quantity  must  be  added  to  a;  2  ^2jx  to  make  it  a  com- 
plete square  ? 

Let  r  be  the  quantity. 

Then  x^+jjx+r  =  complete  square  -  {x+  ^/7~')^ 

=  x^ +2x  yr-{-r. 
Equating  coefficients  we  have 

_p2  /  p 


Or  thus:  Since  {a-}-x)^  =  a-  -\- 2ax  +  x'^^  ;  we  observe,  (See 
Art.  XII),  thsit  four  times  the  product  of  the  extrsmes  is  equal  to  the 
squai'e  of  the  mean, 

4:X^r=j^"^'  ' 

r  =  {-t;-]    >    as  before. 


COMPLETE    SQUARES.  131 

Or  we  may  extract  the  square  root  and  equate  the  remainder 
to  zero .  thus 

P 


p 

px-\-L- 
4 


2T' 

Now,  if  the  expression  be  a  complete  square,  this  remainder 
must  vanish  ;  hence  we  have 

^  =  T  =    (-2 

2.  Find  the  relation  connecting  a,  h,  c,  if  ax^+bx+c  is  a  com- 
plete square. 

Assume  ax-  -{-bx-TC  =  {\/ n.x-{-  \/ c  )"  =ax-  +2-[/(ac).x  +  c. 

Now,  since  this  holds  for  all  values  of  x,  we  have  2  ^ac  =  b,  or 
b-  =  iac,  the  relation  required. 

3.  Determine  the  relation  amongst  a,  b,  c,  in  order  that 

a^x^+bx+bc-{-l>'  may  be  a  perfect  square. 
As  in  Ex.  1,  we  have  Aa^x-{bc'\-h^)  =  h^x'^  ; 

.-.    i-    -  -f-   =    1. 

4u2  I 

Or  thus : 

Assume  a^x^  +hx-\-bc  +  b'^  =  {rix+\/'oc  +  1"^)^ 

=  a'^x^+lai/'h^b^-  +  Z*c+63 . 
Equating  coefficients,  we  have  b  =  2aybc-^h^  ; 

.".    —    _   —    =   1,  as  before. 
4a-  b 

The  same  result  may  also  be  obtained  by  extracting  the  square 

root  and  equating  the  remainder  to  zero. 


132  rOATPT.T'Tr,    SOrAVTi;s. 

4.  Silow  tlmt  U  x'^-ri^'x^-^hx" -^rx-~d  be  %  complete  square, 
the  coefficients  satisfy  the  equation  c^  —a^d  =  0. 

Is  it  necessary  that  the  coefficients  satisfy  any  other  equation  ? 

Extracting  the  square  root  of  x^-^ax^  ^hx^ +cx-\-d  in  the 
usual  manner,  we  have  for  the  final  remainder 


,   -x  +  d-  —lb  - 
2   \  4  /  )  4\ 

Now,  if  the  expression  be  a  complete  square,  this  remainder 
must  vanish  ;  and,  that  it  may  vanish  for  general  values  of  x,  we 
must  have 

a  I         a^\ 

*-y(^-t,'=^ W' 

1  ;       fl2 

^^-Tr-T-'=^ (2); 

Elimmating  h  -   _,    ^re  have  c- -«"(■/ =  0      .     .     .       (3). 

The  coefficients  must  satisfy  the  equations  (1)  and  (2),  and 
therefore  either  of  these  equations,  together  with  the  equation  (3), 
which  results  from  them. 

The  same  result  may  be  obtained  by  assuming 
a-*  +ax^  +  hx^  +cx^d  =  {x^  -f  ^ax+  yd) ^ 
=  x^+ax^  +  2x^yd 

-f  \a-x'^  -r  nx  s/d  -\-  d. 
Equating  coefficients,  we  have  2yd+\(i.^=b      .     .     .'  (1) 

and       ayd=c       .     .     .      (2). 
From  (2)  we  have  c^  —  a"d  =  Q,  as  before. 

5.  What  must  be  the  value  of  m.  and  n  if 
4x*  —  12x'^  +25a;2  _4,»a:+8?r  is  a  perfect  square  ? 

Assume  the  expression  =  {(2.^2  _  3a;+  n/(8».) }  ^ 

=  4a;4-12.'c3-H4«2y(8«)+9a;--Cj-N/(8?0-!-Sn. 
Equating  coefficients,  we  have  <6\/{Qn)  =  Am      ....      (1), 
and4:-/(8?i)  +  9  =  2.5      ....      (2); 
.-.     %=2, 
in  =  6. 


COMPLTSTE    SQU.4IIES.  133 

Or  thus  :  ExtrR,ctIng  the  gqiiare  root  in  the  nrJinary  ivay,  the 
remainder  is  found  to  be  (— 4???-}- 24)3-4-8^—16  ;  .■.  we  must 
have  4m  -I-  24  =  0,  or  m  =  6, 

and     8??  -16  =  0,  or  n—2. 

6.  If  nr^-^bx'+ ex -'rd  be  a  complete  cube,  shew  that  ffc^  _^js^ 
and  b-  =3ac. 

Assume  ax^-{-bx^+cx  +  d=(x+'J}i)-  ' 

Equating  coefficients, 

b  =  Sa^d^ (1) 

c  =  3a^(i' (2); 

dividing  (l)by(2),l   =   ~; 
c  a.: 

ac^=dh^. 
Also,  h^=9a*d^ (3); 

dividing(3)by  (2),  ^  =  3^; 
c 

7.  Find  the  relations  subsisting  between  a,  h,  c,  d,  e,  when 

ax^  -\-hx^ ■\-rx'^ -^dxAr^  is  a  complete /owri/t  power. 
Assume  ax^ -^-hx^ -\-cx'^ -^dx-\-e  =  {(i^x-\-e^)^ 

=  ffx^  +4aVa;3  +  6aMa;2  +  4a¥a;-f-e. 
Equating  coofiacients,  we  have 
h  =  4rt^  e^, 
c  =  Ga^e^, 
d  =  4caie^  ; 
whence  M  =  16fl<'. n\ 

bc  =  2iaie^  =  QaAaiei  =  Qad.       ....       (2). 
cd  =  24:a^ei  =  6eAa'e^  =  &)e.        ....       (3). 

8.  Shew  that  x^+px^  +  qx^+i-x+s  can  be  so  resolved  into  two 
rational  quadratic  factors  if  s  be  a  perfect  square,  negative,  u.nd 

equal  to 


134  COMPLETE    SQUARES. 

Since  — s  is  a  perfect  square,  let  it  be  n^. 

Assume  x^  +px^  -^Q.^^ -\-'>'-''- "~  ■" ^ 

=  {x^  +  vix-{-n)(x'^  -^-m'x  —  n) 
=  re*  +  {m-\-m  ')x^  -1-  mm  'x^  —  n  (w  —  m  ')x  —  n^ . 
Equating  coefficients,  we  have 

m-\-7n'  =  p 
mju'  —  q 

r 

m—m'=  — 
n 

ni^ +2mm'  ■\-m''^  =p^ 

4,mm'  =  4.q    ; 


:.  {in- 

■m')^: 

=p' 

-4j 

j-2 

=  n" 

=  -s. 

'■  ^ 

-  4j0  2 

Exercise  xlvii. 

1.  What  is  the  condition  that  {a  —  x){b  —  x)  —  c^,  may  he  a  per- 
fect square. 

2.  Find  the  value  of  n  vrhich  will  make  2a;- +8x4-",  ^  perfect 
square. 

3.  Find  a  vahie  of  x  which  will  make  x^  -f- Qx^  + 1  Ix"  +  3x  +  31, 
a  perfect  square. 

4.  Extract  the  square  root  of 

.  5.  Find  the  values  of  m  and  ?i  which  will  make 
iz^—4:X^+5x^  —mx+n,  a  perfect  square. 

6.  "What   must  be  added  to  x*  — n/(4x^ -lCx-+16)-4x3  in 
order  to  make  it  a  complete  square  ? 

7.  The  expression  x^  +  x^  —  16x2  —  4x4-48,  is  resolvable  into 
two  factors  of  tho  form  x-'+mx  +  G,  and  x^+nx-[-8;  determine 

the  factors. 

ex 
'  8.  Find  the  value^  of  c  which  v/ill  make  4x4  —  cx^  +  5x^  +  -^  + 1, 

a  complete  square. 


COMPLETE    SQUARES,  135 

9.  Oulaiii  tbe  square  root  of 

4{(a3  -  b-^)cd-i-ab{c'i-d^)}2 -f  {{a^  -  h2)(c^-d^) - iahcd] a. 

10.  If   {a-b):c-  +  {a  +  b)^x+{a^  -  b"){a+b),   is    a    complete 
square,  then  a  =  Sb,  or  b  =  Sa. 

11.  Find  the  simplest  quantity  which,  subtracted  from 
a^x^+iabx-r-Lacx+obc-l-b^c^,  will  give  for  remainder  an  exact 
square. 

12.  a;*  — 4rf3—a;2-f  16a;  — 12  is  resolvable  into  quadratic  factors 
of  the  form  x^  +mx+p,  and  x-  +77x+q^:  find  them. 

13.  Find  the    values   of   m    which  wili    make   x~  +max  +  a'^ 
afactor  oi  x'^  —  ax^-j-a^x^  —a^x-{-a*^. 

14.  Shew  that  if  x^  +  ax^+bx'^+cx+d  be  a  perfect  square,  the 
coefficients  satisfy  the  relations 

8c   =a(46-a2),  and 
64:d=    (46-a2)2. 

15.  Investigate  the  relations  between  the  coefficients  in  order 
that  ax-  -^bij^  ^cz"^  +dxy-{-eyz-\-fxz  may  be  a  complete  square. 

16.  If  x^+ax^+bx-rc  is  exactly  divisible  by  (x+d)-,  shew  that 

-       i(6   -d2)=-^=d{a-2d) 

17.  Determine  the  relations  among  a,  b,  c,  d,  when 
az^  —  ix^  +  cx—d,  is  a  complete  cube. 

18.  The   polynome   ax^  -\-Zbx^  +ucx-\-d   is   exactly   divisible 
by  (a-a;)3  ;  shew  that  {ad-hc)^  =  i.{ac-h^){hd-c^). 

19.  Find  the  relation  between  p  and  q,  when  0:^+^3:2+5',  is 
exactly  divisible  by  {x  —  a)^. 

20.  If  x^+7iax  +  a^  is  a  factor  of  x^-\- ax^-{-a^x^  ^a^x^a'^, 
shew  that  n^—n— 1  =  0. 

21.  li  x^+ax^  +  bx^ +cx  +  d,he  the  product  of  two  complete 
squares,  shew  that. 

(46-a2)-'=64c/,  (46-«3}a  =  Sc,  a>/{Ba^ -2b)  =  Sh. 


136  RELATION    IN    INVOLUTION, 

22.  Prove  that  »*  -rpx^+q^^-^rx+s  is  a  perfect  square,  if 

p'^s—T,  and  q  =  —-  -^  2^/6-. 

23.  If  a.c3+36^'-+3c.i;  +  c/  contain  ax--\-1hx+c  as  a  factor,  the 
former  will  be  a  complete  cube,  and  the  latter  a  complete  square, 

24.  li  m^x^  +px+ijq+q^  be  a  perfect  square,  fiudj:>  in  terms 
of  m,  q,  and  x. 

25.  Find  the  relation  between  p  and  q  in  order  that 

x^+px^-{-qx-\-r  may  contain  (a;-i-2)3  as  a  factor. 
2G.  If  x'^+^j.c2_|_^^^_j_;.  \^Q  algebraically  divisible  by 

'dx'  +  'lijx-]rq,  shew  that  the  quotient  is  u;  +  —• 


Eelatxon  in  Involuiion. 


Art    XXXVIII.     If  aa'  =  ^*6'  =  cc',  prove  that 

1.  {a  +  h'){h-\-c'){G+a')  =  {a'-[-b){h'+c){c'-\-a) 

{a  +  b')xa'  =  aa'+b'a'  =  bb'+b'a'={b  +  a')xb' 
{b+c')xb'  =  bb'+c'b'  =  cc'+c'b'  =  {c  +  b')xc' 
{c-i-a')xc'  =  cc'  +  a'c'  =  aa^-ira'o'  —  [a+c')  xa' 

.-.  {a+b'){h-\-c'){c-\-a')xa'b'c'  = 
{a'+b){b'+c){c'+a)xb'c'a' 

:.  {a+b'){h  +  c'){c  +  a')  =  {a'+b){b'+c){c'-\-a). 

2.  (^a  +  b){a+hl){a' -c){a' -c')  =  {a' +b){a'  +  b'){a-  c){a-    c"). 
{a  +  b)y^a'  =  aa'  +  a'b  =  hb'  +  a'b  =  (b'-i-a')xb 
{a+b')xa'  =  aa'  +  a'b'  =  bb'+a'b'  =  {b+a')xb; 

(a'  —  c)  X  a  =  aa'  —  ac  =  cc'  —ac  =  (c'  —  a)  xc 
(a'  —  c')  xa  =  aa'  —  ac'  =  cc'  —  ac'  =  {c~a)  xc' 

:.  (a+b){a  +  b'){a' -c){a' - c')  X  (aa')^  = 
{b'-i-a'){b+a'){c'-a){c-a)xbb'.cc' 

B\xt  bb'.cc'=(aa')^, 

and  (c'  —  a)(G-a)  =  (a  —  c)(a  —  c') 
:.  la-{-b){a-\- b'){u' -c){a' -c')  =  {u'+b){a'-^b'){a-ti){a -cf). 


RELATION    IN    INVOLUTION.  137 

Exercise  xlviii. 
If  aa'  =  bb'  =  cc'  prove  that 

1.  {a-b')[b-c){c'-a')  =  {b-a'){a~c){c'-b'). 

2.  {b-c'){c-a){a'  - b')  =  (c -b')[b - a)[a'-c'). 

3.  {c-a'){a-b){h'  -c')  =  {a-c'){c-b){b'-a'). 
•    4.   {a-b'){b-c'){c-a')  =  {a-c'){b-a'){c-b'). 

(a-b){a-b')     _     (a—c){a-c') 
^'   (a'-fc)(a'-i')    ~    (a'-c){a'-c'y 
{b—c){b-c')      _     {b-a){b-a') 
^-   {b'-c}{b'-c')     ~    [b^^a^b'-a')' 

{c-a){c-a')      _     (c-b){c—b') 
^*   (e'-a)(c-a')     ~    \c'-b){c'-b'T 

8.  Shew  that  the  seven  preceding  relations  may  he  derived 
trom  the  single  relation 

\a-\-a'){bb'  —  cc')-^{v  -{■b'){cc'  —  aa ')  +  {c-T<:'){c!,a'—bu')  =  0. 


138 


SniPLE    EQUATIONS. 


CHAPTER  V. 


Simple  Equations  of  one  Unknown  QtJANTixy. 

Art.  XXXIX.  Preliminary  Equations.  Although  the 
following  exercise  belongs  in  theory  to  this  chapter,  in  practice 
the  numerical  examples  should  immediately  follow  Exercise  I., 
and  the  hteral  examples  Exercise  III.  Like  those  exercises,  this 
one  is  merely  a  specimen  of  what  the  teacher  should  give  till  his 
pupils  have  thoroughly  mastered  this  prehminary  work.  But 
few  numerical  examples  are  given,  it  being  left  to  the  teacher  to 
supply  these. 

Exercise    xlix. 

What  values  must  x  liave  that  the  following  equations  may  be 
true? 

1.  a;-5  =  0.     a;-3i  =  0.     x-a  =  0.  a:+3  =  0. 

2.  a;4-4i  =  0.     x  +  a=0.     a:  +  3  =  5.  a;-4  =  6. 

3.  x  —  a~h.     x-\-a  =  c.     x-h=  —c.  6  — a;=3. 

4.  8-a;=10.     5  +  :«=ll.     9+x'  =  4.     l-x=-5. 

6.  84-iC-^-6.     fl-a;  =  36.     2rt  =  a;-f-36.     Qa^oh-x. 

6.  2a;-6  =  8.     3a;+8  =  20.     ax  =  a^.     mx=bm. 

7.  Bx  =  c.     ax  =  o.      ax  =  0.     [a-\-b)x  =  b-\-a. 

8.  {a  —  b)x  =  b-a.      {a  +  bx)  =  {a+b)^.      {a—b)x  =  a^-b^. 

9.  {a  +  b)x  =  b^-a^.      {a^-ab  +  b^)x^a^+b^. 

10.  {a^-b^)x  =  a-b.      {a^~b^)x  =  a  +  b.      {a''- +b^)x=l. 

11.  {a-\-x  —  b)  =  {a-\-b).     x  —  a-{-b  =  b~x-ra. 

12.  'la—x  —  x  —  2h.     ax-\-bx  =  c.     ax  —  b  =  cx. 

13.  ax  —  b  =  hx  —  c.     ax  —  ab  =  ac. 

14.  ax  —  a^=bx  —  b^.     ax  —  a^  =  bx  —  b^. 


SIMPLE    EQUATIONS.  139 

15.  ax  —  a^=b^  —  bx;     ax-\-b-\-c  =  a  +  bx-\-cx. 

16.  a  —  bx  —  c  =  b  —  «x-{-cx;     a-{-hx+cx"  =ax  —  b^cx^. 

17.  bx  —  cx-+e  =  ex  —  b-cx";     'dx  =  t;     4.x  =  ^. 

18.  10a;  =  ^  — 1;     ax=  — ;     ax= — . 

c  b 

,  „       .  a  ^7  ac^         ab" 

19.  abx  =  —   -f  — ;     bcx=  —    -i_    — . 

ha  be 

20.  ia;=5;     |a:  =  8;     -50;  =  2;    ■Sx=-Q(j. 

21.  •02x  =  20;     -32;= -2;     -ix^-Q, 

.  .  X  ax 

22.  •18x=l-8;    —    =   6;     -r   =   c 


23. 

T 

0             a;                    ax       . 

—  , =c;     -  =  L 

c         a-\-b              a~{-b 

24. 

a—  0 

a          a  —  b         a-r-b 
b  '       a-hb     ~'b  —  a' 

25. 

a 

0—  't 

a            b  -a         a  —  h 

a — 0          a  +  b          b+a 

26. 

a+b 
—      a; 
a-i-c 

_  a-c  .1            1.2 
a  +  b'      X           -2,  '      X 

= 

3 

5 

27. 

1 

X 

1  .       1            a  ^      a 
ah      ^^x            h          X 

b  , 
c 

7 
a; 

=   1  +  A 

3           4 

28. 

1- 

4     _    33           1  .      ^    j_ 
5a;         5a;          'd         x 

b 
c 

=  0. 

29. 

5 

a;-7 

=6         '    ;         '      -7+ 
a;-7       3a;-4              4 

9 
-3a;' 

30.  (a;-4)-(a;-i-5)  +  a;  =  3;     2a;- (a; -5) -(4 -3a;)  =  5. 

31.  2(3-a;)  +  3(a;-3)=0;    2(3a;- 4)-3(3-4a;)  +  9(2-x)  =  10. 

32.  a(l-2a;)-(2a;-a)  =  l  ;    a;- 5(a-a;)  =  6a;- 5«. 

33.  ma;(3a-4)  +  3ma;-3rt  +  l  =  0. 

84.  a(bx—c)+b[cx  —  a)+c(ax-b)=0. 

85.  a{ax  —  b)  i-h(^cx  —  c)+c{cx  —  a)  =  0. 


^  SnTPLE    FQUATION'S. 

86.    a{hx - a)-\-h[rx  -  h)-^r(ax -c)—-0. 

38.  3(3{3(3a;-2)-2}-2)-2  =  l. 

39.  9(7{5(3a;-2)-4}-G)-8  =  l. 

41-    i{ia{4(a^  +  2)  +  4}4-6)  +  8}  =  l. 

43.  t(l{|(3^:^-li)-H}-li)-H  =  0. 

44.  ^{?T(^{|(fa;+4)  +  8}  +  12)  +  20}-f 32  =  58. 

45.  l{|(t{^-(a:+7)-3}4-6)-l}=4. 

46.  r{g(;j{w(rHa;-a)-&}-c)-rf}-^  =  0. 

47.  (l  +  6x)3  +  (2  +  8a:)2  =  (l  +  10a:)=. 

48.  9(2a:-7)2  +(4a:-  27)3  =  13(4x+  l^)(x+G). 

49.  (3-4a:)24.(4-4a;)2=2(5+4a;)2. 

60.    {9-4x)(9-5x)-{.4.{5-x){5-4x)  =  SG{2-x)9. 

Art.  XL.  In  order  that  the  product  of  two  or  more  factors 
may  vanish,  it  is  necessary,  and  it  is  sufficient,  that  one  of  the 
factors  should  vanish.  Thus,  in  order  that  (x-a)(x-b)  may  =0, 
either  a;-a  must  =  0,  or  x-b  must  =0,  and  it  is  sufficient  that 
one  of  them  should  do  go. 

Hence  the  single  equation  (x-o)(x-b)=0  is  really  equivalent 
to  the  two  disjunctive  equations,  either  ar_a  =  0  or  a;  -  6  =  0,  for 
either  of  these  wHl  fulfil  the  condition  of  the  given  equation, 'and 
that  is  all  that  is  required. 

Similarly,  were  it  required  to  find  what  -alues  of  a?  would  make 
the  product  {x-a){x-h){x-c)  vanish,  they  would  be  given  by 
a;-«  =  0,  ora;-7;  =  0,  or  a'-c  =  0    .-.  x  =  aovborc. 
Hence  the  single  equation 

{x—a)(x—  b)(x  —c)  =  0 
is  equivalent  to  the  three  disjunctive  equations 
x  —  a  =  0,  or  x  —  b  =  0,  or  x—c  =  0. 


simple  equations.  141 

Examples. 

1.  Solve  x'- a; -20  =  0. 

The  expression  =  (x-5){x+4:),  -which  will  vanish  if  either  of  its 
factors  (Joes,  that  is,  if  a:  -  .5  =  0,  or  a-+4  =  0, 
.'.   x  =  5,        or  a*—  — ■^ 

2.  SoUex*—.r^-x^+x  =  0.  • 

This  gives  x'^{x-l)  —  x{x-V)  =  x[x—\\x^-V) 

=  x(a-— l)(a;  +  l)(x— 1),  which  vanishes  for 
a:  =  0,  a"  =  l,  x-=  —1. 

3.  Solve  a;3  4-n2a;2  _crj;  _  aS  =  0. 

This  =  a-(rc2  -  a)  -\-a'^{x'^  —  a) 

=  {x+a^){x^  —  a),  which  vanishes  fa? 
a;4-a2_o,  anda;2— a  =  0,  or 
x=  —  a",     and  X-  =  a. 

4.  Solve  x^-{a-h)+a''-ih-x)  +  h^{x-o)  =  0. 

The  factors  of  the  expression  are  (Ex.  2,  page  79) 

«  —a,  x—h,  a  —  h;  hence  the  expression  vanishes  if 
a:— (7  =  0,  or  a;  — 6  =  0. 

5.  Solve  221ar2-oa;- 6  =  0. 

Here  we  have  the  factors  ITx  — 3  and  13a:+2  ; 
.•.   the  equation  is  satisfied  by  ITx  — 3  =  0,  or  a;  =  fy, 

and  13x+2  =  0,  or  a;  =  -  ^V 

6.  Bolve  2a;4+2a:^  +  Gx;-18  =  0. 

In  this  case  we  have  2{x4  —  9)  +  2x{x^  +  3) 

=  2(a;2+3){x3-3+a:},  which  vanishes  for 
x^+Z  =  0,  ora;2+.,._3  =  0. 

7.  Solve  (x -a)^+  (o. - b) •>  -;- (b  -x)-  =  0. 

The  expression  is  equal  to  B(x  —  a)[a  —  b)(b—x), 
and  therefore  vanishes  for  a;  —  a  =  0,  or  x  =  a; 
and  for  x  —  b  =  0,  or  z  =  b. 


142  Simple  equations. 

Exercise  1. 

1.  If  an  equation  in  x  has  the  factors  2a:  — 4  and  2:c  — 6,  find 
the  corresponding  vahies  of  x. 

2.  If  an  equation  gives  the  factors  2a;-  1  and  3aj  — 1,  vrhat  are 
the  corresponding  values  of  a;  ? 

3.  If  an  equation  gives  the  factors  dx^  —12  and  Ax  — 5,  find  the 
corresponding  values  of  x. 

Find  the  values  of  x  for  which  the  following  expressions  will 
vanish ; 

4.  x''-2x+l]  Ax^-12x+d. 

6.  9x^-4;  x-  —  {a-\-h)2  ;  x^-2ax  +  a^. 
Q.  x^-9x+20;   4a;2-18a;  +  20. 

7.  x^+x-6:  x^-x-U;  9a;3-9a;-28. 

'8.  6a;3-J2a;+G;  6a;2-13a;+6;  6a;2-20x+6. 

9.  6x^-5x-Q;  6a;3-37a;+6;  Gx^-\-x-l2. 

10.  A  certain  equation  of  the  fourth  degree  gives  the  factors 
a;3  — a;  — 2,  and  4:X^  —  2a;— 2,  find  all  the  values  of  x. 

Find  values  of  x  in  the  following  cases  : 

11.  x^-2bx^-Sb^x=-0. 

12.  x'^-ax^+a'^x-a^^O. 

13.  a;3-2.?;+l  =  0;  a;3-8a;+2-0. 

14.  a;*-2aa;3+2a3^— «4=0- 

15.  x^  +  {b  +  c)x^  -bcx-b^c-bc"  =0. 

x  —  ax—b  {a  —  bY         _        x^  —a^ 

■    x  —  b         X- a         {x—a){x-b)  {x—a){x  —  h) 

17.  x^-bx''-a^x-an  =  0. 

18.  3a;3-{-4flte2 -6a3/;2^_ 4(13^3  =,0. 

19.  x^{a-b)  +  a^{b-x)^-b^{x-a)  =  0., 
(x-b){x-c)  {x-c)(x-a)  ^  ^ 

^"'    {a-b){a-c}   "^  {b-c){b-a) 
■x-2a\  3  /2a;- a\  3 


/a;-2a\ 


21.  x\;:rr-7.i  +«(ir+^;  =-^' 


SIMPLE    EQUATIONS. 


143 


cfi  feo-  ax  _       1 

^^'    {l>-a){x-a)   "^   (^^{a-b)   "^  (^a-b){b-x)    ~    a^b 

24.  Form  the  polynome  which  \\ill  A'anish  for  x  equal  5,  or 
-6,  or  7. 

25.  Form  the  polynome  which  will  vanish  for  x  =  a,  or  4a,  or 
3(1,  or   —  4fl. 

26.  Form  the  equation  whose  roots  are  0,  1,  -2,  and  4. 

27.  Form  the  equation  whose  roots  are  1  +  s/-,  1—  \/2,  1  -  ^^'3, 
and  1  +  y  3. 

Art.  XLI.  In  solving  fractional  equations,  the  principlea 
illustrated  in  the  section  on  fractions  may  frequently  he  appHe«! 
with  advantage,  as  in  the  following  cases. 

When  an  equation  involves  several  fractions,  we  may  take  two 
or  more  of  them  together. 

EXAJIPLES. 

1  o  1       8a; -t- 5         7a;  — 3         4a;+6 
1.  Solve  4- =    — -^ — 

14      ^   6a;+2  7 

Here,  instead  of  multiplying  through  by  the   L.  C.  M.  of  the 

denominators,  we  combine  the  first  fraction  with  the  last,  getting 

at  once 

7a;- 3  7  1 

6xT2    =    14    "    T        •■•   7x-8  =  3a;+l,  anda;  =  l. 

2  2a>f8|   _     13a;- 2  ^   _    Z^    _  =^+16 
~9~~           17a;   -  32   ■*■    3     ~    12  "36 

In  this  case,  taking  together  all  the  fractions  having  only 
numerical  denominators,  we  get 

8a;+34+12a;-21a;4-a;+16   _    13a; -2 


36  17a;  -  32  ' 

25  13a;-    2 


or 


lb         17a; -32' 

.-.    425a;— 800  =  234a; -36,  hence  x  =  4. 


144  SIMPLE    EliUATlONS. 

It  is  often  advantageous  to  complete  the  divisions  represented 
by  the  fractions. 

4x-17    _    3|_-22u;   _  A  /         i^ 

9  33  y;   \  C>4 

Here,  completing  the  divisions,  we  have 

4x         17  1  2x  6  rr, 

lOa;  X  ()  6 

—TT— -2  =  a;+-7-    —   —    .-.     —  2  =  — — ,    or.e  =  3. 

V  d  X  X 

ax-\-h         cx-\-d 

4.    +  =    «-fc 

X  —  III  x  —  n 

ani  +  b  i-n-ird 

a  -\ +  0  + —  «4-c 

X  —  III  x  —  n 

{am.  +  h){x—  n)  -\-{cn-\-  d)  (^x  —  vi)  —  0 

{aiiL-\-b  +  cii-\-d)x-  («4-c-)//m4-i/<+(/nt. 

5.  Similarly  may  be  solved 

ax  +  h         ex  +  d  ex^  +/x  —  g 

x  —  m  x  —  n  {x  —  m)[x  —  n) 

ain+b         cn+ d  {e{iii  +  n)-^-/}x--emn  —  g_ 

"      X  —m  x  —  n  {x  —  vi)(x-n)  ~ 

{am.-{-b){x  —  ny\-{cn-\-d){x  —  m)-{-  {e{m  +  n)-{-f}x  —  emn-  f/  =  0. 

{{a+€)in-\-b  +  {(■-\-e)a-\-d+f}x  —  {a  -tb-\-c)in.n  -\-  hn+dm-j-;i. 

3x+l      ~     x-1 

43  13 

•••  4^  -   3:h^1   +   ^   +  JT-i    =    52,  or 

13  43 

:,    -    -s — ^-^',    •'•    39x-  +  13  =  43a;  — 43,  and  jc— 14. 

x  —  1  3x-fl 

_     25— Xx           lGx+4:l  .  23 

7. !L_     . = i    =    0-1-         -. 

ic+1      ^     3.i;+2  x-tl 

Taking  the  last  fraction  with  the  first,  and  multiplying  the  rC' 
suiting  ei^uatioo  hy  15,  we  have 


SIMPLE    EQUATtONS.  145 

240r-J-63    _    r-r     ,    5.r— 30  . 

.-.    SO   -    J^   =    75  +   5   -   -^,  or 
3^  +  2  x  +  1 

o.r-f-2  a'-f-l 

X-  a         x  —  h         X  —  c         ,; 
o-irf  a  +  e         h-\-c 

.-.    ^^^  -   1   +--I1^   -   1   +:■--:   -   1    =.   0; 

''■ff  ffl  +  c  "^  h+a  ~     ' 

which  is  satisfied  bj^  a;  —  (r/.  4-  ft  +  c)  =  0  ;     .-.  a;  =  a +/)-{-  p. 
m  +  71 


9.    -^    + 

'  X  —0  x  —  c 


x  +  n         x—h  x  —  c 

m(.r  —  r)  n[x  —  r)    _ 

x  —  c.  x  —  b 

which  may  be  solved  as  in  Ex.  1. 


10, 


nx-Iro  4a::4-9    _    15r4-_7  IS^r^-l? 

x+l     ~    2x+l    ~     3.r+'l'   ""    ^oTfT  * 

I-    -J^-    -  2  -    —±—   =  5  +    _^_  _  4  -        -        or 
^+1  2a;+4  3,r+l  3a: +  4 

2  12  1 


x+l         2x+4:         Pjx  +  1         3x+4' 

3a:  4- 7         _  3:r  +  7 

2a;2  +  6.i-+4    ~    9a;2  +  15a-+4' 

This  can  be  divided   by  3ar+7,  givini?  3ar-f7  =  0,  or  x=  ~^. 
The  result  of  the  division  is 

1  1 

= ,  or 

2x3  +  G.f-f4         9a:2  +  15a:  +  4 

9a-- +loa;+4  =  2x2  +  Ga-  +  4,  or  7x^  =  —9x,  which  we  can  divide 
by  X,    :.  x  =  0  ;  the  result  of  the  division  is  7a:  =  —9,  or  »:=  —  5.. 


146  SIMPLE    EQUATIONS. 

Exercise  li. 
lOx+n         12x+    2         5a;-4 


1. 


13. 


15. 
16. 


18  18.C-16  9 


6x+13         9a;+15  2x+ 16 


15  5a;- 25  ^  5 

Tx+l    _   ^   X   ^-^ 
a-1    ~     9  a;+2 


3.     -t: — T    =   —   ^    :7T-o   +   3^ 


4a;- 7         2-14x       '  H+x  _  10  -  3fa;   _    19 
^-      2^^  +        7        +      14      ~    ^     2  21* 

2a;  +  a'  Sx-a      ^  ^ 

^-      3(a;-a)   "^   2(j;+«.)  ~'^^' 


a; 


-4         3x— 13  1 


6-      6a; +  5   "*"   18a; -6  3 

3a;  +   1    _      a;-ll  _  a;-9         ^5  ^  ^ 

"^^      2a;-15    ~    2a;- 10"     '     a;-5   "^  a;-8~"" 
^       a;-12         a;-   4      ^         7    .  3a;- 19         3a;-ll 
^-      x3^  +  ^^2  =2+  x37'  -^^^13  +  -^-7  =  ^- 

a;- 2  a;-l 5^.       a;+l  a:  +  4     ^    ^ 

9-      2:^+1    +   3(^8)    ~     G  '    4(a;+2)   +   5x+ 13  ~    £0' 

6(2a;2+3)         5-7a;     ^       ^  3  14 

!«•    -Vfi        +   2:^='-^-^'     ^7   +  x-^  =  X— 8 
7a;+55         3a;  _        Sa;^  +  8 .      17  15  32 


2x+   5   ~    2  ~        2x  -4     a;-16  ^  a;- 18  ~  a;- 17 
3-2ia;     _   28- 5x-  _   IC 
"    14(a;-l)    ~         3"^ 
2+2ia;3-^a;3  1  6 


l-25a;  3-2ia;     _   28- 5x-  _   10a;  -  11  x_ 

12.    —^  14(a;-l)    ~         3"^  30         +3 


6-5a;+a;3  '2  x-5 


^__  60+8^    _     4^    ^   ^^ 

~  '         ■  ^       "^     a;+3      ~   a;+l 


5j;2+a,._3         7a;2_3^._9 


4 

7a; -10 

a;- 9 

a;  +  l         a;-8 
~    a;-i   "^  a;-6' 

SIMPLE    EQUATIONS.  147 

17    x^-3x-9         x^-lx-ll         a;3-6x-15 

x-6        ^         x-d  ..         ^ 


18. 
19. 
20. 
21. 
22. 
23. 
24. 
25. 
26. 


ixA-1^         4.g+9    _  4a;  +  6 

ix-\-Q  "^  4a:+7    ""  l.c4-~4   "^ 

2^-3         2x-4    _  2.t;-7 

2^-^  '"   2^-5    "  2a:- 8 

7a: +  6         2.r  +  4|-  x_ 

28  ~   23a;-6    "^  T   ~ 
a;2_5         a;3-ll  a;3-7 


a;-8 

4a: +10 

4.r  +  8 

2a,- -8 

2a: -9 

11a:         a;-3 

21             42 

a:2-9 

+    "2       iA- 

a;3-6   ^  a;--2-12  ~   a;3-8 

X  - 1|4  2-Ga:                  5.<;-^(10-3a:) 

2  ~T3~  "  ^  "              39 

1— 2a:  1+x  1 


3(a:2-a;  +  l)   "^  2(a;2  +  l)   "^  6(^^-+l)    ~    9(a;3+l) 

2.^2 +a;- 30         x-+4.x-A    _   x^  -  17         2a;2+7x-13. 
2a;-7        "^         x^^O         ~     a:-4     "'         2x-3 

a;-a         a:-?>  {a-hY         _    2(a-a;) 

a;  — 6         a;-  a         (a;  — a)(a:  — i)    ~"      a-\-x 

12a:+10a   j.   28a: +  117^/.    ^    ^g 
~3a:+^      '    ~2^+9^ 

^^     13|x-5  13.^a;-ll     _    13;a:-7  13^a;-9 

"'■    ISJ^^T^  "^   13Sa;-12    ~    l^^^  "^  134a;- 10  ' 

<,Q  1  1  X  16a; 


2(a:-l)2    "*"   2(a:-l)  2(.'c3+i)    "    (^_i)(a;2  +  i) 


29.    i(|x+4)-   Zi 


3 

OQ     3»  _         81a;2-9       _  q     _  i_     ^a;^-!    _   57 -3a; 
*    T         (3a;-l)(a;+3)  ~  '"^        2  '     a:+3  2      ' 

81     1  +     ^^L  i?+i_    _   ^L+^'t?  _ 

2(x-l)   ~   2(^+1)    ~   a:2_2a:.fl 


148  BI^JPLE    EQUATIONS. 

7;c-30  5x-l         2 -21a; 


?.2 


10|-  ^x--3  21 

42x-171        _    .      2a:-'J  1 

7 


6-3- -  10  +  03- iT.  - -(*  -  ■ 


18^-22  l-t-Hx  101-Gi.>; 

4-9x         5-12.C  __^^*l~^_ 

8a;4-25         16^  +  93   _    IQx+m         6x+ 26 
^^'    2;^^-^  "*"  ^2x+Tl    ~   "■2x  +  '"9  ■*"  2.^+   7* 

1  1  1  1  ^ 

gg I . ! _}_     —  0. 

x-\-a-rii         x—a-\-b         x-r-a—b         x  —  a—b 

Art.  XLII.     The  results  deduced  in  Section  III.,  Chapter 
lY.,  may  ofien  be  ax:)X5hed  with  advantage. 


1. 


Examples. 

ax  -\-h          in 

ex  +  d           n 

(ax  +  b)d-(cx+d)h 

rnd—oih 

(cu;+'i)i;t— («x+  Ij)c 

~   'fuo  —  me 

ind  —  nh 

na  —  mc 

ax^-\-bx-{-c           a 

inx''^-^7ix-hp          vi 

<i.age  123). 


(ax'+bx+c)j-ax^      ^     n_      ,  ^^^^  ^^2^ 
{mx'^ +nx+2j)  —  ■nix'^  m      ^^^    ■       '■ 

hx  +  c  a 

nx-\-p  lib 

3x+7    _    3x-  13 

33+4     "        X—     4 


SIlfPLE    EQUATIONS.  149 

By  (5)  each  of  these  fractions  = 
difference  of  numerators  20         3,r4-7  5 

difference  of  denominators'       "    ~S     ~     x-r^~        x+V 

or  -^   =    — -,    -.  ^-=0. 

vix-\-a  +  h         mx-{-a-\-G 
nx  —  c  —  d    ~    nx  —  b—  d* 

vix-\-a  +  b         nx  —  c  —  d 
•■•      ^+r+c   =    nx-b-  d  '  °-    ^y  4'  ^""S^  122, 

i)ix  +  a  +  b         nx  —  c  —  d 

I =    — ; ;  or  ()i  —  m)x 

b—c  o—c  ^  ' 

=      a-\-h-\-c-\-d,   .'.  a;  =  &c. 

^/{a■\■x)  —  ^{a  —  x)  ~ 
Here  by  (6),  page  122,  we  have 

2y(a  +  x)         rt+1 

2  /(a  —  x\   ~   ~a^  '  °^''  ^^^'^^^""o  ^^  2  in  left  hand  mem- 
ber, and  squaring, 

^^x   ^   (^zryiF' whence,  again  by  (G), 

2^    _    (a+l)--(a-l)2    _         4a         _       2a 
26t    ~    (a  +  l)34-(«_l)2    -   2(a2+l)    ~   a^^V 
2a3_ 
■•*       ^=  asqpi' 


6 


^/(x  — a  +  6)  — ■j/(a;  +  a  — 6)    _    a-b 
l/(j;-a  +  fc)  +  v'^('C+«  — ^)    ~    'iVi* 
.     i/ix-a-^-h)    _     a  . 
y^Xx^a^h)    "   T' 
squaring  and  again  applying  (6), 

'■•    ~   2(^^6)   ^  a3~6"3"'   and  u;  =  ^-^ 


150  SIMPLE    EQUATIONS. 

Exercise  lii. 

1     Lt? 

9       0-+X  fi{b-\-x^        , 

b-'r2x  '     a  -X     ~      '    a  —  x 

3.    'L±^   = 
a  —  x 

^     a-j-hx 

a-\-b     ~.     f  +  c/- 
g     2^3 _ 5a; -1-6   _   u;2-7a;  +  5 
2a;2-7a;+3   ~"   a;3-9a;i-2* 


9. 
10. 


ax~h  +  c         (6  +  c)2 

i^.  i/(«^+?/)+i/(x-.v)  ^  ^^  ^i^g^  ^1^^^.  ^:^ 

-\/{x+y)-^{x-y)  y  x-y 

1x-l    _    x+1  .    4a; -5     _     lOx-32 
2a^^   ~   x+li'  2a;+10   ~    ^S^c'-^r' 

57a; -43  30a;- 7  .    28a;+5|  36x-7 


19a;+13    ~    13a;+2o     115.x-- 29         180a;+23 

210a;-73    _    21a;+7-3 .   mx-a-b    _    mx-a-c 
310a; -66     ~    ^Ia;'+8" '    'rix^^^^c^d    "^    nx-b-d 

11     3a;+y(4a;-a;2)    _   ^     y(12a;  +  l)+ s/(12a;) 

'    3a;-y(4a;-a;3)    ~   "'  ■,/(l2a;+l)— /(12a;)    ~   "^^ 


12. 


x^-{-ax'  —bx+c         x'^-\-ax—h 
x^  —ax'^ -rbx+c    ~   x^—ax+b 


13  l/(2a--a;-)  +  V(2^-a^)    _     A  +  ?> 
|/(2a2_a;3)-6-/(2a-a;)  ^/a-^" 

14  T/(x3+a^)+A/(a;^-^^^)     _ 
'     ^/(x^-+a^)-V{x^-a^) 

8a;3  +  12.x3  -  8a;  +  5         4a;3 4-6a;-  4 


15. 


8a;3-12a;3+8a;;+5         4a;2-6a;  +  4 


SIMPLE    EQUATIONS.  151 


16. 
17. 
18. 

18. 
20. 


f{x  +  l)-^{x-l)    _     1 

f{x  +  l)-{-^{x-l)    -     '1 

28+ya;   _    9  +3 ./a; 

28-v/a;   ~   ^  +  2^x 

083.3  +a2^^.2  _  acx-\-d         a^x^+abx—e 

a^z^—a^bx^ +acx+d    ~    a"x^  -ahx+e 

5^/(2x-l)  +  2i/{Bx-d)    _    ^^^^ 

4:i/{2x-l)-2-i/{3x-S)    ~   ''^'^' 

^/2x+y'(3-2x)    _     3 

\/2x-\/(W^^,:)    -   T" 


2^(3a;+3)  +  r (7^+  8)  _ 
•    2^/(3a;+3)->'(7a;+"8)  -^• 

22.  33{13-2i/(a;-5)}  =  3{13+2;/(a;-o)}. 

23.  (i/»+l){i/(«a:+l)  -  ynx)  =  {i/n-l){i/{nx+l)^y'nx} 
l/jz+c)  +  ^/b         i/cc  4- 1/« 


24. 
26. 

26. 


V^+2.3  _  ]/«+38.  1^2^+17  _   1^2^+27 

l/x+  4  ~  i/a;+  6'  f2x+  9   ~   ■^2a;+15* 

yx+2a  _  i/x+Aa,         3a;  -  1         1  +  ]/3a; 

iT^n*  ~  v/^+36'  v^Sx  +  l    ~         2 


v/a--l/(ft-a;)  _    _     ■/il+J'^^ 
l/a+^/{a—x)~    '     \/x  —  -Jb 


28. 


aa;  +  l  +  v/(a-a;3-l) 


29  ii:y{ijiv^(i-^)}_. 


30. 
31. 


'a+x                fe+l  .     l+.6-+a;2 

62          1  +  a; 

v/(2«x+x-3)   ~    6-1'     l-ai+a;-^ 
5x*  +  10a;2  +  l          rt^-flOa^+l 
x*  +  lUa;»4-5.t;    ~    5a*  +  10a-  +  l' 

~   63    ^    1-a; 

Art.  XLIII.     Various   other  artifices  may  be  employed  to 
simplify  the  solution  of  equations. 


152  SIMPLE    EQTTATIONS. 

ExA3IPI,ES. 

1.  Solve  24-7(4..^ -0^4-8) -2;r=0  :  here  there  is  bnt  mie 
surd,  and  it  is  convenient  to  make  that  svird  one  side  of  the  equa- 
tion and  transpose  all  the  rational  terms  to  the  other ;  this  gives 
n/(4.t3  —  9a; -1-8)  =  2a;—  2 ;  squaring  both  sides, 

4^2— 9.(;+8-4.7;2_8x+4,    .-.    a;  =  4. 

2.  V{a  +  x)-\-\/{a—x)  =  'l\/x.  V/e  might  square  this  as  it 
stands,  but  the  work  will  be  simplified  if  we  first  transpose,  thus 

V{(i-\-x)  =  2  Vx~  '/(a  —  x)',  ^iow  squaring, 
a-\-x  =  4:X  +  a  —  x  —  4:V{ffx—x^),  or 
x  =  2\/ (ax  —  x^).     Again  squaring, 

x^  = 'iax  — 'ix- ,  whence  x  =  0,  or  — 

5 

3.  Clea^'  of  ra.'icals 

f'^x  -}-  ^^y + j^z  =  0.     Transposing, 

f/x+f/y  =  -  i^/z :  cube  by  formula  [0] , 

•''^+?/+Bi^/.r//f]^«+]^/.y)=  —z;  and  substituting /or 

■^x+^y  l.s  val'i;>  — -^/z,  t^'is  becomes 

x-\-y —  Q-^^xyz= —z,  or 

a;4-.y+z  =  3^^a;//z;    .*.  caLlng  again, 

(aj+y  +  s)^  =  21xyz. 

a+x+\/{'2rix  +  x^-) 
a+x 
Dividing  and  transposing,  we  have 

division  in  leflhand  member, 

+  1  =  (^,_1)   ...  _^=^^i_(/,_i)n,or 


(«+a-)2  '  a+x 

a 
—  4-  1   =  f!-r. 


SIMPT-T5    EQT'ATIONS.  153 

5.  Solve  A/(4a-3  4-19)+v/(4a;2- 19)=  a/47  +  3. 
We  have  the  ideutity 

(4.c2  4-19)  -  (4.r2  - 19)  =  38  =  47  -  9. 
Now  dividing  the  member?  of  this  identity  by  those  of  the  given 
equation,  we  have 

\/(4a:2  4- 19)  -  v/(4r2  _  19)  ^  ^47  _  3.    Adding  this  to  the  give«. 
equation,  then 

2v/(4«2  +  19)  =  2a/47,    .-.    43:5+19  =  47,  and  a.  =  ±  ^7. 

6.  f/{25+x)  +  xy{'lo-a')  =  2. 

Cubing  by  formula  [6] ,  (See  Ex.  8),  we  have 

25+x+25-x+G]V{252-x^)^8,  or 
1^(625 -«3)=_7,  or  (62o-a;2)=-343; 
.-.      x^  -  525+453  =  9G8,  and  a;  =  ±  22|/2. 

Exercise    liii. 

3     V{x  +  i)  +  x/(x-S)  =  7. 
2.    y(3^  +  l)  +  ,/(4.r  +  4)  =  l. 
S.  ■,/(2a-+10)  +  i/(2a:-2)=6. 

4.  i/(i}ix)  —  -i/(nx)  =  ni  —  n. 

5.  V  '{hx)  +V{ab  +  Ox)  =  y/x. 

0.    ^,+  y(,+  3)  =  ^_^-^. 

7.  V{ax-{-x'^)  =  i\-hx). 

8.  ^{Mx-2G)=   A. 

0.  y^x-y^{a+x)=  ^^ 

10.  h  +  x-x/{h^+x"-)  =  c^. 

11.  ■^/(8  +  .r)-|/x  =  2y^(l+3;). 

12.  ■i/(2a;-27a)  =  9|/a-|/(24. 


154  SIMPLE    EQUATIONS 

13.  ^/(l-,.)-f-^3/(i^^.)  =  ^/3. 

14.  ^{3  +  x)  +  ^{3-x)  =  -^7. 

15.  f^{.c  +  l)-f(x-l)  =  f'n. 

16.  ^{a+x)  +  f(a-x)  =  ^b. 

17.  f/{l  +  ^/x)  +  ^{l-j/x)  =  2. 

18.  ^/x-^/{a-^/{ax+x^-)}  =  }^^a. 

19.  Clear  of  radicals  -^/a-{-^b--^c. 

20.  Solve  x+i/(a^+x^)  = 


21 .  Clear  of  radicals  -\/x+  •\/y  +  y^a;  —  \/''n. 
Solve  the  following  equations  : 

22.  y\i  +  x)  +  ^/{\+x^-^{l-x)]=i/il-s), 

24.  ^{l+x+x^)  +  v\l-x+x^)  =  7vx. 

25.  i/{a^-x'-)+xi.^'{a^-l)  =  a^{l-x^). 

Oft =    -'^ — a. 

VM+c  n 

27.  'v/(2x2+5)+\/(2x2-5)=  \/15+v'5. 

28.  ^/(3.c3  +  10)+^/(3^•2-10)=  /17+ a/3. 

29.  V(3x2+9)-\/(3.^•2-9)=^/34  +  4. 

30.  ^/(3a-3&+a;2)^  ^(2a-26+a;2)=  A/a+ v'j;. 

81.  V(4a3  -352  -  2x2)4-/(3rt2  -362  -x^)  =  a+x. 

82.  Clear  of  radicals,  ir(2x)  - 1^(27/)  -i^(2z). 

33.  ■i/(rt+-T)  +  \/{«-  -x)  =  2.r  -:  i/{  '/  +  ^'(^'2  +^t') y. 

84.  y(^.  +  2,«)+,/(..-^-2„x)=    -^J^^^-^^■ 
86.    y{(2a+x)2-l-i2|+^f(2a-a;)2+i2}=2«.. 


SIMPLS    EQUATIONS.  155 

Art    XLIV.     Sometimes  a  factor  can  be  discovered,  and  the 
principle  of  Art.  XL.  applied. 

Examples. 

X  -a 

Factoring  we  have 

{x^+ax  +  a^)(x-^-ax+a'^)  ^  (^_  j)(^..,  _^^,^  +  ,,,)^ 
x  —  a 

or  x^  —  ax  +  a^  =  (x  —  a)(x—  b)  ; 

.*.    (a-^h  —  a)x  =  ab  —  a-,  and  x  =  (i  —   — 

6 

2     Sabc  b.i-  a"h-^  b^x       2n^-|-^ 

^^    ~    H   ^   {a-\-byi    ^    ^^^  a    '  {aVb)^' 

Transpose   —  and  factor,  then 
a 

ab    \  ^  ah 


J  ""   [  a\  {a+bfl  I 


f  Q         I         ^  "'1 

=   X   '  oc  +  . r 

=  x  I  3.    + 


(^bj2] 

ab 
a  +  b   ~ 
x+a  x  —  b  X — c  b-^c 


'{a—b){G-a)     {a—b){b-c)     {b-c){c-a)     {a- b)[b-'.:)[c -a) 
Add  term  by  term  the  identity  (Th.  iii.,  page  5-1). 
x—a  x—b  x—c 


{a-b}{c-a)  ^  {a-b){b-c)     ^   (b     c){c-a) 

2x  _                 b+G 

{a—b){c  —  u)  ~  (a—b){b  —  c){c—a) 
_    1       b+c 

''•  -^'^  T'  b^c 


156 


SIMPLE    EQUATIONb. 


4.  (x-j-a+b)^  +{a-\-h)^  —  {x+b}^  -  {x-]-a)S-{-x^  +a^+b^  =  abc. 

The  left  hand  member  vanishes  for  x  =  i),  and  .".  by  symmetry 
for  a  =  0  and  6  —  0;  /.  it  is  of  the  form  mabx  in  which  m  is 
numerical. 

Put  x  =  a  —  b,  and  m  is  found  to  be  6, 

.•.    the  equation  reduces  to 

Qahx  =  abc,    .".  and  z  =  ^c. 

jx  —  a\  3         x  —  2a  +  b 
5. r         =    -77-; — ;    let    X  —  b  =  7n,  x  —  a  =  n,    and     .". 

\x  —  bj  x  —  Ab  +  a'  ' 

vi  —  n  =  a  —  b,  then  we  have 

m^  n  —  {m  —  n)         2n  —  m 

n^  ?H  +  (wi — n)    ~    2m  —  n 

:.     2m^—nm^  =  2n^  —7i^m,  and 

2{m'^  —  71^)  —  mn[m^  —  n'-)  =  0,  which  is  divisible  by  m^  —  «', 
.•,     m^  —7i~  —  0,  or  m-{-n  =  0 ; 
'Bntm+n  =  2x  —  a—b  —  0,    :.    x  =  ^{a-\-h). 

1      ^3_4a;  +  2       1      a;3-4.i-+3     _2_     x^ -4x-\-B      5 
^-      y    x^-Ax-1  "^  (5      jj2~4a;_'3~"9"'    ^2"T4^36  =  18' 

Let  2/  =  .«^  —4^,  then  this  equation  becomes 

1      V/+2      1      v+3      2      ?/+3      5 

•^- -f  TT-    ' 5-77"    /^  =  Tu'    or  by  division, 

1     .     _1_         J^         _1_          2  ^      _    5 

"3"  "*"  v/"^  "*■  T  ''"  ^^  ~  y  ~  ^;rr6  =  i8'°^ 
112 

-^  — — -   _  =  0  ;  this  may  be  written 


y—1  y-'d  y-Q 

1111 

^  +  ^  =0'    •••    5^-15+8</-3  =  0,  or 

2/  =  2i  .-.  ^2_4;c^-2|,  or  «3_4^  +  4  =  4_|_2i, 
•    and  a;  —  2  =  +  §.     We  might  assume  (j3  —  2)  ^  =  ;y,  when  the  ^iven 
equation  would  take  th^  form 


SIMPLE    EQUATIONS. 

//-2           1      y-\           2      y- 

-    1 

5 

y—5          0      y-7          y  '    !/  - 

-10    " 

lb 

157 


And  reducing  as  before,  we  should  find 

</  =  6|  =  (x-2)2,  .-.  X-  — 2= +^,  as  before. 

Exercise    liv, 
i-     —f^ =    x'^ -{u-b)x'^-\-{a2  —  ab)x-{-a^b. 

2.    fi±±!.^   =  x-^  +  2a{a-b)x+{^a-b)x--'-2a^b  . 
x  +  b 

a^  +  uo+b--^    ~    a^-b-'i    ^    ll^b    ~    '^^■^' 

i.    ~r~    _  i-   _   1   _   i-    =    2ab\:x^b)xK 
a-\-b-{-x  a  b  X 

1  1 


{x-b){x-c)     '     [a+c){a+b) 
1  1 


(a4-c)(j;-c)   ^   (a+//)(a;-6) 

.     6x-           'dab             a-//-  6^0?       2a  — b 

^- r    +    7 TTT    =    oo;    - 


x^+2«-c  ^  — rt 

x*  -llj;2a>+a4    ~   X''^3ax-~a^" 

.      1   jjj  — rt\  ^       1/         .  I    ,i      ■<^-» 
o«.     —    — —       -  i(    -ui  +  u)=  . 

2  \^  +  rt/  ^+a 

3.  x^-{a  +  b+c)x+{a-2-{-b-^+c^)x-}{a3+b'^-\-c3] 
=  [x—a)(x — b)(x  —  c). 

io.  1  +   1   ,  1  =  Ua+k+c)^  -  i-fiL+A+ M. 

ax         Ox  ex  2   \ofa;     acx     abx' 

,,      l-r/.c  1-^.^  \-cx  12  2  2i 

11.    — ,--      + +     — ,—    =       —   -{-   —   4-   — \x. 

be  ac  ah  \a  b     ^      ^ 

]P,.  x^-\-{b-\-c)^  +  ^b{b-^t)x  =  b^. 


158  SIMPLE    EQUATIONS. 

14.  x—a-S^(abx)  =  h. 

15.  11x^  +  10x^-4:0x^116. 
X  ac  c 


16 


+ 


a  +  6)3     '     (a-b)^  {a^-b^){a  +  b)          {a     b)^ 

,„        „          a  —  h  2cx^                     a  —  b     l  —  cx 

a+b  l+6'.f;                    a-iro     l-i-cx 

2x-{-a 

19. ' +  '                                '"^ 


a;2_lia;+28    ^  x2-17.6'+70         x^-Ux  +  AO 
20-     -.-4-^  + 


x^-6x+5    '    x-2-14.i'+45         ft;2-i0x  +  9 
„,  x+a  x-b  x-{-c 

(a^b){c-a)    ~    {a-o){b-'c)    ~^   {h-c)(c-^) 

a-\-c 
{a-b){b-c){c-a)' 

22.  {x-aY-\-{a-b)^+{b-xy^=x^-ai. 

,,C+2rt\3  /«+2.c,  3 

23.  x\  +   a    [- =%i. 

\x  —   aj  \a  —    xj 

21.    {x+a)^  -{a  +  h)^  +  {}>  -  j^^  ={x  +  a){x  +  b)ia  +  b). 

25.  x^  -{x-b)^-{x-a-{-by-^--a^+{x-a)^  +  {a-h)^+b^ 


■b)c^ 


20.  (x+ 0^  -  {x+by^  -  (-c - 6)'^  -  (2^()-!  +  (:e-  o)3  +  (a  +  Z>)3  4. 

«-f-a  x  —  a  a^ 

^''^'    x^+'tx+a'^    ~   x^-(ix  +  a^    ~    a(Ti^'-^x^~Va^)' 

28.  {x  +  a+b)'^  -{x  +  a)'-  -{x  +  by+x*  -  (a  +  b)-^ +a^ +h^ 
=  V2'ib{x^  +  {u  +  by-}. 

a  —  x  b  —  x  c  —  x  Sx 

29.  -^—r.   +    i:¥—7..   + 


a^—bc         b^  —  c<(  c^ —ab  ab-j-bc  +  ca 

30.    x^{b  -a^)-{-((,^{x-b''i)  ■\-b'^{a~x-)  +ub.,{ab.c-  1) 
=  (a-x2)(63-a4). 


SIMPLE    EQUATIONS. 

31.  (l+x  +  X  =  j=     =^^.{1+U^+X*). 

^  '  ax—  i. 

32.  J|^^      -    ^~^     -1. 


159 


x-'b  '2f+6  +  c/ 

34    i/(.c2  4-27^-+ 180) --i/(a:2+26ic  + 168)=  ^f^±i^\ 

V\x+12  / 

35.  {(--a.  r|    (.<:2+2ax-  +  6-^)}3  +  {3;+a-  ,/(a;2  +  2aic+i2)|3' 

=  1'±(^   4-a)3.     (Sec  page  17,  Ex,  1). 

36.  {x+«+/(x2-2rtx-2Z»2)}-  +  {:t  +  a--/(a;^-2«ar-2i2)}2 

=   2_ft3  ..2rt(a     6). 

-b]   "    x--a-2// 
38.  (5x-7)*-(2,c-4)3  =  27(a;3_l). 

39     J_     ^^-6-^-1         JL     a:2  -  6a;  -  4  ^      a;2-6a:-7 

3  \,;2_tj^_4  +  y  a;a_6ar-9    ~    y   ■a;--Cx-i6 

14  4 


45     '     x^ — 6u;  — 9 
^^      1      x^'-'lx-?*  1      a;2--2x-15  2      a;2-2.r-35 

5      ^•-•_2.r-8   ^    y      :i;-'-2a;-24  13     a;^-2x-48 

_        2_ 
~     585" 
41.    {■  +  a-h         (   2     „2_i2);3_ 

a;+a-6-  v(,«2     a^  ~  b^)^  =  8{.c-\-a-by. 
1  1 


42. 


a)2  ^    -'     •       .-i)S 

1  1 

+ 


8x-     57          0,-68 
39, ~j-    -!-    5- 

\    x-i-A  x-to 


l60 


SIMPLE    EQUATIONS. 


44.  51    -  ._[_ Tl^    +863  = 

,     x-1  a;  — 4    / 

^^"0^=^    -      .-3-)- 

45.  ix-]-a){z  +  Sa){x+ia){x-^Ga)  =  x^  +  Qa^{x^  +7aa;4-6«s). 

46.  _i_      .        2  _  3  _    _6__ 

x+Ca         x-oa~   x+'2a         x+a 

Exercise  iv. 

1.  a{h  —  x)-tb{r  —  x)  =  h{a-x)  +  cx. 

2.  (a  +  hx){a—b)~{ax-b)  =  nl{x+i). 

3.  {a-b){x-c)  +  {a-{-b  {x-\-c)  =  2[nxi-ad). 

4.  (a  -  ^')ix-c)  -  (a  +  i)(a;  +  c)  +  2a{b+c)  =  0. 

5.  (a_6)(rt  _c)(a4-.r)  +  {('+h){a +  c){a-x)  =  0. 

6  (    ~b){a-c-^x)+{a  +  b){a-^c-x)  =  2a^. 

(solve  in  {a;— c}). 

7  (»;+fl  (a+6  — a;)  +  (a— «()(^  -a;)  =  a()H-f  6). 

8.  r;?(a  + '  —  .T)  =  n(a:  — a  — i). 

9.  {m+n)(m  —  n—x)  +  m[x  —  n)  —  n(x  —  m)  =  m^  -w'. 

_^     m  —  x        n  —  x        p—x 
10. [-   —  +  ^^^  =3. 

P  -^-=0. 

6  — a;         c  —  x 

12.  -y—  + +  -^=0. 

Oc  ca  ab 

1  —  ax         1  —  bx         1—  ex 

13.  --T— -    + +       —     =0. 

be  e.ii.  ab 

(Deduce  the  solution  from  that  ox  I>;o.  12;, 

a—  hx         h  —  ex         c—  ax 

14.  — ,—   + +  ——  =  0. 

be  ca  at) 

^  a  —  0  a-{-b  a—o 


SIMPLE   EQUATIONS.  IgJ 


Sahc  an^_  {2a+b)b^x   _    (b  +  Sac)x 

'    a-\-h   "*■   (a-^l)3    +     a{u  +  by^'    "       "^ 

,,     10  4  9  2       /  1, 

,8.   1   +  1  =   ^4zf  +  ^   _  1. 

a;  d  ox  12,         4:x 

19     I.  _^.  i!  =    2(5.^-12)    _   17         10 

3  iJx  8x  2U  ' 


X 


20. 
21. 


l^Tlf        2^+^   _   7£-f-266        4.C+17 
y       "^       7       "     x-t-21  21 


23. 


24.    r: ^  =  1. 

6-    ., — 

e- 

d- 

X 


25.  (.^-lXr-2)-(.c-3)(^-4)  =  3; 
(x-3)(x-4)  =  (..-2)(x-G). 

26.  2(x-4)(3a:+4)  +  i2.c-3):;::3x+2)-6(.c-2)(2u;-3)=0. 

27.  {a-x)[b-x)=x^  \  {a-x){x-b)=x-  -c'^. 

28.  (a-a;)(/-'+-c)  =  63-a;- ;  (-c-a)(u;-i)  =.c- -a^. 

29.  {a+x){b+x)  =  {a-x){b-x); 
{ax  +  6)(^x+a)  =  {b-ux){a  -  bx). 

30.  {a-x){^b-x)i-ia-c-x){x     i/+c)=0. 


Ifl2  SIMPLE   EQUATIONS. 

31.  {a-x){b-x)-{c-x){d-x)  =  (c  +  d)x-cd. 
82.  {x-a)(x-b)-(x-c){x-d)  =  {d-a){d-b). 

33.   {{a^  -b-2)x-ab}{a--ia+b)x}  +2ah''x  = 
{{a+b)^x+ab}{b-{a-b)x}. 

34  {x+l){x-\-2)(x+S)  =  (x~ 3){a;+4)(a;+5). 

35  {x+l)ix-\-2){x-\-d)  =  {x~l)ix-2){x-S)  +  3(z+l){ix-}-l) 

36.  (a;+l)(ar+4)(a;+7)  =  (a;4-2)(.^+5)3. 

37.  (a;+2)(x+5)2  =  (a;+3)2(x+6). 

38    (a;-l)(a;-4)(a;-G)-a;(a;-2)(aj-9)  =  lB6 

39.  (a+a;)(6+a;)(c+a;) - (a-x){b-x){c -x)  =  2{x^+abc). 

a;  — (i 

41    x{x-a)^  -{x~a-^  b){x-a+c){x-b-c)  =  (a^  +  hc){b  +  r'). 

42.  (a;— a+&)(a;-&+';)(a;  — c  +  rf)-a;-(ac-a4-^/)  =  5f(c^-«). 

43.  (a; -a+&)(a;-& +  (■)(«'- «+''0-^(^-"  +  '')(^-c+^) 
=  bc{d  —  a). 

44.  (ic-2a)(a;-2t)(a;-2c)-(x~a-6)(a;— &-c)(a;-c-«) 

45.  x^-{x-a+b){x-b  +  c){x-c+a) 

=  {a-i-b  +  c){a'^  +  b^+c^)~2{a^b  +  b^c+c^a)-3r,bc. 

\  « /   \  a;  /    \  X  I        x^  X 

47.  {x+a){x+b)+(x+c){x+ct)r=(x+b)(x+d)+(:r+d){x+c). 

48.  (rta;+i)(aa;  — c)  — a(&-a;)(</a;4-i)=«^(a  — t•)(^~^)- 
a(«a;— c)(6-— x). 

2a;- 3         3x-2   _    5x^,-29x-   4 
49-      2._4   +   lc_8    ~      K3-12a:+32* 

^"'ZL    _     3a;  +  2     _   « 2 -3^+2 
■    SfT^+Y)   ""    2(a;-l)    ~        6a;3-6 


SIMPLE   EQUATIONS.  163 

.,    ;b:-7  _  n(.v-+i) ii-^+3_ 

■'      :Lx-d   ~   2[x-\-S)    ~    2a;2-oa;-27* 
7x-r,         8«-7  lOx+7' 

ro I  4-     -  —  =5 

•    3x-'2  ^   3x-l    ^   y*2-lb;4-:i 

2.V+7     iix-{)     h{x-Jj  _nx-9.     5x-  S     '2.r 4-2 
^^'    'dx-'7'^'2x - 5 ^  yx -25  "  2x-6'^' ii^c-25  '^.  -dx-f 

4.x^-3z       3.C       4x3+2.1-    x  —  a    x-b      ^ 
54.    —^. -:; =— ^^- .    ; =0. 


55. 


66. 


<2,{x-l)         x+Q   _  3(5a;+16) 
^'^'     ~a:-^7     "^  a;-4    ""   ~6x-2d' 


68 


z,^     ox  +  h  bx  ax  (ax^  —  2b)b 


61 


cx  —  d  {hii  +  fhn)x  —  {hq-{-dp)  a  c 

vix  —  p         nx  —  q  {mx—p){nx  —  q)        ~    m  n 


po        tn  n  p  in  n  p 

02.    _L. +    -  = +    +    —    -. 

x  —  a         x-b         x-c         x  —  c         x  —  a         x—b 


J  64 


SIMPLE    EQUATIONS. 


go     ax  -  2a          ax  —  1h^    a            ^  _              x 

ax—2,b     ~    ax  +  2a  '•  "J"*""™"-^  '        ^^T 

1 a  +  — 

ax  X 


2x^  -3a; +5    _  2 

7a;2-4.j;  +  2    ^  T' 

??2uj2— ?2a;+^    ~  m  '    mx^  ~nx- +mx  —  q    ~    mx  —  n 

65.    iz^  +   1   =.  _^   _  J_. 

^+x   ^     4  ^-f-o;  4   ' 

f^-l  2  2      ,    |x+f 


l-x  b  3 


G6.    J^l Zl_   -      21 7^ 

a— 98         a; -94         .'^+44         a; -52 

^,,7                 8                 9  1 

67.    -—5  +   — TT   =   ^    + 


a;  — 6   "^  a;-ll         a;-7         x-12' 

9 9 2_  2 

a;_51         a;-15    ~    x-81    *"  a;+81* 

_5_  _4 8_  1      . 

^^'   x-Q   ■*"  a;-9   ~  a;-7   "^  a;-10' 


a;— 6         X— 3         a;  — 2         a  — 5 

w.  —  n  rf  —  h 

69. 


70. 


w  —  n 

a-h 

~      X  —  h 

x—n 

h+fl 

x  —  a  X  — m 

a  +  b         rt  +  c  b+fJ  c-i-d 


x  —  b         x—c         x—{a-\-b+2c  +  d)         x-[a+'ib  +  c-\x> 

71.  {x-a  +  b)^  -{x-a)^  +{x-b)^  -x^  +a^  -{a-b)^  -b^ 
=  (a  — i)c2. 

72.  {x+a+by  -{a  +  by  -{x+hY-{x-\-aY+x^  ^a'^^b'^ 
=  lQabxl^lx^a  +  b){x-ira  +  b). 

{m-n){x-a)         {n-r){x-b)         {p-m)(x-c)     ^ 

^^-         b+c        +        t:^        +        a+b  "=^' 


SIMPLE    EQUATIONS.  165 

ax—1  l>x~l  cx  —  1  dx 

74.  z^t—tta  +  n 


n^r  +  h)   ^  h2{c+a)     •     c^{a+b)         ab  +  be -j- ca 
x~2a  x-2b  x  —  2c 

h  +  c  —  a   "^   c+a-b         a  +  b—c~'^' 


x~2a  x-2b  x-2c  Hx 


b-^c-a         c+a—b'a  +  b—c         a-\-b+c 

a-x  b~T  c  —  x  rf 

(7.     .o — 7-_    + 


a^  —  be         h"  —<  c     '     c^  —  ah         a  4-  b  +  c* 

x+2ab  2at)-x     _      x~2ah  ji:^2ah 

a-\-b-\-e  b-\-c  —  a    ~    a—h-\-c         a  +  b^^' 

a  b  a  —  e  bA-e. 

79     4-  =    •  _  —  — . 

x-\-b—^         x-\-a  —  c         x+b         x-\-a 

80.    — +   — ^  +    — + 

x~m  x—n  x—p 

q  [pd.  +  {n—p)c-{-{in  —  'n)h—  ma] 

x  —  q  ~ 


81. 


(a;-2)(x-5)(a;-6)(a;-9)  +  (a+2)(a-4)(rt-5)(q-ll) 

X 

(6+l)(^>  +  5)(i  +  8)(^  +  12)  .        ...       ^.,       ^^^ 

^.v_L_A '^ n /    ^    (.T-4)(a:-7)(a:-ll)  + 

X 

(a8-l)(a-8)(«--10)4-(&  +  2)(6+3X^>  +  10)(fe+ll) 

X 

Art  XLV.  Employing  the  language  of  algebra,  the  princi- 
ple illustrated  in  Art.  XL.  may  be  stated  as  follows  : 

Definition. — Any  quantity  which  substituted  for  x  makes  the 
expression  f{x)  vanish,  is  said  to  be  a  root  of  the  equation  f{x)  =  0. 
Thus,  if  a  is  a  root  of  the  equation /(a;)  =  0,  then /(a)  =  0. 

By  Th.  I.,  if  a* -a  is  a  factor  of  the  polynome  f{xY,  then 
/■(a)"  =  0,  and  a  must  be  a  root  of  the  equation  f{xY  =  0  ;  hence  in 
solving  the  equation  we  are  merely  finding  a  value,  or  values,  of 
X  which  will  make  the  corresponding  polynome  vanish.  Sup- 
po.se/(a;)"  =  (a;-a)<?)(x)"~^  =  0,  we  are  required  to  find  a  value,  or 


166  SIMPLE    EQUATIONS. 

values,  of  x  which  will  make  (a;  — «)?)(a;)"~^  vanish.  The  pciy- 
nome  will  certainly  vanish  if  one  of  its  factors  vanishes,  whether 
the  other  does  or  not,  and  will  not  vanish  unless  at  least  one  of 
its  factors  vanishes.  Hence  {x—a)<^{xf'^  will  vanish  if  a;-a  =  0, 
quite  irrespective  of  the  value  of  f  (a;)"~^  Also,  if  ^(a:)"~^  =  0,  the 
I)olynome  will  vanish,  irrespective  of  the  value  of  x  —  a.  It  fol- 
lows, therefore,  that  if  /(ic)"  can  be  resolved  into  two  or  more 
factors,  each  of  these  factors  equated  to  zero  will  give  one  or  more 
toots  of  the  equation  /(x)"  =  0. 

When  there  can  he  found  two  or  more  values  of  x  which  satisfy 
the  conditions  of  given  equations,  tliey  are    sometimes   distin- 
guished thus  :  a^i,  JCg,  iKg,  &c.,  to  be  read   "  one  value  of  aj,"  "a 
second  value  of  a;,"  "a  third  value  of  a;,"  &c.     Thus,  if 
(x  —  a){x  —  h){x  —  c)  =  0, 
.'.  XY=a,  x^  =  h,  x^=c. 

Examples. 

1.  Solve  2a;3-13a;2+27a;-18  =  0. 

Factoring, 

(x-2)(a:-3)(2a;-3)-0, 

2.  x^-{a-^h)x+{rt+c)h  =  (a-ic)c, 
:.    x^-{a  +  b)x-\-{a  +  c){b-c)  =  0, 

:.   a;2-{(«-fc)+(i-c)}a;+(«+c)(6-c)  =  0, 
...    |^_(a+c)}{a;-(&-c)}=0, 
.".    X'^=a-\-c,  x^=b—c. 

3.  x-{a~h)-\-a^b-x)  +  h^{x-a)  =  0. 

.-.  x^{a-b)-x{a^-h^)-^ah(a-b)  =  0,  \ 

.-.  {x-a){x-b){a-b)  =  0. 

If  a  —  b  =  0,  the  given  equation  holds  irrespective  of  the  values 
of  a;  — a  and  x  —  b,  and  therefore  of  the  values  o{  x;  but  ii  a~b  is 
not  zero,  x^  =a,  x^  =f>- 


SIMPLE    EQUATIONS.  167 


4  _    ia^+b^)x-(a^-b^)_ 

.T+1         a-{x—l)  /■'^+-'-\  "       "^ 


0. 


X  = 


■1    ~    b^x  +  l)      ••   \x-l}  h'    ~    ^' 

'  '    a;^  —  i  6     ~  '  *      ^    ~   a  —  b' 

x^-\-l  a  ^  a  —  b 

-^-^   +  =    0    .'.    a;o    =    —-r 

Xg  — 1  h  ^         a  +  6 

(a-a;)g+2(a-a;)(6-a;)  +  (6-.r)2    _      2(49) -34 
\ri-x)'-i{ii-x){b-x)^{b-x)'^    ~    3(34)-2(49) 
((a-x)-{-{b~x)]^ 


=   16, 


,       —     -''•2      =     0 

[{a-x)-{b-x)] 

{a-x^)  +  {h-x^) 
.-.     r —    4    =    U, 

a  —  b 

{'l  —  X..)-{-{h  —  X.-,) 

^ -^_-^ ^~    +4   =   0,    .-.  ajs  =  4(5«-36). 

g     (g-cr)(3;-6)         (x-5)(a;-fi)   _ 
•    {e-a){c-b)   "*"  (a-i)(«-c)    ~ 

Subtract  term  by  term  from  the  identity  (See  page  53), 

{x  —  a){x—b)  {x  —  b){x  —  c)  {x—c){x  —  a) 

lc-a){c-b)   "^  {a^b){^c)    "^    {h-c){b-a) 
..    {x  —  c)(x  —  a)  =  0,    :.    x^—c,  x.^=a. 

7.  Find  the  rational  roots  of  x^  -  12..-3  +  51a;3  -90x+56  =  0. 

Factoring  the  left-hand  member  by  the  method  of  Art.  xxviii., 
(ic-2)(a;-4)(^--^-Gx  +  7)  =  0 
a;i=2,  x-3=4,  ora;"— Ga;4-7  =  0. 

Since  x^  — 6^+7  cannot  be  resolved  into  rational  factors  we 
know  that  it  wiU  not  p'ive  rational  roots,  .".  Xj  =  2,  Xg  =  4  are  the 
only  vaiueti  that  meet  tiie  condition  oi  the  problem. 


168  simpJjE  equations. 

Any  literal  equation  of  the  second,  third,  or  fourth  degree,  ana 
many  equations  of  the  higher  degree  can  be  resolved  tnto  a  series 
of  disjunctive  equations.  A  full  analysis  for  the  first  four  degrees 
will  be  given  in  Part  II.',  meanwhile  the  following  special  forms 
of  the  Theorem  in  Art.  XLV.,  will  enable  the  student  to  solve 
nearly  all  the  equations  commonly  proposed. 

(A).  In  order  that  two  expressions  having  a  common  factor 
may  be  equal,  it  is  necessary  either  that  the  common  factor 
should  vanish,  or  else  that  the  ppoduct  of  the  remaining  factors  of 
one  of  the  expressions  should  be  equal  to  the  product  of  the 
remaining  factors  pf  the  other  expression,  and  it  is  sufficient  if 
one  of  these  conditions  be  fulfilled.     In  symbols  this  is 
li  {x—a)f(x)=J^x—a)ip(x),    :.    x^  =  a  or  f{x)  =  (p  (x). 
(B).  If  an  equation  reduces  to  the  form  [mx  +  n)'^  =c* 
:.      {mx+n)2-c2=0, 

c—n 
m 

—  c  —  n 
lit, 
(C).  If  an  equation  reduces  to  the  form 

(vix-^n]  2         ii''^ 
\px  +  q)      ~    b'^' 

qa  —  nb  —  qa  —  nb 

thenic,  =  — i '   «o    =    — ,  , (See  Exs.  4  and  5  above). 

^       mb—pa       ^  mo -{-pa       ^  ' 

(I)).  If  an  equation  appears  under  the  form 

{a  —  x){x  —  b)  =  c,  (1) 

then  Xy  =\{(i-\-b^r),  x.^  =\{ti-\-b  —  r), 

in  which  r'^  =  {ci  —  bY  -  4^'- 

From  the  identity  (a  —  x)-\-{x  —  b)=:a  —  b 

we  get  {a-x)^+2{a-x){x-b)  +  {x-by  =  {a-b)^  (2) 

(2) -4(1)  .-.    {a-x)^-2{a-x)[x-b)  +  {x-by 

=  ia  —  by^  —  4:c~r'^  say 

.-.    {[a-x)-{x-b)]'^-r^  =  0, 

:.    {{a—x^)-{xi-b)]-\-r  =  0,  and  .-.  jj^  =4-(rt+ft  +  '-") ; 

or  {{a-x^)  —  {x^—b))-r  =  0,  Q.n(i  .-.  x^=^{a-\-b- r). 


{_)nx^-\-n) — c  =  0  and  .".  x^ 
or     {mx.,-\-n)-\-c  =  0  and  .".  x^ 


SIMPI.E    EQUATIONS.  IG'J 

ft  ,        1  1  11 

X  a  ax 

...  = Applying  .(.4), 

1  ax 

.•.  X  — a  =  0,  or  rtx  =  l> 

1 
a 
0.  {x-{-a  +  b){x  +  h  +  c)  =  {x-Za^h)y'lx-da^1b-c)', 
x+a-{-b  2x-3a+2i-c 


x  — oa  +  6  x-\-h  +  c 

X  —  Aa-\-b  —  c 
~  'da-\-c 

2(a;  — a+6)  x-a->rh 


Page  122,     (5). 


*■      x—Qii-{-b  3a4-c 

,-.    (J)     x^-a  —  h 

\{x.^—3a-\-b)  =  'd<i'{-c    :.    x^=9a-b-\-1c. 

10    ^  ^ — —  =  — >    .'. — = n 

x^  -  'Ix  b  m{x  +  -l)'->rn[x^-1x)         ma+nb      ^^ 

But  (C)  cau  be  applied  if  //t  aud  ?i   are  so  determined  that 
vi{x+2,)^  +  n{x^  —  '2x)  is  a  square. 

Tills  requires  that  4im[vi+n)  =  {2i)i,  —  n)^, 

.*.    S)n  =  n. 
Assume  m  =  l,  then  n—S,  and  (1)  becomes,  on  substitution  and 
redaction, 

(g  +  2)2 a__    ^ 

(c3x— 2)3    -    a  +  86~'    '  '^^y 

_  2(L-M  _2(/-^l) 

•■•  ■^'^   ~  V-1  '    •^■'~"'l  +  3/-' 
(a;  +  l)^  _   _1  (•g^+2.c  +  l)3         _   _« 

^^'    (x3  +  l)"(x-l)2    ~     b'        •'•   {x'' +l){x^ ~'lx+l)    ~    b' 
For  .<;-  -|-1  write  xz 

(.t-g4-2.f)3     _     a_       ^      (z-t-2)3    _     a_ 
xz{xz  —  '2xx)    ~     b       '  '     z[Z  — '1}  b 


170  SIMPLE    EQUATIONS. 

This  'equation  was  solved  in  Ex.  10,  hence  z  may  be  treated  as 
known. 

^     ,    X2  +  1                                   .a;3_[_2a;+l               2  +  2 
cut =2,        .•.  -s X — — r     = Fj- 

-  — ,         =        >c,  a  formed  solved  in  (C). 

12.  (a-a;)4  4-(6-a04-c.  , 
In  the  identity 

LetM  =  ffl — X,  v  =  x  —  b,  .•.  if+i'  =  rt- 6  and  M^+'iJ*  =  c, 

.;.  {a-h)^^c  +  i{a-by{a-x){x-b}-2{a-x)^x-b)^ 
Write  z  for  {a  —  x){x  -  b) 

.-.    z2-2(a-i)2z  +  (a-6)4  =  i|c  +  (a_i)4}=:«3^  say, 
,'.     ■!z-(a-&)2}2  =  j2 

.-.    by  {B)         Zi  =  {a  —  b)^-t;z2={a—by  +  t,.:  z  is  known  ; 
But  {a  —  x)(x-  b)-z 

.'.    by(D)         a:,=l(a  +  h+)^;x^  =  U^+b-r)  (1). 

in  which  r^  =  {a  —  b)^  - 4z, 

or    (a-b)^  -  4{{a  -b)^  +t}  =  -it-  'B{a-b)2 1  ^^ 
and«2  =  i{c  +  (a-Z^)4}.  (3) 

Hence  x  is  expressed  in  terms  of  a,  b,  and  r, 

r  is  expressed  in  terms  of  a,  b,  and  i, 

t  is  expressed  in  terms  of  a,  b,  and  c, 
i,nd  the  expressions  for  r  and  t  are  cases  of  (B). 

13.  (a-a;)(6  +  :c)4  +  (a-a;)4(6+a:)=a6(rt3-f/;3) 

Let  a  — a;  =  « -z  and  6-|-a;  =  ?i  +  z    .".    n  =  ^(u  +  b)  (!)• 

The  equation  reduces  to 

(»2_z2){(w 4  z)3_|-(n-5)3}=ai{a34-6S) 

/.     (n2-22)(2w3  +  G«z3)  =  «i((f3+i3) 


SI?.tPLE    EQUATIONS.  171 

z~  may  now  be  fonud  by  (/)),  and  from  the  result  z  may   be 
found  by  (/>'),  and  from  (1)  x--\{ti  "h)+z  ; 

82^  =  -I (a  -  b) 2  or'  I ( 1  Oab  -a^-b^) 
.'.  x  =  0,  01- a -b,  ov  l{n-h)  +  ^x/(3()ab-'6a^-dl>^). 

14.  {y{„^x)+  y{a-x)\^V{o  +  x)+  y{a-x}=2cx. 
Divide  tlie  terms  of  the  identity 

\/{a -f  x) 4  -  y (<f -x)*  =  2x 
by  the  corresponding  terms  of  the  equation, 

•'*  \  U- J   ~  f-i'   ■*'  c-x  ~   \c-i/  ' 

(r  +  l)4_(c_l)4 
••     ^     -     «-(c4-i)4  +  (c_])4- 

15.  f/{'>-xy-  +  f/{{a-x){b-x)}  +  f/{b-xf  =  f/(a2+ab  +  b^) 
Divide  the  terms  of  the  identity 

f/(a-x)^-f^{b-x)^=a-b 
by  the  corresponding  terms  of  the  equation. 

Cube,  using  the  form  (u—v)^  =u^ —v^—Buv(ii  —  v). 

a-b 


(a-x)-{b-x)-  S^{{a-x){b-x)} 


ir(«2 +«*+>) 


(a~/>)3  _  _8ab{a-b) 


~    a^-hab  +  b^                           a^j^ab  +  b- 
.-.  ^Ua-x)ib-x)}= 

:.   (a  -x){b  —  x)= 

a  form  solved  m  (D). 

^^-       V{a-x)-^/{x-b)     ~   ^^ 

Assume  -i/{a  -x)=z  ^/  {x  --  b) 

.:  {a-x)  +  (x-b}  =  iz^-{-l){x-b). 

a-b 
,*.  X-  b  =     .,-7^- 


172  SniPLE    EQUATIONS. 

The  proposed  equation  now  becomes 

{x-b){z-iy 


(Z  +  1)4  C 


a  form  solved  in  Ex.  11. 


••     (22  +  l)(«-l)2         a-// 

17.   {x-2){x-5){x-Q){x-9)-L(;j  +  2){y-i){y-5){y-ll)  + 
{z+l)[z  +  5){z+S){z+l2}=x{x-i){x-7){x-ll)  + 

{y-{.l)(y-l){y-8){lf-10)-\-{z-j-2){z  +  B)iz  +  10){z  +  ll). 

Let  a: '  =  3:3-11.7:,  y'  =  y^-%y  and  «'  =  22 _|_132, 
.-.  (a:'-|-18i(a;'+30)+(?/'-22)(7/'+20)H-(z'+12)(z'+40)  = 

^'(:«'+28)  +  (y'-10)(2/'  +  8)+(z'  +  22)(5'  +  30) 
...  a;'2  4-48a;'  +  540  +  ?/'2-2?/'— 440+2'2  +  52s'  +  480  = 

a:'2  +  28a:'  +)/'3-2//'-    80+z'2  +  52z'  +  660, 

.♦.  20x'  =  0,     .-.     a;2-lla;  =  0,      .".      iCj  =0,  jCg  =  H. 

Exercise    Ivi. 
What  can  you  deduce  from  the  following  statements  ? 
1.  A'B  =  0.     2.  A-B-C=0.     3.   (a-i)^  =  0.     4.  12a;!/ =  0. 

5.  What  is  the  difference  between  the  equation 

(a;-5?/)(a;-4?/+3)  =  0 
and  the  simultaneous  equations 

a*  —  5^  =  0  and  a;  —  4?/  +  3  =  0. 
What  values  of  x  will  satisfy  the  following  equations  ? 

6.  x{x~a)  =  0.     7.  «a:(a;+i)  =  0.     8.  {x-a){hx-c)  =  0. 
9.  fla;2  =  3ra.      10.  x^  =  {a-\-h)x.     11.  .tO-rS  _a2)  =  0. 
12.  rt2a;3  =  ft2a;.     13.  a;^ +(a-a;)2  =flr2. 

14.  a;2+(a-a;)2  =  (a-2.r)2.     15.  {a-xY  +  {x-bY  =a^ ^h^ . 

16.  {a—x){x-h)  +  ah  =  Q. 

17.  (a-x-)2-(a-a;)(a;-?>)  +  (a;-ft)2=a2+flZ,  +  63. 

18.  x^-{a-h)7-'-(0'  =  0. 

19.  a-3-(rt+6+c)a;2  ,    a&-f  6c+m)a;— a^c  =  0. 


SniPLE    KQUATIONS.  173 

If  ar  must  he  positive,  what  value  or  values  of  z  .vill  satisfy  the 
following  equations  ? 

20.   (x-rj)(x-Jri)  =  0.     21.  a:2  +  29x-30  =  0. 

22.  a;2-17.c-84  =  0.     23.  3a;2 +  10x+3  =  0. 

24.  rJ -13x^+36  =  0.     25.  a;3 -2a;2  -  ox+6  =  0. 

Solve  the  following  equations  : 

26.  (a-xy-+(x-by={a-h)^. 

27.  {a-x)^-{a-x){x-b)-h-{x-hy  =  (a-h)i 

28.  a2(a-x)^=b2{b-xy.     29.   a2{b~xy^  =b2(a-xy. 

80.  {x~a)^+(a~hy  +  {b-x)^=0.     31.   (x-1)- =a(rc3 -1). 
oQ     rt  — a:         x  —  a         o^     «+&-«         a  —  c-\-x 
x  —  b         c-{-x  a  —  c  —  x   ~   fl-f-c— a; 

34.  (x-a  +  b)(x-a+c)  =  {a-b)^-x^. 

35.  (x-fl)2-i24-(«-f6-a;)(6+c-a;)  =  0. 

36.  (a+/>+c)j;3-(2a+6+c)a;+«  =  0. 

87     ^+^~^   _    fl  +  6  — c 
c  a; 

38.  (^-x)2-f(a-6)3  =  (rt  +  6-2a-)2. 

39.  ar(a+i-a;)+(«  +  ft  +  Of  =  0. 

40.  {n—p)z^  +  (p—m)x+m  —  n  =  0. 

.,      rta;2— &a;+c    _     c  .„      ax^  —  bx-^c    __    a-h-^e 

nix^  —  nz+p  p  mx^  —vx-tp         m  —  n-}-p 

43.  4a;2  +0-2  _  ^2  _  2(a  +  h)x  =  {a- x)(/;  +  x)  -  {a-\-x){b-x). 

44.  (2ffl-fc-a:)2+9(«-&)2  =  (rt+?)-2a;)2. 

45.  {2a-\-2c-xY  =  {2b+x){Sa-b-{-Sc - 2x). 

46.  (3rt-5Z^+x)(5rt-3i-a;)  =  (7a-6-3a:)2. 

47.  (3a-5+.r)(33  +  />-a:)  =  (5a+3i-3a;)2. 

48.  a{a-b)-b{a-c)x-{-c{b  —  c)x^=0. 


174  SniPT.E    EQUATIONS. 

49.    (ab-^hc  +  rn)(x^  +  x-\-l)-\- [n—b]^  =  {2ac -^b2){x^  +  x  +  1) 
-T  (a  —  r)^x. 

no.    (,r  +  l)(.';  +  3)(.r-4)(.r-7)  +  r.r-l)(.'?;-3)fx+4)(r  +  7)  =  96. 

51.    (x-l){.c  +  S){x-b){x-i-d)  +  {x-{-l)ix-'6){x+5)x^v}-t-  IB 
=  0. 


52. 

1 

a;-f    —   =  3i. 

X 

53. 

1          a+h         a-h 
X           a—0          u+b 

54. 

1            a 

X            b 

b 
a 

55. 

a  +  x         ^>+^_^-^ 
b  +  x         a-\-x     "2 

66. 

a—x          x—b 

X—  b          a — X    ~ 

13 

67. 

a  -X         b-\-x 
b-i-  X         a  —  x 

m        n 
n        m 

58. 

a            X            m 
X      '     (.     ~    n 

59. 

x^  -\-ax+  a^ 
x^  —ax+(i^ 

60. 

x-+n^ 

61. 

X-  +^'^ 

X-  —(ix  +  a~ 

62. 

ia  —  xMx  —  b) 

5 

63. 

a—x         x—h 
x—b         a—x   ~ 

m. 
n 

64. 
65. 
66. 


(x  +  a)^+{x-b)^  a^  +  b' 


{x+ay-{x-by  2ab 

(a  —  x)^  —(x  —  b)^  iab 


{a-x)ix-b}  (a^-b-) 

(a-x)^  +  {a-x)(x-b)  +  (x-h)2    _   49 
{a-x)^ -{a-x){x-b)~+{x^b)^    "    19" 


2a^  +  n{a-x)-\-{a  +  x)^  c  +  1 

67.  o  9  I — 7 — I — TTT ^   =   ?•    (Also  for  c  =  5). 

'za^-\-a{a+x)  +  {a—x)^  c  —  1      ^  ' 

68.  (5-a;)4+(2-a;)4  =  17. 

69.  x4  +  (a-x)4=c;  a;4  +  (a;-4)4  =  82. 

70.  {n-xY^{x-bY  =  {a-bY.     71.   {a -xY^-{x-lY  ^c. 

72.  x-^^{a-xY^a^  ;  ^- "•  -^ ,  6  -  a;) -^  =  1056. 

73.  (a-a;)^(a;-fc)2  +  (a-a;)2(a:-6)3  =an^(a-h). 


SIMPLE    EQUATIONS.  175 

7-k.  {a-x){h+x)^  -^(a-x)^{b  +  xy-  +  {a-xy^{b-{-xy^  + 

^"-  {a-xy^+{x-by'  -  20  ^"-^''^• 

(a-x)^+{x-hy    _   211 
"^G.    -^^_^)4  +  (^_i)4    -   1,7-  («-^)- 


7/. 
78. 
70. 


(a-.X-)4_|,(a;_Z,)4     ^     a4+fe4 

(a  — a;)5  +  (a;  — ft)^         a*— i 


(a-x)^          (b-x)^  ff3          ^3 

80.    —, 4-   ^ =    -T   +   — 

b -X              a—x  b            a 

a—x               x—b  a            b 

{n-xy  +  (x-b)i'    _  a^h^ 

{a  +  b-'2xj^      ~  [a+b)^' 

(a-x)-^-{-(x-b)^  a^-b'^ 

{a  +  iy^x)"~     ~  0H^T)3' 


82. 


83. 


(a-x)  — (ic-i)      ~  («— a;)(x  — i) 

^- A__Z; 1      =  c(a—x)(x  —  b). 

87     {«-^)^M^-W    ^  _      J 

(a-xj-^+liC-i)*  (a-x-)(a;-i)' 

88.    ^l+x2)»  =  (x3-3)3. 


85. 
86. 


176  SlilPi^E    EQUATIONS. 

x^  +  1       _    a_         QQ         (a;  +  l)^(a;S4-l)  ± 

2^iy^4-l)    -    6'  '     {x-l)Hx^--x-\-l)    "     6 

91        (^-1)'-^       _   i!.         g..      _(5c^±^+l)i_   ^   ^ 

•  (a;2-a;+l)"2    ~     />  '  "'    (a;+l)3(a:2 +  1)  6' 

93     >!+llL   _   iL         94     i^tm    =   ^ 
x-(a;+l)2  ^  '  ■    x(a;-+l)  b 

g-     ic(x  +  l)3  (J  a;2_^  2.4.1     x^x—1    _     a 

~{x-l)^   ^   T'  '     (a;+l)-     ■  I^-l)-~    ~     b 

97     ^^~a;^  +  l    _   _«_         93     x(x^  +  l) a_ 

(x^-l)^   ~     b'        '    '    (x'^-l)-    ~     b' 

99     (^  +  1X^^  +  1)         ^  iQQ      (^  +  lH.g^-l)     ^   _^ 

*  (u;-l)(x3-l)    *     6'  *    '{x-l){x'^  +  l)  b 

101      (-^  +  ^1'    =   -^  102     ^^+^)'    =    ^. 

x^  +  l  b'  x^-hl  h 

103.  2(a-a-)4-9(a-a;)3(x-i)  +  14(a-x)-'(.i;-6)3- 
9(a  -  «)(.«  -  i)3  +2(x-  i)*  =  0. 

104.  4(a-a;)*-17(a-u;)  =  (a;-6)2+4(a;-/>)4  =  0. 
Find  the  rational  roots  in  the  following  equations  : 

105.  x-t  - 1 2^3 -H40.c--78u;+ 40  =  0.  [lucx.  z^x-^  ~Zx], 

106.  x'^-Qx^+lx^+Qx-S. 

107.  x4- 10x3 +  35x3 -o0x+24  =  0. 
,108.  32x^-48x3- 10x3 +  21x+ 5  =  0. 

109.  x3- 6x3 +  5x4-12  =  0. 

4                9                 4  5^ 

a         ic  — 2a         X— oa  x— 4a 

5                4_    _       14  5 

x-rO         X— 4         X— 55  x  — 40         x- 

x+8a  X 

X  — a  X— 2a    ~ 

a;+5t/         2x  — oc* 


110. 

5               4 

X              X- 

111. 

14 

x+20   "^ 

112. 

2x4- 5a 

X 

X  —  (I 

<c  —  '6a 

177 


113. 


x-^ri  x-f2  x+4:  x4-S     _     x-1  x-S 

x-\-'2     '        X  x  —  \    ~    X — 2  x  —  'd         x  —  o 


,.,      1  31  20  8  20  31       , 

114.      _     4- _L      — _L _      -U 

x  x-l   ^  x-2  ^   x-'S  ^  x-4:         x-5     ' 

x  —  b 

116  i/(^i±^Hv>izM  = 

l/{a^-^2x)-'-i/{a-^-2x) 

??>-x3      j/(  OT2£-f2H-i/  ( m  2  .r  -  2  ^ 

117.  '/(a.'2-fl.2)4-^(x2_i2)  +  ^(a;3_^3)  =  ^. 

118.  {V(a-a-)+V(&-ia;)}{V(«-iK)-V(6-x)}»«. 
119     r('^-x)-f^(x-b)    _   a  +  b-2x 

■^^{a-x)  +  f/{x-b)  a-h 

120.  V(a+x)  +  V(a-a;)=y(2a). 

[Write'  u  lor  -^(a  -  x),  and  v  lor  f /(x  —  6)]  . 


178 


SIMULTANEOUS    EQUATIONS. 


CHAPTEE  VI. 


Simultaneous  Equations. 


Art.  XLVI.  There  are  three  general  methods  of  resolving 
simultaneous  linear  equations,  1°  by  substitution,  2°  by  compar- 
ison, 3°  by  elimination.  The  last  is  often  subdivided  into  the 
method  by  cross-multipliers,  and  the  method  by  arbitrary  multi- 
pliers. 

In  applying  the  elimination-method  the  work  should  be  done 
with  detached  coeiEcieuts,  each  equation  should  be  numbered, 
and  a  register  of  the  operations  performed  should  be  kept. 


Ex.  Resolve 

v-j-v+x  +  y+z 

=  15 

u+2v  +  4:X+8i/-{-16z=^. 

57. 

u+3v+9x-^ 

27: 

/  +  81z  = 

179. 

«-Mi!-hl6x+64//-f- 

256z 

'  =  45S 

, 

tt-f  5t'+25x-fl 

25?/-f62 

:52  =  9^ 

■5. 

Eegister 

71 

V 

X 

.'/ 

2 

1 

1 

1 

1 

1  = 

=    15      (1) 

i 

t 

4 

8 

16 

57     (2) 

1 

3 

9 

27 

81 

179     (3) 

i. 

4 

16 

64 

256 

453     (4) 

1 

5 

25 

125 

625 

975      (5) 

(2)-(l). 

1 

3 

7 

15 

42     (6) 

(3) -(2). 

1 

5 

19 

65 

122     (7) 

(4) -(3). 

1 

7 

37 

175 

274     (8) 

(-5) -(4). 

1 

9 

61 

369 

522      (9) 

!7)-(6). 

2 

12 

50 

80    (10) 

(8) -(7). 

2 

18 

110 

152   (11) 

(9) -(8). 

2 

24 

194 

248   (12) 

(11) -(10). 

6 

60 

72    {!?>) 

(12).  (11). 

6 

84 

9G    (14) 

(14)- (13). 

24 

24    (15) 

(15)-f-24. 

I 

1    (16) 

^{(13)-60(lfi)}. 

1 

2   (17) 

^[(10)-{12(17)  + 

50(10)}]. 

1 

3    (18) 

(6)-{3(18)-4-7(17 

l  +  iT;!  :(?)}. 

1 

4    (19) 

(l)-{(19)-f(18)  +  (17)-i-(lG)|. 

] 

6    (20) 

SIMULTANEOUS    EQUATIONS.  179 

An  examination  of  the  Register  will  show  how  easy  it  would 
have  heeu  to  shorten  the  process,  thus  (10)  is  (7)  — (0)  which  is 
(3)  +  (l)-2(2);  similarly  (11)  is  (4)  +  (2)-2^a;;  .-.(1^)13(4)  + 
8(2)  -  3(3) -(1),  &c. 

A  general  systematic  arrangement  of  *^he  elimination-method 
will  be  given  in  Part  II.  For  two  or  three  simultaneous  ec[ua- 
tions  it  may  be  stated  as  follows. 

a^x+b^ij+c'i  =0 
<i<^x-\-l).2!/-{-c.-^—0. 
Arrange  the  coefficients  thus — 

«1        />!        Cj        «j 

tto  ^3  Cg  rtg. 

Form  their  products  diagonally  from  left  to  right  downwards, 

thus "l^2        ^1^2        ''l^S- 

Form  their  products  diagonally  from  right  to  left  downwards, 

thus —  ^l«2      "^l^S      ^1^3. 

Subtract  the  latter  products  in  order  from  the  former,  thus — 

a^b^—b^a^,     b^c^—Cyb.-,,     c^a^—a^c^. 
Divide  the  2°  and  3^  remainders  by  the  1°  remainder,  the  first 
quotient  will  be  the  value  of  x,  the  second  quotient  will  be  the 
value  of  y. 

[Writing  R-^,  A'g,  h^  for  the  three  *  remainders  '  respectively, 
the  general  result  is  {tnx-i-nij)R^  =mR^  +nR^  . 
Ex.  1.  Solve  11x4-5^-68  =  0 

6x-ry+Bl=0 
11  5       -68         11 

0-7  31  0 


-77 

155 

-408 

30 

476 

341 

-  lijT) 

-321 

-749 

3 

7 

1«0 

SI 

JIULTANEOUt 

Ex.  2.      1? 

X 

25 

y 

_    1 

22 

X 

80 

+  — 
y 

=    17. 

12     -5 

15 

-   1         12 

22         30 

-17         22 

360 

425 

—   22 

—  550 

-30 

-204 

910) 

455 

182 

1 

1 

Y 

T 

li 

11 

1 

1 

X 

y 

.-.  x  =  2 

and  y  - 

-5. 

2°  Let  the  equations  be 

ArrflTifTfi  the  coefficients  thus 


"1 


Ji     Oj      — t/j  —a,  — r»j 

^3     '^■2      —^2  -^'a  ""^^z 

«3      />3      Cg       -(/g  -^3  — '>3 

«!      6|      Cj       — cZ,  — aj  — /^, 

«2        ^2        ^2         ""^3  "'^'2        ^2* 

Selecting  the  first  three  cohimns  form  the  ^agonal  products 
from  lel't  to  right  downward;^-,  thus  : 


SIMULTANEOUS    EQUATIONS.  181 


a,      b^     c^ 

\ 

\ 

giving 

uJk.c.^ 

a.,     60     C3 

a.,0._^ci 

\  '\ 

«3       '^3       ''3 

a^O^c^ 

\    \ 

«1       ^1       '^l 

\ 

«2         ''2        <^2 

Form  the  diagonal  products  from  right  to  left  downwards,  thus: 
«i      ^,      (-1  giving  c, 6. a 3 

/ 

«2       ^3       <^a  '^S^S'*! 

/    / 

«3       ''3       '^S  C3^l"» 

/       / 

a^     b^     c, 

/ 

flo         ''2         '^2 

From  the  sum  of  the  former  products  talre  the  sum  of  the  latter 

products  obtaining  a  remainder,  which  call  it^. 

Similarly  form  a  2°  remainder,  Pt,^  from  the  2°,  3°  and  4°  columns 
a  3°  "         A'g         "        3°,  4°  and  5° 

a  4°  "         i24         "        4°,  5°andG° 

Thena;  =  J?2-^2?i,  7j  =  B^-^E^,  z  =  E^-^n^, 
and  generally  I 

Ex.  3.  3x4-2?/-4^+20  =  0 
5.c-7y-G2-  1  =  0 
7«+5?/  +  5z-24  =  0. 


182  SIMULTANEOUS    EQUATION: 


3 

2 

-4 

-20 

-3 

—  2 

5 

-7 

-6 

1 

-5 

7 

7 

5 

5 

24 

-7 

—  5 

3 

2 

-4 

-20 

-3 

-  2 

6 

-7 

-6 

1 

-5 

7 

105   -288       28    -500       (3x -7x5= -105 
100       700     432         14        5  <  ox -4  =100 
84-20     500-504        7x2x-G=-   tt4,  &c) 


196 
-90 


-280       392 

i     960 

-990 

196       600 

-90         10 

50       672 

-    15 

480 
-840 

240       ( 
980 
15 

-4:X  —7x7  =  : 

-6x5x3  =  - 

5x2x5  =  50 

156     1282 

-375 

1235 

-445)  -890 

"Hsf- 

^2225 

2 

-3 

ii 

5 
z 

« 

Exercise  Ivii. 

Solve  the  following  systems  of  equations  : 
1.     2a;4-S//  =  41                               2.     5x+77/  =  17 
3a;  +  2//  =  39                                         lx-5i/=    9. 

3.     11a; +  12// =100 

9;«  +  8//  =  80. 

4.     18a; 
15;c 

-35//-f-13  =  0 
+  28?/ -275  =  0. 

5.     3a; +7// =-7 

5x  +  Si/=  -36. 

6.     3a;+16//-5  =  0 
28//  =  5x+19. 

7.     5a;+3//4-2  =  0 
8.^+2/7  +  1  =  0 

I 

8.     21a; 

23// 

+  8// +  66  =  0 
-28a; +  13  =  0, 

9.     10a;-l-7//+4  =  0 
6a;  +  5//  +  2  =  0. 

10.     23a;  +  15//-4i  =  0 
82.f+21//-6  =  0. 

11.    ia;+l//  =  6. 
3a; -4// =  4. 

12.     4a;- 

SIMULTANEOUS    EQUATIONS.  188 

13.     ^y  =  l7-l.  14.  ^x+pj  =  n. 

i>j=lx-l.  fa:+|^  =  19. 

15.     l-Sx-2y  =  l.  IG.  7a:=:107/-f  1. 

2-5x-Sy  =  G.  nx=lGy  +  -l. 

17.      Bx-4:y+l  =  0.  18.  ■lGx-04:y  =  l. 

l-7x-2-2y-{-7-d  =  0.  •19j;--lli/  =  l. 

19.     3-5x  +  2i7/=.13+4|rc-3-5?/. 
2ja;+'8^  =  22^+-7a;- 3^.V. 

21. 


23. 


20. 

1           1 

X            y     ~ 

5 

1             1 

1 

X           y 

6 

22. 

1-6         2-7 

X     ~      y     ~ 

1. 

•8         3-6 

X         y    ~ 

5. 

24. 

X           5 

T  +  7  = 

4^ 

a;          10 

T  +  7  = 

21- 

26. 

|a;-^(?/  +  l)- 

rl. 

i(a:+l)H-i(2/- 

-1)  =  9. 

25. 


27. 


28.  =   -^^ 29. 


Ax-Z         ly-Q 

80.    l^i^2  :=  8  31. 

45-?/ 


[S4:  SIMUIiTANEOUS    EQUATIONS. 


32.    '    -^   =    A.  83. 


2x-y  +  l 

Sx-y  +  1 
x-jf+'B 

x+1         y+2 
3                4 

^x-y) 
6 

x-2,         y-n 
4               3       = 

2y-x. 

2 

34.    _^_i^(±lL    ^  30.  85. 

•4x+-5?/  — 2-5 

•8x+.-l2/+-6   _     1 
5x+3^-23    ~   T* 

86    ?:^LzI(±_^  _  ^--y+^  _  4. 

"^  '  3  4  ' 

3a; -47/+ 3         4a;-2v-9 

' 4-   — ■ '- =    4. 

4^3 

37.  20(x+l)  =  I5{y  +  1)  =  12(x+y). 

38.  (x-2)  :  (7/  +  I)  :  {.c+y-V>)  ::  3  :  4  :  5, 

39.  (x-5)  :  (.v+9)  :  (:«+?/  +  4)  ::  1  :  2  :  3. 

40.  "^    =   ''l±^.  41.  (r-4)0y+7)  =  (.r-3)(2/+4). 
a  +  l         2/  +  5 

^•^-^  ^^-^  (x  +  5)0/-2)=:(a-+2)(2/-l). 


2(^  +  1)         5i/+7 

42.    {x-l){5>/-3)  =  B{Sx  +  l).  43.  (a;  +  l)(2//  +  l)  =  5x+9?/  +  l. 

(x-l)(4y  +  3)  =  3(7a;-l).  (a;+2)(8i/+l)  =  9x+13i/  +  2 

44.  idx-2){5y-hl)  =  {5x-l){y  +  2). 
(3x-l)(,//  +  5)  =  {.i-+5)(7i/-l). 

45.  «^7/  =  37.  4G.     2.r  +  2?/  =  7. 
y+z  =  25.  7.r  +  ;9z  =  29. 
z  +  a;  =  22.  7/+82;=17. 

47.     l-3a;-l-9y  =  l.  48-     5x+3?/+2z  =  217. 

17//-l-lz  =  2.  5a;-3?/        =   39. 

2-9z-2-la;  =  3.  8y-2z-    20. 

49.     ;^a:--:i7/  =  0.  CO.     Ilx+lhj  =  l0. 

lx-lz  =  l.  2lx  +  Q.}z  =  20. 

lz-iy  =  2.  8i^+3.:2  =  30. 


SIMULTANEOUS    EQUATIONS. 


195 


51. 

x-hy-z  =  n. 
y-\-z-x=  IB. 

z-[-x-y=   7. 

62. 

x+y+z  =  9. 
a;+2?/  +  4z  =  lo. 
x+3//+y2  =  23. 

53. 

x+y+z=   3. 

54. 

7x+Gy+7z  =  100. 

2x+4y+  83: 

=  13. 

a;-2//+   2  =  0. 

'dx+dy+21z-- 

=  34. 

Bx+  y-2z  =  0. 

55. 

3x+2//  +  3z  = 

110. 

66. 

x-\-y+z  =  9. 

5a; -f  ?/— 42  = 

0. 

x+2y  +  3z  =  U. 

2x-By+  z  = 

0. 

x+3//+62  =  20. 

57. 

a;+2»/  +  32  =  32. 

58. 

x+y+2z  =  B4:. 

'2x4-By+z  =  ' 

12. 

a;-f2?/+2=33. 

Sx+  y-r2z--- 

:40. 

'2z+y  +  z  =  B'2. 

59. 

Sx  +  Bij+  t  = 

=  17. 

60. 

x  +  2y-z=    4-6.- 

Bx+   2/  +  3z  = 

=  15. 

^  +  22-x  =  10-l. 

x  +  3i/  +  3z  = 

=  13. 

z+2x-?/=    5-7. 

61. 

a;  4- 2.'/ --72: 

=  21. 

62. 

x+y  =  nz+8. 

3x  +  '2y-   z 

=  24. 

y-\-z^2ly-U. 

•9x+7y-22 

=  27. 

z+x  =  B^x-B2. 

63. 

'^x+y  +  \z  = 

=  36^. 

64. 

2lx+By  +  ^z  =  U{i, 

ix+\y+lz-- 

=  27. 

dlx+^y  +  5iz=  17^, 

^x+y+\z-- 

=  18. 

2|x+3|?/+4|2  =  loV. 

65. 

y+l 

z+1  ~  • 

z+3 

x+1    "   ^• 

66. 

Bx+y  _ 
■     z  +  1     =   ^ 
By+z  _ 
x+1    -   ^• 
82  +  .   ^_    ^ 
y+l 

67. 

■     2/-Z 

■It!  =  9. 
a;-// 

I/+2    _   J 

68. 

^+'  _  2. 
y  +  z 

y±^  =  1. 

x  +  z 

196  SIMULTANEOUS    EQUATIONS. 

69.      ^     -    ^  =    1.                   70.    1  +  Jl  +  ji   =   4. 

^            y                                              X            y  z 

2.3  385 

—    -   ~7  =   '^-                             —   +   —   +  —    =    4. 

2                                               X            y  z 

1           „                                 5           12  10 


4 

3 

c 

y 

2 

3 

X 

z 

3 

1 

y 

2 

xy 
x  +  y 

1 

5 

yz 
y+z 

1 
~    6" 

zx 

1 

z-\-x 

7 

71.  -    =   — •  72. 


6           4 

—   +  — 

X            y 

5 

+    — 
z 

3           8 

—    +    — 
X            y 

5 

+  — 

3 

.3_^lr 

10 

X            y 

z 

xy 
hj-'dx   " 

20. 

xz 

2x-€7.      ^' 

15. 

yz 
4y  —  02 

15*. 

73.  (x4-2)(%-i-l)  =  (2a:+7)?/. 
Cr-2)(32«^l)  =  (a:+3)(82-l). 
(7/  +  l)l2  +  l'      =(7/  +  3)(  24-1). 

74.  {2x-l){y  +  \\  =2{x  +  l){y-l). 
(.«  +  4)(2+l)    =(.r+2)(z  +  2). 
(^-2)(2+8)    -(y-l)(2  +  l). 

75.  (.r4-l)(52/-3)=:(7.r+l)(2y-3). 
(.Ix- l)(2+l)  =  (:c+l)(2z-l). 
(//+3)(z  +  2)   =(3//-6)(32-l). 

76.  21«+31y+42z=115. 

6(2a;+2/)  =  3(3x+s)  =  2(2/+2) 

77.  15(a;-22/)  =  5(2a;-3«)  =  3(y/-f2). 
21x-|-3l7/  +  4l2  =  l&.n. 

78.  Qx{y+z)  =.  Mj  {z-k-x)  =  Bz{x  ^-  \/j. 


79. 


1       1       1 

—  +  —  +  —  = 

X           y           z 

=    9. 

3a;  +  ?/+z  =  20. 

60. 

.r+2-f8.y-33 

3'^4-a;^-4^/  =  30., 

5u  +  y-\-z  =  li. 

%u^Qx-\-z^AO. 
5?t+8(/+;:2=50 

4»4-a;4-2  =  lL 
3?c  ^i'  +  ?/  =  li. 

SIMULTANEOUS    EQUATIONS.  187 

81.     u-{-x  +  rj  +  z  =  lU.  82.     «4-a-4-i/+2  =  2-i. 
tt+2.c+2y  +  2z  =  267.  u+2x+Si/-dz  =  0. 

ii-\-2x  +  'Bi/-\-3z  =  o5d.  'du-x-  5t/+z  =  0. 

z<+2x-  +  3//+4z  =  410.  2i(-^Sx-Ay-^z  =  0. 

38.     u  +  x+i/+z=GO.  84.      u+x  +  y-^z^l. 

M+2x-|-3^4-4z  =  100.  2u+ix  +  Si/  +  16z  =  5. 

u+Sx+Gu  +  10z  =  150.  3jH-9x-+27»/  +  81z  =  15. 

u  +  4x+10?/  +  202  =  210.  4«+lG.c  +  64y/  +  25Gz  =  35. 

B5.     U+i!J-lz  =  l.  86.     lu-lx-h-ly-]^^^l. 

i-c-iy--^»  =  i.  i«+i^4-iy-^i^  =  37. 

lx+'iz-lu  =  l.  ^u-:^x+\y-^r.  =  l7. 

Art.  XL, VI I.  The  principle  of  sj'mmetry  is  often  of  use  in 
the  solution  of  sj-mmetrical  equations.  For  from  one  relation 
which  may  be  found  to  exist  between  two  or  more  of  the  letters 
involved,  other  relations  may  be  derived  by  symmetry  ;  also, 
when  the  value  of  one  of  the  unknown  quantities  has  been  deter- 
mined, the  values  of  the  others  can  be  at  once  written  down,  &c. 
1.  {x-{-y){x+z)  =  a. 

{x+y)(y+z)  =  b. 
{x+z){y+z)=c. 
Multiply  the  equations  together  and  extract  the  square  root, 

•■•    {x-\-y){i/+z){z+x)  =  i/{abc). 
Divide  this  equation  by  the  third. 

.•.   x+y  =  V_l^_£i,  and  therefore,  by  symmetry, 
c 

l/(cibc) 

a 

b 
Kence  we  get 

_  nb  —  bc-\-ca 
2\/{abc} 
whence  y  and  z  may  be  derived  by  symmetry. 


188  SIMULTANEOUS    EQUATIONS. 

2.         x+y-\-z  =  0 (1). 

ax-\-bi/  +  cz  =  0     (2). 

bcx+cai/-\-ahz-\-{a  —  b)(h  —  c){c  —  a)  =  0 (3). 

<;x(l)-(2)  gives  (c  — a)a;+(c-%  =  0. 

.-.  y  =  \ L,  and  similarly, 

b—c 

2  =  (a-b)x 
b-c 
Substitute  in  (3)  these. values  of  y  and  z,  and  reduce, 
.-.  x{a  —  l){c—a)  =  (a  —  b)(b  —  c){c  —  a), 
.'.  ovx  =  {b  —  c),   .'.  y  =  c  —  a,  z  =  a  —  b. 
8.  a[yz— zx — xy)  =  b (zx  —  xy  —  yz)  =  c {xy  —  yz  —  zx)  =  xyz. 
Divide  the  first  and  the  last  equations  by  axyz  ; 

.  ■.   — ^   s;   __   _   —  _  — ,  and  hence,  by  symmetry, 

axyz 

Jl   _    ^         i_        i- 

b  y  z  X 

1  _  2.     i_      1 

c  z  X  y 

,'.  -i-  —   =    —   — ,  and  by  symmetry, 

be  X 

L     L  -      A 

c  a  y 

1      i_  _  _  A. 

a            b                    z 
i.         ((x-tby  +  cz  =  l  (1) 

a^-x-{-b^-y+c^z  =  l (2). 

a^x  +  bhj+c^z  =  l (3). 

cx{l)  — (2)    gives   a{c  —  a)x+b{c  —  l>)y  =  c-l (4). 

cx(2)-(3)        "        a^{c-a)x-¥b^{c-b)y  =  c-l (5). 

6x(4)-(5)       "       ah{c-a)x-a^{c-a)x  =  h{c-l)-{c-l), 
or  a[a  —  b)[a  —  c)x=  {c  —  1)(6—  1), 
...  ^    =      {l-h){l-c), 
a{a — b)ia—c) 
Tvlien4je  y  and  z  may  be  derived  by  symmetry. 


SIMULTANBOrS    EQUATIONS. 


189 


6.  Eliminate  x,  y,  z,  u  (which  are  supposed  all  different)  from 
the  following  equations  : 

x  =  bij-\-rz-{-cIil. 
y  =  cz-\-du  +  ax. 
t  =  du-^ax+by. 
u  =  ax-\-l)i/-\-cz. 
Subtracting  the  second  equation  from  the  first, 
.•.  x  —  y  =  by — ax,  or 
(1  +ri)x={l  -f  b)!/  =  (by  symmetry)  (1  +c)z  =  (1  +d)u. 

These  relations  may  be  also  obtained  by  adding  ax  to  both 
members  of  the  first  equation,  by,  to  both  members  of  the  second 
aquation,  &c. 

Now  divide  the  first  equation  by  these  equals. 

1  h  c  d 

1  +  ^'    '' 
1 


1  +  b   "^    1+c   "^   l  +  u' 


And  since =    1  —  ,  we  have 

l-\-a  1+a 


1    =    I _L     ^^ 


1-fd 


Exercise  Iviii. 


1.  Given  ax  -\-  by  =  c 

a'x+b'y  =  c' 

2.  Given  hx  —  mj 

dx-\-md  =  cy-\-ni 

3.  Given  axi-by  +  rz  —  d. 

a^x-\-})^y  +  c~z  =  d'^ 


and  that  a;  = 


b'c-bc' 


b'a-ba' 
derive  the  value  of  y 

a(dm—  en) 

and  that  x  =  —i — -, — » 

be  — ad 

derive  the  value  of  y. 

and  that  x  = 

a{d-b)(d-e)        .^    ^ 
,  write  down 


d{a  —  b){a  —  c) 
ai^x+l'^y  -hc^z~d^      the  values  of  ?/  and  z. 


190 


SIMULTANKOUS    EQUATIONS. 


4.  Tliere  is  a  set  of  equations  in  x,  y,  z,  u,  and  ic,  witt  corres 
pending  coefiacients  (a  tea;,  &c.),  a,  b,  c,  d,  aude;  one  of  tiif 
eciuations  is 

ic  =  lJl/  +  cz-i-d>(  +  (nv,  write  down  the  otliei*3. 
Solve  the  following  equations  : 

K      ^      ,      ?/  '/  z  ,     X  z 

'"  n  n  p  m.  p 

6.  x+aij  +  hz  =  )n,   y-j-  az  4-  hx  =  n,  z  f  ax+hy  -p. 

7.  x  +  ay  =  l,    tj-Y(jz~iii,  z-{-cit=it,    ti.  +  dw=p,   w  +  ex  =  r, 

8.  Eliminate  x,  y,  z,  (supposed  to  be  all  different)  from  tb{ 
following  equations : 

X  —  hy  -f  .■  z,    ,j  —  cz  +  ax,  Z  —  ax  +  hy. 

0.    Eliminate  x,  //,  z,  from 

■^      =    i       ''■'      -  \,      J_ 
y-\-z        '' '   z-\-x    ~    '*    ^^+y   ~  ^' 

10.  Having  given 

*  =  %  -f '  2 + '^»  +  fi'', 
y  =  cz  +  <ln+eir-\-(ix, 
Z  =dii  +  en--\-ax  +  i>y. 

u  =  CIV  -\-ax-{-  by  +  cz, 
ic  =  a  x-x-liy-\-iz-{-  d  tl , 

Shew  that    -—    +     — r   +   v-j-  +   ,-,-,  +  t-t-   =   1. 

.   Art.  XLVIII.     Eesolution  of  Particular  Systems  of  Liueai 
Equations. 

Ex.   L                                x+y-\-z  =  a  (1) 

y+z  +  ti  =  b  (2) 

z-^u  +  x  =  c  (3) 

u+x+y  =  d  (i) 

(l)  +  (2)  +  (3)  +  (4)              S{H  +  x-\-y-j-z)  =  a  +  h-^c-\.d  (5') 

3(1)                                                Si^x  +  y-i-z)  =  3a  (6') 

H(5')-(C')}  w  =H-2<,+/>+r+'/.) 


SIMULTANEOUS    EQUATIONS.  191 

The  values  of  x,  y  and  z  may  now  be  written  down  by  sym- 
metry. 

The  following  is  a  variation  of  the  above  method,  ai)plicable  to 
a  much  more  gfineval  system. 

Assume  the  auxihary  equation 

.-.  (1)  becomes  s  —  u  =  a,  (a\ 

(2)  "  s-x  =  h,  (7) 

(3)  "  !i~y  =  c,  (8) 

(4)  "  .s-z  =  f/,  (9) 
(5)  +  (G)  +  (7)  +  (8)  +  (0)                     is  =  s-\-n  +  h^c-^d. 

s  is  now  a  known  quantity,  and  may  be  treated  as  such, 
in  (G)  giving  ii  =  s  —  a 

"  (7)     "  x  =  s-b 

«'  (8)     "  y  =  s-c 

"  (9)     "  z  =  s-d. 

Ex.    2.  7/2  =  «((/  +  z),  (1) 

zx  =  b{z+x),  (2) 

xi/  =  c{x+ij),  (3) 

111 

(D^aijz,  1 =    _, 

y           z  a 

{2)^bzx^  1 =    -7-' 

2  X  h 

111 

{Z)---cxy,  ^  _    =   _. 

x  y  c 

This  may  now  be  solved  like  Ex.  1,  using  the  reciprocals  of  a 
6,  c,  X,  y  and  z  instead  of  these  quantities  themselves. 

Ex.  3.                           a,M+i,(x+?/+z)  =  Cj  (1) 

a^x-\-h^{y+z  +  u)  =  c^  (2) 

a^y  +  h^{z-\-u+x)  =  c^  (3) 

<tiZ-^h^{n->rx-[-y)  =  c^'  (4) 

Assume  the  auxiliary  equation 

«  +  x+v/+z  =  s.  (5) 


192  SIMULTANEOUS    EQUATIONS. 

^1)  becomes  5 ^s  —  (/!^j —rti)«  =  Ci 


— s  -  a  = (G) 


Similarlyfrom(2)  b-^.'-'^^hT^  C^) 

(5)  +  (0)  +  (7)  +  (8)  +  (9)    (^A_+^:^+  ^-^';s__^_*,_j  . 

C-i  Co  Co  Cj 

=  s  -f-1 — "^ —  +  -, -— +7 — ^—  4-  -, — * —  (10) 

From  (10)  we  can  at  once  get  the  value  of  s,  which  may  there- 
fore be  treated  as  a  knowu  quantity. 

6 ,  s  —  c  ^ 
m  (G)  giving  u=,    _ 

and  tlie  vakie  of  x,  y,  and  2  may   be  obtained  from   (7),  (8j  and 
(9),  or  they  nvay  be  written  down  by  symmetry. 

Ex.  4.  .     aa;+6(?/+2)=6-  (1) 

aii-\-l){z  +  iC)  =  d  (2) 

oz-\-h{u-\-x)  =  e  (3) 

au  +  h[x^,j)^f  (4) 

Assume  u+x-\-y  \  z  =  s  (5) 

(l)  +  (2)  +  (3)-f(4)  («  +  26)s       =c  +  J  +  .+/  (6) 

Hence  s  is  a  known  quantity  and  may  be  treated  as  such. 

From  (1)  and  (0)  hs—hu-\-{a  —  h)x  =  c, 

bu  —  (a  —  b)x=.bs—c,  (7) 

Similarly  from  (2)  and  (5)    bx-{a  —  h)y  =  bs—d,  (8) 

«'      (8)     "      "      hy-{a-b)z=:hs-e,  (9) 

'■      (1)      "     "      Lz~{a-b)u  =  bs-f,  (10) 

b{7)  +  (a  - h){8)  b~ H -  {a  -by-y==  ahs -bz-{a- h)d,{ll) 

fc(9)  +  (a-6)(10)  b-^y-{a-bfu:=ahs-hi~{a-b)f,(\'l) 


SIMVXTANEOUS    EQtlATIOXS. 


193 


b-{U)  +  {a-b)^12)     .  {b^-{a-b)^}n  =  abs{h^  +  {a-hy-} 

-a{b2d+{a-b)y^}-b{b^{c-d)  +  {a-b)^e-f)}   (13) 

The  values  of  x,  y,  and  z  may  now  be  written  down  by  sym- 
metry. 

Ex.  5.  a^  +  a^x-\-aij+z  =  ^. 

The  polynome  t^  ^-xt^  -\-yt-\-z  vanishes  for  t  =  a,  t  =  b,  t  =  G\ 
.*.  by  Th.  II.,  p.  46,  for  all  values  of  t. 

t^+xt^-\-yt-\-z  =  {t-a){t-b){t-A 
=  t^  -{a  +  h-\-c)t-  +  (nb->rbc  +  ca)t-ahc. 
^.  Th.  III.,  p.  53,  x=  -{'(  +  b  +  c), 

y  =  ah-i-bc-{-ca, 
2=  —abc. 


Ex.  G.  x+y+z  +  n=-[, 

ax + liy-\-rz  -\-  da  —  0, 
a-x-\-b-2y  +  c^z  +  d2!i  =  0, 
a^x  +  b^y-\-c^z-\-d^  21  =  0. 

Employing  the  method  of  arbitrary  multipliers, 

(4)+Z(3)  +  w(2)+n(l)         a^x+  b^'y+  c^  z  4-  d^ 
+Z<|2j    +/62|    +ic2     +id-2 

+ma    + '.'ji    +  mc     +md 
-\-n \    ■{■  n\   +  n      -\-  n 
To  determine  x  assume 

c^+lc-  -{-mc+n  =  0, 
d^+ld^-j-md  +  n  =  0, 
n 


0) 

(2) 
(3) 
(4) 

(5) 


(G) 
(7) 
(8) 

(9) 


a^  +  la^  +ma-\-n 
But  the  system  (6),  (7),  (8)  has  been  solved  in  Ex.  5,  from 
which  it  is  seen  that 

1=  —(^/)-^c  +  d),  vi  =  bc  +  cd-\-dh,  71=. —bed, 
and  a^-\-a'l-^am-^n  =  [a  —b){a~c){a  —  d\ : 


194  SIMULTANEOUS    EQUATIONS. 

.'.  using  these  values  in  (9) 
—  hcd 


(^a  —  b)[a—  c)[a  —  d) 
The  values  of  y,  z  and  u  may  now  be  written  down  by    sym- 
metry. 

Ex.  7.  -±-  +  -^  +   -^-    =   1.  (1) 

+  _^   +   _J_    =    1.  (2) 

n  —  b  71  — c 

■+   ^   +    ^^—   =    1.  (3) 

p — 0  p  —  c 

Assume  1  -  -^ ^  _   ^_  ^   tS-^Bt^~  +  Ct+D 

t—a  t  —  b  t  —  c  {t  —  a){t  —  b){t-cr 

But  in  virtue  of  equations  (1),  (2)  and  (3),  the  first  member  of 
(4)  vanishes  for  t=rti,  t  =  n,  and  t=p,  and  .'.  t^+Bt^  +  Ct+D 
vanishes  for  the  same  values  of  t,  and  .•.  Th.  II.  p.  46, 

t^+£t"  +  Ct+D  =  {t—m){t—oi){t-p), 


m 

X 

■a 

n 

X 

a 

P 

— 

a 

X 

.'.  (4)  becomes  1  — 


y  z 


t  —  a  t  —  b  t  —  c 

^^  (t-m){t-n){t-p) 
~  (t-a){t-b)(t  —  c)  ' 
To  obtain  the  value  of  x  multiply  both   sides  of  this  equation 
by  («-«), 

t-a-x-  y^tZ^   _    <^~-^)     ^    it-m)(t-n)(t-p) 
t-b  t-c  {t-b){t-c) 

Now  t  may  have  any  value  in  this  equation  ;  let  i  =  a, 
(a—m)(a  —  n)(a  —  p) 
~  {a—b){a  —  c) 

The  substitution  (x)jz\abc)  will  give  the  values  of  y  and  z, 

Ex.  8.  x+a   ^   j/^   ^    z_+c_  ^^^ 

p  q  r 

Ix  +  my  +  nz  =  s  ^  (2) 


SIMULXANEOUB    EQUATIONS.  195 

By  Art.  XXXYIL, 

x+a  ij  +  b  z-\-c  l.v-\-imj-\-?iz-\-Ja-}-tnb-\-nc 

V       ~       1/  r  lp-\-t:iq-\-nr 

/ov  s'^-\-la  +  nib  +  nc 

i^J  =    — r— =   R,  say 

Ip  +  mq  +  nr  •' 

,',  x=pB,  —  a,   ij  =  qR  —  b,  z  =  rR—c, 

Ex.9.  yz+z-^-+x^={a+b  +  c)xyz  (1) 

yz  +  zx   _   zx  +  xj/    _   xy+yz  .^. 

{l)^xg»  A   +  -L   _^  ^    ^    a^b^e  (^) 

(4) 


Page  122  and  (3) 


a 

b 

c 

1 

X 

+ 

1 

+ 

1 

z 

= 

a+6-h(? 

+ 

1 

y 

= 

1 

+ 

1 

z 

1 

_       2 

+ 

1 

fl3 

a 

c 

2 
a; 

+ 

2 

2/ 

2 

H 

z 

2 

(5) 


(4)  and  (5)  .-. 1_  —   =    2a,    —    +    —    =    2?>, 

X  y  y  z 

-  +  -   =    2c.  (G) 

z  X 

111 

(3)_.(G)  -~    =    a-b  +  c,  —    =<t-\-h—c,  y    =    -n-^rb  +  c. 

E..  10.  £+f   +  »+^   =   2.  (1) 

a-\-b         a\-c 

x  —  by  —  c^  ,ns 

+  -^—i   =    2.  (2) 

a—c         a—b 

(1)  .       ^i    _    1    =    1    _    '^Jt^ 


a;  — a  — 6+c  a  +  r  —  b  —  y 

a-\-b  ~  a+c 


(3) 


^^^  SIMULTANEOUS    EQUATIONS. 

Similarly  from  (2)      x-a-b  +  c    ^    g-h  +  c-y 
a—c  a—b 

(3)  and  (4)  ...     x-a-h-^c  =   "L+^U-b+c-y) 

a  —  c  ,   ,  . 

But  unless  _    J ,  this  cannot  be  the  case  except  for 

a+c         a-b  k  ^ 

a-b+c-y  =  0, 

in  which  ease  x  -  a  —  b-^c  =  0  also, 

giving  ■x  =  a-t-b  —  c  Sind.y  =  a-b+c.  (5) 

Tj-      (i  +  b  a  —  c 

^      ^7    =    ^[ZTf  ■'■   a^-b2=a^^-c^  (6) 

^*2_c2=0,  ov  {b  +  c)(b~c)--.0, 
b  =  c,  or  b=  —c. 
But  if  i  =  4-c  or  — c,  (1)  and  (2)  are  one  and  the  same  equation  ; 
hence  if  (1)  and  (2)  are  independent,  (6)  cannot  be  true,  thus 
leaving  only  the  alternative  (5). 

Ex.11,  2ax={h  +  c-a){y-hz),  (1) 

2hy  =  {c-i-a-b)(z-[.x),  (2) 

(a;  +  i/  +  z)-'+x2  +  (/2-fz2=4(«2+5=i-f2)     (3) 

(1)  and  page  122^5) ?_    =   JL±1_   _   «+i/±f  (A^ 

b  +  c-a  "la       ~    b+c  +  a  ^  ' 

(2)  "  «»        ?/ _     a;+z     _   z+y-\-z  ,„. 

c  +  rt-i    ~       2i       ~    c-\-a-\-b  ^°' 


<,4),  (5)  and     "    ...^H:^   ^ 


y 


a-f/y  +  f  b-\-c  —  a         c-^a-b  a-j-b-c' 

(x+y+z)^+x2+y2+z^ 


(6+c-a)3      {a  +  b  +  cy  +  {b  +  c  -a)2 -^(c-i-u  -  by  ^(a-\-b -c)2 

^  '  4(rt''- 


Ked>,otionana(8)=    (:E±SH^)l+i^,M:.:   ^ 

^    ^  4(rt2  4-i2^c2)  -^* 


SIMULTANEOUS   EQUATIONS.  197 

111 

Ex.  12.  flx  =&7/  =  c«=  —  +  —  +  -—  (1) 

u^  y  z 

rt     _      6     _     c  _      a+b  +  c 

{l)^xyz         :.     —   =   ~   -  ^  ••   -  ^yz+zx  ^"^ 

a  1/1-1       1  \        xy+yz+zx    ■'    ^ 

Usofrom(l)-;i^^5  —    = h— +—    =    ^^2T3-       3 

^  -'       "^     yz  xyz   \x       y       z  j  x-yz' 


(2)x(3) 

Ex.  10. 

(1) 
then 


a-\-b-\-c 


y^z^          x'^y'^^z'^ 

a^x'^    =    a+bJrC. 

y+z-x         z+x- 
a         ~          b 

-y         x+y-z 
c 

(1) 

xyz  =  vi^ 

(2) 

z               x 

y         "» 

(3) 

a+b    ~    b-j-c    ~ 

c  +  a'  -    r    s^PPOS® 

xyz 

=  7i 

(a-{-b){b  +  c){c  +  a) 

.-.     r^  =  {a  +  b){b  +  c){c+a) 

Hence  the  value  of  r  is  known  and  from  (3) 
rx—Vi{b+c). 

Ex.   14.                  y-^z  =  1axyz  (1) 

z-^x  =  1bxyz  (2) 

x^y  =  1cxyz  _             (3) 

•   '          1/+Z         z-\-x  x-\-y         x+y  +  z 


2a  '2b  2c  a  +  b+o 

xyz 


h-\-c  —  a         c  +  a  —  b         a-\-b  —  c 

2 xyz 

:.  X  y  z   =  (i_,.c_a)(c  +  a-^)(a  +  i-c) 

.,  ^  , 1 

..  x-y  «-=  (^ij^(;-a){c+a-b){a  +  b-c)' 


(1) 


198  SBIULTANEO'JS    EQUATIONS, 


Hence  tlie  value  of  x~i/~z-  is  known,  call  it  -^  and  substitute 


in  (4) 


1                        X 

^ 

r     ~    b-\-  c  —a 

:. 

rx  =  b  +  c—a. 

in  wnich 

r2  =  {b-{-c-a)(c-\-a-b){a  +  b-c). 

Ex.  15. 

y^+z-  -x(y  +  z)  =  a 

(1) 

z-  -\-x^—y{z-\-x)  =  h 

(2) 

x'-+y^-z{x+y)=d 

(3) 

(l)+(2)+(3) 

'2.{x-+y'^+z^—xy-yz  —  zx)  =  a-\-b-{-c 

(4) 

(1)  may  be  written 

a-3  -j-y'^  -\.z'  —x[x  +  y-{-z)  =a 

(5) 

(2) 

x^+y^--i-z--y{x  +  y  +  z)  =  b 

(6) 

(3)         - 

x^+7/-+z"-z{x  +  y-i-z)  =  c 

(7) 

.-.  x  +  y+i 

^_a  — &          b  —  c          c  —  a 
y-x         z-y          x  —  z 

.-.  (x+y  +  z)' 

^_    ^a-bY^^-[h-cY^{c-aY 
[y-x)--\-(z-yY-^{x-zY 

a;3_}-2/^+z2  -xy-yz  —  zx 

(4) 

1{a^+o-+c^-ab-'bc-ca) 
a-\-b-\-c 

(8) 

2(a3  +  63+c3~3a6c) 

(9) 

Write  r2  for  'l{a^  +  h^+c'  -dabc). 

(9)              ''■-+y^^  =  -^j^  (10) 

Eeturning  to  (8)  {x+y+zY  =  '^^^'^^'t+V+t  "'"'"^  ^'^ 

(4)                     2{x"-+y^^z"^-xy-yz-zx)=i^^±t^  (11) 


BIJIULTANEOUS    EQUATIONS.  1^9 

i{(8)  +  (ll)}  ^2+,/=+3^    =    ^+^I±''.^  (12) 

(5)  and  (10)  a;-+v-+s2-  -— --    =    n 

a  +  b+G 

(12)  =a^+b'+c^-a{a+b-^c) 

=  b'^+c^-a{b+c). 

(5),  (6),  (7)  are  symmetrical  with  respect  to  {xyz\abc);  (10)  sho-ws 
this  substitution  does  not  affect  r,  and  consequently  the  values 
of  y  and  z  may  be  written  down  at  once  from  that  of  x. 

Exercise  lix, 

1.     ax+hij  =  c,  2.     ax-^hy-c, 

vix  +  ny  =  d.  mx  —  ny  =  d. 

3.     cix  +  by  =  c,  4       a;  ?/     _ 

7?/a;  4"  n  ?/  =  c.  «.  6 

x+y  =  c. 

5.       -    +  Jl   =    1,  0.     4  +   f   =.    1. 


a 


6  '  '      a     '      b 


i     +    «     ~    ■^'  b     ~     a' 

7.     rt«  +  6c- =  ft  ;/  +  ac.  **      1      ''     _ 

8.     —   +  —    ^1 

x  +  y  =c.  .-  y 

b  a 

—  4-  —   =  n. 
js  y 

9.     {a  +  c)x—{a  —  c)y  =  2ab,  x—c   _     a 

(a'{-b)y-(a-b)x  =  2ac.  '     y-c   ~     b 

x  —  y  =  a  —  b. 

X  a  12      ^+?/    _   <^<  +  ft+g 

~^   ~    17'  '     ?/+i    ~    a  — 6  +  c 

a;  +  »i  c  2/-1    _    a-b-G 

y^n    ~   ~d'  x-\-l    ~    a-^rb  —  c 


11. 


200  SI.Ui;L.TAIvEOUS    E:iUATIONS. 

13.      ^-=^'   =   ±.                    14.     "+'  +  y+t   ,  2. 

y-a  +  h           c                                  a+b   ^  a+c           ' 

V  +  b^    _    c^+a                                      x—b  y-c 

x  +  c    ~    h  +  a                                     a  —  c  a  —  b   ~      ' 


15. 


__f_      ,        V       _  16.     x+y+z  =  0, 

m-a  "^  m-b    ~      '  {b  +  c)x+{a-j-c)y+{ai-b)z 

X           y         .  =0. 

n  -a         11  — b            '  bcx-{- acy ^abz  =  l. 

17.     x->ry+z  =  l,  x-a         y  —  b         z-c 

1  Q  __      " 

nx  +  by  +  cz:=m,  P               <1               ^ 

_j^__     , V__      ,        t_    _  l{x-a)-\-m{y-h)-\-n{z-c) 

l-a  ~^   l-b    ^   I-  c    ~  =1. 

--«  __  y_^^  _  ^Zi  ^^-    (t{x-a)  =  b(y-h)  =  c{z-c), 

P                q,                r  ax  +  by-{-cz~m^» 
lx-\-my-^;iz  =  l. 


19. 


21.     x-\-y  +  z  =  a  +  b  +  c,  22.     a;+2/+2  =  0, 

6x+e?/  +  r/2  =  a2-j-i2^_(.2^  ax  +  by-irCz  =  ab-{-br  +  ca, 

cx-{-ay+bz  =  a"  -{-b"  -\-c^ .  (l,-.  c)x  +  {c  —  a)y-{-{a  -b)i 

=  0. 

23.     x-{-y-{-z  =  m,  2-1.     aa;-i-%  +  c2  =  r, 

X  :  y  :  z  =  't  :  b  :  c.  mx=  my,  qy=pz 

25.     a:y + ?/2  f  ?.<;  =  0,  ays  4-  hzx -f-  ra;?/  =  0, 

bcyz-\-acxz-\-abxy  -j-  (a  —  Z>)  (6  —  c)  (c  —  rt)x//2  =  0. 

26.  {a  +  b)x+{b+c)y  +  {c+a)z  =  ab-\-bc+cay 
(a+c)x  +  {a+b)y-\-(b+c)z  =  ab-{-ac+b<;^ 
(b-\-c)x  +  {a  +  c)y+{a  +  b)z  =  a-''+b^  +  c^. 

27.  nix  +  7iy-\-jz  +  qu  =  r, 

x  y  z  u 

abed 


28. 


SIMULTANEOUS    EQUATIONS. 

p.r„,L~)         2/{x+z)         z{x  +  >/) 


201 


1   . 

a  +  b--c 


111 

—    + h   —    =    a  +  b  +  c. 

X  y  z 

29.   (rt - h){x+c)  -ay+hz ^  (c- a){y  +  h)-cz-hax  =  0, 
x+y+z^'^{cc-^h  +  c). 

30. 


^2. 


84. 


ar  +  by  =  l, 

31. 

Z?/  +  7?7.r  =  n, 

by-\-cz=  1, 

72a;  +  /z  =  w7., 

cz  +  ax=i. 

mz+}iy  =  L 

x+y  =  a. 

33. 

VI  n 
2/  +  z-^  =  -p' 

y+z  =  b, 

In 

z-\-x~y  =  — , 
m 

x-\-z~c. 

Im 
x  +  y-z  =  _, 

71 

I        1\ 
y        *\ 

2a. 

.V 

1                  1 

2 

a 

1       1 

z           a; 

26. 

1 
z 

1                1 

X          y     ~ 

2 
T 

1           1 

aj           y 

2c. 

1 

a; 

1           1 

7/             z     ~ 

2 
c 

86.    (^H-''y^  +  (a-5)^  =  2/;e, 
(c  -\-a'^z-\-{c  —  a)y  =  2ab. 


y  + 


=    h. 


38.        2-    +   -4-    =    ^-"^ 


X  y 


c+«  a+b 


202  SIlHiLTANEOUS   EQUATION'S, 

39.    X-+.'/— 2  =  ''',  40.    u-i-v  —  x  =  a, 

y+z  —  v—h,  o+x—y  =  h, 

z+v  —  x=c,  x-t-y-z=c, 

v-\-x  —  y=d  y-\-x  —  u  =  (l, 

z  +  u  —  v=:e. 

Exercise  Ix. 

Resolve 
1.  {a  +  b)x  +  (a-  b)y  =  2{a-  -\-b-)   2.     x  +  y  =  a, 
{a-h)x  +  {a+b)y  =  2{a^-b'^)  x- -  y^  ^h. 

3.     2x  —  dy  =  m,  4.     {a-b)x-\-{a-\-b)y=^a-\-h. 

2x^ —Sy"  =7i^-\-xy.  X  y  1 

a  +b         a  —  b    ~    a-\-b 

a  +  b  +  1  •  ^       {a-\-b  —  c)x  —  {a  —  b-^-c)y 

=  4«(i  — c), 
.c  a-\-b — c 

y  a  —  L-i-G 

X — a         a-~h 
y  —  fi         (1+6 

X  «^ —b^ 


\_u  -  u)x-r 

■y-      a+b    ' 

a—b-i^l 
»  +  («  +  %=  ^_5  • 

•c+y 

a 

x-y 

b-c 

X+C 

ai-b    ~ 

y+b 

x-y+1 
x-y-l 

=    a. 

x  +  y+l 
x+y~l 

=   b. 

x-y  +  1 
x+yl 

-  a. 

x+y+1 
x-y-l 

=  b. 

{r)  +  r)x-{- 

■  [a  —  c)y  =  2ab, 

(a+b)y- 

(a—  b)x  =  ^Aac. 

8. 


10. 


y  a^-\-b^ 

x  +  y  +  1  «-t-l 

x  —  y  +  1         a — l' 
x+y  +  1         1  +  b 


x-y-l  ~  1-6 

11.         /     .    -    a,  12.    — _  +   .^  =  a+b. 

x+v-1  ,       a-\-b         a  —  b         ' 

\-  ^  —  2a. 

a  b 

13.     {o  +  n)x+{a-c)y  =  2ab,  14.     a^+ax-\-y  =  0, 

b^-{-hx  +  y  =  0. 


SIMULTANEOUS    EQUATI0X3.  203 

15.     y  +  z-x  =  a,  IG.     ".r-{-ll>/  +  z  =  a, 

z  +  x-y  =  />,  •          7^-(.iiz_(_^;  =  /,^ 

x+i/-z  =  v.  7z+llx+y  =  c. 

X            IJ            z         "     '  • 

L  ^  L.  _   1.   =<2,hc  {c-a){y+h)-cz+ax  =  0, 

y         ^         X      "   ' 

—   +   —   —    —    =  lea.  ^               ' 
z            X            y 

19.     r^  +  -^  =^a  +  h,         20.    ^  +  _^ ?_^0 

b-\-c          c—a  h-irc          c —a          a- b       * 

.'/         ,         z  X                 y                 z 

+    7    =b+c, —    — :^ —    4- 0 

c  +  a         a—b  b-c         c~a   ^  a-^b'    • 

z                X  X               ri               4 

a-\-b^b-c          ^  b-\-c^  c-a^  a  +  b~^^ 


21.    ^  +      JL_    +    _^L_    =    1  22.    -3L 


=   a. 


«  a— 1  «— 2   .         '  x+y 

X  y  z  ■    yz 


c-1  c  — 2     "'■•  jj_f.a. 


oo        ^  V  Z  X  1/  Z 

a  i>  c  0  c  (c 

X  u  z  1  1  1 

c  a  0  a  b  c 

24.    —    =    -^    =  — -    =    — ,  25.  ax  =  by  =  rz  =  dii, 

a  l>  c  d 

mx  +  ny-\-pz-{-qii  =  r  .  y2_23_3,_^^ 

26.    y-\-z  =  au,  27.  3;+// =  771, 

a; +  -:  =  /)?«,  ?/-|-z  =  j», 

x-{-y  =  cu,  .  z+t(  =  a, 

1  —  x         a  u  —  x==b. 


204  SIMULTANEOUS    EQUATIONS. 

28.    11x+9i/'\-z-u  =  a,  2d.  .c  +  mj  +  a"z+a^u+a*  =0, 

lly  +  Qz+u-z  =  b,  •      x+bi/-^b2z  +  b^u+b'^  =  0, 

llz+9u+x  —  y  =  c,  x+cy  +  c^z-\-c^u  +  C^  =  0, 

llu-\-9x+y  —  z  =  d.  x+dy  +  d-z  +  d^u+d^  =  0. 

80.         x  +  y  =  a,  31.         x-{-ly=a, 

y-bz  =  b,  y  +  mz  =  b, 

z+u  =  c,  z  +  nu  =  c. 

u  -\-v  =  d,  u  -\-pv  =  d, 

v-±x  —  e.  v+qx  —  e. 

82.         x-\-y-\-z  =  a,  83.         x  —  y+z  =  a, 

ytz  +  u-b,  y  —  z+ii  =  b, 

z  +  w4-f  =  c,  z—ii,+v  =  c, 

u-\-v-{-x-  d,  u—v-\-x  =  d, 

v-{-x-\-y  =  e.  v—x  +  y  =  e. 

84.         x  +  y+z  —  tc  =  a,  35.         x+y  +  z  —  u—v  =  a, 

y-\-z  +  u  —  v  =  b,  y+z  +  it^v  —  x  =  b, 

z  +  u-i-v  —  x  =  c,  z-\-u+v  —  x  —  y  =  c, 

u  +  v+x~y  =  d,  u+v  +  x  —  y  —  z  =  d 

v+x+y  —  z  =  e.  v+xi-y  —  z~u  =  e. 

86.    2x-y-z+2io   -v  =  3a,  87.  v-2x+Su-2y+z  =  a, 

2y—z  —  u  +  2v  —  x='ib,  x — 2^  +  3^- 2z  +  u  =  6, 

2z-u  —  v  +  2x  —  y  =  dc,  y—2z  +  Sx—2a  +  v  =  c, 

2u-v-x-^2y-z  =  dd.  z-2u  +  3y  —  2i--i-x  =  d. 

2u  —  x  —  y-{-2z  —  u  -~  3e.  u — 2t'  +  32  —  2x  +  >/  =  e. 

Exercise   Ixi. 

Kesolve  the  following  systems  of  equations  : 


i+y+y-         '  Z/+1  >y-i 

1+y+x^  ^  xM-x+1    ^    ^^jx-l\* 


i-^x+y^         '  y--hy+i  \y-il 

{l+x)(l+y)    _   l±a  x  +  y     _    a^-oc^ 

{\-x){\-y)   "   1-a         4.     i+j;^   -    «2_f_^8' 


SIIIULTANEOUS    EQUATIONS.  205 

(l+x){l-y)    _    1+^i  X-1J     _    h^-B'' 

{l-X){l+l/)     ~     l^b         ■  l-XIJ     ~     6-+p2' 

l+xy   ~    64-c'  '    1— x?/         l-a2 

x  —  y  b  —  c  J^—y  26 

7      ^+^     _       2rta  g     1-f-a^  >c+y     _    2a 

a;-y    _       2hQ  l-xy  x-y     _    26 

!+.'•//   ~   6^-/3-'  a;  — y  1-xy   ~    n 

g     y(l+a:3)    ^  ^ij     y  +  2  =  2ra'/2, 

Z(l+^2j  '  X+2  =  2/^.cz/^, 

y(l  —x^)  x  +  y  =  2CXI/Z. 

n      i'+~  — ^'         2+a;  — y         a;  +  2/  — 2     12.    ax  =  hy  =  cz, 

a  "  h  ^  c~~'  111 

■  = ! +  — . 

xyz  =  m^.  X  u  z 

13.    y'-  +z-  -z(y  +  z)  =  n, 

x-+y^-z{x+y)  =  c. 

14     '2ax={b+c  —  a){y-\-z), 

2/,y  =  {c  +  a-b){x+z}, 

(u;  +  //-i-2)'-'+x2  4-//2+22=4(a3+63+c3). 
,-     a;— 1    _    (i  —  l  ^,,     x^  -\-xy-\-u-     x^-^y^     xy 

2/  — 1         6—1  x^—xy-i-y^"      a       ~  b 

x^-1    _    a3     1 

S/3-1    ~   6^31' 
17.    x^+x-'^s^z/'^^rt,  iQ 


j;2+j;^4-?/2=6. 


.1 


19.    =cy    +j    =    a(x^  +  ^=)    20-     x3=a(x^+^'-')-6.^y, 


20b  SIMULTANEOUS    EQUATIONS. 

21.    4rU-2+l)=(rt  +  6)(a;-7/)2, 
^c{y-^-\)-={a-h){x-y)K 

h-Vc 


3          3             *- 
2,3  _„3     ^     _ 

-c 

'+-f^+i/ 

')(^+2/)- 

23. 

a-+.r2 

24. 

a;2  -f  ?/2 

-   a, 

xy 

1-^xhf-         ^ 
xy 

25. 

a;(?/  +  2)  =  a, 
?/(z+a;)  =  6, 

2(a;+?/)  =  (,-. 

26. 

{.r+y){x-\-z)  =  a, 
{l/+z){>J-{-x)  =  b, 

(2-hx)(z  +  //)=C. 

27. 

a;(a;+?/+2)  =  a- 

-2/2. 

28. 

x^~{y-z)^=a. 

y{xAry-r'z)  =  h- 

-sx, 

y^-{z-xY-=b, 

z{x-^y+z)  =  c- 

•  a?/. 

Z^  —  (x  — ?/)2  =c. 

29. 

^.2_|_^3  —  rt2^ 

30. 

1            1           2a 
x^    -^  y^    =    z^' 

a:+?/  =  ^2, 

1            1           26 

a:^          ?/■■'         r^ 

x  —  y  =  cz. 

1            1            1 

x          y    ~    c' 

81. 

.(•2  —7/ 2  =^,2^ 

32. 

2-1 

•^             2+1 

(x-2,-)(2+l)  =  2^, 

a;— j/  =  6'z. 

{x^-y-)(z+lYz=^ht. 

KXAMINATIOxN    PAPISRS.  207 


CHAPTEK  VIL 


ExAjnNATiON  Papers  :     Education   Department   and    UNrsraRsixr 
OF  Toronto, 


I. 

1.  State  the  rules  for  the  addition  and  subtraction  of  Algebraic 
quantities.     Express  in  the  simplest  form 

(b+c  —  a)z+  {c-\-a  —  b)i/-\-(a-\-h  -  c)z 
{c-\-a  —  h)x  +  {a-{-h  —  c)y  ■\-  {h-\-c  —  a)z 
{a-\-h  —  c)x-^{b-\-c  —  a)ij-\-[c-\-a  —  b)z 

2.  State  and  prove  the  Index  Laws.     Assuming  these  to  be 
general,  interpret  x~"'. 

Find  the  products  in  the  following  cases  : 

(1)  (x3  +  6x3j,  +  l2a;?/2+8v3)(a;S-6a;2y+12x?/2  -8^/3). 

(2)  (^a  +  h-\-r){b^c-a){c+"~b){a^h-c). 

3.  Prove  the  rule  of  signs  in  Division. 
Divide  :    [Apply  Homer's  Method  to  (1)] 

(1)     x-6-22.c4+60a;?- 55x2 +  12x4-4  by  x2  +  6.r+l. 


(2)     a;*  +  9-|-81x-4  by  a;2  _  g  +  g^c-a. 

(3)     a;"'  -  1  1: 

4.  Find  the  square  roots  of 

(1)    4a;4'"-  —  ^""    +   TT^^"* 

(2)    -nj   +   —   4-   —  -  2—   - 
b^           c          a^              c 

-   2f  +  2I. 
1}              a 

6.  Distinguish  between  an  algebraic  equation  and  an  identity. 
Solve 

(1)    ^(l-2.i)-H|^(l4-2x-)  =  3, 


208  EXA5IINATI0N    PAPER3. 

(2)    "-Z^   4.-'^    -    2  '^. 
^   ^    x-j-2   +   x-2    -     _-x--d 

6.  A  person  bought  a  certain  number  of  oxen  for  $o20.  If  be 
had  been  able  to  purchase  four  more  for  the  same  sum,  each 
would  have  cost  him  $4  less.  Find  the  number  [of  oxen.  Ex- 
plain the  negative  result. 

7.  (1)  If  ^   -   -1  shew  that  «_!±M±8L^    ^   ^i^^. 

(2)    Find  the  value  of  x''  -200a;5+198a;4 +200x3  -  197a;2 
-397a;  when  a;  =  199. 

8.  Three  towns,  A,  B,  C,  are  at  the  angles  of  a  triangle.  From 
A  to  C,  through  B,  the  distance  is  82  miles  ;  from  B  to  A,  through 
C,  is  97  miles  ;  and  from  C  to  B,  through  A,  is  89  miles.  Find 
the  direct  distances  through  the  towns. 

II. 

1.  Prove  x"  4-  a"  =  a:"""". 

Simplify  {a+h-\-c)- ~d{a  +  b  +  c)^c  +  5{a-{-I>-^c)c^  -e». 

2.  Prove  the  rule  for  finding  the  L.  C.  M.  of  two  quantities. 
Find  the  L.  C.  M.  of 

a^Jl.])^j^c^-Qabc,  and  (a  +  h)^ +2{a-}-b)c+c-. 
a  c  ac 

8.  Prove -^   ><   "J  =   fc^* 

/1-.t3  1-x     \        I'  l^x  l-x^\ 

Simplify  [^^^  +  xz-^q-^.l  -^  Ji^^-s  -  i:^]- 

4.  Eeduce  to  their  lowest  terms  — ^r^^. — ^ — ^,  and 

5.  (1.)  li  a^ —pa" -'rqa  —  r^O,  then  x^ —px" +qx  —  r  is  exactly 
divisible  by  a:  —  a. 

(2.)  Prove  that  {a  +  h  +  c){bc-{-ca  +  ab)-{b-irc){c+a){a  +  b)i& 
divisible  by  abc.     Is  there  any  other  divisor  ? 


EXAM  IN' ATI  ON    PAPERS.  209 


la  +  b\  ?^    ,        ,  a^-b^ 

6.    If^=     M""'"'^^^"i.^+6^(v^-^-i--^) 


7.  Solve  the  equations — 

3-2^         5--2x   _  4a;- -2 

(3 )  !!l±?    _   ^±^    _   i^±^  _   l^-g+17 
a;+i    ~   x+2   ~    2x'+7  ~     G^  +  IO' 

8.  A  pei'son  going  at  the  rate  of  jy  miles  an  hour,  and  desiring 
to  reach  home  by  a  certain  time,  finds,  when  he  has  still  r  miles 
to  go,  that,  it'  he  were  continuing  to  travel  at  the  same  rate,  he 
would  be  q  hours  too  late.  How  much  must  he  increase  his 
speed  to  reach  home  in  time  ? 

9.  Of  the  three  digits  comprising  a  number,  the  second  is 
double  of  the  third  ;  the  sum  of  the  first  and  third  is  9,  and  the 
sum  of  the  three  digits  is  17.     Find  the  number. 

10.  A  owes  B  $a  due  m_  mouths  hence,  and  also  $b  due  n 
months  hence.  Find  the  equation  which  determines  the  time  at 
which  both  sums  could  be  paid  at  once,  reckoning  interest  at  5 
per  cent,  per  annum. 


III. 

1.  Ifa;=10,  y  =  ll,  2=12,  find  the  value  of 

{  z^  -(y-{-z)-  r  X   ■^r — r  ;  and  subtract 

{y—z)a'^  +  {z-x)ab-\-{x  —  y)ii'^  from 
{y  —  x)a^  —  {y—z)ab  —  {z  —  x)h^. 

2.  Divide  a +  (rt+&)j;  +  (rt-ii/;  +  c)a;3  ^{aJ^h+ifx^ +  {h-\-c)x* 
+  cx^hy  \-'f-x+x^-\-x^  ;  and  find  the  square  root  of 

9  -  2-lx+58a;3  -  llGu;3  +  129u;*  -  140.(;5+100.i;6. 

4x  +  5         x^5         2.C  +  5         a;2_io 

3.  Solve   (1)  ---^   4-  ^^   =   — --,   -   -^^:3-  +  x. 


£10 


STAMIXATION    PAPERS. 


(2'.   ^:,-fT,,^.T,^o,  ^x  +  iy-lz=-ll 

4.  A  boy  bought  a  number  of  oranges  at  the  rate  of  45  cents  a 
dozen  ;  if  he  had  received  20  oranges  more  for  the  same  money 
the  whole  would  have  cost  him  only  40  cents  a  dozen.  How 
many  did  he  buy  ? 

6.  A  farmer  took  to  market  two  loads  of  wheat,  amounting  to- 
gether to  75  bushels  ;  he  sold  them  at  difierent  prices  per  bushel, 
but  received  on  the  whole  the  same  amount  for  each  load  ;  had  he 
sold  the  whole  quantity  at  the  lower  price  he  would  1  ave  received 
$78.75  ;  but  had  he  sold  it  at  the  higher  price  he  would  have  re- 
ceived $90.     Find  the  number  of  bushels  in  each  load. 

6.  Show  how  to  find  the  square  root  of  o  +  y  b. 
Find  the  square  root  of  1-f  ^/(l— a^) 

^^+^         4a;— 1         7a; +1 

7.  Solve   ^ ^  -+-  5   =   — ■- — -  ;  and  find  the  value  of  i 

^X —  <  X — £i  X  —  o 

when  ax^  —  ^Qx  +  S\  =0,  has  equal  roots. 


^^^^a-b    ~    -i/(.7c)-  V{bd) 

9.  SliQw  thfit  a^{b  —  c)  +  b^(c  — a) -\-c^ [a  — b)  is  exactly  divisible 
by  a+b  +  c ;  and  resolve  the  expression  into  its  factors. 


IV. 

1.  MnltiTplj  a^+h^-c^+2ab  hy  u^  -b^ +c^ +  2ar,  and  divide 
the  product  by  a^  —b-  —c-  +'2bc, 

2.  Simplify 

x+y '    "    t    7(c+4      "     I  21^2~    -^    «(a;2  -  y^)l  f 


EXAMINATIOX    PAPSRS.  211 

3.  Find  tbeL.C.M.  of4^2-9^'^  Ax^ -lOxij  +  6;/^.  and  6x«  — 
lSxi/+6>j^,  and  the  G.C.M.  of  l+x'^+x-fa;'  and  2x  +  2x'  + 

■i.  Obtain  the  square  root  of  i  — |i/^-,  and  find  the  value  of  c 
when  Ax*  —  12x^i/+cx'^i/^  —  12x!j^-j-4:!/^  is  a  perfect  square. 

5.  Distinguish  between  an  eqnntion  and  an  identUy.  Give  an 
example  of  each.  Wbat  value  of  m  makes  (x  -  3)2  _  (x  -  l)(x  —  6) 
=  III  an  identity  ?     Can  any  value  of  in  make  it  an  equation  ? 

6.  Keduce  to  its  simplest  form 

l/(2+a;)-T/(l+x)     ^     l  +  i/{l -1  ^  (1 +a;)} 

i/(i+x-)-|/a:      "^  1  +  y  {i-fi-^i+^n 

7.  Solve  the  equations 

.  1 V     2j;  -h  5         2x—5         ix  —  5 

(2)    7i',y~5x=[x-5i/){x  +  '6y), 

'!_   _    _^ T_ 

x—5y         x+'6i/    ~    83* 

8.  A  person  performed  a  journey  of  22|  miles,  partly  by  car- 
riage, at  10  miles  an  hour,  and  partly  by  train,  at  36  miles  an 
hour,  and  the  remainder  by  walking,  at  1  miles  an  hour.  He 
did  tiie  whole  iu  1  hour  50  minutes.  Had  he  walked  the  first 
portion,  and  performed  the  last  by  carriage,  it  would  have  1*.iken 
him  2  hours  30^  minutes.  Find  the  respective  distancci;  by  car- 
riage, train  and  walking. 

9.  Solvu 

x-t^         x  +  1    _    4a?+9  12a;-}-17 

x-fl  ~  x+2  ~  u+7  ~  'ex+To' 

10.  What  value  of  y  will  make  2x*-\-3xy+Qy^  ezacily  divisible 
bya;-3? 

If  a  and  h  are  the  roots  "of  the  equation  x^  -\-x  +-1  =  0,  show 
that  a3_ 63=0. 


212  EXAMINATION    PAPERS. 

V. 

1.  Multiply 

Prove  that 

{lx  —  i/)^  —  (x  —  hj)^  is  exactly  divisible  by  x+y. 

2.  Express  in  words  the  meanine:  of  the  formula 

{x  +  n)(x  +  h)=x^+{a  +  h)x  +  ah. 
Retaining  the  order  of  the  terms,  how  will   the  right-hand 
member  of  this  expression  be  nffected  by  changing,  in  the  left- 
hand  member  (1)  the  sign  of  b  only,  (2)  the  sign  of  a  only,  (8) 
the  signs  of  both  a  and  6  ? 

a  Simplify  (flt  +  t)'^ +  («-&)* -2(a2-&2)2  ;  and  show  that 
(^a+b-\-c){h  +  c-a){a-{-c-b){a  +  b-c)  =  4.aH3 
when  a^+b^  =  c^. 

a  c  ad  ' 

4  Prove  that  y  -  "^   =   ^• 

Simplify 

[~2air  "^  ■^/    U"3T^"3)    '■    a^-ab+b*' 

5.  I  went  from  Toronto  to  Niagara,  85  miles,  in  the  steamer 
"  City  of  Toronto  "  and  returned  in  the  "  Eothsay,"  making  the 
round  trip  in  5  hours  and  15  minutes  ;  on  another  occasion  I 
went  in  the  "  Eothsay  "  (whose  speed  on  this  occasion  was  1  mile 
an  hour  less  than  usual),  from  Toronto  to  Lewiston,  42  miles,  and 
returned  in  the  "  City  of  Toronto,"  making  the  round  trip  in  6 
hours  and  30  minutes  ;  find  the  usual  rates  per  hour  which  these 
steamers  make. 

6.  Solve 

3  2  1  2.1  2 

(1) =   — ' =   — • 

^  '     x  y  a  X  y  a 

(2)    a;2+5a:=5^/(a:3  +  5.r+28)-4. 

7.  Find  three  consecutive  numbeis  whose  product  is  48  times 
the  middle  number. 


ErA^nXATION    PAPERS.  218 

8.  If  OT  and  n  are  the  roots  of  ax^+bx-^c  =  Q,  then 

nx^  +hx+c  =  a(x  —  m){x  —  n). 

Show  that  if  ax-  -{-hx+c  =  Q  has  equal  roots,  one  of  uitm  is 
pven  by  the  equation 

{'I'l-  -lalAx  +  ah-b"  =Q. 

9.  If  —    =   — aud-r-  +  Vtt  =1,  prove  mat 


YI. 

1.  Simplify 

2.  Divide  a'  — ^3_c3_3a7,c  by  a  —  b  —  c,  and  show,  without 
expansion,  that 

(1  +x+jr-  ;. 3  _  (1  „ a;  +  x3 )3  -  C.r^^*  +a;3  +  1) - Bx^=0. 

3.  Resolve  into  factors  a;'^  — Ja;^./"  +  ?/*,  and 

Ix"^  - 6?/2  -  a;,/  +  19x  +  33;/ -  3G  ;  and  prove  that 
b^{c-^a)+c^{a-\-h)  —  c(-(b-'rc)+abc  is  exactly  dis'*sihle  by 
b-\-c  —  a. 

4.  Apply  Horner's  method  of  division  to  find  the  value  of 
5a;6 +497x4 +  200x3  +  190x2 -218a; -2000  when  x= -99,  <ind 
the  vaue  of  Cx'^ +5x4-17x3 -Gx^+lOx- 2  when  2x- = -3x+l. 

6.  Find  what 

V(n+x)+V(a-x)  ,  ,  2ab 

— -^ '—1 1: -'.becomes  when  x  = . 

\/{a+x)—  \/{(i  —  x)  1  +  6* 

6.  If  a  and  b  be  any  positive  numbers,  prove  that 


1  a  ,      a  b 

T  +  r+-a  >  '•  T  +  -„■  > 


.+.'-•■    -  ^- 


214  EXAMINATION    PAPKSS. 

7.  Solve  the  equations — 

(1)  cr-  +  /  =  5,_ 

1  I 

5  5 

•»     +  y      =  |. 

(2)  a;+22/+82  =  U, 
2a;  +  3i/  +  z  =  ll, 
3x+?/+22  =  ll. 

(3)  (x+l)(x  +  3)(x  +  4)(x+0)=rI3. 

8.  There  are  three  consecutive  numbera  such  that  the  sum  of 
their  cubes  is  equal  to  1G|-  times  the  product  of  the  two  higher 
numbers  :    find  the  numbers. 

9.  (1)  Form  an  equation  three  of  whose  roots  are  0,  y  (— 3), 

and  1-1/2. 

(2)  If  one  of  the  roots  of  the  equation  x'--\-px-\-q  =  Q,  is  a 
mean  proportional  between  p  and  q,  prove  that 

10.  Two  trains  start  at  the  same  instant,  the  one  from  B  to  A, 
the  other  from  Aio  B;  they  meet  in  1^  hours  ;  and  the  train  for 
A  reaches  its  destination  52^  minutes  before  the  other  ti-ain 
reaches  B  :  compare  the  rates  of  the  trains. 


VII. 


1.  Give  some  application  of  the  ''rule  of  signs"  in  Algebraic 
Multiplication  and  Division. 

2.  Find  the  numerical  value  of  the  quantity 

bc(c  —  a)(a  —  b)  —  ca{a  —  h)(h  —  c)-{-ah(b  -c){c  —  a), 
when  a  =10,  6  = -01,  t;  =  0;  and  pr6ve  that  if 

H,  = ,  then  will  {a-\-b)  .        ' 


a-\-h  a-\-b  —  c-\-x 


EXAMINATION    PAPERS.  216 

8,  Inveptigate  a  method  of  finding  by  inspection  the  remainder 
after  dividing  any  rational  and  intogral  function  of  x  by  x-\-a. 

Show  that  the  quantity 

ifl  divisible  by  each  of  the  quantities  x-\-i,  x-\-b,  a—^x.  h-x, 

4.  Investigate  the  rule  for  finding  the  H.C.F.  of  two  algebraio 
quantities,  showing  under  what  limitations  factors  may  be  intro- 
duced or  suppressed  at  any  step. 

Find  the  H.C.F.  of 

(1)  6.r4  _  7x3  _  i3a;2  +  I9a;_  6  and  x^  +  2a-3  - 1. 

(2)  {x+y){ax^-h!i-^)—xy{a-h){x-iry),  and 
{x-y)(iix^-hj^)  +xi/{a-b){x-y). 

5.  Prove,  by  general  reasoning,  that  the  value  of  a  fraction  is 
not  altered  by  multiplying  or  dividing  both  the  numerator  and 
denominator  by  the  same  quantity. 

13  7  X     i 


Simplify       (1) 


12(2a;-3)  12(2j,-+3)         Ax^'+d' 


(2)     1 1 +  ^L__^    I    . 


1  1 

+ 


[{x+a)(x+b)  {x  —  a){x—b) 

Solve,  with  respect  to  x,  the  equations 
,,.    z-lS         2.r-24  lla;-34  7 


(2) 


4         '         11         '  ii2  44 

5x^+x-B         7x3 -8a;- 9  a;- 3 


5a;-4  7a;- 10  35a;3-78a:+40 

(3)    x^  =  ax-\-hy,  and  ?/3  =  hx-'r  ay. 


VIII. 
1.  Define  the  terms  "  power,"   "root,"  "  index,"  and  '*  coeflfi- 
cient ;  explain  also  the  reasoning  by  which  it  is  shown  that 
a  —  {h  —  c)  z=  a  —h  -^- c. 


219  EXAMINATION    PAPEK3. 

2.  Multiply  (x2 +.<•// +  2/2) 2  by  ix—y)'^. 
Find  the  values  of  a  and  h  whicli  will  make 

X'  +ax+b  divisible  by  x-^p,  and  also  by  a;  +  j. 

y.  Divide  x^+7/^-{-2x^ij^  by  (x^yy,  and 

4.  Investigate  a  rule  for  the  extraction  of  the  square  root  of  any 
algebraic  quantity,  and  deduce  the  rule  for  the  extraction  of  the 
square  root  of  a  number. 

If  to  any  square  number  be  added  the  square  of  half  the  num- 
ber immediately  preceding  it,  the  sum  will  be  a  eomplete  square  : 
viz.,  the  square  of  haK  the  number  immediately  following  it. 

6.  Find  the  square  root  of 

(1)  a^x'^  +  '2abx'^  +  (b^ -i-2ac)x^ -i-c^x-'^  +2bc. 

(2)  ix'-i:<:"  +  j/+i./-K'  +  A^'- 

6.  If  x^-r-ax+h  and  x^+a'x-b  have  a  common  measure,  it 

will  be  x-{-  — o — »  a,ud  the  condition  that  they  may  have  a  com- 
mon measure  is  ib^a-  —a'^. 

Find  the  H.  C.  F.  of  x'*' +2)^ x^ -}-p*  and  x^ +2px^  +p^x''  -p^. 

Find  the  L.  C.  M.  ot  2\{x^-^x-20),  d\{z^-x-ZO),  and 
4i(x3-10x  +  24). 

7.  Find  values  of  a  and  h  which  will  render  the  fraction 

Zx^-{ia-{-h)x+a  +  2b^ 
bx"  -  (Qa+b)x- a^Ab^ 
the  same,  for  all  values  of  x. 


d.  Solve  the  equation  2  +  |/(d;-i-l)(x+6)  - -j/(a;-l)(x-f  5)  =  0, 
and  account  for  the  circumstance,  that  the  values  of  x,  determined 
from  it,  apparently  do  not  satisfy  the  equation. 


EXAMINATION    PAPBR3.  217 

IX. 

1.  Prove  that  a{2ni-l){a''  +7r7TTl) - "(2" -M)(«^  +«•«  +  !) 

=  (rt  — n)'. 

2.  If  a,  b,  and  c  are  positive  quantities,  and  if  a>^  and  c>a-  6, 

prove  that 

e  -  {a  —  b)  =c — rt-fy. 

Assuming  this  equation  to  hold  good  when  a,  b  and  c  are  unre- 
stricted, prove  that  the  expression -(  -a),  occurring  in  an  algeb- 
raic operation,  is  equivalent  to  +a. 

3.  11  x^ -^ax- -^h  and  x3+7"-  +  5'  have  a  common  measure  ot 
the  form  of  x^+mx+n,  then  a^hq  =  {h-qY 

4.  Find  the  H.  C.  F.  of 

a'^-h'^-abxij-^ahx-^y-^,  and  a-x^ -b^y-^ -\-a-bx'^y-b^-X]r''* 
;>.  A  and  B  are  two  numbers,  each  of  two  digits.  The  left- 
hand  digit  of  A  exceeds  that  of  Bhj  x;  the  excess  of  A  above  B 
is  y ;  but  the  sum  of  tlie  digits  of  B  exceeds  the  sum  of  the  digits 
of  -4byz.  Pi-ove  that  y^z  =  'dx;  and  give  an  example  of  two 
such  numbers  as  A  and  B. 

^     -^l  a.   -   —    =   — ,  prove  that  each  of  these  ratios 

■^a        ^    ,  a+b-\-c 

7.  Solve  the  equations 

x±a         x—a    _    b-^x   _   b—x 
^  '    x^a   ~   x+n   ~    h  —  x         b+x 
(2)    a(x2+y3) -6(3-3-2/2)  =  2a 
(rt2_fe2)(a;2_j,y2)         =inb. 

8.  A  farmer  buys  a  sheep  for  $P  and  sells  6  of  them  at  a  gaia 
of  6  per  cent.  ;  at  what  price  ought  he  to  sell  the  remainder  to 
gain  10  per  cent,  on  the  whole  ? 

9.  The  sum  of  three  numbers  is-70  ;  and  if  the  second  is  divided 
by  the  first,  the  quotient  is  2,  and  the  remainder  1 ;  but  if  the 
third  is  divided  by  the  second,  the  quotient  is  3,  and  the  remain- 
der is  3  ;  what  are  the  numbers. 


'^'•°  BXAMfNATION    PAPERS. 

X. 

bya;+?/+2.  ''^     ' -^^ 

_    2.  Prove  that  if  x^^px^+qx+a^   be  divisible  by   a:^-!     it 
IS  also  divisible  by  x^  ~a^.  ' 

3.  Explain  the  reason  for  introducing  or  suppressiug  factors  in 
tae  process  of  finding  the  H.O.P.  of  two  algebraical  quantities. 

Why  is  the  name  "  Greatest  Common  Measure  "  objectionable  "> 
Find  the  H.C.F.  of  x*_^.3_^2  _^_2  and  Sx^ -Ix'^+s^.^^, 

4.  A  traveller  leaves  A  for  B  at  the  same  time  that  another 
leaves  5  for  A ;  the  former  walks  at  the  rate  of  3  miles  an  hour 
till  he  has  performed  half  the  distance  ;  he  then  rests  for  an  hour  • 
after  which  he  resumes  his  journey,  walking  now  at  the  rate  of  4 
miles  an  hour  ;  the  second  traveller  goes  at  the  rate  of  4  miles  an 
hour  tiU  he  has  got  over  one-third  of  the  distance  between  B  and 
A;  he  then  rests  for  40  minutes;  after  which  he  resumes  his 
journey,  walking  now  at  the  rate  of  3  miles  an  hour  The  tra 
Tellers  reach  A  and  B  respectively  at  the  same  time.  Find  the 
distance  between  A  and  B. 

5.  Show  by  examining  the  square  oU+b  how  the  squave  root 
Oi  an  algebraical  quantity  may  be  found. 

Find  the  square  roots  of 

(1)  25x^~B0ax^+A9a2x^  -2ia^x+Wa^,  and 

(2)  $  +  ^-     l±^JL]y2  +  ±. 

y       ^-       \y       xr        2 

OT 

6.  Show  that  a"  =  Va^,  when  m  and  n  are  integers,  and  m  is 
divisible  by  n;  and  state  the  principle  on  which  you  would  main- 
tain the  truth  of  the  equation  for  all  values  of  m  and  n. 

7.  Solve  the  equations 

(1)  ^^^+^-^   _   7^2 -8a;- 9 

5a; -4       ~    "   7»~-T0~"' 

(2)  {Bx-l)^  +  {ix-2y^=(5x~8)^. 


EXAMINATION    PAPERS. 


219 


8.  Two  regular  polygons  are  so  related  tliat  the  number  of 
their  sides  is  as  2  to  3,  and  the  magnitude  of  their  angles  as  3  to 
4  ;  find  the  figures. 


XL 

1.  State  in  words  the  several  operations  to  be  performed  m 
order  to  obtain  the  result  expressed  by  the  following  algebraical 
expression : 

*  jma^  -\-nb^ 
m-r'H 
Also  find  its  value  when  a  =  6  =  4. 

2.  Two  men,  A  and  B,  dig  a  trench  in  8|  days.  If  A  were  to 
do  more  work  by  one-thu'd  than  he  does,  and  B  more  work  by 
one-half  tliau  he  does,  they  would  dig  the  trench  in  2|§-  days.  lu 
what  time  would  each  dig  it  alone,  at  his  present  rate  of  work  ? 

3.  Perform  the  multiplications  in 
(1) 

/  2  J  +  3/y^  )  (  2a:^  -  2^/'  \Ux'-h  Gx^/+  ^y^  )  (  4a;*  -  Qx^i/  +  Oy^  \ 

(2)     ax^+ixy-fi!/^)i^x^--ixy-\-%y^). 

4.  Divide 

(1)  x^  +  d  +  Slx-*  by  x^-B  +  9x-^. 

(2)  x'*^  —  {n-\-b+p)x^-{-iap-{-hp  —  c+q)x^-{aq-{-bq-cp)x  —  qc  by 
X-  —px+q. 

5.  Show  that  a;"'" +^  —  aj^^-i  is  always  divisible  by  x±il,  m  and  n 
being  any  positive  integers. 

6.  Define  a  fraction  ;  and  from  your  definition  prove  a  rule  for 
adding  together  two  fractions  with  different  denominators. 

Add  together  the  fractions, 

a^ —  he  6- — ca-  c^—ab 

(a  +  b)(a+cy    {b  +  c){b-^ay    [c  +  a){c  +  b)' 


220  KXAillNAl'lON    i-APi-KS. 

7.  Solve  the  following  equations  : 

(1)  ^'^  +  2a;4-2         x"  +  Sx+20-_   x^+Ax  +  6         x^  +  6x+12 

'~x  +  l        "^         x+i  ~        ^+2        "'  x^       * 

(2)  (x^+y^)-^   =    ^^    (a;2-^2)J^   ^   ^. 


XII. 

1.  When  7n  and  to  are  whole  iiumbers,  and  m  greater  than  n^ 

a'"  1 

show   that  —  =  a"*""  and  that  —^  is  correctly  symbolized  by  a~" . 

2.  Multiply  (a -i)(a+6)(a-+/>2)(a4_^^4)  .  .  ,  to  (n  +  1)  factors. 

3.  Divide  1  — tc  by  1  — 2a;,  to  5  terms,  and  write  down  the 
(?-+l)th  term,  and  the  remainder  after  (r+1)  terms. 

4.  If  the  number  three  be  divided  into  any  two  parts,  show 
that  the  difference  of  the  squares  is  three  times  the  difi'ereucQ  of 
the  numbers. 

5.  Find  the  L.  C.  M.  of  1  -8x+nx^+2x^-2ixA,  and 

l-2x-rSx-^+S8x^-2ix^. 

6.  What  relation  must  there  be  between  the  coefficients  /«,  h, 
■p  and  q,  in  order  that 

{x^  -^mx+JiY -\-j)X-^  +  qx 
may  be  an  exact  square  for  all  values  of  x  ? 

7.  Solve  the  following  equations  : 

l+a;3  i-x' 

(1)  (rq-jy2  +  (i_;,)2  -  ^' 

ax  —  h^  ^{ax)  —  h 


(2) 


^/(«x)+& 


(3)    — r     =  1,  -jr   =  2,  and  -^—   =   3. 
^   '    x-\-y  x-^-z  y-t-z 

8.  Given  a; +.V+Z  =  «•«  =  %.  find  (x+y+z) -^z. 

9.  Find  a  number  expressed  in  the  decimal  notation  by  two 
dibits,  whose  sum  is  10  ;  and  such,  that  if  1  be  taken  form  its 
double,  the  remainder  will  be  expressed  by  the  same  digits  in  a, 
reversed  order. 


EXAMINATION  PAPERS.  221 

XIII. 

1.  Find  the  value,  when  a  =  2i,  fc  =  3i,  c  =  4^  of 

2.  Show  that  the  vahie  of  the  expression,  in  the  preceding 
question,  is  not  altered  hy  changing  a  into  a+x,  b  into  b-{-x,  and 
c  inte  c-\-x. 

3.  Multiply  (1 4- «i  a;)  (1  +  "  2^)  (1+^3^)  •*•  (l+^na-')  to  3  terms. 

4.  A  speculator  borrows  a  sum  of  money  at  the  yearly  interest 
of  7  per  cent. ;  part  of  the  amount  he  Invests  at  8^  per  cent.,  and 
the  remainder  at  9 ;  and,  at  the  end  of  the  year,  he  finds  that  he 
has  made  a  profit  of  $75  ;  but,  had  the  former  part  been  invested 
at  9  per  cent.,  and  the  latter  at  8^,  his  profit  at  the  end  of  the 
year  would  have  been  only  §65.     Find  the  whole  sum  borrowed. 

5.  Given  ax-{-l'i/  =  c,  a'x  +  h'i/  =  c',  determine  the  value  of 
mx-\-ny,  and  find  tiie  conditions  under  v^'hich  the  value  becomes 
indeterminate. 

a„  a^  a- 


thenwilla, -|-nf„+r?,+  .     .     .   +  a„  = 

7.  Eliminate  x  and  y  from  the  equations 

a  a  a 

a;     -r   y     =    « 

a.    =   x+dx^y' 
/5    =    y-^-^x'y^. 

8.  li  ax'  -^-hx+c-^O  a'  d  7j,r2 -f-5^x-|-Cj  =0,  then  will 

9.  Find  that  number  of  two  figures  to  which  if  the  number 
formed  by  changing  the  placos  of  tiie  digits  be  added,  the  sum  is 
121 ;  and  it  Lai^  bame  two  numbers  bo  subtracted,  Ui©  remainder 
is  U, 


222  EXAMINATION    PAPERS. 

XIV. 

1.  Simplify 

a(b  +  cy  +  h{c  +  ny-i  +c{a  +  b)^  -  {{a-h){a-c)(b+c)  + 
{b~c){b-a){c  +  a)  +  {c-a){c-b){a-\-b)}. 

2.  State  tiie  law  of  Indices,  and  prove  it  for  positive  integral 
indices  ;  and  assuming  it  to  be  general,  interpret  the  expressions 

x~"\    X  ,  where  m  and  n  are  positive  integers. 

8.  Having  given  the  equations, 

x-\-]j^z  —  Q,  «  -f/z'+z' =  0, 

prove  that  a^{yz—y'z')  +  h'^ [zx  —  z'x')+c'^ [xij ~x'y')  =  0. 

4.  A  traveller  P  sets  out  to  walk  from  A  to  B,  proceeding  at 
the  rate  of  3  miles  an  hour ;  and,  82  minutes  afterwards,  another 
traveller  Q  sets  out  to  walk  fro^^i  B  to  A,  proceeding  at  a  uniform 
rate.  They  meet  half  v/ay  betwixt  A  and  B.  P  then  quickens 
his  pace  by  1  mile  an  hour ;  and  Q  slackens  his  1  mile  an  hour. 
Q  reaches  A  at  the  same  time  that  P  reaches  B.  Find  the  dis- 
tance between  A  and  B. 

5.  How  are  equations  classified  ? 
Solve  the  equations — 

(1)  nmx+dmnrzn^x-j-mu'^ . 

(2)  x*-x^+y^-y^  =  84:, 
x-+x"y^+^j''  =  i9. 

6.  What  two  numbers  are  those  whose  difference,  sum  and 
product  are  to  each  other  as  the  three  numbers  2,  3,  5  ? 


XY. 
1.  What  is  the  meaning  of  the  symbols  a,  a~,  a'^     .     .     ? 
Show  a  prion  that  a°  =  1  ;  how  do  you  know  that  ab  =  ba  ? 
How  is  it  proved  that  the  multiplication  of  hke  signs  gives  a 
positive,  and  that  of  unlike  signs,  a  negative  result. 


EXAJnNATION    PAPERS.  223 

2.  FiuJ  the  valne  of 

(b-cy  +2{c-ay  +  (a-L)^  -3{b-c){c-a){a~b) 
Avlien  a  =  1,  b=  —  J,  c  =  |. 

3.  Simplify  the  following  expression  : 
(ac-b^'){ce-d-2)-\-{ae-c^)(bd-c^)-{ad-bc){be-ra) 

4.  P  aud  Q  are  travelling  along  the  same  road  in  the  same 
direction.  At  noon  P,  who  goes  at  the  rate  of  m  miles  an  hour, 
is  at  a  point  A  ;  while  Q  who  goes  at  the  rate  of  n  miles  in  the 
hour,  is  at  a  point  B,  t\YO  miles  in  advance  of  A.  When  are  they 
together  ? 

Has  the  answer  a  meaning,  when  to— n  is  negative  ?  Has  it  a 
meaning  when  m  =  n  ?  If  so,  state  what  interpretation  it  must 
receive  in  these  cases. 

6.  Show  how  to  find  the  Least  Common  Multiply  of  two  or 
more  algebraic  quantities. 

(1)  x^  --ax-2a^,  x^+<ix^  aud  ax^-x^. 

(2)  x^-x-i/-a^x  +  a'-^ir^^^x-^+('x^-xy--ay3. 

In  what  algebraic  operations  is  the  Lowest  Common  Multiple 
of  two  or  more  quantities  required  ? 

6.  State  and  prove  the  principle  upon  which  the  rules  of  Addi- 
tion and  Subtraction  of  fractions  are  founded. 

Simplify  the  following  e:icpressions  : 

(a^b-c)^  -d^  {b  +  c-a)^-d^  (c+a-b)^-d2 

(1)  (^6)2_(c+cZ)2  +  (6  +  c)2-(a+f/)3    "^   {J+af^-{b+df' 
rj.ij^yi-z^  +  2.xij     a^-[-a^h         a{a-b)  2ab 

(2)  x--y^-z--^2ijz     aH-b^   ~   (a+h)b   ~   a^-0^' 

7.  U ax-~hij  +  o{x-y)  =  {a - b){a-{-h  - c), 

by  -  cz+aiy-z)  =  {b-  c){b+c-a), 
cz  —  ax+b(z  - x)  =  {c  -  a){c  +  a  —  b) 
then  will  a^{h-c)+b''{c-a)  +  c\a-h)  =  0. 

8.  P  is  a  number,  of  two  digits,  x  being  the  left  hand  digit,  and 
y  the  right.  By  inverting  the  digits,  the  number  Q  is  obtained. 
i?rove  that  11  (x+y){P-  Q)  =  9  {x-y)  {F+Q), 


224  EXAMINATION    PAPEKS. 

XVI. 

1.  Show  that 

{{nx+hijy  +  {a7j -bx)'  \  {(nx+hij)^  - (ay+hx)-}  = 
(a<i-64)(:c't-2/4);  and  that 
2{a-b){a-c)-{-2{h-c){b-a)-i-2{c-b){c-a) 
IS  the  sum  of  three  squares. 

2.  If  s  =  a  +  i-(-c-|-(i;c.  to  n  terms,  then 

s  —  a  s  —  b  •"'  —  <^,p  1 

-1_     J.      +     &C,      =     77   —   1. 

S  .V  S 

3.  Show  that  a  —  b,  b  —  c,  and  c—a  cannot  be  all  three  positivo 
or  all  three  negative. 

4.  Extract  the  square  root  of 

4.c«  +  Ox^  -  12j:*  +  lGa;2  +9  -  2xiGx^  -  8a;4  +  ^x^  - 12). 

5.  Gis-en  ab  -  \{fi-{-b){p  +  q)-{-pg  =  0, 

find  the  val'.ie  of  p  —  q,  and  show  that  if  eitlier  a  or  b  is  equal  to  e 
or  d,  then  p  is  equal  to  q,  unless  a  +  b  =  c-\-d. 

6.  Find  the  value  of  — ,  having  given 

y 


7.  Prove  that  {a  —  b){b—c){c  —  a)  is  a  common  measure  of  the 
quantities 

(„2  _t2)5_^(/;2  _c2)5  +(^.2  _^3y5^ 

8.  Find  the  conditions  that  a^x+b.^y  =  c.y,  a^x+h„y  =  r^,  and 
a  a;_|-/i  T/^Cg  may  be  satisfied  by  the  same  values  of  x  and  ij. 

9.  Two  persons,  A  and  B,  start  at  the  same  instant  from  two 
stations  (c)  miles  apart,  and  proceed  in  the  same  direction  along 
the  line  joining  t}\e  stations  with  velocities  (a)  and  (b)  miles  per 
hour.  Find  ihe  distance  (x)  from  the  stations  where  A  over- 
bakes  B,  and  interpret  the  result  when  a  z  6. 


EXA^^^•ATION  papees. 


225 


XYII. 

1.  Express?  in  symbols  the  result  of  snbtractinfr  from  unity  the 
quotient  obtained  by  dividing  the  sum  of  a  and  h  by  their  product. 

2.  Multiply  to;^ether  x  +  y/a  +  Z',  x—\/a-^h,  x-\-i^/a-h  and 
X—  Va-  b  ;  and  divide  24^f3  ...'■2SLa-b  +  ^a'^c  —  5ah~-\--llahc  —  o\ac^ 
+  G63-22/>2c4-lG6c3  +  8c3  by  'da-1h+4.c. 

3.  If  x+a  be  the  H.  C.  F.  of  x^-\-}:x-\-q  and  x-  ■\- p'x+q', 
their  L.  C.  M.  will  be  {x+a){x+p  -  a){^x  -^ p'  -  a). 

Show  that  the  difference  between 

X  x  x         ^    a  be 

-f   7   + and + ;   +  -^ — - 

x  —  u         x  —  b         x  —  c         x  —  a         x—o         x  —  c 

IS  the  pame  whatever  values  be  given  to  x. 

4.  Prove,  if  the  four  fractions 

bx+nj+(Jz       cx+chj-\-az       dx+<nj  +  hz       ax  -j-b]/  -\- rz 

b+c-i^d—a       c  +  d-ru  —  b'      d+a  +  b~c       a-}-b+G  -  d 

are  equal  to  one  another,  their  common  value  will  be  ec^ual  to 

-—'- —  as  long  as  a  +  b  +  e+d  does  not  vanish. 

6.  What  do  you  mean  by  solciivj  an  equatioit.  Show  that  3  is 
a  root  of  the  equation 

8  +  if(x-2) 

6.  Eliminate  x  between  the  equations 

x3  +-73  +  3  x  +  — )  "=  "h  and 

7    If  4.  —  _  —   =    — — , ,  a,  6,  c  are jaot  all  different, 

a  b  c  a-{-  0  —  c 

8.  A  cas'k,  A,  contains  m  gallons  of  wine  and  n  gallons  of  water; 
an  another  cask,  B,  contains  p  gallons  of  wine  and  q  gallons  of 
water,  how  many  gallons  must  be  drawn  from  each  cask  so  as  to 
produce  by  theii-  mixture  b  gallons  of  wine  andc  gallons  of  water  ? 


226  EXAMINATION     PAPES3, 

XYIII. 

1.  Multiply  together  tlie  factors 

1  —  x,  1+x,  l+x^,  1+x*,  and  1+x^, 
and  sho-w  that  if  7i  is  any  uneven  number,  the  S'lm  of  the  nth 
powers  of  any  two  numbers  is  always  divisible  by  the  sum  of  the 
numbers. 

2.  Find  the  numerical  value  of  the  expression 

c       */a+  */c 
b       */a—  -v/c 
where  a,  h,  c  are  connected  by  the  equation  a''b  —  c)^  —  c(b+c)'^=0. 

3.  A  has  a  younger  brother,  B.  The  diffei'ence  between  their 
ages  is  §•  of  the  sum  of  their  ases.  By  adding  twice  -B's  age  to 
5  times  ^'s,  we  obtain  the  age  of  the  father ;  and  by  subtracting 
twice  -B's  age  from  5  times  .4's,  we  obtain  the  age  of  the  mother. 
Show  that  the  age  of  the  mother  is  y\  that  of  the  father. 

4.  Find  the  H.C.F.  of 

z^-{2a+b)x^-ha{2a  +  l>)x-a^(a+b),  and 
xi-{2b+a)x^+b{'Ih+a)x-b^{b+a). 

5    If  J_  4.  _   =   — ,  shew  that 
be  a 

^a^],_c)s^2{b  +  c-a)^  +  {c  +  a-b)3  =  2(b+c)\ 

6.  Show  fully  how  the  rule  for  finding  the  square  root  of  a 
o-iven  number  is  obtained.  If  n-\-l  figures  of  the  square  root  of 
a  number  have  been  obtained,  prove  that  the  remaining  n  may  be 
obtained  by  division. 

Extract  the  square  root  of 

x^x^-+y^+z^)+y"z"-+1x{7j+z){yz-x^). 

7.  Find  the  value  of  the  expression 

x—v      ^  a+b  b 

"t £-  when  x  — ,  y  =  — 

14-xy  a-b  a 


ESAJIINATION    PAPERS.  227 

8.  Solve  the  equations  : 

(1)  ^{x-2a)  -Ux  +  Sa)  +  l(x-ea)  =  0. 

(2)  ^/(2a;3  4-l)+V(2.r3+3)  =  2(l-«). 

9.  Divide  21  into  two  parts,  so  that  ten  times  one  of  them  may 
exceed  nine  times  the  other  by  1. 


XIX. 

1.  Multiply  together 

Divide  this  product  by 

and  extract  the  square  root  of  the  quotient. 

1  1  1  ,         ,    . 

2.  If  a;-fv+2=  —  +  —  4-  —  =  0,  shew  that 

^  X  y  z 

(x^+y^+z^)-i-{x^+y^+z^)  =  xyz. 
8.  Find  the  H.  C.  D.  of  20x4+a;3-l  and  7oa;*  +  15a;3 -3a;-3  ; 
also  of  (x+yY  —x^  -y''  and  (x^  —y^Y' 
4.  Given  that  rti-(«4-fe)(a;-f2/)H-4a;2/  =  0, 
cfZ-(c+J)(a;-2/)  +  4a:£/  =  0, 
find  the  value  of  (x-  — ?/)3. 

fi.  Having  given 

x^  =7/2  4^2  —<2,ayz 

y-  =z^  +x'^  —  2lzx 
z2=x^+y-—2cxy, 
a;2  ^2  ^2 

Show  that  j3^^   =    3;_r^    =    j_^s- 

l+x+/(2r4-x'^)    _ 

7.  Determine  a;  in  terms  of  a  and  6  in  order  that  x^-^2ax^-\- 
Sb'-x^  —  'La^x+Ah*^  may  be  a  perfect  square. 

8.  A  company  of  90  persons  consists  of  men,  women,  and 
children  ;  the  men  are  4  in  number  more  than  the  women,  and 
the  children  exceed  the  number  of  meu  and  women  by  10.  How 
many  men,  women,  and  children  are  there  in  the  company. 


228  EXAMINATION    PAPERS. 

XX. 

1.  Divide  (l-{-w)x^  —  {m-\-n)x>/{x  —  y)  —  (n—l)y^  by 

x^  —XjJ+ll^. 

2.  If  x^+px^+qx+r  is  exactly  divisible  hy  x^-^-mx+n^fherx 
nq  —  n^  =  rm. 

3.  Prove  that  if  m  be  a  common  measure  oip  and  q,  it  "will  also 
measure  the  difference  of  any  multiples  of  p  and  q. 

Find  the  G.C.M.  oi  x^ -2->x^  +  {q-l)x- +i>x-q  and 
X"^  —  qx^  -\-{p  —  l)x^  +  qx  —  p. 

4.  Prove  the  rule  for  multiplication  of  fractions. 

Simplify  ^Jji-^^z:^  X  y'-^::^^^^  x  ""^^y)'^: 

a  a  a3  2a3  _^,3._.ai3 


a2_|.i2  a2_^2    -r   (^a-h){a^+b^)  a^-b^ 

5.  What  is  the  distinction  between  an  identity  and  an  eqmition  ? 
li  x  —  a  =  i/-\-b,  -prove  x  —  b  =  y-\- a. 

Solve  the  equation 

16x-lS         40^-43    _    32a;-30         20a; -24 
4a;— 3     "*"      8a; -y      ~    ~8x'^     "^     4a;-5  ' 

6.  What  are  simultaneous  equations  ?  Explain  why  there  must 
be  given  as  many  independent  equations  as  there  are  unknown 
quantities  involved.  If  there  is  a  greater  number  of  equations 
than  unknown  quantities,  what  is  the  inference  ? 

Eliminate  a;  and  2/ from  the  equations  ax  +  by=c,  a'x-\-h'y  =  c'. 
a"x  +  b"y  =  c". 

7.  Solve  the  equations — 

(1)  ■^/(n^x)-^-^{ii-x\=m.. 

(2)  3a;+?/+z=13,  3?/+2+a;=15,  32  +  a;  +  y=17. 

8.  A  person  has  two  kinds  of  foreign  money  ;  it  takes  a  pieces 
of  the  first  kind  to  make  one  £,  and  h  pieces  of  the  second  kind  : 
he  is  offered  one  £  for  c  pieces,  how  many  pieces  of  each  kind 
must  he  take  ? 


EXAMTVATTOy    pvrrRS.  220 

9.  A  person  starts  to  -walk  to  a  railway  station  four  and  a-half 
miles  off,  intending  to  arrive  at  a  certain  time  ;  but  after  walldng 
a  mile  aud  a  balf  be  is  detained  twenty  minutes,  in  consequence 
of  wbicb  be  is  obliged  to  walk  a  mile  and  a  balf  an  bour  faster  in 
order  to  reach  tbe  station  at  the  appointed  time.  Find  at  what 
pace  he  started. 

10.  (a)  If    ^   =.   ^  then  will    '^-±^'    =    "'''. 

(b)  Find  by  Horner's  method  of  division  the  value  of 
a:^+290a;4+279a;2-2892aj2-58Gd;-312  when  x=  -289. 

(o)  Show  without  actual  multiplication  that 
{rt  +  /)+c)3  -  („  +  Z>+c)(r/2  -ab-\-b2-bc-\-c''-ac)-3ahc  = 
d{a-{-b){b-\-c  j[c  +  a). 


Note. — In.  Ex.  6,  p.  87,  after  proving  that  a-^h-^,'  is  a  factor, 
we  may  proceed  as  follows  to  discover  the  remaining  quadratic 
factor : 

Tbe  quadratic  factor  must  be  of  tlie  form 

m{a^+J>-  +c^)  +  n{ob+hc+ca), 
in  which  m  and  n  are  independent,  being  either  zero,  or  a  positive 
or  negative  number.     To   determine  them   put  c  =  0,  then  the 
given  expression  gives 

{a^  +  b^-\-Bab{a  +  b)}^{a-\-b)  =  n^  +  b'^-^2(tb, 
but  also  ='m{ri^-\-b^}+nab.      :.  ?«  =  landw=2. 
,*.    a»-¥b34.c^  +  3(a  +  h)(h-}-c)(c+a)}^{a  +  b-{-c)» 
a^+b'+c^  +  2{ab  +  bc  +  ca)  =  {a-\-b-i-c)^. 


230  EXAMINATIOJJ    PAPERS. 


XXI. 


/ 1    .    1  \        /I 


1.  Find  the  value  of  ic' —    —  +  ~r) -f^' +  {'1 ]^"^~P 

■wheu  rt  =  i,  h  =  ^,  x  =  2y    Simplify 

2.  Find,  by  symmetry,  the  sum  of  {a-\-b+c)^ —(a-^h  —  c)^ - 
(a-b  +  c)^-{b-a  +  c)^,  and  oi  {a^-Aa^x  +  Sa'^x^ -2ax^ +  dx'^)^ 
and  {a*  +  4:a3x+Su^x^  +  2ax^-{-3x^)-. 

'6.  Explain  and  illustrate  the  signs  >,  < 

Prove:  .-c^ +?/2  >2.r//,  (.c+^Z+z)- >3(u;?/4-//z+z.v;),  and 
x^+l/^-\-z^  >  Sxyz. 


5     3 


4  ,  ^  h  h 


4.  Determine  the  value  of . <;+?/  — s  + 3a?  >/  z  ,  when  a;  +ij'  -z 
0,   &c.  :  of  a''  +lax'-^  +  Sx'  —  Sa^  —  {x*  +  lax^  —  8x^  —  3a-),   when 
x=  -1. 

p  mp 

5.  Show  that  (a"*) «"  =  a7  . 

Simplify  j  (-^y  *l~'  X  (  yj     '  X  */  (256),  and  divide 

a;  — Qax  -f-Scf  ic  +  Sa^aj"  —  2a   by  a;   —  2rt  x+a  . 
G.  If  u  =  }j  ix+'~-\  and  v  =  il  >/+    ~]  prove  that 

7.  Gold  is  19J  times  as  heavy  as  water,  and  silver  lOi  times. 
A  mixed  mass  weighs  4,160  ounses,  and  displaces  250  ounces  of 
water.  What  proportion  of  gold  and  silver  does  the  m^.ss  con- 
tain ? 

8.  Shew  that  l-{-px-\-qx"-\-rx^  is  a  perfect  cube  if  ^^2-3^^ 
and  q^  ='di)r. 

9.  Solve  the  equations  : 

IX— 2  ix  +  2 

(1)  "^x^  +    ^^^2  =  4- 

(2)  {x^  +  i^'^y-+x^yHx-'-y'r^+x''-;/^  =  32S,  x---y^  =  3. 

x^         2x+y  y^+x 

(3)  —  +  -^  =20  - ,     x  +  8  =  4:ij. 


•BXAMTNVTrON    PAPV,nS.  281 

10.  A  person  bnys  two  bale?!  of  clotli,  each  containing  80  yards, 
for  $240.  By  selling  the  first  at  a  gain  of  as  maeh  per  cent,  as 
the  second  cost  him,  and  the  second  at  a  loss  of  as  much  per 
cent.,  he  makes  a  profit  of  $16  on  the  whole.  Find  the  cost 
price  per  yard  of  each  bale. 


SECONI)  CLASS  TEACHERS,   1880. 


xxri. 

1    Find  the  value  of  x'^+u;^~166x^ -lGGr^+81x  +  8l  when 
J--  —-7  ;  and  the  value  of  x^  —  Zpx"^  +  {'dp'-^  +q}x—pq  wheii 
jC  =  ii-\-p.     (Arrange  the  latter  result  according  to  powers  of  a). 

■2.   What  is  the  conditioa  that  x-i-b  shall  be  a  factor  of 
ax'^  +bx+c  ? 

Find  the  factors  of 

{a).     (rH^-rt6)  +  2(63-«/;)  +  3^3-/;2)4.4(a-/;)2  ;  and 

(b).      {ax+b){bx  +  c)  {cx-\-a)  —  {'ix  +  c){bx-{-a)(cx  +  b). 

3.  What  must  be  the  relation  aoiong  a,  b,  c,  that  ax-  -^hi--\-G 
may  be  a  perfect  square  ? 

(a).  Extract  the  square  root  of 

(^a~b)^-4c{a''+b^){a-by+4:{a'^+b*)  +  8a^l,'. 

(h).  If  5  be  subtracted  from  the  sum  of  the  squares  of  any  four 
consecutive  numbers,  the  remainder  will  be  a  perfect  square. 
(Prove  this.) 

a  c  <•  li  I  n 

■4.  If     T~   =   "T   =   —F-^^^~l'    ~    —    =    — 
u  <(  J  h  in  p 

{a+r-{-e){h  +  l-\-n)  ah  +  cl-J-en 

prove  that     jir+dJ^k+^u:+p)    =    l^'dJn+f^- 

■     ab{x^ -;,■■') +xi/{i>^-h-') 
5.  («).  Reduce     , ,   o  , — jt~; T^TTirT  to  ifc.-i  lowest  terms. 

(i).  If  xi/-{-ijzi-zx=l  prove  that 

X  >i  z  4xiiz 

+  v--:f^  + 


\-x'    ^   l-y'   ^   1-a-    ~    {l-x''){l-y-){\-z^) 


202  EXAMINATION    PAPERS. 

6.  Prove  that 

2{x+2-f/(x2-4)} 

J-  7'  -^ 

(b)     {b+c  -  a)a''  +  {c  +  a  —  h]b^  +  («+/>  —  cy  = 

i  .1.  1  s  a  2 

(a  +  h-\-c)ia  +  //"  +C'')  —  2(t<"  +  i"  +c') . 

7.  Solve  the  equations — 

(a),     (i-  c)(^  -  a)  3  +  (c  -  a)r.y;-  /;)»  +  («-  i)(^-c) 3  =0. 
(b).    x+y  =  4x// ;  .y  +~  -  %2  i  s +^  -  ^2^-. 
((■).     x+>j+z  =  0. 

ax-i-bij  +  cz  =  0. 

bcx  +  ca>j-\-cibz-{-(a  —  b)(b  —  c)[c  —  a)  —0. 
x—1         x—B 


FISST-CLASS  TEACHERS,  1870. 


XXIII. 

1.  Investigate  Horner's  method  of  division. 

Divide   x^ --^x*" -SW +'25x^ +Bx^ -8x^ +  19x'' +Sx  +  10  by 
3a;*  —  21^;^  -i-Qx—Q,  showing  the  "  final  remainder." 

Find  the  value  of  2x^-{-80'dx^  -3dSx-'  +  l60ox-  -120ix+4:22, 
when  x=  — -i02. 

2.  Iif{x),  a  rational  and  integral  function  of  x  is  divided  by 

,,  .,      ■    {f{a)-fi0)}x+a.m-0Aa) 

'£-+7>x  +  q,  the  remamder  is ., > 

^  -"  a  —  i:> 

where  a,  ^  are  the  roots  oi  x-+px+(j  =  0. 

Examine  the  case  where  p-  =4:q. 

a.  Show  without  actual  expansion  that 

'^^b-'cy+b-^ic  -  a)  +c^a  -  b)        '^ 


EX.UIINATION    PAPERS.  233 

4.  Find  the  value  of  x  aucl  y  that  will  render  the  fraction 
..  „  ,  , 7- :t— ; 3-7  the  same  lor  all  values  oi  z. 

5.  Show  how  to  find  the  sum  of  n  terms  of  a  series  in  Geo- 
metric progression. 

(1)  Show  that  the  sura  of  n  terms  of  the  series 

l  +  /-  +  Vl  +  20(l  +  r)  +  (l+3r)(i  +  ;-)^+  .  .  .,  is n  {l  +  r)\ 

.11  1 

(2)  Sum  to  infinity  the  series  ^.  p+A.p.o  +  g  g.i  a+  •  •  •  • 

6.  Explain  the  notation  of  functions  :  prove  that  if 

f  (w)  =  l+mx+  "^^^^:^-^-  +^^  '  *^eu/  {,n)  xf  (n)  =f{iii-i-u). 

Show  that  in  the  expansion  of  (1  -\-x)"  the  sum  of  the  squares 

1-2-3  .  .  .  .  2/i 

of  the  co-efficients  =    rrrrs w" 

(1-2-3  •  •  •  •  n)- 

7.  Solve  the  equations — 

/-.s    x  —  a         x—b         x—c 

^  ^    bT'c   "*"   a'+c   "^  a  +  b  ^  ^• 

(2)    x4-10x3  +  0ou;2-50.c+24-0. 

_^_Jl 1 

8.  Give  a  brief  account  of  mathematieal  induction,  and  show 
that  a  square  of  a  multinomial  is  equal  to  the  square  of  each  term 
together  with  twice  the  j)roduct  of  each  term  into  the  sum  of  all 
that  follow  it. 

Find  the  sum  of  the  products  of  the  jSirst  n  natural  numbers 
taken  two  and  two  together  ? 

X  >/  z 

9.  If  —  -y  +  z,  -r-  —  z  -i^  X,  -^  =  X  +  y,  prove 

1.1         1  \+a       \+h        1  +  c 

^•'^      V  '  IT  '  T    ^    T^^)  ■  l^^c.  '  l  —  ra 
c-  y-  z- 


(2) 


a(l  —  he)     b{  1  —  ca)     c(l  —  ab) 


V'l-bc      Vl-ca      '^1—ah      ^  1— be    Vl  -  ca     Vl-ah 


234 


EXAMINATION    PAPFRS. 


10.  AB  is  divided  in  C,  so  that  AB,  BC=AC^  ;  from  CA  is 
cut  off  a  part  CD  equal  to  CB ;  from  DO  is  cut  off  a  part  DE 
equal  to  DA  ;  from  ED  is  cut  off  a  part  equal  to  EO,  and  so  on 
ad  inf.  Show  that  the  points  of  section  coritinually  approach  a 
point  C  such  that  AC  =  BG. 

14.  Ehminate  ;c,  y,  z  and  w  from  the  equations 

a^x  +  h^y-\-c.-,z+d.^u  =  0. 

a^x-{-b^y-\-CgZ  +  d^u  =  0. 

a^x-i-h^y-{-c^z-hd^^l-0. 
12.  A  railway  train  travels  from  Toronto  to  CoUingwood.  At 
Newmarket  it  stops  7  minutes  for  water,  and  two  minutes  after 
leaving  the  latter  place  it  meets  a  special  express  that  left  CoUing- 
wood when  the  former  was  28  miles  on  the  other  side  of  New- 
market ;  the  express  travels  at  double  the  rate  of  the  other,  and 
runs  the  distance  from  CoUingwood  to  Newmarket  in  1-^  hour ; 
and  if  on  reaching  Toronto  it  returned  at  once  to  ColHngwood, 
it  would  arrive  there  three  minutes  after  the  first  train  ;  find  the 
distance  between  Toronto,  Newmarket  and  CoUingwood. 


FIEST  CLASS  TEACHEES,  1877. 


XXIV 

'4- 


x~{y—z)+y-{z — x)-{-z^(x  —  y) 


-  1      X 


x^y-  +x^y^+x^z^  +x-z^+y^z'--lry^z^+2x-y^z- 
ax+m -\-l         ax-\-n  ax  +  m         ax-i-n+1 

2.  Solve  (1.)  —^^.^4-     ^       o  =  ~  ~ — ^  +  —5 i- 

^    '  ax  +  m—1      ax-\-n—A     ax  4- m  —  J,     ax-f-n—l 

(2-)  fr^/x+fi  -  y.'c  =  2. 

3.  A,  B,  and  C  start  from  the  same  place  ;  B,  after  a  quarter 
of  an  hour,  doubles  his  rate,  and  G,  after  .  walking  10  minutes, 
diminishes  his  rate  one-sixth  ;  at  the  end  of  half  an  hour,  ^  is  a 
quarter  of  a  mile  before  B,  and  half  a  mile  before  C,  and  it  is 


IXAMINATIGN    PAPERS.  235 

observed  that  tlie  total  distance  walked  by  the  three,  had  they 
continued  to  walk  uniformly  from  the  first,  is  6|  miles.  Find 
the  original  rate  of  each. 

■i.  {l}  Investigate  the  relations  that  must  exist  between  the 
con.stant3  in  order  that  Ax'+By-  -{-Cz^+a/jz  +  lixz+cxi/  shall  be 
a  perfect  square. 

(2)  Find  the  conditions  that  the  values  of  x  and  y  derived  from 
the  equations  ax-'rbi/=  —-^  —  =  c'^  maybe  rational. 

5.  li  x'^+px+q -And  x^-{-mx-{-n  have  a  common  factor,  then 
will  {n  —  q)'^-\-u{)n  —  p)^  =  iu{))i—p)[n  -q). 

6.  Prove  («'")"  =  ^t""*,  whether  m  and  n  be  positive  or  negative, 
integral  or  fractional. 

Show  that  (a^-^+a;-")"'"  =x"^^  "  x  (a;'"-"  +  x"-'")m« 

7.  (1.    ^^-r   =    -T  tlien    \---- —   =     M 

^    '         b  d  c'^'^  +  rf-"         \c-d/ 

(fj'd" 5"c"  «"c"  —  h"cl" 

1 

of  these  fractions  =  —  (rt"4-/^"+c"+ti"). 

8.  If  X  be  very  small,  show  that — 

(l4-2a;)^+(l  +  3x-)^ 
T —  =  2  -  4a;,  very  nearly. 

2  +  ox-(l+'J:x-) 

9.  Prove  that  1  -  71^  +     ^     32     ~         1^    2^    P""       +■  =  ^ 

10.  If  a  debt  $a  at  compound  interest  be  discharged  in  n  years  by 

a 
annual  payment  of  S— ,  show  that  (l+r)"(l— mr)  =  1,  where  r 

is  the  interest  on  $1  for  a  year. 


233  EXAMINATION    FAPiSAS. 

11.  Solve— (1.)  8a;'^-2a:?/  =  55. 


5  5     

1  3    ?    n-t-(7  i 

(8)  rt-5-a;"  —  irt'i'ajspi  =  (a  — ('^)-a;'' 


FIEST  CLASS  TEACHESS,  1878. 


XXV. 

2  /     ' /— — \   2 


1.  Simplify  fV"±i-_V-j;-)    -  (ViL_^(Jl 
'.        X  a  A-  X  \      a  x 


X' 


a  -^  X  \      a  X  ■         a{a  —  x) 

x^^j-{ll-zY         y^-{z-x)^         z^-(x-y)^ 

2.  Divide  — -  1  —  — --^4-  —  +-^  by  .r-a  ; 

J 
shew  that  ( -  %a^)   =^{  v  (Or;  1  -f  v  ( -  Or/)}. 

???.       n        r  x^      (/-      2- 

8.  If  —  =  — = — ancl-:r+TT+— 7=1,  prove  that 
X        y        z  a^      b^      <:- 

^  ~^b^ ^~^^  'x^'+y'^z^ 

4.  Fiiid  the  relations  between  the  roots  and  eo. efficients  of  the 
equation  ax^+lx-\-c  =  0. 

If  in  and  n  are  the  roots  of  the  equation  ax^  +  hx-{-'--^0,  show 
that   the   roots   of  the   equation   acx'--^-{2ac—b^)x+nc  =  0   are 

VI  n 

—  and  — 
n  m 

6.  Solve  the  equations  : 

(1)  x^-\-2\/x^^^^=2x+8. 

x^     w3  ;      X        y 

(2)  —-—  =  104,  —  -  —  =  ^. 
^        y       X         ^    y       X      '' 


(3)  xz  =  y'\  x+y+z=12,  x^+t/^+z^  =  91. 


EXAMINATION    PAPERS.  237 

6.  Two  men  start  at  the  same  time  to  meet  each  other  from 
towns  which  are  28  miles  apart ;  one  takes  five  minutes  longer 
than  the  other  to  walk  a  mile,  and  they  meet  in  four  hours.  Find 
each  man's  rate  per  hour. 

7.  If  P,  Q,  R  be  respectively  the  ;jth,  5th,  rth  terms  of  a  G.P., 
shew  that 

12       3 
Sum  to  infinity  the  series  — 4--^+— :t4-  &c. 

8.  Find  the  amount  of  ^f  at  compound  interest  for  n  years,  r 
being  the  interest  on  ijl  for  one  year. 

Supposing  $p  to  be  withdrawn  at  the  end  of  each  year,  what 
will  be  the  amount  at  the  end  of  n  years  ? 

9.  Detei  mine  the  number  of  combinations  of  n  things  taken  r 
together. 

The  number  of  combinations  of.?i  things  taken  two  together 
exceeds  by  6  the  number  of  combinations  of  w  — 1  things  taken 
two  together  :  find  n. 

30.  (1)  Find  the  limit  of  (l-|-^)*when  a;  increases  without 
"imic. 

(2)  Find  the  (r-fl)th  term  in  the  expansion  of  (3  — 5a:) 

a;2  _  3^.  _  3 
11.  Determine  the  limits  between  which  lies  o  ,0  ,  o    ,1  for  all 

possible  values  of  x. 


FIRST  CLASS  TEACHERS,  1879. 


XXVI. 

7.    Prove  that   2{{a-hy+{h-cy-{-(c-ay}=7{a-h){b-c) 
{c-a){(a-by-^{b-cy  +  {c-ay}. 

2.    Extract  tlic  square  root  oi  ab  —  2ai/{i-ib  —  a^),  and  find  the 
simplest  real  forms  of  the  expression 

l/(3+4v'-l)  +  y/(3-V     !)• 


238  EXAMINATION    PAPEHs. 

3.  Solve  the  equations : 

(1).  2z^  +  x^  -llx^+x-^2  =  0. 

(2).  x^+'y^+z-=a2 
yz-\-zx-\-xxj  =  b^ 
s+  tj-   z=c. 

(3).    y(a;3+5a;+4)  +  i/(a:-+3a;-4)=-a;4-4. 

4.  Prove  that  the  number  of  positive  integral  solutions  of  the 

c 
equation  rta;+%  =  c  cannot  exceed  -y  +  1. 

In  how  many  ways  may  £11  15s.  be  paid  in  half-guineas  and 
half-crowns  ? 

5.  If  xy  =  ah[a-\-b),  and  x''^  —  xy -\-y'^  =  a^ -\-b^ ,  shew  that 
:  X       y\    I  x       y 


a       0  j   \  0       a 

6.  Given  the  sum  of  an  arithmetical  series,  the  first  term^  and 
the  common  difference,  shew  how  to  find  the  number  of  terms. 
Explain  the  negative  result.  Ex.  How  many  terms  of  the  series 
0,  10,  14,  &c.,  amount  to  96  ? 

7.  Find  the  relation  between  p  and  q,  when  x^  +px  +  x  =  0  Las 
two  equal  roots,  and  determine  the  values  of  m  which  will  maKe 
a^  +  m/tx-\-a^  a  factor  oix* —ax^+a^x^ —  a^x+a^. 

8.  lu  the  scale  of  relation  in  which  the  radix  is  r,  «hew  that 
the  sum  of  the  digits  divided  by  r  — 1  gives  the  same  remainder 
as  the  number  itself  divided  by  r  —  1. 

9.  Assuming  the  Binomial  Theorem  for  a  positive  integral 
index,  prove  it  in  the  case  of  the  index  being  a  positive  fraction. 

Shew  that  the  sum  of  the  squares  of  the  co-efficients  in  the  ex- 
pausion  of  (l+x)"  is  ]^h-(|_^)2,  n  being  a  positive  integer. 

10.  Sum  the  following  series  : — 

(1.)  l-f3.c-f-5:c2+7a;3  +  &c,  to  n  terms. 

1  1 

(2.)   Q^   o+o — To-f  &c.  to  n  terms,  and  to  infinity. 


11.  Shew  that 

nhc(<i-\-b-{-c). 


EXAMINATION    PAPKS9.  239 

be,  —  ac,  —  ah 

/>2  -  c3,  «2  4-  2ar,  -rt^  -  2nh       is   divisible    by 
c",  C-,       (a  +  i) 


FIRST  CLASS  TEACHERS,  18S0-Geade  C. 


XXVII. 

1.  Ifiuax^-j-2bxi/  +  c)/^,  kii  +  lv  be  substituted  for  a:  and  nnt  +  uw 
for  ?/,  the  result  takes  the  form  Ait^  +2Bhv  +  Cv'^ .  Find  the  value 
nHB^  -AC)^{b^-ar)  in  terms  of /,•,  I,  m,  n. 

2.  Iiesolv(sa{b  —  c)^-\-b{c  —  a)^-{-c{a-b)^  into  factors. 
Prove  that    — = 

uvw  XIJZ 

iiu  =  x{Bi/^-Cz3),  v^^i/{Cz^—Ax^),  w  =  z{Ax^-By^). 

S.  Extract  the  square  root  of 

(a-/>)"-J(6-c)2+(/;-c)2(c-a)2-f-(c-a)3(a-&)3, 
HU(.i  thfl  cube  root  of 

^ia-b)^+{b-c)^+{c-a)^-S{a-b)^{b-c)^c-a)^\. 

4.  EHminate  x,  y,  z  from 

a         b         c 
ax-\-by-\-cz  =  \   —  =  —  =  — 
'    '^  X        y        z 

k{x^- -\-y^  +z^-)+2{lx^-my  +  nz)  -\r  h  =  0. 

5.  Simphfy   "^J^^^^,  {l/(4  +  3i)  +  v/(4-3j)}3, 

/-l+;v^3\2      -l-i-yV3 
and    [—^ ]   + 2 —  +  ^' 

in  which  j=  v/(  — 1). 

6.  Given  the  first  term,  the  common  diflference  and  the  number 
of  terms  of  an  arithmetical  iDrogressioft,  find  (i.)  the  sum  of  the 
terms,  (ii.)  the  sum  of  the  squares  of  the  terms. 


240  3EXAMIXATI0N    P.'.PESg. 

7.  Solve  the  equations 
(i.)      (a^.r)3  =  (x-Z.)3. 

a         b 
(11.)      ax-^bu=~+~=-\. 

—1  —1  —1 

(iii.)     x{y-\-z     )—a,  7j{z+x     )  =  b,  z{o'+ij     )  =  r.. 

8.  What  value  (other  than  1)  must  be  given  to  q  that  one  of 
the  roots  of  x^—2x-{-q  =  0,  may  be  the  square  of  the  other. 

If  a,  b,  c  are  the  roots  of  x^  —  yx'^+qx  —  r,  express  • 

2a  2 /;  2  _|_  2/;  2 1: 3  _}.  2c  2  «  3  _fl4_^Z,  4^_C4 

'2iih  +  2i/c  +  2ca-«3  _  /,3  _'c 2 
in  terms  of  p,  5  and  r. 

9.  A  vessel  makes  two  runs  on  a  measured  mile,  one  with  the 
tide  in  m  minutes  and  one  against  the  tide  in  n  minutes.  Find 
the  speed  of  the  vessel  through  the  vrater,  and  the  rate  the  tide 
was  running  at,  assuming  both  to  be  uniform. 

10.  Five  points,  A,  B,  C,  0  and  P  lie  on  a  straight  line.  The 
distances  of  A,  B,  and  C,  measured  from  the  point  0,  are  a,  b, 
and  c ;  their  distances  measured  from  the  point  P  are  x,  y,  z. 
Prove  that  whatever  be  the  positions  of  the  points  0  and  P, 

x^-i^b  -  c)  +y-  (c  -  a)  +« -'(a  -b)-jr{b  —  c){c  -  a){a  -  6)  =  0. 


APPENDIX. 


Section  I. — Elementary  Theorems  on  Polyncmes. 
(See  page  39,  et  seq.) 

Theorem  I.  If  the  polynome/  (x)"  hs  divided  by  x  —  a.  the 
remaiuder  will  be. /"(«)". 

D'AIcmbert's  Proof.  f{xY  is  the  div-idend,  x  —  a  is  the  divisor* 
let/i(x)"~^  be  the  quotieut,  which  is  necessarily  a  polynome  o* 
degree  »i— 1,  and  let  R  be  the  remainder.  Then,  since  the  pro- 
duct of  the  quotient  and  the  divisor  added  to  the  remainder  re- 
produces the  dividend, 

/(.;)"=  fa; -«)/•,(.^•)"-l+/^\ 
But  R  does  not  contaiQ  x,  hence  it  will  remain  the  same,  not 
merely  in  form  but  in  actual  value,  whatever  value  be  piven  to  x.. 
Take  the  case  x  =  o,  then  {x—a)f.y[xY~^  vanishes  for  its  factor  « — a 
does  so,  heuco  R=/(a)".  Thus  the  remainder  is  the  value  of  the 
dividend  when  x  has  the  value  which  makes  the  divisor  vanish. 

It  has  been  objected  to  the  above  pi'oof  "  Division  can  be  per- 
formed only  when  there  is  an  actual  divisor,  therefoi-e  in  assum- 
ing R  to  be  the  remainder  of/(.c)"-=-(x— a)  it  is  assumed  that  ajis 
not  equal  to  a,  and  although  R  will  remain  unchanged  for  all 
values  of  x  that  fulfil  this  assumption,  it  cannot  thence  be  inferred 
that  it  will  do  so  if  the  contradictory  assumption  be  made.  In 
such  case  the  only  legitimate  conclusion  is  that  there  being  no 
divisor  there  is  neither  quotient  nor  remainder.  Therefore, 
although /(a)"  may  be  the  remainder  in  the  case  in  which  x  is 
not  equal  to  a,  yet  the  above  ai-gument  does  not  prove  it."  This 
objection  confuses  arithmetical  or  numerical  division  with  alge- 
braic or  formal  division,  division  by  a  definite  quantity  with  divi- 
sion by  an  undetermined  or  variable  quantit}'.  The  following 
proof  does  not  involve  the  assumption  x  =  a,  and  consequently  is 
not  open  to  the  foregoing  objection. 


242  APPENDIX. 

Lagrange's  Proof.     Lemma,     jf^    -  a"  is  divisible  by  *     »,  if  n 
be  a  positive  integer. 


By  actual  division   — ; =  ^""^  —  a  • 


x  —  a 

:.  orJ"  —a"  is  divisible  by  .c— a  if  a;"~^  — a"~^  is  so  divisible, 
hence  x""^  -  a*^^  "  "  x  —  a  "  x''~'^  —  ci"~'^  .... 
Thus  vre  can  reduce  the  exponent  unit  by  unit  until  at  last  we 
arrive  at,  x^—a^  is  divisible  by  x—a  iix—a  is  so  divisible.  But 
x—a  IS  certainly  divisible  by  aseK,  .'.  x^  —a^  is  divisible  by  a;  —  a, 
.  x^  —  «3  ig  aigQ  divisible  hj  x  —  a,  :.  so  also  is  x'^—a^  and  thus 
we  may  go  on  to  any  positive  integral  exponent  whatsoever. 

Theorem.  Writing  f(xY  m  polynomial  form  arranged  in 
ascending  powers  of  x, 

/(./;)"  =  A^  +A,x  +  A^x^  +A^z^+  +  A„x^, 

:.  f{a)-=.A^  +  A,a-\-A2a^  ^A^a^-^  +4„«", 

•••  fix)''  -/{nr  =  A,{x-a)~\-  A^(x^  -a^)  +  A.^{x^  ~a^)+  .  .  .  . 

+^„(-«"-«")- 
But  every  term  of  this  polynomial  is  divisible  by  x  —  a,  and  the 
highest  power  of  x  in   tlie  quotient  is  a;"~^  got  from  the  term 
-^ni-*"  —  "")>  so  the  quotient  may  be  represented  by.fj(x)"~S 
.-.    {f(x)''-f(a)-}~{x-a)=J\[x}''-^ 

f(xY  f(a) 

x  —  a     -^  I  ^   {        'x  —  a 

Theorem  II.  If  the  polynome /(.«)"  vanish  on  substituting 
for  X  each  of  the  n  different  values  a^,  a^,  a^,  .  .  .  .  «„, 

thQnf{xY  =A{x—a^){x—a^) {x-'-i-n), 

in  which  A  is  independent  of  x  and  consequently  is  the  coefficient 
of.«"  in./'(ic)". 

Since /(ai)=0,  :.  f{xY  ={x-a^)f  [xy-^.  In  this  substitute 
a,  for  X,  :.  siucef(a^Y  =0,  it  becomcd  0=  (ctg  —  «i)/, (as)""^-  Of 
this  product  the  factor  a^  —a^  does  not  vanish  since  by  hyjjothe- 
sis  a.2  is  not  equal  to  a^,  therefore  the  other  factor  f^ia^Y'^  must 
vanish  that  the  product  may  vanish,  and  consequently /i(^-)"~^  is 


APPKNDrX.  243 

divisible  by  x—a^.     Let  the  quotient  be  denoted  by  f^{xy~^, 
fixY' ={x  —  ai){x—a„)f^{xy-~-.     Substitute  a 3  for  .c  and  proceed 
as  befure,  and  it  will  be  j)roved  that  x—a^  is  a  factor  of  /(a;)". 
Continuing  to  n  factors  we  get  a  quotient  independent  of  x,  since 
each  division  reduces  the  exponent  of  x  by  unity,  .v  finally 

f{xY=A{x-a^){x-a^) (.6--«"). 

Cor.  lijXx)"  and  <p{xY'  b(!th  vanish  for  the  same  ??  different 
values  oi  j-,j\xY  is  algebraically  divisible  by  <p(j;)"'. 

Let  Mj,  a2>  ^3' ^m  be  the  m  different  values  of  x  for 

which  the  polynomes  vanish, 

.-.  f{xY  ={x-ay){x-a^) (x— a,„)  F(a;)"-"' 

9.n^<p{x)"'  =  A{x-a^(x~a,^) {x  —  a„) 

.'.  f{x)"  ~0(x)"'  =  F{xy-'"^A, 
which  is  an  integial  function  of  x  since  A  does  not  contain  x. 

Theorem  III.  If  the  polynome/(x)"  vauisli  for  more  than 
n  different  values  of  x  it  will  vanish  identically,  the  coefficient  of 
every  term  being  zero. 

Let  flj,  a„.  a^ «„,  a„^i  be  n  +  1  different  values  of  x 

for  which  /(a;)"  vanishes, 

Substitute  rt„^.i force,  and  since /(a„+i)"  =0, 

But  none  of  the  factors  «ri+i  —  «n  ««+i  —  «2'  ^^*  vanishes, 
.'.   A  must  be  zero,  or 

f(xr  =Oix-n^){x-a„)(x-a^) (a:~«„) 

and  the  factor,  zero,  will  be  a  factor  in  the  coefficients  of  every 
term. 

Theorem  IV.  If  the  polynomes  /'(x)",  <p{xY'  {n  not  less  than 
m)  are  equal  for  more  than  n  different  values  of  x,  they  are  equal 
for  all  values,  and  the  coefficients  of  equal  powers  of  x  in  each 
are  equal  to  one  another. 


244  APPENDIX. 

::f{xr-<p{xr  =  A,-B,  +  iA,-£,)x+{A,-B,)x^-h 
{A^-B.)x^+   ....   +(^,,-7;>"' 

+  J,„+ia;"^  +  i+^„j+.a:"^+2 +A„x\ 

and  tins  is  a  polyuome  of  degree  n  at  most.  Biit/(.c)"  =(p(.'c)'"  for 
more  than  ?i  different  values  of  x,  that  is  /(x)"  —  (p(:i')"'  vanishes! 
for  these  values,  .-.  by  Theorem  III.  f{x)"-  -  (pix)""  vanishes  identi 

cally,  and  the  coefficients  ^o  —  i?o,  J. i  —  />\,  ^2  "^'2' 

A^-B,^,  ^m+2, -4,»+2, A,,  are  all  equal  to  zero, 

.•.A^  =  B,,A^=B^,A.^=^B.^,...A^  =  B,,,A,n^^=Q,  A,n+2  =  0  ... 

Note  to  Art.  XVII.     To  find,  where  such  exist,  the  factors  of 

ax"  +bxu  +  Cxz-{-ei/^  +gi/z  +  hz^. 
Multiply  by  4a 

Aa-x^  -{-iahxif  +  iacxz-i-icei/^  -\-4:agijz-\-4.ahz^ . 

Select  the  terms  containing  x  and  complete  the  square,  thus 
Aa^x"  -\-4:(ibxij-\-4:acxz  +  b-!j~  +''2bcxz  +  C-z' 
-  (63  -  4«e) »/2  -  2{bc - 2afj)yz  -  (c^  - 4:ah)z"  = 
{2ax+by  +  C^) 3  -  { (i3  -  Aae)i/^  -\-%bc-  1ag)ijz  +  (c2  - 4ali)z^ } 

If  the  part  within  the  double  bracket  is  a  square  say  (?>'-?/ +  ?^)3 
the  given  expression  can  be  written 

(2rta;  +  %  +  Cz)2-(?H7/  +  ?7z)3 

which  can  be  factored  by  [4] .  Factor  and  divide  the  result  by 
4a.  If  the  part  within  the  double  bracket  is  not  a  square,  the 
given  expression  cannot  be  factored.  If  ft  and  c  are  hoik  even, 
multiply  by  a  instead  of  by  4a  and  the  square  can  be  completed 
withoit  introducing  fractions.  If  e  is  less  thau  a  it  will  be  easier 
to  multiply  by  A.e  instead  of  by  4a  and  select  the  terms  containing 
y.     A  similar  remark  applies  to  h. 

This  method  can  evidently  be  extended  to  quadratic  multino- 
mials of  any  number  of  terms. 


appendix.  245 

Examples. 

1.  Eesolve  a;3  ^-. ry +2-^3  —  %"+ 7.'/2- 822  into  factors. 
Multiply  by  4 

4^2_i..i.c^4_8//2-8//3  4-28//3-  12z3 
Complete  the  square  selecting  tjrms  in  x, 

(2.i-  +  //  +  2z)--(3.v-4z)2  = 

{(2x+i/+2z)  +  (3^-4^)}{(2x+//+2z)-(3»/-42)}  = 
(2u:  +  4^-22)(2x--2^V+ez)  =  4(a;  +  2y-;^-Xx-^+32) 
.-.  tlic  factors  are  (.<;  +  2y  — 2)(.e— ^  +  Ss). 

2.  Ga2  -7((^+2«c-20634.G'k-486--. 
Multiplygby  4  x  6  =  24 

144«3  _  lG8a6  +  48«c  -  480^3  4;153G5c  -  1152c2  = 
(12a  -  7^+2'-) 2  -  520^2  +1564ic  -  llGGc-s  = 
(12rt— 7i  +  2c)3-(236-34c)2= 
(12«+166-32c)(12a-306  +  3Gc)  = 
24(3a+46-8c)(2a.-5/>  +  Gc), 
.-.  the  factors  are  3a +46  -8c  and  2a—  5/)  + 6c. 

3.  a:2  4-12^z/+2^-2+2G^2_  8,^3 -933  = 

(x-  +  12x^  +  2a-Z  +  3G^2  + 12/72 +2-)- 10^2-20^2 -1022  = 

(x+6z/  +  2)--{0/+2)v/10}'-'  = 
{^+(G+x/10)?/+(yiO+lM  < 
{^  +  (G-yiO).y -(1/10 -1)2} 

4.  8a2  +10'(.'>  -  14^"?+12«f^  -  8/^2  _  8W+8c2  -  8cfZ. 
Multiply  by  3,  not  4x3,  since  the  coclEcients  of  tho  other  terms 

in  a,  are  all  even, 

9« 2  +  30a6  -  42ac+3Ga(?  -  2462  _  g^^ci  +  24c2  -  %icd. 


246  APPENDIX. 

Select  the  terras  containing  a  and  complete  the  square 

706c-84//^-256'2+60cfZ-36c/-^  .-= 
(3«  +  5/) - 7c+Gf<) 2  -  (7i -  5c  +  6d)3  = 
i^3«  +  12Z/-12c  +  12c/)(3a-2i-2c)  = 
3(/i +  46 -4c +  4(0(3^' -26 -2c), 

,'-  the  factors  are  «+46— 4c+4(Z  and  3a  — 26  — 2<?. 

Work  Exercise  XXIX  by  this  method- 


Section  II. — Indices  and  Scp.ds. 
The  general  Index-laws  are 


m           p 

an  .act 

=    an      '     q 

(1) 

m          p 

m  _  p 
=    an           q 

(2) 

m 

(ab)" 

•    m            m 
=   rt«     .    6"n 

(3) 

{a  -f  6) « 

m            m 
=  «  »   H"  6  »» 

(4) 

(a^)7 

iiip 

(5) 

Tho  law  connecting  the  Index  and  the  Surd  symbols  is 

oJ^  =  lYiiV)  (6) 

[The  indices  ,},  ^,  J,  &c.,  are  generally  used  to  denote  *  cither 
square-root,'  '  any  of  the  cube-roots,'  '  any  one  of  the  fourth- 
roots,'  &c. 

The  surd  symbols  ]/,  ^,  4/,  &c.,  are  by  some  writers  re- 
stricted to  indicate  the  arithmetical  or  absolute  roots,  sometimes 
called  the  positive  roots.     Thus 

V4  =  2,  but  4*  =  ±2,    .-.  4^  =  ±  y4 
Also,   v'{(-2)2}=  \-4  =  2. 

^'27  =  3,  but  27^'=  3  or  8/ =f^    j  •.  8' =  (P).3/^'^ 

4/16- 2, but  16*=  ±2  or  ±2j,  .-.  16*  =  (1*)V16. 


APPKNDrx.  24V 

Witli  this  restriction  the  general  connecting  formula  -^onld  be 

In  the  following  exercises  this  restriction  need  not  be  observer.] 

Exercise. 
] .  Wliat  is  the  arithmetical  valne  of  each  of  the  following  : 

"■^     Cir-,^     -i/>*      on'        .5       ni      nrr^     n  a^      OO"      fll"      d"^       /OSX^ 


S6\  27^  3  6*,  32',  a\  8^,  27^  0^,  32%  64',  81%  (Sff, 
(5:VA    (IfV)',  (-^S)-,  (-027)1   4!)'^   32"',   8l'"' 

-2  -2  1 


2.  Interpret  a-"^,  a^,  a-   ,  (a-)     ,  a^     ,  a  %  (a  ^)  *,  a",  «-*. 

3.  "Wliat  is  the  arithmetical  value  of 

36""^  27~*,  (-16)"%  (-0016)""^  {i)~',  U%f^,  (y%)~K  (5,-^,)~* 

4.  Prove  («"■)"  =  («")"';(«"')"=:  (a"  )'^';  «-">=  (a-^)"* ; 
avid  express  these  theorems  in  words. 

6.  Simplify  a\J,  c^ .c\nf.nrK  n.n~^'\{'7ifi2i)^-{di;)^ 

a       e        a        e  6  i  i  li 

"V    1'    >'    "T"    ^=4'    (2|)    -(6*)  ^(i) 
a       c      a  e         a; 

C.  Eemove  the  brackets  from 

(a«)%  (/;)^,  (cf )     %  (/)  %  (.^)^  (/    -') 

1       a  «  in  1  2  1       a  4  ft 

(a=62)  ,  (a  6  )    ,  (rt2c   i)      ,  («  -c  )         (x  ?/      )      . 

7.  Eemove  the  brackets  and  sim])lify 

(.?•        )  (a;        )  (x        )  ;  X  x  x  ; 

h         h  -hi 

i;  «  ;  a;  a?  : 

—  2  — 2  2  ^ 

{a;2     H-a;-2      }{.c<-2)  --a:-^    }. 


248  A|>PENDIX. 

1  a  2. 

(-a;)       {x  ^-x      } 

9.  Deteimiue  the  commensurable  and  the  surd  factors  of 

12^  24*,  18~*,  (-81)*,   12^   64',  {^^J,  (6J)~^. 

(The  surd  factor  must  be  the  incommensurable  root  of  an 
integer.) 

10.  Simplify  8*+18*-50^;  72*-}-(^y-5/'-(Tl5)~*; 
{(6+2^^)(6-2^)}';  (2*  +  8^)'+(2--3V; 

(2  +3  )(4  +9  -6');  (7'--3')  (7  +3")  ,  ■ 

[{{a^x){x+h)]^ -  {{a-x)ix-h)fy^  ; 

Express  as  surds, 

11    a  ,  X  ,  p    ,   c      .  /i 

7(  +  ^       — w  +  5       -25       — 'i  +  OT 

12.  ic        ,  y  ,  a     ,  b 

a  7n-3  72 — a 

13.  (aa;-&)*,  (x3-4;r+l)   4   ,  {p-gx) 
Express  mth  indices, 

14.  ■,fa^   \/c^,   \/x:\  ^Vy"-",   V(r/a;),  |/a-*. 

15.  ■,f(a3+i3),   ^/(«3+/,3)3,    /^(«3^_i3)p^   .>/|(a_j)a;|, 
;/(a  -  /«)"-\   V («"  -  6" )™-"' 

16.  (/t  {b~^f,  {o'~h~^,  (xfK  (ct-'x)'^,  {a-^x~^r^, 

7   —i  14 

{x  y     )   • 
Simplify   the   following,    expressing  the  results  by  both  nota- 
tions. 


APPENDIX.  249 

17.    rt.a      ,  «".«      ,  a    .a      ,  a.a      ,  a      •}/«,«  ^a^,a  \/a 


l  i  6  i  i  — ±      s  — i  * 

a  0  c   .  a      0      c 

^1  i 

a  a        !yc^      ^x^      F  ?/~"      ■|^(24rt~3^     c(ah)  —ac 

i/a     ^a      Wc         as  — i  2V«  ,         /  ,v 

^       •  *  *^  X  y  bc—c{ab) 

1  1  H  3  Sn  Sn 

a  +rt  a'— a    ""      a~  ^  —  a"      «-+l  +  a~"^ 

19.  -J       ZTf       -^       =1'     —3;^        3^.     a  +  l  +  ar^ 
a   —a  a  —a  a      a  +  a  ^ 

on    -n-    •;!  v        -        -       I  ,     9  I  ,     I ,      a:S  .     i  4        4 

20.  Divide  a;-i/  by  x"  —y;  x  +a  x  +a   by      -r"  .<;  +«  ; 

i  \  ^         h       i      i 
x+;j+z—3x  y  z    hj x  +y  +2' 

*      i      i 
2ai+2k-+2ca-a3_ft2_c2  by  a  +&  4-c 

Exercise. 

1.  Express  tlie  following  quantities  L  as  quadratic  surds,  ii 
as  cubic  surds,  iii.  as  quartic  surds. 

2.  Eeduce  to  entire  surds, 

x^/x,afa,  V-f/b\  81^3,  4f2,  V^,  W^>  W^y  H'^ 

^V(!).^V(r'fr(.V). 


{x-yi' '^ix^-+2xy^y-')~\   {x-x~^)f{x^--\-l)\ 


250  APPENDIX. 

3.  Reduce  to  their  simplest  form  1 

,/12,  yS,  v^50,  if  16,  4i3/-250,  V^,  f^h  V^\,  5ir(-320),  ^ 

V(l-A)>   l/«^   l/(('^^0>  if"-',   3iif(54a;9),    4/(a;'?/'^9), 
,/{a3(l_.,-2)>,    ^|a2(«2_i^4}^    ^(,,i)^    ^,,.+1^     ;ya'"+«, 
y«^"^^  >,,'"'-S    v'(«2a;  +  a3),   if(a3+2r.4;«  +  asx^), 
i/{(x-1)(.a;2-1)},   f/{(«3  +  2r/u,-  +  a,---')(rt3+a;3)}, 

/(8a;2- 16x4-8),  ir{(a;2-2+;c-2)(a-*-2x-3  +  l;;, 

l/2x_-2+2^j     l/3.r3-6a;2+3.^.^    I /(a^ -aZ»)3+4a3j  \ 
^\   *+2+a;-i   /N\27a:3+18x-  +  3N\  ^1^6 j 

4.  Compare  the  following  quantities  by  reducing  them  to  the 
same  surd  index : 

2  :   VB;  2  :  ^f  9  ;  t/2  :  ,f  3 ;  i/lO  :  ^f  30 ;   2  /2  :  if  22  ; 

fl3  :   >/a3  .    eyx:^^i/;  ^x  :  ;///  ;  ^x^  .  "jx^;  i/a  :  ^yb  :  l/c  ; 

ya:^^b:  ;^c  ;  '>/"  :  ^^-'^ :  ^o'". 

5.  Eeduce  to  simple  surds  with  lowest  integral  surd  mdex 

v^(r«),  r(Vi),  r(i/t-),  rd/^-i),  v(r^-^).  ^(r-.-^"), 

r(^-«^^^)>  if  (l/27),  ^/(if  81),  4/(if  81),   i/(«^/a), 
^(ai/a),    -/(.rifx),  -^(^s  y,)^  -^{5^/5),  |/(3if3), 

V(3if3),  t\x^^x),  y{«y(/>yc-)},  a;/(..-iyx-i), 

6.  In  the  following  quantities,  combine  the  terms  involving  the 
same  radical  ; 

3/2  +  5i/2-7v'2;  i/8-V2;   ifl6+8if2; 

^16+ /2;  a^/x-^x;  al/x-hl/x; 

8i/fl +5 '/a;-7i/a+ v'(4«)  - 3  v/(4a-)  +  4  ^/(9x) ; 

|/a;+3i/{2x)-2v  (3a;)  +  i/(4a;)-  v/(8a;)+  v'(12^-); 


APPENDIX.  251 

Ix -  3  i/.r  +  5^x -  2  Vj;3  +  ^.c3  ; 

AV{a^-x)  +  2y\h''-x)-S^ {{a  +  hy-x] ; 

V{{a-hyx]  +  y''J,{a+byx\-V{a-x)  +  x/{{l-aYx)-Vx; 

^/{a-h)-r-^{lQ>a-lQb)^-^{ax^-hx")-  V {^{a-b))  \ 

■,/(a3-|-2r(3/j  +  a/v3)_  ^ [u^  --la^h^ab"-)  -  -^ (Ub^-). 

7.  In  the  following  quantities,  perform,  as  far  as  possible,  the 
indicated  multiplications  and  divisions,  expressing  the  results  in 
their  simplest  forms : 

V2.T/6;    a/3,  a/12;  ^/U.  a/35.|/10;  y'ayiSa); 

v/c.  a/(12c)  ;    V{ex).i/{Sx);    v'l/^Vy';^!/'-^'/'; 

i/a"+V«"''';  iri"-^^iri-^""^  yi2--^/3;  ^/(ex)-!/^^^); 

(«+x-)H-V(a+a;);  (a-' -x2)-f-V(a-a;};  (a;^ -l)^if  (ic+l)2  ; 
(3v/8-5\/2+v/18+v/82+v/72-2a/50).-/2; 
(7i/2-5a/6-3v/8+4  a/20)  a/18);  (N/54-i/3)(i/5-y  3)  • 
(y/24-l)(s/6- v/3);  (3-v/2)(2-f3v/2); 
(5v/3+v/6)(5i/2-2)  ;   (v/a-  ^/i)(^  ,,  +  ^/>)  ; 
(ay'6-j-i\/a)(6  \/a  —  ay/b)  ; 
{V{x+l)+V{x-l)}{V{x+l)-V{x-l)}; 
{  v/(3«  -  b)  +  -^{Bb-a)}{^/{Ba-b)  -  ViSb-  a)}  ; 
V(a+Vb).V{a-  Vb);    a/(iA:+  v///).  A/(v/a;-  \/v/)  ', 

ir{x-v^(a;3-l)}.^{a;+,/{x3-l)}; 
^{V«-l/(«^-^')}-#'{v/(a3-a;3)+av/4; 


252  APPENDIX. 

y(B  +  3A/7).y(8-3v/7);  (Va+i/b)^;  {^a+Vh)^; 

f     \l'2a\         |/S6\j2 

[{V(«4-a')(^-^)}+V{(a-a:)(:.+/.)}]2;-;4T;;;;-4,^)| 

[\/{(r?  +  .r)(a:+&)}  +  y{(a-rc)(.c-M}]2; 

{V(V104-1)-V,V.10-1)}2; 
[V{a4-V(«2-a:2)}4-V{rt-V(«2_a.2)|j2. 

(Vj:  +  V//)*  +  (Vx-V»/)4;   (a^+(,?V24-62)(a2 -^6  A/2+i2) ; 
(V^H-  V/'  + Vr)(V/>  + Vc  -  ^/a){^/c  +  \'a  -  ■^h){ya-\-y/h  -  Vc)  : 

(^,,+^i  +  ^,){^a2+if6-^+^.'2-2if(6c)-2i3/(c«)-2ir'y^)}- 

8.  Fiud  rationalizing  multipliers  for  the  following  expressions, 
and  also  the  products  of  multiplication  bj'  these  : 

a-\-\'h,   Vct  +  Z^Vr,   (iVh  —  hVa,   «  +  V(a^  — a;^), 

V(a-ic)  -  V{a+x),     V(«2  +  v<-)  +V(rt3  -Vc), 

V{8+V(24+V5)}-V{8  +  V(24-V5)},  Va  +  Vi  +  Vc, 

3  +  V2+V7,  V6  +  V5-V3-V3,   ^a  +  ^h  +  ^c  +  ^d, 

f/«2— ^o2,   V«+^/c,   ^a~f/b,   ^Ja^^a,   y^  +  Vy^ 
fa  +  fb-{-f/c,  a  +  ^b  +  ^c. 


APPENDIX.  253 

9.  Eationalize  the  divisors  and  the  denominators  in  the  follow- 
ing, and  reduce  the  results  to  their  simplest  form : 

1^^2-V3),  3-i-{d+y'6),  5--(v/2  +  V7), 

(,/3  +  ^/2)--(^/3-^/2),  (7,/5+5t/7)-^(,/5+ ^/7), 

a^x  +  bVi/  2V&  i+3,/2-2\/3 

cVx-eVy^    ^2+i/3-  V5*    i72  +  ^73+|76' 

V^6— 1/5— 1/3+\/2 2 

2c H-aH-v^+x2^) 

^/{n-\-x)-\-Via  —  x)  1 


y^{{l+a)(l  +  b,}-V{{l-a){l-b)} 
y{il+a)(l+6)}+i/{(l-a)(l-^)}' 

(a  -  X)l/{b^  +  ,/2)_(6-y)  i/K+^ 

(rt+x)i/(t2  +  i/2)  +  (6  +  Z/)V(«^+*^/ 

V(l  +  a)^l/'(l -«)+;v/(l+i)- V(l-  6) 
y'(l  +  a)+V(l-«)  +  v/(l+/>j+V(l-/.)' 

x/(x+fl)-v/(3;-a)-  v/(a;+/^)  -|-  \/  (aj  --  6) 
{/{x^a)+\/(x-a)  +  ]/{x+b)+  V{x-by 

y/a     i/b         j  |«+a;\  I  /^-^i       '  |t  +  y/x         Wx—Vy 

1         1        Va     -Jx 

lY-f-,/(rT2_l)       s/x~V^j      Vz~Va 

•vl«r-i/t«^-i)'  2_    il'  ^^  ^^* 

Vx     \///     Va;     Va 


254  APPENDIX. 

10.  Fiud  the  values  of  the  following  expressions  for  ?;  =  !,  2, 
3,  4,  5,  respectively. 

1  |/i+i/5r    /i-i/5n 

V^Sli      2      i         12/1' 

J._ J  (2+t/6)"+^- (2+V6)     (2- a/6)"+^-(2-V6)| 
2v/6t  l  +  i/G  ~  l-v/6  I 

11.  Show  that 

^       [{^•+l/>'-l)}*"t^+{x-s/(a;^--  l)}"-'  +  2] 


2(a;-l) 
is  a  square  for  n=  1,  2,  or  8  respectively. 

12.  Extract  the  square  roots  of 

x-^-ij  —  'l  v^(xi/),   a-^c  +e  +  2^\/ {'(c-\-ce), 

a  +  2c  +  e  +  2^/{{a+c){c+e)},    2a  +  2V (a^ -c^), 

Vx  +  2  +  Vx-'^,  x  +  dx^  +x'^-^2x^/x-]-2x^ V X, 

x^  -X1J  +  \i/^  +  \/{4:xhj  -  Qx'hj^  +x>j^),  2x+s/  {%x^  - y^), 

6-2v/6,  10+2a/21,  9+4\/5,  4-V15,  7  +  4v'3, 

12-5v/6,  70  +  3|/4ol,  4-|/15, 

9  +  2|/6+l(i/2+ V3),  15.25 -o\/.6. 

13.  Find  the  value  of 

(a  +  h)xtj  .  a  \fa  h-^/h 

y(a;-+.v3),  given  a;=vK(rt^c)  y  =  f/{a^e); 

V(l+a;)-V(l-a;)      .  2ah 

V(l+x)+V(l-«:)'  S^^^''  •«  =  .T^TP  ' 

2«V(l4-a;2)      .  f     Y         \'\ 


255 


:.h+c  =  0. 

ir    c-      IT  iO+V5)x-2  x(l-V5)a;-2 

15.   bimpMy  ^,  _,^+ ^g^i  +  ^rr.  ,  ^i  _  v5)..+i- 


Complex  Quantities. 


Quantities  of  the  form  a-{-h\/  —  l  in  which  neither  a  nor  h 
involves  \/  —  1,  are  called  complex  quantities.  The  letter^'  (or  i) 
is  frequently  used  as  the  symhol  of  the  diteusive  unit  V  —  1,  so 
that  a  +  by/  —  l  would  be  written  n  +  hj.  So  also  V  —x=j\/x, 
\/  —  x.V  —y  =p  V  {xy)  =  —  Vxij,  and  J  3  =  —j 

Exercise. 

Simplify  the  following,  writing^'  for  -/  — 1  in  any  result  in 
which  the  latter  occurs : 

1.  v/-4,  1/-36,    |/-81,    V-S,    iZ-ia,    1/-72,    if-S, 
^_5.^_6,     v'-6.|/-8,     \/-8.v/12,     -/_8.|r-8, 
V/ -5.1/ -20. 

2.  ,/-»,    i/-a;3,    v/-^<3^     i/-^'^",     N/(-a)^     ■i/(-«)-\ 
n/5.  \/  —  rt. 

3.  i-,  i^  i*,  i',  i^  j'\  j'\  j''.  p\  ./n  /"+S  /"+^  /"+», 

4.  "i-'y,  jVx.j\/ij,   5j,  j-x/o,  Ji/-  a,  jY-a^,  Ji/a.^  -a. 

6-  V-p,  V-p,  V-J^'  v/-i^  -/-i^",  i/-j4". 

y'  _  6        V-C  y/B  J|A_       l/     rt  1 

*^'   "V3  '      y-3'      ;/-3'      v^-T      736'     TZl' 
a  a^  i/(  — «*)        —  V  -  1  aS 


^_e3'      .j/(_«)2«-i' 


256  APPENDIX 

7. 


111-11         1         -1         1 

J  ji  jS  J  ji.         jin-rl       jin^l      yn-l 

a^j  X  —  y  cj^ 

l/-«^'      JVx      j\/-y'^'      ~  V  -  c^ 
8.    V{a-h).V{b-a),    ViSx-4:y).V{iy-3x),   {3  +  5j){l  +  Aj), 
(8-9;-i(8-7;),  (7-i^/5)(7+i /lO),  (v'3~jva)(V2-jV6), 
(a  +  /./j(c'  +  .;),    {u  +  {a-l)j}{a  +  {a  +  l]j\, 
(l/«+iT/6)(i/«-J/Vc)   (a+/y-)(a-/y-),   (a;'  +  &) («;'-&), 
(  \/« +i }//')(  y«-i\/^),   {(t  V Ij +CJ V x)iaY/'b -cjW x), 
^/{l+j).V{l-j),    V(3+4j)V(3-4y-), 
v/(12+5;-)VU2-5;),  (l+Jj^    (ya-jVi)2,   (5-2;V6)2, 

(«+^yr+(«-^jP,  («+^;J^-(«-/>y'j^  («+^!;')24-(ry-^/)^ 
n+jvd\\  /-i+iV3f    /-i-iv3|3^  /1+jy^  (^-"-^y, 

{a  +  bj}^-{a-bjy,  (l+j)5-|.(i„^-)..,  (l+jV2)^  +  (l-iV2)« 

[A{  7(30- G  a/5)- l-/5}  +  i;{N/15+  ^/3+ v/(10-2v''5)}]» 
for  all  positive   integral  values  of  n. 

4  64  21  5  1-20/75 


9. 


14.JV3     l-iV7     4!+a;V6     V2+JV3       7-2yV5 

1+JV3'  1-f    1+i'    1-i'  (i+j)^'    1-i'  ^--i// 

a+iVx    jVo  +  V— ^     <t  — /^'     a-\-jV{l-x^) 
a^\/x     V-«-iW/    o/'+i'    a-yv^l-a;'-^/ 


APPEXDIX.  257 

y/(x-y)-V(!/-x)        1    _^J_     l±i,l-i 

1  1       -  1  1  X+  llj       X  —  If} 

■  ■+ 


x  +  yj     x  —  yj     Vx+jVy     Vy+j^x 
a  4-  hj ~a  -  hj     V.c  -  jVy     ^y  -jVx' 

y(i +a)  -iva  - «)  -v/ii - «)  -7va  +«)' 

10.  y'(S  +  iJ}  +  y{S-AJ),     ^/(3  +  4;-)-  V(3-4j), 

v'(4  +  3j)+v/(4-3;-),    i/(H-2Jv"G)  +  ;/(l-2;VG) 

v/(5+2;VG)±v'(5-2yV6), 

v/ (2^/15 +  3q;>±:  v/ (2  v/ 15  -  30;), 

v/(y3+iv/105)+ v^d/^-i/lOo), 

V{a+j\/{z--a^)}±\/{a-jV{x^-a^)}, 

A/{«-+.MV'<-^-'+2(/2)}±-^/{«3-J>y(x3  4-2r(3)}. 

11.  Prove  that  both  A(-l+.y|/'3)  and  i(-l  -;V3)  satisfy  the 

equation  — 171"  =  0, 

that  (x4-u7/  +  (f2z)3  =a;3^^3_^2n_|_3(a.4.ii.y)(y.|.j<,2)(2_^„.a;) 
and  that  {x+y+z){x-{-wy-\-iv^z){x+w^y  +  wz)  = 
2..3_|_y3  _|_23 — 3x^z,  iu  which  w  represeuts  either  of  the  pre- 
ceding complex  quantities. 
Hence,  prove  that 

(i)   {2a-b-c+{b-c)jVB}^  =  {2b-c-a+(c-a]j^S)}^  = 
{2c  —  a-b  +  {a-b)jy/'S}^  ; 

(ii)  u^  -^v^ -\-w^  —Suva-  =  {a^  +b'^  -\-c^  -  Sabc) X 

(z^+y^ -[-z^ -dxijz),ii  u  =  a;v  +  by  +  cz,    v  =  ay-{-bz+cx, 
w  =  az+bx  +  cy,  or  ii  u  =  (ix-{-cy+hz, 
v=zcx  +  by-}-az,     w  =  hx  +  ay  +  rz. 


258 


APPENDIX. 


12.  Prove  that  i  {\/5  +  l+j  v  (^10  —  2  v'5)}  satisfies  the  equation 

x^  +  1 
x+1  "^- 

Writing  w  for  the  preceding  complex  quantity,  prove  that 

and  {x  ■^y-i-z)(x+tc^y  —  tc^z)(x  -  w^i/  —  icz){x  -  u-z-\-  w^z) 
{x+W^y  +  ic-z)  =x^  +y^  +z^  -  5x''^yz+5xy"z^. 
Prove  that  {4a  +  (/j-c)(i/5 -1)  +  (64-c)J|/(104-2a/,5)}»  = 
{l(a  +  b){-l-tjV{]/5  +  -2)}  +  {a~b){V5+jy{s/o~2)}] 
X  y5-4c}\ 


Section  III. — Pure  Quadratics. 


Examples. 

^        x+B{a  —  b)     a{3x+9a  —  7b) 
x^dja'^b)  ^ b{8x-la+\9b)' 

m        p  >n-\-ii.     p-\-q , 

Apply,  if  —  =  — ,  •"•  = » 

i:i-  J^        n         q  m  —  n     p  —  q 

X  3.-c(ffl  +  &)  +  9a3-14«i>  +  9i2 

•*•  3(a_^)=       Qx{a-b)-^9{a^--b^) 

Dividing  the  denominators  by  3(a  — i) 

.-.  a;{+3(rt-l-i)}=3.c(a+^)4-9t(3_l4«i  +  9i^ 
.-.  a;2=:9«2_l4a6  +  963 

'a;-2rt  +  4//\  ^     5x-9a-\-8h 
a;+4a  — 26/     ^  5a;  +  3a  -  96* 

m       p           n  —  m     q—f 
Apply,  if  —  =  — ,   .'.  =  —   ,  and  factor  the  numerator 

rr  .; '  ^  ^    '  ^^  p      ' 

(a;+4a-26)2-(x-2a  +  4i)3, 


APPENDIX.  259 

12(x-\-a+b)(a-b)        12{,7-b) 
(a;+4a-26)3       "5x+3ft-96' 

x+a+b         x-h'4:a-2b       3{a-h)       ^       ^,. 
"•  x+Aa - U  =  5^+3^7396  =  i^r^^^T^-   ^7    taking    differ- 

ence  of  numerators  and  difference  of  denominators.     To  the  first 
and  third  of  these  fractions,  apply  if 

rn        ij  m  p 

n    "  q  '    " n—m~q—p 

x  +  a  +  b  ^a  —  b) 

"   3(a  -  6)~4x'-4a  — 4Z>' 

.-.  4{a;2-(a  +  ^)2}=9(a-6)2, 

...  a:-2  =  j|4(a  +  6)3}+9(a-/.)3}. 

^-       ,/(3a;-2_l)_y'(3-a;2)-  ^  ' 
Sx-  — 1     rt4-6 


^ 


3_a;3  -a-6' 


3-a;-   -{a-by^ 

4.   7/7V(l+a;)-wV'(l-a:)=\/{??i2+n3)  (1) 

Square  both  members  and  reduce 

.-.  (7n2-w2).r-2/nr?v/(l— a;2)  =  0.  (2) 

Transfer  the  radical  term  and  square  both  members, 

.•!    (7ra2_n2)2a;2=4„i2,j3(i_a.i3)  ^3) 

.-.     (TO2^„2j2a.2=4,„.2„3  (4) 

+  2?nn 
x=    ~  ■     o.  (4) 

The  above  follows  the  usual  mode  ol  solvluo:  equations  involv- 
ing radicals,  viz.,  make  a  radical  term  the  right-hand  member 
gathering  aU  the  other  terms  into  the  left-hand  member,  square  each 


260 


APPENDIX. 


member,  repeat,  if  necessary,  until  all  radicals  are  rationauzed. 
This  method  is  couveuient  but  it  does  not  explain  the  difficulty 

-\-2mn 
that  only  one  of  the  values  of  x  in  (4)  satisfies  (1)  viz.  — ^     ^.j- 

—  2mn 
The  other  value,  —5— — ^  satisfies  the  equation 

m  a/(1  -\-x)  +n  \/{l~x)  =  |/(m2  4- ?,3). 

The  explanation  is  simple.  Squaring  both  members  of  (1)  is 
really  equivalent  to  substituting  for  (1)  the  conjoint  equation 

{)nV{l+x)  -  nV(l  - x) -  V{m''^+n^)} 

{mV{l-\-x)  +  nV{l-x)-V{nt-^+n'')\=^0  (5) 

which  reduces  to  (2)  above. 

Treating  (5)  or  (2)  by  transferring  and  squaring  is  equivalent 
to  substituting  for  it,  the-  equation 

{m\/{l  +  x)-n\/{l-x)-  \/{m^+n^}'^  X 

{ms/(l  -ha:)  -'«|/(1  -x)-^  V {m^ +n^)}  X 

{m-i/{l+x)-\-n  V{l-x)  -  V{m^+n^)}  x 

{m\/{l  +  x)  +  ni/{l-x)  +  i/'{m^+n^)}  =0  (6) 

which  reduces  to 

{(m2  -n^)x-2mnv{l -x^)}  {m'^  - n^)x  +  2m?z  ^/(l  -x^ }  =  0       (7) 
which  further  reduces  to  (3) 

Thus  the  whole  process  of  solving  (1)  is  equivalent  to  reducing 
it  to  an  equation  of  the  type  A  =  0  and  then  multiplying  the 
member  A  by  rationalizing  factors.  Thus  instead  of  solving  (1) 
we  Freally  solve  (G),  i.e.,  a  conjoint  equation  equivalent  to  four 
disjunctive  equations.  (See  page  140,  Art  xl )  Now  the  values 
given  in  (4)  will  satisfy  (G),  the  positive  value  making  the  first 
factor  vanish,  the  negative  value  making  the  third  factor  vanish, 
while  no  values  can  be  found  that  will  make  either  the  second  or 
the  fourth  factor  vanish. 


APPENDIX. 


261 


Hence,  if  one  of  sucli  a  set  of  disjunctive  equations  is  proposed 
for  solution,  the  conjoint  equation  must  be  solved,  and  if  there  be 
a  value  of  x  which  satisfies  the  particular  equation  proposed, 
that  value  must  be  retained  and  the  others  rejected. 

(This  process  is  the  opposite  to  that  given  in  Arts.  XL.  and 
XLV.  :  there  a  conjoint  equation  is  solved  by  resolving  it  into  its 
equivalent  disjunctive  equations.  The  two  processes  are  related 
somewhat  as  involution  and  evolution  ai  e) . 

Furtlier,  it  should  be  noticed  that  just  as  there  are  four  factors 
in  (6)  while  there  are  only  two  values  in  (4),  it  will  in  general  be 
possible  to  form  more  disjunctive  equations  than  there  are  values 
of  a;  that  satisfy  the  conjoint  equation,  and  consequently  it  will 
be  possible  to  select  disjunctive  equations  that  are  not  satisfied  by 
any  value  of  x,  or,  in  other  words,  whose  solution  is  impossible. 
This  will  perhaps  be  better  understood  by  considering  the  fol- 
lowing problem. 

Find  a  number  such  that  if  it  be  increased  by  4  and  also  dimin- 
ished by  4  the  difference  of  the  square-roots  of  the  results  shall 
be  4. 

Reduced  to  an  equation  this  is 

y(.,+4)-i/(:c-4)  =  4  (8) 

Rationalizing  this  becomes 

{4-v/(a;-t-4)  +  v/(x— 4)}{4-i/(a;  +  4)--,/(x-4)}x 
{4  +  V(a;+4)-f;/(a;-4)}{4  +  |/(a;+4)->/(a;-4)}=0     (9) 
which  reduces  to 

{2t-8|/(a;-F4)}{24+8T/(a;-f4)}=0 
ie.  9— (a;-F4)  =  0,  or  a;  =  5. 
Now  a;=  5  satisfies  (9;  because  it  makes  the  factor 
4-|/(x+4)-  v/(x-4) 
vanish  and  it  is  the  only  finite  value  of  x  that  does  satisfy  (9),  or, 
in  other  words,  there  are  no  values  of  x  which  will  make  any  of 
the  factors 


262  APPENDIX. 

4--i/(a;+4)  +  |/(a;-4),  4+  ^/(x+4)+  v/(a;-4), 
or  4+  \/{x+4:)-V{x-4) 
vanish.     There  is,  therefore,  no  number  that  will  satisfy  the  con- 
ditions of  the  problem. 

[It  will  be  found  that  as  x  increases,  1/(0; 4- 4)  —  |/ (a;— 4) 
decreases,  hence  as  4  is  the  least  value  that  can  be  given  to  x 
without  involving  the  square-root  of  a  negative,  the  greatest  real 
value  of  ]/(a:+4)—  \/(a;  — 4)  is  1/8  which  is  less  than  4.  We  see 
by  this  that  our  method  of  solution  fails- for  (8)  simply  because  (8) 
is  impossible]  .  , 

5.  V{{a  +  x){b^x))-x/{{a-^){h-x)\  = 
^{{a-x){h^x)]  -  ^ {{a+x){h-x)}  (1) 

Collecting  the  terms  involving  -\/{a+x)  and  i/{a,-x)  respec- 
tively the  equation  becomes 

{x/{a  +  x)-V{a-x)}{i/{b+x)+\/{b-x)}=Q  (2) 

This  is  satisfied  if  either 

y'ia+x)-i/{a-x)  =  0  (3) 

or   Vib  +  x)+\/{b-x)  =  0  (4) 

The  rational  form  of  (3)  is  {a  +  x)  —  {a  -  cc)  =  0  which  is  satisfied 
by  a;  =  0  and  this  also  satisfies  (3). 

The  rational  form  of  (4)  is  (b  +  x)- {h—x)=0  which  requires 
x=0,  but  this  does  not  satisfy  (4).  Hence  the  second  factor  of 
the  left-hand  member  of  (2)  cannot  vanish. 

Therefore  the  only  solution  of  (2)  and  /.  of  (1)  is  a;  =  0,  derived 
from  (3). 

6.  f'{a+x)+^ia-x)  =  f^{2a) 

Cube  by  the  formula  (u+v)^  =  u^  +v^ +Suv{u+v) 

:.    {a+x)  +  {a-x)  +  nf{2a{a^-x-)}==2cL. 
.-.    2a(a3-a;3)  =  0, 
.".   X  =  ±«. 

Both  these  values  belong  to  the  proposed  equation. 


APl^NDIi. 

The  rationalizing  factors  of 

are  -^{a  +  x)-{-u^{a-x)-u--^[2a), 

and  |/(a+a;)-f  oj-|^(a  — x)  — w|^(2a).    See  page  257. 

The  remarks  on  Ex.  4,  will  apply  mutatis  mutandis  to  equations 
of  this  type. 


'■      f[a+x)^--^{a2-x^y,+  ^{a-x)^ 
Assume  -^(a+x}  =  u  and  -^[a  —  x)  =  v 
.'.  u^-{-v^  =2a  and.  u^ —v^  =  2x, 


u-^  —  V^        X 

and  .".     o   ,     -  — 

Also  (1)  becomes 

u^-\-uv+v'^ 

u^-uv-\-v^~'^ 

u  —  v 

Multiply  both  members  by 

u  +  v 

M3  |;S                 11  — V               ^ 

T      tO\ 

X  u  —  v 

u^  ->rv^~  "^  u  +  v^  '  '   "•'  ^"'     a    ~      u-\-v 


(1) 


(2) 


(8) 


(4) 


Again  adding  and  subtracting  denominators  and  numerators 
in  (3) 

m2^j^2       c  +  1 

My       ~  c  -\ 

Adding  and  subtracting  2  (denominators)  and  numerators  in  this 

M-— 2MV+t;3       3  — c  iu  —  v\^       3  — <T 

or 


w3  +2 WW  +  r2  ~  8c-  V  "      \« +«/         8c  -  1 

.c^  8-c 

.-.  substituting  by  (4),  "^=''■^3^^113' 


|3-r 


264 


APPENDIX. 


8.    \\/{x+a)  +  Vix-a)}^r{x-ha)  -  V(x~a)}  ^1c  {1) 
Assume  ?6=V(x+«)  aud  v=\/{x  —  a),  and  (1)  becomes 

(ii+v)^{i(,-v)  =  2c  or  (u+v)^{u^-T^)  =  2c  (2) 

Also  u^-v^  =  2a  or  (u^+v^){u^ -v^)  =  2a  (3) 

and  u^+v^  =  2,x.  .  (4) 

From  (2)  and  (3),  (u-v)2{i(2 -v2^)  =  u.-2c  (5) 

.'.    (2)X(5),    («2_t.2)2(„3_.j;2)2  or  (,t2_^.2)4  =4^(2(1-0)         (6) 

Also  (3)2 +  (6), 

or  (u4+i;4)(if2  _r3)2  ^2(«2+2ac-c2) 
Substituting  by  (4)  and  (6) 

2xi/(2flc-c2)  =  rt2+2ac-c2. 

Exercise. 

1.  {x-\-a-^b){x-a  +  b)-\-{x+a-b){x~a-h)  =  0. 

2.  (a  +  6a;)(/;  — rtx)  +  {6+oa;)(c— ?jx)  +  (c4-aip)(a— ca;)  =  0. 

3.  (a+bx){ax—b)  +  (6-|-ca:)(6a;  —  c)  +  (c  +  aa;)(c»  —  a) 

=  ^-(a2  +  i2+c2). 

4.  (a  +  a;)(6-a:)  +  (l+aa;)(l-6a;)  =  (a  +  6)(l+x-2). 

5.  (a4-.c)(i  +  a;)(c-,v)  +  (a+a;)(/>-x)(c+x)  +  (a— a;)(6+a;)(c  +  ^) 
+  (a-a;)(6-a;)(c+a;)+(a-a;)(6+.t,-)(c-a;)  + 

(a  +  a;)  (6  —  x){c  —  x)  =  5abc. 

6.  (rT,  +  a;)(?>  +  .T)(c  +  a;)  +-(«+x)(6+.c)(c-a;)  +  (a+a;)(/;-Jc)(c-f-a;) 
+  {a—x){b  +  x){c+x)-\-{a+x)  (b—x)  {c—x)  +  (a—x)  {b  +  x)  {c—x) 
+  {a-x){b  —  x){c+x)  +  {a-x){b-x){e-x)  =  8x^ 

7.  {a+5b+x){5a-^b-{-x)  =  3{a  +  b+x)^. 

8.  {a  +  nb+x){na+h+x)  =  9{a  +  h-\-x)^. 

9.  (9a~7fc  +  3a-)(96-7a+3a;)  =  (3a4-36  +  a-)=^. 


APPKNDIX. 


265 


ub                  cd  ^      x  —  a     x-\-a 

iO.    ,-T36^.-H,T3rfF->=0.  11.    ,+!+,— ,  =  2c. 

a+x     x+b  ax+b     ex+d 

'    a  —  x~x  —  b'    '  ■    a-\-bx~c+dx 

a-x       1  —  bx  ^      a—x        -k-x 

14.    :, =  -, .  15.    1 =  '. /— 

1-ax      b-x  1  —  iix     1-bx 

x+a  +  2h     b-2a  +  2x  a+U+x     Sh-a-{-x 

^^'    x  +  a-'lb^b  +  'Ia—'Ix  a-4:b+x~ 3b  +  a-x 

x-Loa  +  b      x-n  +  b  a—lb-\-x     a+5b+x 

■JO .  JQ  =         .    -  _  _ 

x  —  'da+b     a  —  x+'db'  '    la-b  —  x     5a-\-b-j-x 


Sa-b—x     5b-da+x        ^,      3a-2i  +  3a;       x-a+2b 
a  —  36  +  a;  ~  5a  —  36  4-;»' 

3rt-264-3a;_  x~7a  +  8b 
a  —  2b-\-x    ~  3a;— 5a +46 


^^'    ^-^6  +  a;'^5a-36+^-'  a-26  +  a;   ~3a;-3-i  +  26' 

22. 


5a_G6  +  u;     3a  — 55+3^  ff  +  6-a:       3(r?.-6-!-.'c)^ 

a+x       ~     rt  +  6+a;    *         '    3a  — 6  — 3a; ~  a— 56+a; 

la  +  b  —  x       3(fl^  — 6+a;) 
6a-{-  36  -  3a;  ~  a  -  176  +  a;" 

5a-b-\-x       2(2g— 6+a;) 
2(rt  +  26-sc)^  a+116-a;  * 

'ja-b-\-x     a{a  +  5h-\-x)  x-hn-h     a(x+a  +  ljb)^ 

27.    fbZi;[:^=  bij^i  +  b+x}'  x-a+b~b{x+5a+b)' 


25. 
26. 


29. 


5fl-36+x\  2     7a -96  + 3a; 


/5a- 
\56-: 


3a+a;/        76-9a+3a; 
/a  +  56+3;\  ^     a  +  176+x 


/a  +  56+3;\  ^  _  a  +  176+x 
30-     [^a+b+xj     ~17a+6+a; 

7a- 6+a;,  3_17a  +  6-a; 


31. 
32. 


\76 -a+a;/         176  +  a— a; 

17a +6-X  _  a  2  (a + 176-M)  ^ 
0+176^^  62  (17a +6  +  x)' 


2G6  APPENDi::. 


33. 


{5x  +  3a-llb){x-a  +  nb)     5x+7a  —  59b 


g^     ( 1  +3x+\5x^){x^  +  '3x+5)  _9^ 
{l+2x+Sx^){x^-{-2x+'3)~  4 


35. 
36. 
37. 
38. 
39. 
40. 


N/(l+a;2)  +  y/(l-.e2)       g 

V{l+X^)-y/[l-X^)-    b 

^(l+a;2)  +  ,f(l^a;2)  g 
-^(lJrx')-f{l-x^)~  b 
t/(l+^-")  +  t/(l-a;^ )  _  a^ 

V(l+a;2)  +  y(l-;e2)  _  _«_ 
e/'(l  +  .«^)-y(l-u;3)-  /; 

V(l+^2 )+ Via;3-1)  _  ji 
y(>f3+l)+y(x3-l)       a 


41 .  ■,/(4rt  +  A  -  4x)  -  2  v/  (a  +  Z.  -  2x)  =  i/&. 

42.  ^/(3rt-2/;  +  2a;)  -  %/ (3a -26-  2..:;)  =  2  v  a. 

43.  >v/(2«-/>+2.6-)-  -/(10a-96-6a;)  =  4N/(a-&). 

44.  -i/(3rt  -  4/v  +  o;c)  +  ]/(«-«)  =  2  \/(.t;+a). 

45.  \/(3a.-4//+5x)  +  i/(a;-«)  =  2v'{2.c-26). 

46.  i/(5.c-3r,4-4i)+^(5.e-3«-46)  =  2v/0c+aV 

47.  -i/(2a+5  +  2.f)  +  i/(10^<H-96-6.c)=2v/(2a  +  ?>-2a;). 

48.  2'/(2rt+6+2vC)  +  i/(10a+6  -  6a;)  =  v/(10a+96-6.-,;). 

49.  |/(2rt-13^  +  14a;)+v/{3(/>-2a+2a;)}=2v/(2«-6  +  2x-) 

50.  |/{3(7a4-6+a;)-N/(«+76-a;)  =  2,/(7a  +  6-x-). 

61.  >/{{a^x){x-^b))  +  V{{a-x){x-b))=^^{ax). 

62.  ^/{(a  +  x)(.c-4-6)[-i/{(a-a:)(x-6)}=2|/(M. 

63.  a/(«:c+x-)-  \/{cix-x^)-  v/(2ax-a2). 


a/.    ^ 


58 


APPENDIX.  267 

1  1  _  ^ 

^^-    l4-^(l-«)+l-i/(l-x)-  9*' 

x+ \/{ax)     a-\-i/{ax)     x  —  a 
«  —  n/  {ox)     X  —  ■\/{ax)  ~     a 

_     T/{(«+a:)(.-c  +  fc)}+A/{(«-a:)(.r-&)}         |^ 
{{a-{-x){x  +  b)}  -V{{a-x){x-lj)}      <b' 

i3rt-2/.  +  2.r     {x/g-h  \/{2a-2b)}' 
■^3(1  —  2b  —  2x  ~  '2b— a 

59.  ^{a-\-x)  +  f{a-x)  =  2^a. 

60.  ^(a+x)2-if(a2_^.3j_,_^(„_^)2^-^3/,j3. 

62.  f(l+x)'-' +1^(1 -^•)'  =  2i  1^(1 -^•-). 

63.  1^(3+^) +  i^^3-x)  =  1^6. 

64.  |^(l+x)3+|^(l-x)3  =  5{^(l  +  a;)  +  i;^/(l-x)}2. 

65.  ^{U+xy-  -^{VJQ-x"- j  +  f'iU-xy^  =1. 

66.  {^(9+a.0+p^/(9-x)}ir(81-a;3)  =  12. 

67.  {^(14+.i-)2-if(14-x)2}{i^(14+a;)-i^(14-a,-)}=16. 

68.  {f^{57+xy-  +^/{51  -xy-} {^{57 -x)+  ^{57 -hx)}  =100. 

69.  5{4/(41+a;)4-V('il--'^-)}^=8{V(41+.r)+]/(41-x)}. 

70.  {t/(x+5)+^{x-5)}m/(x+o)-t/{x-5)}=2. 

71.  {V(^+l)4-Ma'-l)}{v/(x+l)+v''(x-l)}  = 
26{t/(a;+l)-t/(^-l)}. 

72.  ^\±^^  +  f^]^^  =  a.      [y  +  r^=«]. 

73.  2{ir(l+a-)2+^V(l_x-2)}  =  (c2  +  l){^(lf.f)-v^(l-a:)}2. 


268  APPENDIX. 

74.  f^{a+x)  +  f'{a-x)  =  ^c. 

75.  {f{a  +  x)+f{a  -  x)}^{a^ -x^-)  =c. 

76.  ^{a+x)"-  -  ir(rt-  -a;3)4-ir («-«)-  =  fc\ 

77.  {#/(«+■«)'— if («-^)'}i]r(''4-^0-ir(«-^)}=c. 

78.  {^(a+:c)3  +  ^/(«-x)3}{^(a+.^)-f.^(a-x)}  =c. 

79.  (a+a;)|f(a-.T;)-(a-:«)irOf  +  .c)  =  ^{l^(a  +  a;)— |r(a-a;)}. 

80.  (a  +  ;^)|r('f+-«)-(«-^)lf  {«-*•) -''■{#^(«  +  ^)-ir(«-u^)}. 

81.  {if(«  +  .t-)3-y?/(a3_.,3)  +  ^(a_x)2}2  = 

c{if(a+x-)4-ir('^-.^')}- 

82.  {:t/(a+.'«)  +  M«-^-)}'  =  (<'  +  l){  /('^+-«)+  \/(«-:c)}. 

83.  {t/(-^-+«) -t'(.«-a)}{^/ (.*;  +  «) +  i/(.c-a)} 2::= 
c{:/(.f+a)  +  V(aJ-«)}. 


Section  IV. — Quadratic  Equations  and  Equations  that 

CAN    BE    resolved    AS    QUADixATICS. 


Examples. 
1.  x^  +  {ab  +  iy-  ={a^  ^h'^){x^-  +1)  +2{a^~b^')xi-l, 
^.    x^+n^b"-  =  {a^  +  b^)x''  +  2{a''i  -  b2)x  +  {a-  by 
:.    x^  +  2abx'^  +an^  =  {a-\-byx''  +2{a^  -b^)x  +  {a  -b)'- 
.-.    x''+ab=±{{a-\-h)x^-{a-b)], 
or  x^T{(i'-\-b)x  +  ab= +{a  —  b), 
:.   a;3qi(a4-^)cc+i(rt  +  6)3-i(a-&)3±(a-6), 


APPENDIX.  269 

2    («-^)V(«-a;)+(a;-fc)W(a;-^)_^^_;. 

{a  —  x)i/{a~x)-'[-)x  —  b)\/(x  —  b) 

Writer  — 6  in  the  form  {a—x)-x-{x~b)  and  multiply  by  the 
denominator  of  the  left-band  member, 

.-.    {a-x)-i,''(a-x}  +  {x  —  b)^V{x—h  = 

{a-x)'-\/{u-x)-\-  {a  —  x){x  —  b){\/(u-x)+  y(x-b)\-L 

{x-b)W{x-b), 

:.    {a—x){x-h){s^{a-x)+  /(x-i)}=0, 
.".    («  —  x)  =  0,  or  x  —  h  =  Oy 
or   s/ {a  —  x)+V{x  —  b)  =  Q. 

x'l  =«,     o-'g  =  b. 

The  equation  v'(a— x) -I- V(a;-6)  =0  has    no   solution   for  the 
sum  of  two  positive  square-roots,  cannot  vanish. 

The  solution  x=  •K'''  +  ^))  belongs  to  the  equation 
<^ya-x)-\/{jc-b)  =  Q. 

ax-\-b     mx—n 

3.      ,— J-  = 

bx-^  a     nx  —  m 

Add  and  subtract  Numerators  and  Denominators 

{n+b){x  +  l)      {in+n)(x-l) 
{a  -  b}{x  —  l)  ^  {m-n)[x-{-iy 

lx-{-l\  2  _(a  — 6)(m-fn)_ 


.        /x-hl>'^_(a-6)(m-fn)__, 

••       [x-lj         {a+b}{m-n)  ^' 


S-fl  _   Sj-1 

•••    a^i  =  ^^,  x^=  ^_|_^- 

6-1- u;         a  —a; 
Square  both  members,  subtract  4  and  extract  the  square-root. 


270  APPENDIX. 

•■•    v''^^  =  ^{'-±V(f2-4)}-^say, 
a  —  X  2x  —  {a-b)     1  —  e^' 


.-if 

■  —  n 


(a 


Or  thus,  cube  both  members, 
a  —  x  h-\-x       o 

0  -\-x  a  —  x 

{a-xY  +  {h^-xY  ^ 

2{a  —  x){b  -^x) 

f  (/^+x-)-(a-a;) \  3  _ 0^-30-2  _  (c^l) " fr -  2) 
•"■    uT+^i^(^^-)j     "  c 3  -  8c  +  2  "  ^c^l)  2  (^t  +  2) 

1x-{a-b)     c  +  l        |c-2 
*  ■         a  +  h       ^  c^\     'nIc  +  2* 

1-^3     c  +  1         Ic-2 
(Prove  «iat^:p-^3  =-]3     ^^T^g'  ^^  2.  =  .±v/(c3  -4). 

5.      -r, r-^vn: (  =  — ^^— — —-   Eationahze  Denom. 

{V{a-x)  —  V(b-z)]^  _  V{{a-x){b-x)} 
[a — b)  ~  c 

{V{a—x)-Vib-x)}^     a-b 
o^       ~'V{{a-x){b-x)}~  "     c    ' 

(V(a-a;)i- V(6-g)p_      a-b      . 
'■  \v{a-x)  +  V{b-x))        a-b  +  4c  '    - 

V{{a-x){b-x)}         I     g-fe  .    . 

c  ~N«-i  +  4c'  ^    '' 

Also  from  (A), 

a-^h-2x  a-b  +  2c 

V{{a-x){b  —  x)}^        'e        ' 


(^) 


APPENDIX.  271 

Multiply  (B)  and  (C)  member  by  member 

a:2  4-20 
6.  x4  -  4  =  -o-  ,-,  ;  x«  -  2u-A  ~  5^2  -12  =  0. 

Find  the  rational  linear  factors  of  the  left-hand  member  by  the 
method  of  Ai-t.  XXVII.,  page  90. 

.-.    {x~2){x+2){x*  +  2u>^--r3)  =  0, 

:.    a;-2  =  0,  orx4-2  =  0,  or  a;-i+2.c3  +  3  =  0. 

The  last  of  these  equations  may  be  solved  as  a  quadratic  giving 

x-  =  -l±2V—2,   :.  x=±l±V-2, 
.-.  x,=2,  x^_  =  -2,  X3  =  l  +  V-2,  x^  =  l-V-2,  ' 

a;5=-l  +  V-2,  a;g  =  -l-V-2. 

N.B. — In  solving  numerical  equations  of  the  higher  orders,  the 
rational  linear  factors  should  always  be  found  and  separated  as  dis- 
junctiie  equations,  before  other  methods  of  reduction  are  apj)lied. 
Such  separation  may  always  be  e£fected  by  the  methods  of  Arts. 
XXVII.  to  XXX.,  and  unless  it  is  done  the  appHcation  of  the 
higher  methods  may  actually  fail.  Thus,  if  it  be  attempted  to 
solve  as  a  cubic  the  equation, 

x^-dx-10  =  0 

the  result  is  x=  {5  +  ^-2}  +{5-V  —  2}  ,  which  can  be  reduced 
only  by  trial.  The  left-hand  member  can  however  be  easily 
factored  by  the  method  of  Ait.  XXVII..  and  the  equation  reduces 
to 

(a;+2)(.f2_2a:-5j  =  0, 

which  gives  x  =  2  or  \±yQ, 


272  APPENDIX. 

i'actor,  (See  No.  20,  p.  89),  rejecting  constant  factors, 
.-.  a;(a;-2)(.i,-2-2a;4-4)2=0 
.-.  x  =  0,  or  x-2  =  0,  ov  x^--2x  +  4:  =  0. 
The  last  equation  gives  x=l±:\/  - 3. 

Exercise. 

Solve  the  following  equations  : 

1.  {x+a  +  b)^=x^+a3+b^.  2.  (x  +  a  +  b)^  =x'^ i-a^ +  b'^. 

3.  (a—b)x^  +  {b-x)a^  +  {x-a)b3=0. 

4.  {a-b)x^  +{x-b)a-  +  {x+a)b^  =2abx. 

5.  (x-a)^  +  {a-b)^  +  {b-x)^  =0. 

6.  (x-ay  ■h{a-by+(b-x)''  =0. 

7.  {a^-b)x'^+{x^-a)b't-{-{b^-x)a*=abx{a2b2x^  -1). 

8.  {x—a){x-b){a—b)  +  {x-b)(^x-c){b-c)  + 
(x — f)  {;x  —  a)[c  —  a)  =  0. 

x^-1  x^^-1     ^ 

X—1  X^  —  i. 

11.    ^,--p^o.  12.  -^i3r-«- 

13.  x""  +  5a;3  -  lOx-^  4-20.r- 16  =  0.     (See  Art.  XXII.) 

14.  a;4_3^.3_|_5^2  4.6.,.4.jt  =  0. 

15.  (a:-«)4+a;4+a4  =  0.  16.    2x^  =  {x-6y. 

17.     4.c-2)2(a;+2)  =  2.  18.    (4x3 -17)u;  + 12  =  0. 

19.  x^  +  (ab^iy  =  {a''  +  b^){x^  +  l)  +  2(a»-b^)x  +  l. 

20.  a;=(a;-169)2-{-17a;  =  a;2-3540. 


APi-ENDIX.  873 

21.  6x(x2  +  l)3  +  (2x-5  +  5)3  =  150x+l. 

22.  2x{u;-l)2  +  2  =  (x+l)3.         23.    x^  =  12x  +  5. 

24.  5.«;-t  =  12.<;3  +  l.  25.    (a;+-l)3  =  3(2x--l)2. 

2G.  V(.f2  4-»t3)-f V{(n-ic) 3 +;/t2}=v](;<; --^91)3  + (A/iV3 -«()}• 
27  (a;+l)-*  w  28      (-^+1)^        ^^ 

'*        (j;2  +  l)(.«--  l;3-    -rt   *  *     ;c(.<;3  +  1)  "    >i    * 

oj       (x-+l)(.r^  +  l)      w-^  g2.  (a;3-l)3  m 


33. 


84. 


8G. 


x3(x-+l)  n  x(a;- +  !)(.<;— l)-*       « 

x{x-¥l)^  n{n — ni) 

(^H-  lT(I--l)-2  =  2>ii{2in-ny 

(x'^  +  ps  4m2 

a;(x3  -  1)-  ~?n3  —  ?f3 

(a;-l)(x3+l)^  _2{m  -v)^^ 
{x^  -  IXx-i-iy-  ~       rim 

x^  —1  2m 


{x  +  l){x'^-  1)     2m -n 


^         ^x^-l){x-{-l)^  m  +  n  (x-\-l)(x^  +  l)     m  +  n 

ax—b  ax  —  b 

39.      a;3=^ .  40.    x^  =  . 

ox  —a  ox  —a 

ax-h  ax^+bx+e 

41.      x'^=j .  42.    a;4=— — -; — ; ^^ 

ox— a  a-\-bx  +  cx^ 

43.     x^  =  {x-l)'{x^-{-l).  44.    a~x^  =  {a-xy{a^-x^). 

45.  X3^(j._„)2(a;2_l). 

46.  aVix'^  +  l)-xi/{x^+a2)  =  cx. 


274  *  APPENDIX. 

47.  ■^(a3  4-.i'3)-|-^(a3_a:3)  =  -^/(ae_3.6)2, 

48.  m{x+m  ~-n)(x  —  m  +7n)^  =  n{x  —  m+n)(x-\-lyn  —  7i)^ . 

49.  ni-{x  +  m  +  17n){x-m-5n)^  =  n°{x+llm  +  n){x-57n  +  7i)^, 

50.  m^{.r-\-m  +  lln){x—m  +  ln)^=n^{x  +  nin+n){x  +  "m-n)^. 

y/\x  —  n)  +  -^/{x  —  h)         \x  —  a 

''^^'  V{x  -  a)  -y{x-b)^  ^J^) 

g9  V{x-a)-^V(x-b)         la 


\'{x  —  a)-\-V{x  —  b)         la -a; 
V{x-a)-V{x-b) ""  -Nx  -  h 


\a  —  x         \h-\-x  \a—x,\h- 

53.         ri— -^ =c.  54.       , — +V— 

^A—x      ^J>+x  sl/a  —  xX^       ,  (b  — 


-  =c. 


.a— .r      ^  J)  +  X  a/''—x     a /^  —  X 

'  r-r-  +  V  -^  =  c.          58.    V I V : 

0  -f-x         a  —  X  0  —  X         a  —  X 


59.       Vr~+V- =  c.  60.     V; -e  rrC. 

b +x         a  —  x  b  —  x         a  —  x 

t,,"—x     f.  ,h-{-x  „^  ^,a  —  x     r,b  —  x 

61.      Vr-r-+V-^— =(?.  62.    V, -V r  =  c. 


63. 

64. 


h-\-x'^  ^  a-x       '  "''         b  —  x         a  —  x 

V(a~x)^+  V{b-x)^ 
■l/{a-x)  +  y{b-x} 

^/{a-x)^+i/{b-x)s 
{V{a-x)+y'{b-x)}^ 


\/{a-x)^+  l/{b-x)^  _ 
^/{a—x)-\/{b  —  x)    ~ 

gg       {V{a-x)+^/(b-x)}^_ 
^     '         ■y/{a  —  x)  -  V{b  —  x) 

^{a-xr+V{x-hy 
^''         V{a-x)+]/{x-b}    -"• 


APPENDIX.  276v 


^/(a-x)''-Vix-b)^  _ 
^^        ^(a-xy+K^{x  +  hy 

^/{a—x)^  +  V{x  +  h)^  _  {a  +  b]2 

"  V{a-x)  +  V{x  +  b)     ~-iV{{a-x){.c-^b)y 


70. 


x^+{a-x^)V{a-x^) 
x  +  V{a  —  x") 


7ti. 


x+V{a^-x^)  ^  ' 


73.      ^/{a-x:)-^~^{{a-x){x-h)\^^\x\hY=f{a^-ah^h^) 
^  b-^/[a  —  x)-\-a\f{x  —  })) 

^_        a\/{a-x)  +  bi/{x-h)    _ 
l/{a  —  x)  +  y{x-h)       ~ 

r-Q       v/(a;-a)+  \/ (x+a)  - -/ (2a)    _    yX+c 
^{x  —  a)  —  ^{x-\-a)+^{'la)    ~        x  —  c' 

78.  ir(a-x)2-if{(a-x)(.c  +  ^)}+ir(^+M2  =  -,^(rt2-«5  +  i3). 

79.  {^{a-xY-  -^[[a-x){x-h)-\  +^{x-b"~Yy^  =     . 

(r,.-i){Tf(a-a;)  +  i3/(a:-ft)}. 

80.  {f{a-xY+f'{b+xYy-={a  +  h){^^{a-x)  +  f/{bJrx)]. 

81.  if(a-x)  +  ^(x-6)  =  if6-. 

82.  iK(a+x)3--|f(a-a:)3  =  #^(2r.r). 

83.  f{a-xY-^^{{a-x){b-x)]+f/{b-xY  =  f/cK 

84.  i^/(a-x)---^{(a-a;)(a;  +  /.)}-^i3/(a;  +  i)2  = 


276  APPENDIX. 

85.  {^(a-x)  +  f/{x  +  b)}f/{{a-x){x  +  h)}=c. 

86.  ^/(a-xy+^{x-br-=c{f/{a~x)  +  f/{x-l)}K 

87.  x  +  f/{a^-x^)  =        ^/^ 

X'^{a^  —x^) 

gQ         a^ x+f/{2h^-x^) 

89.  {a+x)^{a+x)  +  {a-x)^{a-x)  =  a{ir(a  +  x)  +  ^:a-x)]. 

90.  (n  +  x)V{(t-x)  +  {a-x)::/{a  +  x)  =  a{^{a+x)  +  t/{a-x)}, 

91.  4/('-^6-.x-)  +  t/(.c-10)-2. 

92.  {^{a-x)+t'{x-b)}^"-=c{V{a-x)  +  ^/{x-b)}. 

93.  (a-a;i:^  a-;c)H-(a;-/>):y(.t;-&)  = 

94.  {t/{a-x)4-4y{x-h)}^  { ^/(a-x)  +  i/(x~-h)}  =  r{a  +  b  -  2x). 

95.  {^{a-x)+^ib-x)}{V(<.i-x)+Vib-x)}''  = 

c{y{a-x)-if{b-x)}. 

96.  fl\/(l  +  .r-)-a;i/(;c2+a-')  =  e. 

97.  {a-x)iV{x-b)  +  {x-b)f/{a-x)  =  c{f/(a-x)  +  ^{x-b)}''. 

98.  {ir(a-x)+ir(6  +  ^)}"=f{ir(a-a:)3  +  |r(6  +  ^)2}. 

99.  {^{a-x)  +  ^{b+x)}'=c^/{{a.-x){b^x)}. 
100.   -,3/(a-cc)^-ir(6-a;)3=ri3/(a+i-2a;). 

101.  V(^-^')+i/(^-^)  =  :!/'^- 

102.   ^(a-x)  +  ':/(x-b)=^c. 

(a  -  x)  i/(n-x)  +  (x  -  h)l/(x-b) 
^^^-       {^t^t/{x-b)+{x-b)t'[a-x)  =  ''' 


104. 
105. 


(a  -  x)^'h  - x)  +  (h  —  x)^(a  —  x) 
i/[a-x)-1/{h—x) 

i/{a-x)  +  ^{x-b)  c 


i/{a-x)~t/{x-h)     a+h-<2.x 


APPENDIX.  27T 

^^-  i/{a-x)-t/[h-x)      -'■ 

107.  [a-xyif{a-x)-{x-hy:/{x-h)  =  c{':/{a-x)-  ^{x-h)]. 

108.  {a-xY;/{x  +  b)  -{x-^hY:/{x-a)  =  c{i/{a~x)-^j^x+h)]. 

109.  {'^{a-xy^';/[X-bY]-:y{{a-x)ix-b)\=c. 

110.  {!y{a-x)-l/{x-h)y{-:y{a.-xy-%f{x-bY\=c. 

111.  {■y(a-.'c)3-^(a;-Z,)2}3{y(a-a;)  +  ^(a;-i)}=c. 

112.  {^(«-u;)3  +  y(a;  +  t)3}2=c{v(«-aj)  +  ^(.c+/0}- 


Section  V. — Quadratic  Equations  involving  two  or  more 

VARIABLES, 


1.  {x^-y){x^+y^)  =  a,  I. 

x^!/+xy^  =  e.  11. 

I+2IL  .-.  {x+y)^=a  +  2c 

.-,  x-\-y  =  f^{a+2e).     (Any  one  of  the  three  cube-roots).     III. 

I  —  II  x-+y^    ^   — ;  .-.  i^~^\  ^  ^   fi-^c 

xy  c  '         \x+yi  «-f-2c' 

n     TTT  l/(^  ~  2c) 

Also  ;.+j,   =    Y-^l 

v/(a+2c)  +  v/(a~2c) 


y  = 


2V(«  +  2c) 
\/(«+2c)-i/(a-2c) 


2V(«+2c) 

(Not  any  one  of  the  siy.  sixth-roots  of  a+2c  may  be  used  indiffer- 
ently in  the  denominator,  but  only  any  cube-root  of  whichever 
equare-root  of  a4-2c  is  used  in  the  numerator.     Thus  if  the  radi- 


278  APPENDIX, 

cal  sign  be  restricted  to  denote  merely  the  arithmetical  root,  if  k 
be  defined  by  the  equation  Ic^  —  k-\-l=0,  and  if  m  and  n  indicate 
any  integers  whatever,  equal  or  unequal,  the  value  of  x  may  be 
written 

{^'2™  \/ia-\-2c)+k^^-^  ^/{a-  2c)} H-2:y ^a -f  2c). 
2.       8x2-5a;?/+3?/2  =  9(x  +  ?/)  1. 

lla;2-8x(/+5?/2  =  13(a:+?/)  II. 

1st  Method.     Eliminate  {x+])). 

.-.  104a;-  -  65a;//+3%2  =  OQa;^  -72a;^  +  45^»» 
.-.  5.i;2-|-7a;//-6^2^0, 

.-.  x  =  %y  or  —2?/. 
Substitute  these  values  for  x  in  I. 

.-.  72.?/2  =  360y  or  45^3  =  -% 
.-.  y  =  0,  or  5,  or  -^, 
and  z  =  0,  or  3,  or  f. 

2nd  Method.     Take  the  sum  of  the  products  of  I.  and  H.  by 
arbitrary  multipliers  Jc  and  I, 

hiQz-'  -5a;?/+37/2)+Z(lla;2  -8X//  +  5//2)  =  (9A-  +  130(a:+?/).     HL 

Determine  k  and  I  so  that  the  left-hand  member  of  III.  may, 
like  the  right-hand  member,  be  a  multiple  oi  x+y.     This  may 
be  done  by  i^utting  x-  -y  in  III.  from  which 
16A"  +  24Z^0,    .-.    2/.-=-3Z 
;..   if  yl'  =  3,  /=-2. 

Substituting  these  values  in  III.,  it  becomes 
1x^-\-xy  —  y^=-x-^y 
■'•   (x  +  y)[2x-y)  =  x  +  y,  or  (x  +  y)(2x-y -1)=:0, 
:.    either  x  +  y  =  0,  or  2x  —  y-l  =  0, 
;,    y=^  -X,  or  2a; -1, 


279 


Substituting  these  values  for  y  in  I.,  it  becomes 
lG.r2=0,  or  10x--7x  +  3  =  '27a;-9, 
.*.  x  =  0,  or  3,  or  |  ; 
and  i/  =  0,  or  5,  or  —  ^. 

.rS-f-y''        flS  +  Z^S 

a:^  +  ?/^  ~«^+  f>^ 


I.-IL, 


(a;- +2/-)-  +x^y^ 


"      (x^+y^)^ -x-y"^' {a^+b-^y -a^b^ 

xy  ah 

Write  z  for  -;r~; — 7,  and  Z;  for 


a;2+2/3   """      ^""  a2  +  ^2 

z  A:  1 

"I-    •••    l3^  =  rrp'   •••  ^  =  ^or-^ 

xy  ab  a^  +/;2 

or 


x'^-\-y^     a^-\-b^  —ab 

xy  ctb  a^  -\-h'^ 


o,  or 


I. 


X*  —x'-^y  +  x-y^  —xy^+y^ 


III. 


'■     x^+xy  +  y^     a^  +  ab  +  b-'        a^—ab  +  b^ 

a2+ab-4-b^ 
II.,      .-.     ^//  =  «^,or(«24.,,.)-^_^-^  IV. 

v/ill.+IV.),  .-.  x+y=±{a-\-b} 

V^(II.-3IV.)  and  x-y=±{a-b), 

.a^+ab  +  b^ 

.*.  r=  ±a,    ife  or 


280 


y=+b;   ±ci  or 

M--\-ah  +  is 
lAV{%r'~ab  +  b^-)-j^{^a^-+ab^2h^^))V-^.f~^^^,- 

4  (^.=^+^2)(x3-+y3)  =  a,  I. 

Pat  2-       ^'J  '       ^-^ 


X^  +  l/^' 

1- 

2z'    ~     b 

%IZ^  —  bz~ 

-{a-b): 

=  0 

4:az'^=b±: 

7(8^3- 

-8ai  +  &2-), 

-b+r 

■  say. 

xy 

b+r 
'      4a 

x  +  y 

2rt4-?^  +  r 

a;-y/  'N2rt-i  — r 

.     x_   _    y^(2a4-6+r)+v/(2«-fc-r) 
"    Y    ~    i/(2a  +  i  +  r)  -  V  (2«  -  Z*  -  r  j 

_    {|/(2.<  +  /^+r)  +  N/(2a-^>-r)}'' 
2(6 +  y) 


in. 


IV. 


•       ^io/?/\M32«^(2a+6+r)(4a-6-r)2j    _  ^ 

~     1?//     (32(2a+?'+»-)(4«-^ -»■)'' ^ 

j^  _    {x/(2a  +  6+r)  +  -i/(2a-6-r)}"> 

1024(2a+&+?-)(4a-Z>-r)3 


X 


APPENDIX.  281 

2y{(2«  +  6  +  /-)(4a-6-r)»} 

in  which  r=  ±:v/(8a-  —'6ah-\-h-). 

The  value  of  y  may  be  derived  from  that  of  x  by  the  first  form 
iii  IV. 

6.  x"*  =ax  —  bij,  1. 

y*  =  ai/  —  bx.  n. 
iC.I.  —  7/.II.     x'^ —y^  =o{x^ —  y-) 
y.l.-xlL     xy{x^-y^)  =  h{x^-y^-), 

:.  either  x  —  i/  =  0  from  which  a;  =  ?/  =  0,  or  ^/(«  — 6)  IH. 

or     x'^+x^y  +  x^y^-\-xy^+y''  =  a(x+y)  IV. 

and  xy{x-  +xy  +  y-)  =  b{x+y)  V. 

(IV.+V.)  (^+2/)3(x2+y2)  =  a-|-6  VI. 

V.  (a:-|-2/)4-(a;2+i/3)3=4i(;r+y)  yil. 

1/(VII2  +  4.VI).    (a;+^)4  +  (:c2+y3)2=2i(.«  +  y)  VUI. 

in  which  i=  y{(«+6)3+462|^  jX. 

^(Vn.+Vin.),  .-.  {x+y)^  =  (26+0(x+</) 
:.{x-iyy  =  2b-{-t 
:.(x+y)  =^'i2b+t)  X. 

VI.  .A  ..X  -^y    -^^^2b-tt)  ^^■ 


2.XI.  -X2  .-.  (x—yV'  =  ^("±3^  -^/m^t\?'  -  - 


2a— t 


f\2b+t)    ^^^"-^'^  -f{2b+ty 


V{2a-t) 

X.  and..+  </    =v:(2Ml) 

V(2i+0 


282  APPENDIX. 

and  ^=^^^^+±1:^2^^; 
in  which  i=  \/{a2 +2ab  +  5b^). 

*+'/  2*  2?/ 

Let  2  = ',  .-.24-1  = andz— 1  =  — '—  III. 

x-y'  x-y  x-y 

I. +  11.  x'^+y'^  =  >n[x-\-y)'^  +  n{x-ij)^ 

...       (-  +  l)4  +  ,2  -  1)4  =  16(,H24.^„) 

.-.     (8«i-l)24 --6^2 +  (8/1-1)  =  0, 


=  4 


3  +  y-|9-(8/M-l)(8?^-l)} 
8?rt  —  1 


n   &III.  {2-\y(^x-y)^^-lQ>c*  =  lQn(x-y)* 


IV. 


2c 

•"•'''~^";t/{16n-(2-l)*}  ^^ 

and  a;  +  V  =4  tttt. ~! rr^T" 

-'       ^{lb»-(2  — 1)*} 

<z  +  l) c(;s  +  l) 

•'•   '^-y{16/i-(^-l)4}  -  v{2+l)*-16m"ij* 

C(2-1) 

and  y-  ^.s i6,i _  /gUiW '  ^^'^  ^^®  ^'^^'^^  *^^  *  ^^  given  by  IV. 
7.     :c2+y3  =  i(2»,,+«2^, 
x^  -\-y^  =  m.n. 

.'.  (a;+?/)--2x.v  =  i(27/?,+w») 
and  (x- + y)  3  -  3x//(j; + ?/)  =  nm. 


APPRNMX. 


283 


Let  u  =  x-i-y  and  v  =  xij,  and  the  equations  become 
tt^— 2t;=i(2m+n2); 
«'  —Zuv=^mn. 

Eliminate  r,  .•.  ?/3  — (2w+n2)»+2wn  =  0, 
.-.   M*  — (2/»  +  7^3)^t2_[-2w/i  t<  =  0, 

.*.  jt^  — ?/?  =  ±:(»u  — 7??.), 

.•.  «  =  «,  (the  value  u  =  Owas  introduced  by  the  multiplica- 
tion by  n), 

or  u"  +  n}i  -2??4=0, 
.-.  u=h{-n±V{n-  -^^m)) 
.'.  r  =  A(«2_„,)  ox  \{n^+Qm  +  Zni/{n^+Qm)} 
.-.  u  and  V  are  completely  determined. 
Also  x-{-y  =  u,  x  —  y=-J{u'—-iv) 

If  m  =  7  and  n=5,  the  above  equations  become 
x^  +?/2  =  13,  and  »■'  +y3  _  35. 

Solving,  as  above,  gives 
u  =  o,  or  2,  or  —7, 
2i'  =  12,  or  -9,  or  36, 
.•.  x-\-y  =  5,  or  2,  or  —7, 

x-7/=±l,  or  drv/22,  or  +/|/23. 
.-..  2;=  3,  2,  ^(2±s/22)  or  ^(-7+7V28j; 
?/  =  2,  3,  i(2+v/22)  or  i(-7=P;V23). 


284  APPENDIX. 

•••  il-^=(f-?y')^ 

Testing  this  for  rational  linear  factors  it  is  easily  reduced  to 

.:  y  =  l  or^(-2±-/2); 
x=^  or  i(-l  +  4A/2). 

H.     (2x-y  +  z){x  +  y  +  z)  =  d;  I. 

{x+2y—z}{x  +  y+z)  =  l;  11, 

{x+y-2z){x+y^z)^4:.  in. 

Let  s  =  x+]f+z  and  the  equations  may  be  written 

{s  +  x~2y)s^9  rV. 

(^s  +  y-2z]s^l  V. 

(s-8zV  =  4.  VI. 

1V.+3.V.      {4:S+x+y-Qz)s  =  12,OY  (5s-7z)s  =  12  VIL 

8  VII-7.VL     {(15s-21z)-(7s-21z)}s  =  8, 
.-.  8s2  =  8,    ..  s=+l. 
Substituting  in  I,  XL  and  III.  they  become 

2x-y+z=±9,     x+2y-z=±l,     x-^y-2z=±i, 
.'.  x^  +4,     y^+2,    z=+l. 
10      x^+y^  =  a; 

xy-\-uv  =  c; 

xu  +  yv  =  e. 
Let    t=xy—t('V.- 

.-.  (^x  +  y)^=a+c+t,    .•.x  =  l{./{a+c-{-t)-i-V{a+c-t)} 
{x-y)^=a-c-t,         y  =  lU/{a+c-\-t)-  \/{a-c-t)} 


APPENDIX.  285 

(u^vy  =  b  +  c-t,         ii  =  l{s^(b-}-c-t)  +  r/{h-c-{.t)\ 

(u-v)^=b-c  +  t,         vr=^{^/(b  +  c-t)-V{b-c-\-f)} 

Also  2{xu+i/v)  =  {x+]/)(i(+v)  +  {x  —  y){u—v)  =  2e, 

.:  ^/{{a  +  c  +  t){b+c-t)}+V{a-c-t){b-c-\-t)}=2e, 

.-.   {4.e^-{.{a-c-t){b-c  +  t)-{a  +  c-{-i}{b+c-t)}^  = 

■[Ge-{a-c-t){b—c  +  t). 

.-.   {{a-by-\-4:e2}t2-2{a^-b^)ct-\- 

(^a-^h)2c^—4:e-{ab+c^)-{-ie^  =  0, 

_   (fl^-&-)c±2cv/  [{ab~  g^){(ff-6)2-4(c2-g2)}] 
•'•    '  ~  {a-b)-+ie'^ 

11.  xy  =  nv  I, 

a;3  +  j/3  +  u3  +  r3=63  jjj 

x''+i/^+u^+i-^=c^  IV. 

liet  x+y  =  h{f+z)-    :■  ^i+v=l{a-z).  V. 

Also  let  r  =  xy  =  uv  yj, 

(a;  +  7/)  ^  =  a;3  +  2/3  +  3x?/(a;+ »/) 
(w+v)3  =  u^^v^+Buv{u  +  v) 
.-.  a(3z3+a2)  =  4(fc3+3«r)  VII. 

Also    {x  +  yf  =  x'  +y-'  +5xy{x^ -{-y^)  +  10x'' y^x  +  y) 

.-.  a(5z*  +  10a2z2irt4^  =  10|,.5^5^3,._|.i0f„.2|  yjji 

Eliminating  r  between  VII.  and  VIII, 

45a224  -  30a(a3  +263)52 ^^e  -  20a363  _8066  4-l44,,c«  =  0 

.•.15az2-5(a3+26^)=+2s/{5(a3_|.5i3)2_i80«c-}         IX. 

a3  4  263-|-2y[^{(a3  +  563)2-36ac=}] 
.-.  r=  i/ 3^ X. 


286  APPENDIX. 

VII.  &  IX.      12ar  =  a^-4:b^  +  3az-2 

=  2fl.3  - 2b^ ± 2V [i{{a-'  +5h^)-  - BGac^}] 

5(a^  - b^)±:V{5{a^  +  5b^)^  -ISOac^} 
•■•'■-  ¥Ort  ■^^• 

X.  and  XI.  give  the  values  of  z  and  r  which  may  now  be  treated 
as  knovv^u  in  V  and  V. 

^+y~¥''  -^^)>  anda;^  =  r 
.-.  x-ij  =  ,^v{{a+z)-''-lGr\ 

a;=i(fl+z±V{(a  +  2)2-16r}); 
v/  =  i(a+zqFV{(a+z)^-16r}). 

The  vah;es  of  u  and  v  may  be  obtained  from  those  of  x  and  y 
respectively  by  changing  z  into  -  z. 

Exercise. 

1.  6{(7-x)2+,v2}  =  13(7-x)2/,    x2  + 477  =  7/2 +4. 

2.  10a;3  -  9i/2  =  2a;^    Sx'^ -6//^  =  Vdx. 

3.  a;y  =  (3-a-)2=(2-r/)2.  4.  :c3+7/-2  =  8a;  +  %  =  144. 

5.  a:2+2/2=cc+7/+12,    X!/  +  8  =  2{x-\-y}. 

6.  a;+a;?/+y  =  5,    x2+a,-?/+//2=7. 

35         2S 

7.  x-'»+.V^  =  7a:?/  =  28(a:  +  .y).       8.  a^s+xv+z/^  =  ^^::^  =— • 

9.  s:4  4-a:3?/3+?/4  =  133,    a:^y+a:2?/2 +a;;!/3  :=  114, 

10.  (x4-?/)(a;-+2/-)  =  17a;(/,    (cc-?/)(.v2  -  (/2)  =9ar(/. 

11.  25(x^+y^)  =  l{x-\-i/)^  =  175x1/. 

12.  2a;2-?/3  =  14(a;3-2?/^)  =  14(a;-2/). 

13.  2x^-3x7j  =  9{x-3ij),    S{x^-Sy^)  =  2{2x^-dx!/). 

14.  2u;3-a;?/  +  5?/2  =  10(.T+?/),    a:2+4xv  +  3?/2  =  14(a:  +  .v). 

15.  (2a,— 32/)(3a;+4?/)  =  39(»-2y),  (3a;+2y)(4x-37/  =  (99(a;-2y) 


APPENDIX.  287 

16.  {x-{-2!/){x+3y)  =  3{x  +  y),     ^'2x-\-y){3z+y)  =  28(:x+y). 

17.  x+y  =  8,    x*  +  y*  =  70Q.       18.  x+y=^o,   u;^+^^  =  275. 
19.     x-\-y  =  2,   r6[x^'+y^)  =  V21{x-'+y^). 

•20.  .*;  +  // =  4,    ■il{x^+y^)  =  12-2{x'^+yi). 

21.  x^  —  5xy-{-y^+5  =  0,     xy  =  x-{-y-l. 

22.  x^+y  =  o{x-y),     x+y'^  =  2[x-y). 

23.  8(x-2  +  //)  =  3(a;+.?/3)  =  13xy. 

24.  10(.^3  4.^)  =  10(a:+2/^)  =  13(x2  +2/^). 

25.  .c2+.v=V,     ^+y-  =  ¥-     26.     9(a;-^+?/)  =  3(a;+^3)^7. 

27.  x+-c//+^  =  5,     x^+xy+y3  =  17. 

28.  ^+//  =  2,     (x-  +  l)^+(y-2)^  =  211. 

a;2  4-.c+l     31/.c2-x4-l\ 

29.  3(x-l)to  +  l)  =  4(.+l)fa-l),    j^5Tp^i  =  3sj(^._y|-i) 

1 

30.  x+y=  — ,     x-y  =  xy. 

31.  x+Tz-fl^O,     xc+i/s  4-2  =  0. 

32.  :c+j/  =  l,     3(.c8 +?/«)  =  7. 

33.  •4x^/2  =  5(5 -:«),     2(.^2  _|_,/2)  =  5. 

34.  21xy  =  17,     9{x-+y^)=-8. 

35.  (.^2+^2)2-1-4x3^/2  =  5-127/,      y(x2+2/2)+3  =  0. 

36.  z+y=xy,     x^+y'^  =x^+y^. 

37.  a;4  -  Qx^  V{y--x^)   - 16?/2  =  9a;2^ 
(x2  +5i)2  =4{2+:cV(2/'+i--)  -2/'}. 

88.  x(y3+3//-l)  =  2i/2+27/+3,     i/(a;2  +  3.c-l)  =  2.c3+2a;+3. 


288  APPENDIX. 


^^     ,    V^         o,.3      ^     ,11  .  /  a;  V 


39.  :l-  4.  ^   =    2c-3,    _    ,    iL   ^    e 
rtS    ^    ^3  a     ^     b  \a  b 

40.  rc^  +xi/^  =a,     y^  +  x^y  =  i. 

41.  .«+(/  =  a, -I-  ^    =    c. 

42.  x--^ay'^=  "^         ax^+y^  =  {a--l)y. 

a—1 

43.  x  +  y^=ax,    x^+y  =  by,         44.  x-\-y^=ay'^,    x^+y  =  bx^. 

45.  x^  -y^=a-{x  —  y)',     x^  -x^y+xy" —y^  =  b^{x+y).     . 

46.  (a;+^)(.«3  +  3^2)  =  ,,j,     (aJ- 2/)(.c3+3//2)  =  «. 

47.  x-y- -y{a-x)^  =x[b-yY. 

48.  a;3(6-(/)  =  (/3((i-x)  =  (a-a:)^(i-?/)3. 

50.  x^-y^=a{x-^-y^),     x^- +y'^  =  h{x+y). 

51.  a;  +  ?/  =  (i,     x^+y^^bxy. 

52.  I^   _       !^    =    — ^',     x{c"--\xy)    ^    ^^^ 


Ay  Ax 


a         y{o^-xy) 


53.     x  +  y  =  xy  =  X'-\-y--  54.    x-y=  —    =   x^-y^, 

y 

55.   .x3(l  +  //2)(l+i/4)  =  a,     a;3(l-2/2)(l-^4)=A. 

x^-xy+y^    ~         a  b 

57.     x^y-hxy'^=     ^^    ,,     a;'*(/+a;i/*=  &. 

59.      (^  +  ^)(x+i/)  =  «,     f!  +  ^   =   6. 
\y         xj  y         s,: 


APPENDIX.  289 

CO.     x^  +y2  =  rt.r2//3  =xii{x-\-ij). 
Gl.     abxy  =  a{x'^-'ty^)  =  h{x+!j)^. 
62.     xy{x+y)  =  a,     x^y^[x^  +y^)  =  b. 


63. 


(1    -f   l.](x^-y^)  =  a,       (1    _    J_)(a-3+^3^  =  ft. 
\  X  y  '  \  X  y  I 


64.  a;*  +  ?/4  =  m(.r3  4-?/2),     x^+xy  +  T/^  =:». 

65.  rt6(.c  +  ?/)  =  av/(a4-i),     x^ -fj/S  =a2  +  i'. 

66.  x^  ty^=a{x  +  y).      .r-i +//4  =  6f3r4-«.A" 

67.  a;2_|_,/2^f^^     a;5  4-w°  =i^fx3 +v*». 

68.  a,-y  =  (j,     a;-'  +  ?/-5=i(a;3-f  ?/3). 

69.  {x-y){x-^  +  u^)  =  {a-h){n^+h^),     x- -  y^  =  a"^  - b'-! . 

70.  a;2— ?/2=af^     a-3  _f_y3  =  i(^^_jy^_ 

71.  a;+?/  =  «,     a;4  4-//4  =  6.  72.  x  +  y  =  a,     x'''+y^=b. 

73.  a:+?/  =  a,     x~  +y-  =b^x-y'^. 

74.  x  +  ?/  =  rt+^;,     (a-612(a;4  4-_y4)  =  (^_y)3(rt4_i.^4j, 

75.  a  +  y  =  a,     o(.'"^  +  //4)  =a://(.t;3+^3)^ 

76.  {x+y)^=a{x''-}-y-),     xy  =  c{x  +  y). 

77.  a;2)/^a;^3=rtt^     c^(x^-{.y^)=x^y^. 

78.  x^  =:(i(x--\-y-)  —  cxy,     y^  =  c{x~-^y^)  —  axy. 

79.  a;3-?/2=</2      j.3_^3^c4  I  —  -— |. 

80.  x*-y''=a2xy,      {x- +y^)^  =  h^{x'^ -y^). 

81.  {x->ry)x-y'^=a,     x^+y^:=b. 

82.  (x  +  yjxy  =  a,     x^  +y'^  —  hxy. 


290  APPENDIX. 

83.  x^+y^  =  a{x+7jy,     x^+y^  =  b{x-\-9j)3. 

84.  x^  +  x-i/'^+y'^  =  a,     x^  —xy-\-y-  =  1. 

xHl+x^y^)      a       l  +  xy 

-\/{x'^y^)—x 

86.  x-\-y={x-y)V{xy),     ^^^i^^,s^^y  =  a. 

87.  ^  +  y  =  ^,     i/{l-x)-V{l-y)  =  b. 

88.  x^+y^=a{x  +  y),     x'^  +  y^  =  b{x^ +  y^). 

89.  x^+y^  =  a,     {x  +  7j){x^  +  y^)  =  b{x'^+y^). 

90.  (x^+tj^){x'-^+y^)  =  axy,     {x+y){x^+y'^)  =  hxy, 

91.  {x+yy-{x'^+y^)  =  a,      (x"- -{-y^)^x^+y^)  =  h. 

92.  (a;-?/)(a;2-?/2)(;^4_2/4)^4rt^^^^ 

[x-}-i/)(x^-+y^){x^+y^)  =  b{x-y). 

93.  a;4?/  +a;;y4  =  ^(a;^?/  -^xy^)  =  h{x'^  +//4). 

94.  a{x^-{-y'^)  =  ab{x-\-y)  =  bxy{x^-\-y^). 


95. 


a;^—  2/^     ffl^  —  b^ 

x^  —y^     a^  —  6* 

X'-y^~  a^-b^' 

x*  —  y^~  a'^  —  b' 

x^  +  y^     a^-\-b^ 
:r^-y^~a--b^' 

x^-\-y'^     a^^b* 
x^  —  y^~  a^  —  b^ 

96. 

97 .  .r^  =  ^ax  -by,     y^  =  %iy  —  bx. 

98.  (x+?/)(x3+i/3)  =  a,     {x-y){x^-y^)=h, 

99.  i^?^)^^^'i!±^^^-l    _    8^2, 
(x'^-\-ij^)\x^  —xy  +  y^) 

{x-y)^{x^-xy  +  y^)     ^    _,^^^ 
{x^+y'>){x^-^xy-\-y^) 


APPENDIX.  291 

100.  {x  +  y){x^ -^y^)=axi/,     {x  —  y){x^-y^)^bxi/. 

101.  {x  +  y){x^+y^)-^a{x-^+y-^),       ^x- y){x- -y')  =  b{x'+y-). 

iu2.       (■^+y)Hx'+y^)      ^  ^^2 

{x'^+xy  +  y'){x-+y-) 

{x-y)Hx^-yS)         ^    ^^ 


{x'-xy+y-){x--\-y-) 

103.     i^'^  +  'J^){x+y)^    ^   o,^3         (x^-y^){x-y)^    ^   ^^^^ 
x--^xy+y^  "■'    *  x-—xy+y'-^ 

(j;-'-t-j;//  +  ^-;-  {x-  —  xy+y-)- 

105.  xy(./;  +  ?/)(x3  +  7/3)=a,     xy{x  -  y){x^ -ij^)  =  b. 

106.  4x+(/)(a;+2^)(x4-8//)  =  a3,     (x  +  ?/)2  +  (a;  +  2i/)a  =6. 

107.  N/(:c-.cj;)-i-v/(y-a;^)  =  ^,     >/(x  — a;^)  +  v'(i/-2/-;  =  6. 

108.  (x4-l)(y-l)  =  «(x-l)(y-rl), 

(a-5  +  l)(t/-l)^=i3(.^_l).(^..+l). 

109.  x+t/  =  u,    V{x-lc)-\-V{y-^^)  =  c- 

no.     a:+(/  =  a(l  +  x7/),     (x-  +  ^)4  =  64(l  +  .c4_y4). 

111.  x+y  =  a{l  +  xy),     x^+2/=  ='->^(l+^-'^=). 

112.  (x+l)(y-l)  =  «(a;-l)(//+l), 

(x«-l)(y-l)  =  6^(7/5-l)(.«-l). 

(l+a;)a+?/)  (14-a;)3(i4.,/)3_^ 

i^^-      ^i_^^(i_^;-«'      ^i_^3)(i_^3) 

{c+x){c-\-y)  {c^+x^)(c'^  +  yi)_^ 

{x  +  m){y  +  iu  _  (x^  +  W^)(j/44-/< 4j ^ ^ 


292  APPENDIX. 

^^Q       {x+l){y  +  l)      a         {x^  +  l){y^  +  l)      h 
11Q       ^+^  P         ^(l+a;+a;^) 


122 


"n^/ 


y(l  +  a:^)  y'il  +  x^)_ 

(.a;-i/)(x-^  — i)"~    2(f6     '      (/(a;--lj~a-4-6 
TOO       (^+y)(l+a;.y)  x{l-y')_ 

(a;-yj(l-sc^)     "*'      {x- -y-)[l  -  x^y'^) 

,o(.       (a;4-y)(l+a;//)  (a;3+Z/^)(l  +  .^^y'^) 

'^^-       {x-y){l-xy)-''      {x^-yZ)il-x^y^)-'>' 

(a;2  _ xy  +  y^{l-xy-\-uy^ii^) 
{x-y)'{l-xy)^ 

129.      a;4  -3^3^  +  5«^x+^2=o,     2/^ -^^i/" -2a"x  =  0. 


APPENDIX.  293 

130.  2x(!/^-2x)^=a,     2/(y2_2a;)V(.v'-4cf)  =  6. 
(Hence  deduce  the  solution  of  a;^—6a;2 -|-2  =  0). 

131.  2xy{x^+y^y=a,     {x^ -ij^)ix'' -hy^)^  =b. 

132.  V{x--^i/^')  +  V{{a-x)^+y^}=V{{UVB-y)"  +  {la-x)^}, 

6(a;2 -2/a)  =  a(6a;-2</v3+a). 
Exercise. 
1.     {2x-}-y-4z)(x+y+z)  =  24:,  2.     x^-yf:=l, 

{x  +  2y-2z){x+y+z)  =  6,  y^-xz  =  2, 

{-2x+3y-{-5z){x  +  y+z)  =  nO,  z^-xy^3. 

3.  {x+2y-'dz){x  +  y  +  z)  —  2(xy+yz+zx)=  -12, 
(2x-3y+z){x+y+z)  +  {xy  +  yz  +  zx)  =  61, 
{Bx-y-i-2z){x-i-y+z)-5{xy-{-yz+zx)  =  5. 

4.  x--yz  =  0,  5.     {x^+y^-hz')^  +  (x+y)^=Sl. 

x  +  y+z  =  7,  {x^+y^+z^)^  +  [x+y  +  z)^=129. 

x3+7/3+22  =  21.  i^x+y)2-\-{x  +  y-^z)^^81. 

6.     «-— 1/2=0,  7.     x+yz  =  li, 

x-\-y-^z=21,  y  +  zx=ll, 

(a;-2/)2+(j/-s)3-f(z-x)2  =  126.         2+a;y  =  10, 

8.     x+2/  =  8z,  9,     «;+?/  =  52, 

x-^-i-yS  =  13iz^,  x^+y^  =  Sdz, 

a;2+2/^+z2  =  184.  a;3  4-^3  =  105^2. 

10.     x+y  =  7z,  11.     a:4-j,  =  72, 

a:2+i/2  =  25z2,  ic2+y2^25z2, 

x4  4-7/4  =  67423.  a;«  +  ?/«  =  20272z. 

12.     x+?/:y+2:2+x::a:i:c,  13.     x+y:ij-{-z:z+x::a:b:c, 
{'i  +  b-\-c)xyz  =  '^  {a  +  b  +  c)xyz=2{x+y-iz.) 


^94  APPENDIX. 

14.  ax  =  by^cz=   1    4.  J_   _|.    1.         16.     s /^  4.  ^)  =  a, 

X  y  z  \y      '      x] 

15.  {x-\-y  —  z)x  =  a,  jx  z\       , 

yl i =b, 

{x  —  y+zhj  =  b,  \  ^  -^  I 

(-x+y-i-z).  =  c.  xlJL   +    l.\==c. 

U  i/l 

17.     (y  +  z)(2x+y+z)=a,         18.  x(7/  +  z):y(z  +  x):z{x+y)  = 
{z+x)  {x  +  2y-\-z)  =  b,  6  -f  c :  c+a :  a  +  b, 

{x+y){x+yY^z)=c,  xy+yz+zx  =  {a-{-b  +  c){x-\-y  +  z). 

19.  {a  +  b)x+{b+c)y  +  {c  +  a)z  =  {a+b+c){x-}-y+z), 
a[x+y)  =  c{i/+z}, 

{x  +  y)^  +  iy  +  zy  +  [z+x)2  =4:{a^-^b-'  +c-^). 

20.  c(x+y)-i-b{x-z)-a{y-^z)  =  0, 
b{x-z)  =  {a-c)y, 

X-  +y^  +z^  =  a^  +b^  +  c^ . 

21.  x-\-y  —  az  =  x—by-\-z=  —cx  +  y-{-z  =  xyz. 

22.  {a  +  h  +  c){x-y)^-a{x+z)-b{y  +  z)=0, 
{a  +  b  +  c){x-z)+a{x  +  y)-c{y-\-z)  =  0.^ 

ax^  by^  cz^ 

23.  xy  -\-  —   =   a;      yz  -t  ~   ==    b,      zx  +    —   =*  e. 

z  X  y 

24.  y-\-z:z-\-x:x+y::h-\-c:c->ra.:a-{-b, 
{^x+y  +  z){xyz)  =  {a  +  b+c){xy+yz-{-zx). 

25.  n-  —■yz  =  a,     y^—xz  =  b,     z^-xy  =  c. 

26.  x-+{y-z)"=a-,     y'^ +  {z-x)- =b- ,     z" -^-^x-ryf  =c^. 


APPENDIX.  295 

27.  T-+:n/-{-y'^  =  a',     y^+yz-\-z^  =  h^,     z- +zx-\-X' =c*. 

28.  x^-\-y^ —z^-\-dxyz  =  a{x+y —  z) 
a;3_2/3_|_23  .{.Qxyz  =  b[x  —  y -\-z) , 

—  x^  +y^  +z'  +  dxyz=:  c{-  X  +  y  -\-z), 

29.  a-+?/  +  2flz=--0,                          30.  x-{-y-az  =  0, 
x"+y^-2bH^=0,  y^x^^y'i)^i%^ 

a:"+y" +  2"  =  (-■".  x^+y^  =  c^. 

81.     x(y-l)(2-l)  =  2a,  32.  xfy- 1)  =  «(2- 1), 

a;3(y3  _  1)(23  _  1)  =  6f2=  «3(,/S  _1)  ^  c3(23_  1). 

83.     a;(j/-l)  =  a(z-l),  34.  x{y-l)  =  a{z-l), 

z^{y---l)  =  k^z'^-\),  zHy-^-l)  =  h^z^^-l), 

x*{y^-l)  =  cHz^-l).  x^{y^  +  l)  =  c'^{z^  +  l). 

35.     x{y-l)  =  a{z  —  l),  36.  (ir-?/^  =^^2(.^•+?/), 

a;2(y3  _1)  =  7,2(^2  _  1)^  (a;3  _,^3  ^i2(a;+?/)3, 

87.     x—y  =  a,  88.  a--}-?/  =  rt, 

u  —  v  =  b,  u  +  v  =  b, 

xy  =  uv,  X'-\-H^=c^f 

x^  — 2/'4-«'  — ■"'  =(•(«  +  ?>).  t/-+r2  =e'^ . 

39.     xy  =  itv  —  a^,  40.  xy  =  uv  =  a', 

x+y+^i+x  =  b,  x-^y+u+v  =  b, 

41.     xy  =  au  =  a't  42.  xy  =  uv  =  a^^ 


296 


APPENDIX. 


x^+?/^+u^+v^=c\  (a;+M.)3+(y+i;)3=c». 

43.  xy  =  uv  =  a^,     x+i/-\-ii  +  v  =  b,  {j^  +  u)^  ^(^y  ^v^  =c*, 

44.  x!/  =  uv,  45.     xy  =  uv, 

x  +  y-\-u-\-v  =  a,  x-^y+u+v  =  a, 

46.  xy  =  2(v,  47.  xy  =  uv, 

x+y  +  u-\-v  =  a,  x  +  y->ru-^v  =  a, 

X^  +2/-  +2t-+t'2  =  62^  3.3  _^^3  4.^3  -1-^3  =  6', 

x^+y^-\-u^+v^=c^.  a;4  +  2/4+?t4  +  i;4  =  c4, 

48.     xy-uv  =  0,  49.     a;3+?/2=a2^ 

x:(,+yv  =  a'^,  u^+v2=m^, 

x+>/  +  ii  +  v  =  h,  ux+vy  =  c", 

x^-\-y^+u^+v^  =c^.  vx  +  uy  =  71^. 

50.     x+y+ii+v  =  a,  61.     2Kl+^^)  =  2iC, 

xy  +  2iv  =  b-,  u{l+y^)  =  2y, 

x^-{-y^=m-,  r(l+M2)  =  2M, 

u2+y2=,j2,  iC(l+r2)=2r. 

52.     a;+^+?<  +  ?'  =  a,     (a;+?/)2+(M+r)2=^,3^ 

X        la  —  u        y        2b  — u        z         2e— M 


63 


y-{-z     a-2u     z-\-x     b  —  2ii     x+y     c—2u 


ANSWERS. 


Exercise  i. 

1.  9.  -69,  1,  0,  1206,  -29,  1^.  2.  -160,  106,  41,  108 
8.  -T^,  H.  -25,  125,  ^V  -31,  -4^V0,  -1-  4.  9,8, 
7,  -^V  5.  176,  82,  254|6,  -37H-7if3.  6.  18  each. 
7.     146,14,-72,-270,396.         8.     Each  =  0. 

Exercise  ii. 

1.     -1.       2.      -166542.       3.     100.         4.     -2967511. 
5.     968.  6.      -162.  7.     10.  8.     -8.  9.     0. 

10.  -20.        11.  706440254900.        12.  0  each.        13.  Each  0. 

Exercise  iii. 

1.  0,  16a4.  2.  a,  a^/S.         3.  2a.,  0.         4.  26«e,   -26n^. 

5.     0.         6.  4rt4.         7.     6a4.         8.     f.  9.     c.  10.     0. 

11.  aH-(a+6).  12.  a3c(6  +  2c)  ^62.  13.  a24.i2_1.c2. 
14.  0.  15.  (12a26-24a63  4.2863)^(36-a)3.  16.  0. 
17.  0.  18.  -b^c.  19,  20,  21  and  22,  each  0. 
25.  2{b  +  l)h,  4x^.  32.  ^2  =  3^2.  33.  i=  y^(irf2), 
85.     'jjr',  xir  +  r'){r-r'). 

Exercise  iv. 

1.  2{bz+ei/).  2.     3{ax-b7j). 

8.  a^{z  —  z)  —  ab{x  —  i/)  —  b-{y  —  z). 

4.  {x-^y+z)(a  +  b  +  c).  5.     («  +  6+c)(x3 +  2/2+z2). 

6.  2{x+i/-\-z}x{a'-+b2  ^c^—ab-bc-ca).  7.     0. 

8.  2(ox+hy  +  cz).  9.     aS -f-62  .|.c2. 

10.  2a"(a-26).  11.     a-{-b-c. 


Exercise  v. 

1.     2(V4-9;r4),  4a^h^.  4.     -iia^-b^)^. 

5.     x^+Ax,    -3ix4-4at2?/2+3iy4.  6.     a^. 

8.  x^-6.v^+9x'>'  +  2xi/  -  6xy^  -  dx^y  +  lSx^i/^  +y^  -  6y^  +9y*. 
9.     4x//fx2-?/3),  2(1 +  12;c3 +16.1-4). 

10.  -r'^c^.        11.    a3_262,  8rt&(«  +  5)2.        12.     2{a-c){b-d), 
13.     ix=  +  if/2  +  iz3_|.i(^j/  +  y-.+;.;g).  ig_     {l+x-y-. 

16.     4(.-r.y+?/z  +  0.c)-2(«3+?/3+z2),  17.    3.2. 

18.     (a2+2i3_2c3)3.  19.     I6x^y2^         20.     -4ab. 

21.     4(a  +  64-<*)^-  23.     4(l+a;3+a;'t +a;S). 

24.   {a^x"+b2y"')2. 

Exercise  vi. 

1.  l-4r  +  10.r3-20.r3  4-25s:*-24a;^  +  16.c% 
1  - 2x+Sx^—4:X^  +3.c4  - 2x''  +a;«. 

2.  1  -4x  +  8x-3  -  14.^3  +  Ux^-Sx^+ox^  +6x''  +x^, 
l+6x  +  15x2+20a;3  +  15a;4  +  G.eS-fa;«. 

3.  4r/,2+//-'  +  c4  +  l-4a&-4ac3-4a  +  26c2  +  26-i-2c2, 
1  +a;3  +ij2  -f  23  _  2.C  +  2)/  +  2z  -  2xy-  2xz+2yz, 
ix^+^y""  +  36z2  -  ^xy  +  6a;z  -  4//z. 

4.  a;«-2./;-^'^  +  3a;-i!/3  — 4.i-3i/34.3x3;/4_2.r»/5+y6^ 

a^a;2  +  2a6a;3  +  (2ac  +  62)a;4  4.  2(rtrf  +  6c)x5  4.(26(i+ c2)a;6  4. 
2c(^x^+ci3a;8.         8.     3{a3 -^b^ +c^)-2{ab+bc+ca). 

11.  4rt'^  +^63^2  ^^^c^x-  +4:d-x-  -  2a6a;  — rtcx  +  8ao?a;+  ^kx*  — 
2bdx'^  —  cdx^. 

ExEKCisE  vii. 

1.     (a2_52)2.         2.     iu:4+y/4.  3.     a^^Sa"b2+4:b^. 

4.     a;* -2/*.         5.     ^.-3,         6.     1G,.;^         7.     0. 
8.     4a4-9&*-16c*4-24i3c2.  9.     b'^ -9c^ -4ta^ +  12ac, 

9cS_4a2_62+4a6.       10.    x^'-y^.       11.    a;8+j;4^4+^8^ 


ANSWERS.  lU 

12.     a^-a^i^+b^-l.         U.     «*+?/* +  tV;=2/'. 

15.  a;8+O^.G_j.3a.4^2x-  +  l. 

16.  'ia*x--Aa*x]j  +  aM/'^-a-x^-2a^:)^i/+2ax'^  +2az*ij-x^. 
20.     (.,:3+^--=_2xy-z2)3.  21.     x^  - y\ 

22.     ^-6u2+ 27^-1.         23.     (m+p)3-(«+?)2. 
24.     2.t-+x*  +  2x<^-x«-l.         25.     a8-Z>i6. 

Exercise  viii. 

1.  i-4+4a;3  4-3.^2- 2a; -12,    a;3+*/3 -2.r?/  +  8ar2-8?/2+15z2. 

2.  x-i  +  12x-3-f49.c-+78x-+10,    x*^ +bx^ -a^+Sab-2i2. 

3.  a»+8«e  — lUa4-104«2  +  10o,    a;«+2x« -x^— 2. 

4.  x^  +  ux^i/'-12x-iy^  +  5xy^  +  ij'^. 

x^     V2     2a;     2;/ 

5.  x-2"-2a;"-a2-16a-63,    — +^-j-— +  — -1. 

■(/-     a;-^Y/       x 

6.  7i2a;2+2na:y  +  ?/2  +  10Ha;  +  10?/  +  21. 

7.  (u;  +  a)2+2^(x4-a)-3i/2.     8.     a;*"+2x3»  +  a;2»(l-fr-6)- 
af(a+i)+«6.         9.     ia;8-u;'i?/3  +  j/4_;c4^22/»-8. 

5 

11.  a;4-8a;3+19j;2— 12a;+2. 

12.  (a;  +  6)*-(a2-f-c2)(a;+i)2H-a2o3.         13.     ai  +  ci. 

Exercise  ix. 
1.     2(l  +  3a;4),  2x>/^{3x^+x-y^).         2.    9i(a2  +  ft2^r762)^ 

6(27a2-27a6+7/>2).        3.    (x  +  y)^.        4.    ba^.        5.    8a;3. 
6.     8x3.      7.    ,/3.      8.    27x3.      9.   (2+a^3.      12.    S{x-+y-')^. 
14.     (a3+63)(a;3  +  2/3).       15.    0.       16.    0. 
Exercise  x. 
1.     l-3.r  +  6a;2-7a;3+6x*-3x*+a;6,  a^  -b^-r'!f -3a^{b+c) 
+  Sb^{a-c)+dc2{a-b)  +  6abc,  1  -  6x+21a;2  - 56a;3 -j- 
lllx*  -  174.t-^  +  219x'*  -  204a;^  +  144x«  -  64x^ 


/I        1  \ ^      .  / 1        1  1 

10.    — +—    =^  — +— 

\x       y I  \x       y  I 


IV  ANSWERS. 

5.  0.      t5.   45x^+1682a;*?/2_432a;3//223.       7.    (rta;  +  i?/+cz)3. 

Exercise  xi. 

a;^+7a;e?/  +  21a;57/2  +  35a;4y/ 3  +  85^,3^4  ^-2Lr3?/5+7a;.ye  +t/7, 
a;8_^8a;7^^28j;6i/3+5G;c5vy3  4_70a;4y4  4.56a;3?y5_|_28a;3.^6_^ 
Sxij-'  +y»,  x^  2  4.12^1 1/7  +  66^1 0//3  +  220a;9^-3  ^  495a;8y4_j. 
792^^2/=  +924^6(/6  +792^;^^^  +&c. 

2.  The  signs  will  be  alternately  positive  and  negative. 

3.  a^  -  5a^b  +  lOa^b^  -  lOa^b^  +  5ab^  -b^, 

a*  —  8rt2i+24a-62  —  32a63-(_i66*,  same  as  last,  terms  in 
inverse  order.  4.  l+(!))ii-\-15m-  +  ^Q>n.^ +  15m^^Qm^  + 
m^,  >n^  +5/?i4  JL.  I0»i3  +  10m^  +  5m  +  1,  Qhii^  +  192hi-'-j- 
240^4 +  160m3  +  60»i2-j-i2Ht  +  l.         5.     120. 

6.  x^ -'ix^ii  +  Qx^ij^ -"^x^y^+y^,    a^ -lQa^b"+^Oa^b*'~ 
ma^h^+^Oab^  -  3261",  «i  »  _  12rti  ^^3  +60ai s^g  _ 
160a9/^9  +  240a66i2— 192a36^5_j_64ii8. 

7.  495a8i4_792a7i5, 

Exercise  xii. 

1.  l^-a;3+a;4  4-a;6+a;l^  2.  1 +a;  +  x' 4-2^3  ^x^+^e+x^^ 
a.8^a.9_j.a;is.  3,  a;4  +  2a;3 -85a;2-86x+1680, 
2a;9-3a;6+4a;5+a;4+.x3_2x2-a:+2.  4.  ic«-57a;*  + 
266x3-1.  6.  18^8 +21a;^  + 8^6 +a;5+63x='+96a;2  + 
43x+6.  6.  l-ia;2-ix4.  7.  6x^2  _  4^9  _  5^8  _2a;7  + 
9x«-10x^+a;*-5a;3+5x2+.c  +  4.  8.  aj^^+g^^ +10x  +  ll. 
9  a;4  +  3x3.  10.  a;* -3^3.  11.  a;4+8x3-8a;. 
12.     (1),   -1.  (2),   -1,  (3)    -4.  13.  -1. 


ANSWERS.  V 

Exercise  xiii. 

1.     8x5-2^«-4.r  +  2.  2.     5z^  -  ix^  +  Bx^ -2x+l. 

8.  a'^  +  2a^  +  Sa^+ia  +  5.         4.     x^+2xy>/+3.rij^ +  4y3. 
5.     a^+Sa^x  +  Sax'^+x^.  6.     4x''-\-8x-^7,    -13x-20. 

7.  10x3  +  5x2  +  1,  lOx+lO.      8.     x^-xy  +  y^. 

9.  x3_„3,  10.  x^  +  {l-a)x^  +  {l-a  +  b)x^  +(l-a)x+l. 
11.     8a;3+2.r2+a'+li,  3Kx  +  l).  12.     5x- +lBdy  +  12y^. 

13.  6x*-a;4-a;3+x-2-a;+6,   -1. 

14.  2x4-3x3+4x2  — 5a; +  6.  15.     a  +  b.  16.     x+(/+2, 
17.      lOx-3,   10(x4-20).                   18.     wx3  +  n.r2+a. 

19.     l+x-5fx3- 3x3+9x4.       20.     88.         21.      -4. 
22.      -20.  23.     15?/4.  24.     85x+8.         25.     755. 

Exercise  xiv. 

1.  y3_2,/2-4?/-9,  if//  =  x-l. 

2.  y^  +  Sy  +  S,  if?/  =  x  +  l.  3.     ?/4  +  81,  ify  =  x-2. 

4.  7/4+4?/3-43(/3+92(/-67,  iiy^x+2. 

5.  3j/5+302/4+1192/3+238?/3+249/y  +  106,  if  y  =  a;-2. 

8.  (x-2//-'')-8.v(x-2i/)2_18(/3(x- 27/)  -247/3. 

9.  (:r-?/)^-10?/2(x-2/)3-207/3(x-^)2_lO?/4(^_y). 

10.  (2x+i/)3+2i/2(2x+2/)  +  5?/3. 

11.  512y^-3y-y\\,  ify  =  ix-^\. 

12.  (/4-24(/2+49i/-28,  if2/  =  x+2. 

Exercise  xv. 

1.     a^b,  +au'^-j-a^n  +  b^c-\-bc2+ac^, 

(a-i)2  +  (i-c)2+(c-a)2,    «(/.-c)  +  t(c-a)  +  c(a-i), 
afc(x  —  c)  +ic(x  —  a) +ac(x  —  i), 
a6c(a26  +  a36  +  i2c  +  a62-t-af2+6c2), 


Fl  ANSWERS. 

(a  +  h){c-a){c-b)  +  {b+c){a-b){a-c)  +  {c  +  a){b-c){b-a). 
(a+c)^^b^  +  {b+a)^-c^+{c  +  b)--a'', 
a{b+cy-\-b{c  +  a)^+c{a  +  by. 

2.  abc-\-bcd-\-cda-\-dab, 

a^{b+c-{-d)  +  b^{c  +  d+a)^c^{d+a  +  h)  +  d2{a  +  b  +  c), 

(a^  b)  +(a-c)  +  {a  —  d)  +  {b -c)-\-ib  -  d)  -r  (c-d), 

a-(a-b)  +  b-{b-c)+c"{c-d)  +  d^{d-a). 
13.     xandy.         14.     ax  and  by,  x,  y,  z.         15.    /audfe. . 
16.     X  and  y,  also  a;  and  —2,  and  y  and  —2. 
'/7.     a,  b  RJid  —c.         18.     it;2,    — ^/-ands^.  19.     b  And  c. 

20.     rt  and  c.  21.     a  and  6.  22.     a^  and  2aZ/. 

23.     a-6  and  abc.  24.  a^Z^,  a^c.         25.     a;^,  a^y,  and  x^j/'J 

same  ;  x'^y,  x^y~.  26.     Not  symmetrical. 

'28.     a^,  a36,  a-bc,  abed;  a^,  a^b^.        29.     a^,  uH. 
Exercise  xvi. 
1.     4(a3_|_7/i_j.c2),  2.     3(a3+i3+c-^)  +  2(a6  +  &c+ca). 

3.  4(a2+Z^2^.c2^f/2).         4.     2(a2+i2_|.c3). 

6.  4(a;2+?/2+22+?i2).         6.     2{a3-{-b^ -{-c^)  +  6:Sa^b -12abc. 

7.  14(x2_{_^24.22j_j_2(a;!/+?/z+2a;).         8.     24:abcmnr. 
9.     2aZ;c(a+i-t-c).  10.     aS^s  +  Z/^cS+c^aa. 

Exercise  xvii. 

1.     115.       2.   2;aS-33a5  +  3ra-s.       3.    2.  4.  -17-3533. 

5.     1,  2{Qa-+l).         6.    0or2?/",  2?/",  0.  7.    36. 

8.  -(Z^2^a2)3_ (3^2)3.         9.    _i5a4.  10.    3888a*6*. 

11.     a262(a  +  i),  12.  0.  13.  2a^ -3ai(a-6), 

2A34-6a6(a+Z/),  2(^3  +  ^3). 

Exercise  xx. 

1.     3.       2.    1.       3.    -l±2x/-2.       4.  2.       5.    36.      6.  11. 

7.      _l^^  13.   2^=-q,    q  =  (i.  14.   jL>=-46,  (^  =  14, 


ANSWERS.  Vll 

Exercise  xxi. 

1.     h=-S,r  =  S"2,d=-24:.     2.  c= -20i  (f=-13^,  ^  =  00i. 

8.  h=-d,  c=-10.  4.  a  =  S,  6  =  0,  c=-57.  5.  a=-2, 
c  =  2-H,  ^  =  0.  6.  c=-10G^,  d  =  202^.  7.  rt  =  200, 
b=-810,  c  =  639.  8.    «  =  4,  c  =  — 27,  d  =  7,  e  =  SO. 

9.  399.         10.    x^-{p  +  3)x^+{2p  +  q+d)x-{p  +  q+r+l). 

11.  :c3-(2?-3)x2-(2/)-y-3)a;-(^-2+r-l). 

12.  rx^-{dr-q)x^+{dr-2q+p)x-{r-q-{-p-l). 

13.  a;3-5x3+7>;-x-r2.     14.  a;3-(;)3_2^)x2 +  (9^ -2pr)a;- /-. 
15.  x^ —2qx^ +{pr-\-q^)x  +  r^ —pqr.       16.    ra^  —  (j9^  +  3r)a:;2 + 

(;>3_2^g-  +  3r)a;-(pj-r),      83.   -1.        34.   1.       35.-1. 
SO.  1.     37.    -1.     38and  39.  a  +  6  +  t+c^.     40.-1. 
Exercise  xxiii. 

1.  5?>4  +  15c4.     2.  6.     3.  3.    4.-{ili^6+c  +  rf)  +  ...  +  ...4---.}. 
5.  0.    6.  56*  — 30ai3+30a363_5rt36.   8.0.    9.0.    10.0.   11.1. 
12.  {a  +  l>-\-c  +  d).         13.    -1.         14.  a  +  b-\-c  +  d. 

15.  (rt  +  6 4-c) (« 2  4- i 3  _i_ 034. ((^4. 6c +ea)+rt6c. 

16.  (rt.+6+c)2(a2+i3_|.c3)  +  2rt6c(a+64-c).        17.  a4-h  +  c-Jrd. 
18.  (a  +  6  +  c4-<^)2.     19.  (a  +  6  +  c+rf){(fl+/>4-c-f<0-- 

(a64-<JfZ4-ac  +  6c  +  6f/  +  ctZ)}+«6aZ.        20.  a  +  6-fc.        21.   3. 

22.   -1.       23.  0.      24.  0.       25.  0.       20.  l+^x--kx^ +  ^'sx\ 

27.   1-ix-ix^  -^\x^.  28.  1+x+x-^+x^. 

20.   1— 2a;  +  3a;2-4x3.  30.  l-i-k-c-^x-^+^\x^. 

Exercise  xxiii.    /'a) 

1.      (p-p'+q)^  =  (p  +  l)(p^—pp'  —  q). 

8.  9ip^-q){r'-qt)-{pr-t)^=9{3{p^-q)(qr-pt)- 
{pq-r)ipr-t)}  X  {S{pq-r){r^  -qt)-(pr-t){qr-pf)}. 

9.  x"(4x3  +  3/>.c2  +3<^a;  +  r)  -i-  (a;*  +  4/;x3  +  67a:-'  +4rx  +  iJ. 


Vln  ANSWERS. 

10.  -4p.    (4/9)2-2(6?),    -(4;9)3+3(4j9)(6<z)-3(4r), 
(4^)4  _4(4p)-2(6(^)+4(4j9)(4r)+2(6^)2  -At, 

-(4/j)5  +  5(4/))3(6^)-5(4/>)2(4r)-5(4^j)(65)2  +  5(4j3)«  + 
5(6r7)(4r),  (4jt>)6-6(4;j)4(6y)4-6(4j5)3(4>-)  +  9(4;9)2(6(/)2  - 
6(47^)2i-12(4j3)(6?)(4r)-2(6<?)3  +  6(6?)i  +  3(4/-)2. 

11.  SqS^  — 4siS3+3s',  SQ.Sg  — Gs^Sg  +  lS.SgS^  — lO.Sg,  where  Sq,  »j, 
&c.,  are  the  coefficients  of  the  terras  (taken  in  order)  of 
the  quotient  in  No.  10. 

12.  x^{4:x^ -2Sx  +  l)-^{x^-Ux^+x-S8);  Sj  =0,  Sg  =  28,  s_, 

=  _3,    8^  =  544,    s^=-10,    S6=8683;    2(a-6)4  =  4526, 
2(a-6)6=264122. 

Exercise  xxiv. 

1.       (8m  +  2)2,    (C--1)^  2.       (y3_.3)3,    4^2(2x+y)2. 

3.  (3a&  +  2c)3,  4?/2(3a;-2/)2  4.  (|a;2  _ 4^2)3,  (^^^2 -1^2^2)2^ 

5.  («  +  6+c)2,(3a;4  -1^2)2.  6.  (z-;c+2/)2,Hy)    -(-j    f    . 

7.  (x2-z2)3.     8.     (a;-r/)4.  9.  (a  +  i)^,  (i^;3_4y3)3. 

10.  (a;-2/)2.  11.  4(x2+y2)2.         12.     (.c+^)4. 

,3.  iw".-(±)"r    14.  (.-j+c)^. 


_  6  /      \  rt 

15.  (a2-/;3_c2)3.        16.     (2rt-2c)2.        17.     (2*2  _g5  +  4.)8. 
Exercise  xxv. 

1.  (7«  +  2t)(7a-26).  2.     {Ba  +  lb){3a-U). 

3.  (3a-2fc)(9a2+4i2)(3a  +  26).  4.     (lOx- 6?/)  (10.^+6//). 

5.  5i(a+2x!/)(a-2.r;/)  6.     (3x3-4.y2)(3x3+47/2). 

7.  (3,  +  i)(|c-l).  8.     (22/2-|x2)(22/2  +  fx-j). 

9.  (3a-l)(3a  +  l)(9«2  +  l).    10.     {a-2b){a+2b){a2  ^U^). 

11.  (a-6)(a  +  fc)(a2+t2)(a4  +  64)(a«+&8). 


ANSWERS.  IX 

12  (a-^h-c)(a-h-}-c).     13.   {o  +  2b-Sx  +  4:y){a-j-2h-Sx-\-iy). 

14.  (x2_y2)3     15.  (a;_|.,/_|_92)(a;+_,/_2z).    16.  1G(.(,-  +  1)(1 -a;). 

17.  (x  +  y-\-z){x+y-z)'z  —  x-^y){z-\-x~y). 

18.  Axy{x  +  y){x     y).   19.  (x-  s  +  ?/)(a;-z  — </)(.r4-2+i/)(a;+2- ?/). 
20.  ^{a-Vc){h-^d).  21.     24a:(l  +  2x2).  22.     8ai(a  +  i)2. 

23.  {a^h-\'C-\-d){a-irc-b-d){a-b-c-\-d){a  +  b-c  —  d). 

24.  (x+2/+z)(a;-?/— z)(j;+2/-z)(a:-t/-|-2). 

25.  6a363^ae_3rt3;,3^^G).  26.     (fl3 -i./>3)(,f3  _i3)3. 

27.  (x2  +  j/2+52)(a.2+y2^22-2a:?/-2?/2-2z.r;)- 

28.  (x+22)(a;-2?/).     29.   («  +  fc-c)(«-i+c)(^  +  c+«)(6  +  c-a). 
80.  {x-y^z){x^y-z){x  +  y-\-z)[x-y-z). 

Exercise  xxvi. 

1       (a:-7)(a-+2),  (a;-7)(.r-2),   (a:+4)(a;  +  3). 

2.  (a;-3)(a;-6).  (a;-7)(a;-12),  («- 12)(a;+5). 

3.  2(2x-5)(x+2),  3(3a:-20)(a;-10). 

4.  i(.K  +  12)(ia--3),  5(a;+l)(5x  +  3),  (3a;3 -4)(3x3 -5). 

5.  (ia;  +  4)(ix  +  3).  4(4a:-5)(a;+l). 

6.  {x-a){:x  +  a)(x-b){x-\-b),   {'2.{x+y)~U]{2{x+y)  +  ^]. 

7.  (a;2+7/2-a2)(a;2+2/2+62).         s.     (a  +  6-3c)(a4-6+c). 
9.     (a;+?/)(H-a;+?/){x+?/+(a;-?/)2}. 

10.  {(i-\-b){\-a-h){a-{-b+{a-bY). 

11.  (a;2+x-^+2/2  +  2x  +  2/)x{A-2+x^+?/2-(a:  +  2y)}. 

12.  (a-o6  +  36)(a  +  i-c).  13.     (a;3+^3^^3j2  _  jg  ^^g_ 

14.  (A:2-10x-l2)(a;''-10a;+8). 

15.  (x2-14x+10)(a;-9)(a;-5).  16.     (x^ -2^2)2. 
17.     (z  +  l)(2-l)(22-2;,  (a;2-3)(x2  +  l), 


i  ANSWERS. 

(S:c*-\-5i/^){3x^-2y-^).  18.     (c'»+2)(c'"  -  1), 

19.      {x"'-ay''){x'"  +  by"). 

Exercise  xxvii. 

1.  {x-bij)(bx-y).  2.     3{x+2;/){2x~y). 

3.  4:[Ux-5i/){x-y).  4.  4.{Ux+5y){x-ij). 
5.  {14:x-y){x—20ij).  6.  4(7aj  — oy)(2x  — ;y). 
7.  2(28a;  +  y/)(:«-10?/).  8.     'i{Ux~5y){x+y). 

0.  (8.c-5.y)(7x-4y).  10.  {8x+5y){7x-4:y). 
11.  2(3x+7/)(a;-3y).  12.  (3x-2//)(2a;  +  3?/). 
13.  2i28x+y){x  +  10y).  14.  2(28.«-07/)(a;- 2?/). 
15.  2(28a;-f5?/((a;-2/y).  16.  {5Qx-5y){x-Ay). 
17.  2(4.^--?/)(7x-10.y).  18.  A(Ux  +  y){x-5y). 
19.  3(3./;+^j(4x-5i/).                 20.     (8x+5y){dx-8y). 

Exercise  xxviii. 

1.  (5.r-7)(2x-|-3).      2.  (5.c  +  3)(2x-7).      3.  (5a;-3)(2x+7). 

4.  (2;c-5)(3x-ll).    5.  (4a+l)(3a-2).       6.  (3.s-7)(4x-3)- 
7.  (3x  +  7)(4a;+3).  8    (Sa^  _462)(3a3+5i3). 

9.  (4x+l)(8x-l).  10.  Sy^{x-y){-dx+2y). 

11.  (2«+3i/)(2a;+?/).  12.  ic^(36+a:)(26-3a;), 

13.  (3x3 +72/2)(2a:3 -57/2).        u.  (2xa_9)(a;3  +  5), 

15.  (2x+2/)(2a;-j^)(a;~3?/)(a;+3?/). 

16.  {2.c+4-f.v)(2a;+4-7/)(.c+2-3.v)(a;  +  2  +  3?/). 

17.  169a;y.         18.     (19!/3  +  60x//-6a;2)(35x--12a-?/+30?/3). 

19.  2(4x7-3x2 -32/2)(61x2-49a;?/  +  61i/3). 

20.  2(50^3  +  4x//  +  10//2)(x2  +  10j;^+2(/3). 


ANSWERS.  Xt 

Exercise  xxix. 
1.     (7x+6//  +  B)(.r-2/-~).         2.     (5a;  -  52/-22)(4r+7/+4). 

8.  (Bz^+^i/^  +  13){x^-y^-l).  4.     {4x+5y){5x-^)j+7): 

5.  (9x+8j/-20){8x-7/-l).  6.     {x-\-Sy)ix-4!/-5). 

7.     (4.f  +  3^-0)(2x  +  3.y+z).         8.    (3a:-2»/-22)(2rc-  3//  +  42)!- 

9.  {Sx^—29/^+5z"){2x^+5!/^ -5). 

10.  (15a:2  +  8//2+5zn(a;=*-2//2+82-). 

11.  (2a-5i-7c)(2a  +  3/;-f3c).  ;. 

12.  (a -  i  +  c)(rt  +  b-  c){ci  +  b  +  c){a—h  -  c). 

Exercise  xxx. 
1.     -c'+'^+lv'S,  2a;2  +  ^±^l/5.        2.    ^r^ +  7,?/3^_3,/2  ^5^ 
^\{6x^-i-5y^±y^VlS).  3.  i(4a;2  4-5±v/13), 

TV{C(a:+2/)2+5z3+z3  ,/13}. 

6.  -rV(6-c24-5?/-')(6a;3+ll//2),    {6A-2  +  o)(6a;3  +  llj. 

7.  i(5x2  4-10±3N/10),  (2a3-f3±2,/2). 

8.  {2(x+y)2  +  (3±2V2>2};  ^ 
^{10x^  +  {l0±d\/10)y-}{10x''+(20-6y'10yj^}.                   \ 

9.  -K9-*^^+7±yi3),  A{2x■2  +  (6±l/16)(2,•+2)n• 
10.  ■  ^(2^:2+6+ v/G),  |(7u;2-f  20±,/85).  i 

11.  |{4x2  4-(9±v/28)^2|.  j 

12.  4{7('t-6)2  +  8c2+t;v/29},  -^{3«2 -1-62.^3}.  j 

13.  H^^'  +  l^+V^^)//'}.  i{Ha-\-b)^  +  {B±^/3){a-by}. 

14.  {7«2  +  (6±v/14)i-},  (5«i3  4-9n2)(5;/i-+3/j2). 

15.  {7{vi+n)2  +  {G±  V U){m-7iy} . 


XU  ANSWERS, 

ExEEciSE  xxxi. 

2.  (x-±2xy  +  2y^),  {4:X- ±?xy-}'y^),  {ix^±xy-\-y^). 

3.  (x- ±1/2x4-1),  (iK-  +  ^/6x(/  +  3?/2),   (ld=2!/-4y2). 

4.  (x2  +  3a;+l),  cc2  +  x/6j,-  +  3,  ia:2±2x?/+2/^. 

6.  (2^3+//2±|a:/y),   {x^ +y^-±\xyVSd),   (2a;2  +  l±2x). 

7.  (a:2'"  +  8?/2"'  +  4a;'";/'"),  x^"'  +  2y^"'zt'2'x"'y"'), 
(|a;2  —  ^y^±:xy  V5). 

8.  (2a;2_l  +  2x),    -(^a;2 -6//2 +a;//)(^.*-2  -  6?/2 -a;?/), 
(ic- +a2?/3 -ffla-?/y'2). 

9.  mx'^-ny'^±xy^p),  a;3'"  +  2"'-»7/2'"-+-2"'^'», 

10.  4x2 -3±x,  2x2_2±2a;v/2, 

-  (3a;2  -  2//2  +xy){^x^  -  2.7/2  -xy). 

11.  2x2±4.r^-3?/2,  a;2  +  2x  +  5. 

12.  2(a2+„6-j-/;2)2^     (2rt2_^a_,_l)2. 

13.  {(ir+2/)-  +  3(a;+?/)2+z2||(^  +  ^j2_3(3.+y)^425| 

14.  (a  +  fc)2+|c2±|c2v/5. 

15.  {4a2  +  5a(6-c)  +  2(6-c)2}{4r»2-5fl(6-c)  +  2(^>-c)2}. 

16.  4(a2+5ai-2i2)(i2  +  5f,i„2a2). 

17.  {(x2+y2_a,,/)2_j.3(a;2_|.^2_;,.^)^.,.  +  ^) 

18.  [i^a^^nb-\rh^)-\-l{a-li]^±^{a-bY^^]. 

19.  (4a2+2rt  +  l),  a;2  +  7^44. 

20.  (a;2±9x?/  +  9!/2),  (IdzSz+Sz^). 

21.  4(3a;2-2a;+l)(x2-2a;  +  3). 

Exercise  xxxii. 
1.     (a,-2  +  3Xa:+3)(a;-l).         2.     20i-2  +  3)(.T3  +  .r-3) 
3.     («2  +  4)(a;+4)(a;-l).         4.     (a;+2)(a;-2j(3x2 +a:  +  12). 


ANSWERS. 


5.  ix'-3){5x^+4:x+15).  6.  {x^ +  6){10x^+5x-G0). 

7.  (ia;2  +  TV)U-«'+^0-e-TTj)-     8.  (5a;2  -  l)(5x2  _8a;4-l). 

9.  (5j;2-8)(7i^2_6^_12), 

10.  (3x2-4j(2U-2-13^-28).    11.  (18.«2_|_i)(45^3  4.9^^s), 

12.  {nx'^  +  l){22x^--Bx-2).      13.  (U-^ -|j(^x-'+ix+|). 

14.  8{x^-2y^){10x^-4:xy  +  20y^). 

15.  (2a;2-5y^)(12*2-6a;(/  +  30//2). 

16.  (a;2-16?/2)(2x2  +  .^xy  +  32//2). 

17.  (a;2_  6)^11x2 +10x+^/).      18.  10(^-2 +2)(4x2  +  3:c- 8). 

19.  (a;2_6//)2(l.Sx2-12.cy  +  78^2). 

20.  (x2+4//2)(3.r2+3a;y-12^2|, 

21.  (.c2-3?/2)(5a:=+4x^  +  15y=). 

22.  2(x2-2?/2)(2a-2 -7a;?/+2//2|. 

23.  (x2+ii/2)(/;2+80x2/-^i/2).  24.  (x^- -Gy^){2x^-xy  ^12y^-). 

Exercise  xxxiii. 

1.  (.i;^+3.c  +  27)(x2-9x  +  27).  2.     a;2 +a;(l±|/3)  +  4. 

3.  {.i-2+l+i(l±\/5)x}.  4.     a;2  +  l-a;(2±:,/5). 

5.  2x2  +  2- 3j;±xv/23.  6.     (x2_|_i5_g_  5)(^2_a._5)^ 

7.  (4x2-2)(4x2-6x-2).  8.     (a;2+8x+4)(x2 -3x+4). 

9.  (x2+7x-2)(x2-x-2). 

10.  (x2+5x//+3;/2)(x2-xi/+3y2). 

11.  (x2+l0x-l)(x2  +  2x-l). 

12.  (x2+7xy  +  2/2)(x2-3xi/+2/2). 

13.  2x2  + J-?/ -5^2+ ^^^46.      14.  (x2+7x//-J/2)(x2-x^-y2). 

15.  x2+2?/2+3x2/±xy|/3, 

IG.  {3x2  +  10xy-2y2)(3x2  -4X//-2//2). 

17.  ^\{nx-+22y'^  +  5xy±^^xyi/n}. 


xiY  ANSWERS. 

Exercise  xxxiv. 

1.  {y-z){v/^-y).  2.   {hy  +  c){ax-^hy—c). 

3.  {2-'-{-a){x-\-a){x-a).  4.   (2a;-«)(a;-2/;). 

5.  (.T+3rt)(a;  +  2/>).  6.  (x-b-^){x-a){x  +  a). 

7.  (x-6)(a;  +  M(a.— a)(a;2+rt.x-H-a2).        8.  (2x  +  3«)(4.t+5?)). 

9.  {a-\-hx){a-hx^cx^)  10.   (a-&c)(a  +  &x+c.r2). 

11.  {ax-d){hx'^+rx-f).        12.   (p.r-9)(x3 -ar-1). 

lo_  («_j_c)(«  +  26  +  3cl.       14.    («  +  fl)(.'c-+a:-fl). 

15.  (wx-w)(pa;2+?ic-r).      16.   {x-a){x-h){x-c). 

17.  (cc  +  rO(a:-J)i,'c-c).  18.  (a;  +  a)(a;  +  i)(a;-c). 

19,  (rt3^z)(x-«v/)(a:3-7/).      20.  («6a;4-^^?/-<7/2)(«ic  +  %). 

21.  (^ax-\-c){ax'i  -hx-^c).         22.   (a;-?/)(xH-//)(w?a;-«//+7-z). 

23.  (wa;-w/)(«a;4-%  +  ^'z)-      24.   {?«x  +  r?.)(aa--k"a4-a). 

25.  (c2-xz)('^2_y2)(a2_a;_y).     26.   {x^-m^-x^-a){x^ -n  +  n''). 

27.  (l  +  x-a;2)(l-rta:+/^a;3-ca;3)- 

28.  («a--rf//)(««— t^)(«*'+ci/)-     29.  {wx+q){2rx+n){>n^x-n). 
30.  (»ia;+n^)('»:c-n?/)(/^2a;3+22y2)(a.  +  l). 

Exercise  xxxv. 

I  (^a,JrOc){a-h).  2.    («a:+%)(/w -«'?/). 

3.  (x-«)(a;+«)(a:^ +«»;  +  «-)•    4.   .r(a+a:)(«2  +  rta;+a;2). 

5,  {ax-h){cx  +  d).  6.    (5a:2  -  l)(5cc3 -a:  +  l). 

7,  («,_6)(«  +  i+.x--c).  8.    (rt2 +  />)(«+/,). 

9 


(a:-^/)(«+?/)^.  1^-  {x-y  +  l){x''+xy+y^-). 

11.     (fc-2.v)(2  +  i4  12.  (a;-l)(a;  +  2)2. 

13.     (/?-?)(;j^-2?2).  14.  (a-l)(a3  +  2a+2). 

15.     («^3_i)(3at2  +  l).  16.  (2/-l)2(y+2). 


ANSWERS.  XV 

17.  {a-\-b){2a^  -Bcib  +  2b2).      18.    {b"" -  l){b^'^ +  2b"' +2), 

19.  (^'^2")(ya.._3y.2..^,3»j.    20.    {a-b){a^-\-ab-2h^). 

21.  (a"'-c")(a"'  — '2c").  22.   (aa;-i)(»- -aa-i). 

23.  (5u;"-3«-)(7x"  +  8(t2).  2-4.  («6+6c-ca)(rt6  — 6c  +  ca). 

25.  (?rt-6)(OT,  +  /;)(a-m).  26.  (^ -3a2)(l  _3a)(l  +  3a). 

27.  {x-y-z)(x^  -2x1/+!/^  +z).  28.  (6m- 7w)(4m2 +«3). 

29.  (.c'" +  */")(•*;" +y'')-     30.   (a;2+.cy+«a;+!/2)(a;2+x(/-ax-.y)2. 
Exercise  xxxvi. 

1.  {x-7/){x-[-y){x^+xyi-tj^){x''-xij+y^-),      (x-l){x^-+x+l), 
{x  +  2){x-^  -2a;  +  4),     i2a-dx){4:a'  +Qax+9x-), 

{2  +  (ix){4:-2ax  +  a'-^x^). 

2.  {x-a^){x^+x^a^  -{-x^a^-^xa^  +a»), 
(3rt-4)(9a3-hl2«  +  16),   {a^ -b2){a^ +b^){a^  +  b^), 
{x^-2u){£^+2x\i/+4:X^y'^  +  Sx-!/^  +  16>j'^). 

3.  {a-b).  4.     :«  +  42/.  6.      (.«+7/)(a:2  ^?/2)(u:4  +  ,y4) 
6.     5(//3-x2)(7x*-ll.<;3^3  4.7_y4),    («2-2/>)(a2+26)(a*  +  4i2). 

9.  (<<  +  /;)(m+a)(m2-«?rt+a2). 

10.  {x^+xy+y^){x^-xy+y^){x^-+2xy-y^-). 

11.  (a3+6c)(a4-4a26c+762c3). 

12.  (a:-a+i){(a;-a)2-(x-a)ft  +  i2}. 

13.  (x^-2xy  +  Ay^){x+2y-hix7j). 

14.  (2:c+3//)(2x-3»/)2.  15.     H -2a;)(l+4a;3). 
16.  {a'^  +  ubc+b^c^){a-\-bc)(a^-abc  +  b^c^). 

Exercise  xxxvii. 
1.     S(x+y){y+z)(z+x).  2.  {a-b){b-c){a-c). 

3.     3(a3-/.2)(i2-c3)(c2-a3).  4.   (x  +  y){y-\-z){z-^x). 


XVI  ANSWERS. 

5.  3{a  +  b)(b+c)(c  +  a).  6.   (a-^h-\-c){a-h)(b-c)(c-a). 

7.  (a-i-h){b  +  c)(e-}-a).  8.     (a^ -b)h^ -c)(c^ -a). 
9.  {a  +  b)(b  +  c)(c+a).                   10.     (a-i)(6-c)(c-a). 

12.  (rt2  4.62  +  c2_rt6-6c-m)(a-6)(i-c)(c-rt)- 

13.  (a2  +  &2^c2)(«  +  ;;  +  r). 

14.  (c-i3)(a_c3)(6-a3).  15.    (.c2-?/^)(7/2_22)(^2_22). 

16.  (x  +  y-^z){x-7j+z){y-z+x){z^y-x). 

17.  (a-6)(/j-c)(a-c).  18.  8(a  +  i-fc)3. 

24.  (a-i)(^-c)(«-c)(«'-'  +  63  +  c2+ai  +  /;c+ca). 

Exercise  xxxviii. 

1.  (a-2)(«'--7fl  +  2).  2.  (x-2)(a;-3)(x-4). 

8.  (a;-3)(a;-2)2.  4.  (a:_2)2(a;  +  4). 

5.  {x-}-l){x^+2x  +  d).  6.   {x^-+2x  +  S){x-'  +  2x-\-S). 

7.  (.c  +  2)(a;-l)2.  8.   (x2  4-2a:  +  3)(a;2 -2x-+3). 

9.  (/n  — n)(»j2_2wm — 2)^2).   10.   None. 

11.  {m-n){m-2n)^.  12.   (&+3c)(62  _26c  +  13c2). 

13.  ^(^yn-n)^{m^~vin  +  n).    14.   (rf+26)(a— 26)(a2— 7a/j+462). 

15.  (a;- 5) (a: -3)2.  16.   (a:+2)(a;2H-3a;  +  l). 
17.  («-l)(a2_2a-195).         18.  {iJ  +  2){p-l){p  +  A). 

19.  (a-l)'^(«4-2)(a+3).  20.   (a2»_l)(a2»-2)(a2n_3). 

21.  «2_|.452  +  7rtj.  22.  (./-i)2(a2  +  2a6  +  2i2). 

23.  {p-2){p^-2p^2).  24.  (a:"-l)(a;2»  +  5a;" +5). 

25.  (?/-2)(i/3-3i/2+2j/+4).  26.  None. 

27.  {a-b){a'i  +  2ab-\-db^).       28.  (a"+l)(2r/2"-3^"  +  2). 

29.  (a;-2)(a:-3)(a;-6)(x--7).  30.  ix-y){x-2y){x-Syy. 


ANSWERS.  XVU 

Exercise  xxxix. 

1.  2{x-l)ix''-9.r+10),   {x-2yy'{z-3y). 

2.  (^x  +  3y){3z^-xy  +  y'),  (a;-l)(4a;-2)(2.c  +  3). 

3.  {x-5a){3x^+a-^),  {2x  +  3y){x^+3x!/-y^). 

4.  (6+c)(ft-4c-)(2/>2_6c+c3),   (oa+4:/*)(3a2+7a6-353). 

5.  {2p+q){2iJ-\-3q){p^-+q--'). 

6.  {10x-9y){15x+lQy){x^-5xy+Sy^). 

7.  (2jr-3</)(2.c4-3//)(3.c+4yj(3x-5?/). 

8.  (5a;-2z)(2^3_3^.2^_j.8^y2  +  12y3). 

Exercise  xl. 

4.  (a;+2//)(a;2+8!/2).  5.  l-2:K+3a;2. 

6.  {a-x){a+x)^.  7.  x-  +y-  -Jrz^  +xy+^z-zx. 

8.  (a4-/')(3a  +  6).  9.  (a;-.y)(2^+3^). 

10.  a^'-b^+c^.  11.  7a3_3rti-[-2i2, 

12.  rt-7.  13.   (a-fe)(6-c)(a-c), 

14.  (x-a)2-J(x— rt)+/>2.      15.  x^+y2-\-z^  +  l. 

16.  a:(a;2 -«./•+ 6j.  17.  .t^  +  z/S. 

18.  {x-y){x^+y'').  19.  aS  _/;-2_|.c2  4.i_ 

20.  a3-63-c3.    21.  a-\-x.    22.  (e— fc)(a  +  6  +  r). 

23.  ab-ca-bc.  24.  a;2+?/a  +  l  _a;?,-f  a;  +  ?/. 

25.  (a;3-2)(a;+l).  26.  a3H-5a+3.. 

27.  (2.£-i/ja2-(.c  +  ;/)a.c+:e3.  28.  a(x2+a;4-l)-(a;  +  l). 

Exercise  xli. 

1.  x^-3.  2.  x+r>.  3.  a:2__^_^-L  4..  ax'-'-i-bx  +  c^ 

5.  None.  6.  c^+c".         7.  {a-6)(x4-«).      8.  %+z/). 


9.  (a-b){h~c)(c-a),  10.  o.Z"'  +  \.  ^ 

12.  5{a-b){b-c)(c-a).  13.   {y-l)(x-l). 

15.  {x+l)(x"  +  l){x-l)^.  16.  {x+l){x  +  ^){x-\-'d)(x''r4:).   17.5 

18.  Same  as  given  quantity.     25.  {a  —  l){b  —  c){c— a). 

21).  «4+.t;-'+2x-l-l. 

Exercise  xlii. 

1.  (x-l)--(.'r2 +4^+16),  ic(3.v-7)-=-r/(72/-3). 

2.  (.?;2_«.,;-|-a2)H-(a.-^-«2),    (^4.4)-^(aj_.l)3. 

8.  (a;-l)0T+2)-r-(a;=  +  5.r  +  5),   (./;3  +  2a;  +  3)--(x3 -2.T-3). 

4.  l-(i-2.c),   l-^(a;3_2x-+2). 

5.  5fl3(ft,  +  .r)^a;(a2+a.7T  +  a;2),   (43:24.1)^(53.3  j_2;  + l). 
G.  (a;-2/)-^(.c+y).         7.   (S^^-o^  +  l)--(4«3^4_|.2rta;2  _1), 

(f7j;  +  i)/)-f-(rta;— %).       8.    —1-i-abc. 

9.  -(a  +  ^  +  c)-i-(«--6)(6-r)(c-a).      11.   5  - 7(^-'^ +a;^+?/2) 

Exercise  xliii. 

1.  (4~.r)-f-(5-a:),   (a'' +b-)^2ah. 

2.  a-,  2«-f  (a2  +  l).  3.  ^(l-f  r/)--(l  +  2rr  +  3a2),  x. 
4.  />2-=-a2,   (/)  +  l)-^r762.  5.  (rt(._i(/)^(rtc+M),  A-^a. 

8.  1.  9.     -(a'i+«2^3_|.^,4^_=_,.i(«_ij2^ 

10.  («.  +  /> +  c)2-=-2&c.  11.     1^1     ,    4:a^x^^{a^+x^). 

12.  (x  +  ?/)-f-(.t— ?/).  13.  (a -^) »--(«  + 6)  3. 

14.  {x+y)^{:x-y)  15.  l-=-a;3. 

lb.  l^n.  lY.    ±(l-i)-^(l  +  c^  la.   1-^0. 


ANSWERS.  Xll 

Exercise  xliv. 
1.     (x9a)-L.5.  2.     a-^b.  3.     IGa'a;-^  (a* -a;*)'. 

4.  0.  5.      l^(;c+2).  6.      l-f-(rt4-a;4). 

7.  12^?/-4-(9.r2-4v3).  8.     (43;2+2) -4-a:(16x4-l). 
,9.     l^{x  +  l}{x-[-2){x  +  8).     10,  4(u;4  4-4.'c-i/2+2/4)H-(.c4-?/4V 

11.  (a-i)3_^(a;-frt)2(a;-f6)2.  12.     2a ---X. 

13.     (236-77x)H-18(lla;-8).  14.     l--(a-6). 

15.  15a(3a-.'c)-^(9«+2u:)(«  +  3:r). 

IG.  (10x-7)^(a;-l)(2./;-o)-l-4-(2a;-7)(.«-4).  17.     Z. 

18.  ?/"(«/"-«")■  19.     (a -6)^" +  2. 

20.  0.  21.     4a;-^-^(a;i2_i), 

22.  -  (a2  +o2)(a2  _  ab'\-b-^)  -4-  (aS  -^s^^^a  ^. <,6  +  63). 

Exercise  xlv. 

1.  x-y.  2.     a  +  h.  3.     0.  4.     0.  5.     0, 

C.  {{a+b){c+a)x^  +  2(ah  +  ic+ca)ax-2a^bc}  h- 

{a-\-h){a+c){x  +  a)x{z-}-b){x+c.)  7.      1. 

8.  a+6  +  c.  9.     1.  10.     a;3_,/3  n.     q. 

12.  (rt-i)(6-c)(a-c)-r-(«  +  ^')(6+c)(c  +  a). 

13.  a;3-=-(a:-a)(a;-6)(x-c).  14.     1.  15.     0. 

16.  {b{x  +  a-b)-{-ax}  -^-  {(7t'4-(o -«)(.«— ^)}. 

Exercise  xlvii. 

1.  (fl_fc)2+4c-^=0.  2      8.         '3.     10.  4.     «2+/;«, 

5.  7J?  =  2,  «=1.  6.     2a;2,  or  6.  7.     7/?.=  — 5,  w  =  6. 
8.  ±12.     9.    (a2+&2j(c2+d2).     11.    _36c-46-2  4-i2c2-463. 

12.  (a;2-4a;+3)(a;2-4),    sl]bo  (x^ -Sx-\-2)lv^ -x -6). 

13.  i(-l±i/5).  15.     a-=-c  =  d2  H-e-',  rt-f-fe=/2 -r-e', 


XX  ANSWERS. 

h-^c  =  d-  -^f-.  17.     ac'^^b^d  &\ididad  =  bc. 

19.  i^;3_|_27g  =  o.  24.    _p  =  2?-/i32±2?7i^>/(m3+f). 

25.  4.{p-2,)^q. 

ExERCtsB»--  xlix. 

1.  6,  31,  a,    -3.  2.      -41,    -a,  2,   10. 

3.  a  +  ^  6--^/,  6-c,  3.  4.      -2,6,    -5,12. 

5.  -14,  a -36,  2a- 36,   5i-3a..  6.     7,  4,  a,  6. 

7.  ic,   5-=- a,  0,   1.  8.      -1,    {{a  +  hy^  -a}  -^b;  a+h. 

9.  (/)-«),  a  +  i.         10.      1-^a-b,   l^{a~b),   1  ^  («'-'+ ^/2). 

11.  2i,  a.  12.  a +  6,  c-i-(a  +  6),  i^(a-c). 

13.  (i_c) -=-(«- 6),   /^  +  c.  14.      ,-<  +  /;,   a^+ab  +  b^. 

15.  fl2-«&  +  /^^   1.  IG.      -1,   {a  +  b)-^{a-b). 

17.  (<;  +  &)(e-?>),  2-f-15,  3^14.  18.    -1-12,  b^ac,  a~b. 

19.  {a''-{-b-)-^a-b2)^a^b--',  a{b^ -{- c^)  ^  be. 

20.  10,  12,  4,  \.         21.   1000,  |,  |.         22.  9^^^^,  ai,  bc^a. 

23.  63_^rtc,  c(fl  +  &),  ft(«+6)-=-«. 

24.  a-^h,   {a-b)~{a  +  b),    -{a+b)^-^{n-J))^. 

25.  -1,    -1.  26.   (a2_c3)--(a  +  5)2,  2,  3^. 
27.  rt/^  i-^«,  rtCH-^  12.  28.  12,    -ac^b. 

29.  9,  2.        30.  12,  1.        31.  3,  1.        32.  (2fl-l)(2«  +  2),  0. 

38.  iH-m.     '     34.   1.  35.   {ah  +  bc+ca)-^(a2 +  bc+c^). 
3(3.  (^^^2^l,2J^c-)^{<th-^bc+ca).        37.  a  +  b+c.         38.   1. 

39.  1.         40.    1.         41.  1.         42.  15.         43.   16j.         44.  6. 
45.  5.         46.  {>i])cja+pqb  +  qc  +  d)-i-mnpg.  47.   —^. 

48.  0.         49.    -25^136.         50.  1. 


answers.  xxi 

Exercise  1. 
1.     2,  3*        2.  i,  i.         3.   ±2,  1:^.  4.  1,  1^.         5.   +f, 

±{a+b),  a.  6.  4,  5,  2,  2^.  7.    -3  or  2;  4,  -3; 

2i,    —  li.        8.   1  ;  f  or  | ;  i  or  3.       9.    — f  or  f,    ^  or  6  ; 
for  -|.  10.    -1,  2-,    -i,  1.  11.  0,    -b,  dh. 

12.     <f,    ±«V-1.  13.  1;  i(-l+v5).  14.   ±a. 

15.      +6c,    — (i  +  c).  16.  a  +  2b.  17.  // or  +a. 

18.      -2ab,  ^ab{lztV7).         19.  a,  ^/,    -{a-^b).         20.  a,  6. 
21.     rtorl-«.         22.    -«,    -6,  a -26. 

23.  «,  ^6(l-/y)H-(H-rt-^).         24.  a3-6.c2-37.<;  +  210. 

25.  a;4-4<fx3-13«-'a;:i+64fl3^-48c<4. 

26.  a;(a;-l)(x-T2j(u;-4)==0.  27.  a;* -4.1-3 +.c-2  4-6x-  +  2  =  0. 

Exercise  li. 

1.  4.         2.    -7f         3.    -107.         4.  8.         5.  3a.      6.  ^Vf- 

7.  50-1^,  17.         8.  22,  46A.         0.  7,  3.          10,  10,  10,  11. 

11.  0  or  11;  33.           12.3956^3971.            13.     |(15±v/190). 

14.  3.         15.  3.         16.  4.          17.   If         18.     U.        19.  3^. 

20  4.         21.   ±3.         22.  11.         23.  2and-l±i/-3. 

24.  2^.  25.  0.         26.  3a.         27.   |.          28.   Jf.         29.  3. 
80.  10.         31.  0,  1,  or  (-5±a/-23)-=-8.         82.  102f. 

33.     (-ll±:y'4681)H-20.         34.  2,  i,  |.         35.    -4 
86.     Qov  dz\/{a"+b^). 

Exercise  lii. 

1.  (l-<i)^(l+./),  a(m4-l)-H-(w-l),  b{m  +  l)-^a(m-l). 

2.  a  —  b,  0,  0.  3.      fc,  ijia-i-b,  b  ~  ca.  4.      1,    —1,  0. 
5.  -^or-1.         6.     {c-/v)(t-+c2)^2aic.         8.     14,  4^. 
9.  2,  6^295.       10.     73-- 210,   (./ f  6+r  +  t/) -r- (w+7i). 


ANSWERS. 


SI 


11.  b~a.  12.     h-^a.  13.     «  or  0. 

14.  rhi/rt'+l-^  2.  15.     h  16.     \^.  17.     0  or  4. 

18.  c-^ab.  19.     83i/(2a;-l)  =  100A/(3x-3). 
20.  75 --52.                     21.     8.  22.     34/^^. 
23.  l^»(n  — 1).           24.     a6-^(/>-a).           25.     4,  3f  or  13|. 
20.  a^h^  -~{a-hy~,'d.      27.     4a3-^(l+a)2,    i(ft  +  i)3_j.(a-i)2. 
28.  (1+62) -=-2^6.            29.     ^^(l-a;)  =  2  H- (a  +  i)2. 

30.  ~a±as/ {{l+h+h^)-^-lb]. 

.r+li  '    _        Z^^+l' 
-1/        "        U-li 

ExEBGiSE  liii. 

1.  8.  2.     0.  3.     3.  4.     {Vm  +  y'nY. 

5.  ab^il-'l-Jb).  6.     4^7.  7.     l--(«.-2). 

8.  18962-^-12393.  9.      Vo -^  ( v/'/  +  2). 

10.  (c4-2Z)c2)^(2c3-26).  11.     i.  12.     18a. 

13.  a;2=80-f-81.  14.     zh's^v/ar-  l^-      +Av/-ll- 

16.  ±v/    {«'-  ^^'-^^l^}-  17-     0.  18.     A«- 

19.  (^c-a-hy^^'2nahc.  20.     a:2  =  a2(7i-l)2 -f.  (2n- 1) 
21.  16..7/=0i-4.^'-^)2.  22.     0,    -|i. 

23.  /_^-l)^0.  24.     2v/(l-7«2)-=-mv'(4-m2). 

25.  (rt2_i)|a3-f-2+  V(«--|-lj}H-rt2. 

26.  {cn-an  +  cY~h{n-iy.  27.   ±5. 
28.  2\/(3x2  +  10)  =  (17;/17-3|/3)h-7.  29.    ±5. 
80.  ±,/(36-2a).                     31  ^^l{a^-h^). 


ANSWERS.  XXUl 

32.  (2//  +  2z-2x-)3+216.tv/2=0.  33.     fa/G. 

34.  a(rt2-47i-f-8)  -  (2/i-4).  85.     a- +2a. 
36.  ±y'{'3a^+b^)-i^y"d. 

Exercise  liv. 

1.  - 1^1(2 ^lj2^^a.                       2.   {2a2+b-)^2a. 

3.  {(a-  6)a3  -  2c{a-  +ab  +  b'')}^{a-'  -  2c{a^-b'^)}. 

4.  -  b.  5.   a +  6+6-.              6.  ab-7-[b  —  a). 

7.  a;--3«:K-«-=0,  &c.  8.  a.  9.  ^{a  +  b  +  c)  10.  lH-a6c. 
11.  l±{a  +  b-\-c).  12.  (rt-6)(«c-26)-^(«+6)ac.  13.  -c. 
14.  {fn  +  '^h)3.          15.    ±2.          16.   c^{a-b). 

17.  (a-6)-^(«  +  />).     18.  |a.             19.    ±2.          20.    +2,  &c. 

21.  ^(«+<)-^i"-0-    22.  «,  (3rti-3/^2_^,)^(l_,_3^j_3i) 

23.  ^/.       24.  a,  b,  'lb.        25.  a,  (c-* +  6«6)^6/>.       26.   ^(c  +  6a). 

27.  \a.    28.   a  +  Z'.               29.   (a/;  +  6c+c«)^(a  +  6+c). 

30.  ±6,   ±a.                       31.  i/{l^(^     !)}• 

32  {6(rt-5)-4c(c-6)}--{lc-3/>-a). 

33.  ((•2_a6)^(„+i^2c)  34.  ^{-29±V37). 

35.  (x-)-«)2  =  262-a2.  36.    ^ [b'^  -  ^ab).         37.  !(?>-«). 
38.  3^,  f     39.  a;--6.'c  =  a.     40.   lit:i/19.                41.  b,  b-a. 

42.  (r(2+6--)^(a  +  ^).         43.  a;=-5H-2,         44.  J (5 ±  a/3). 

45.  -2a,  4^,   la.                46.    -3c/. 

Exercise  Iv. 

1.  bc^{a  +  c).                             2.   («2  +  ^,_2a/.)-f(a  +  /^-). 

3.  (ad-ir)--(«-i).  .         5.     -^--^.          6.  c.          7.   .^(a  +  i) 

8.  a-\-h.         9.  0.  10.  0.                  11.  abc. 

12  («2+/>2_^e-3)-=-(a  +  6  +  c).      13.   (a+Z/  +  c)H-(a2  +  fc2-}-6-3). 


14.  {a^+h^^c^\^{ah+hcJrca)    15. —7.  16.    -^• 

17.  U.      19.  4.        20.   -140.        21.  17.  22.  10.        23.  a. 

^■^-  aC+c^-M:     25.  Si,  0.     2G.  S^.     27.  (r^fc-c2)-(«  +  6). 

28.  -&,  rt,       29.  0,  0.       30.  ^(fl-fi-c).  31.    -^. 

32.  (f.       33.  ah~{a^-h^).       34.    -3f.        H.5.  |.        36.    -3|. 

37.  lufinity.  38.  10.         39.  abc-^{ab  +  bc  +  ca). 

40.  {ab-{-hc  +  ca  -  ad  -  5  J  -  cd)-^(a  +b-\-c  -  3(/). 

41.  aib  +  c)^ -^(b'^  +  c^  - ab+bc  -  ca). 

42.  ic(rf-fl)  +  (({-6)(5-c)(c-J)H-(fl/)-fif4r^/-flc^-&^-c2). 

43.  bc^  —b^c-ac--\-b-d-ah(l  +  acd~-{ab-\-hc-ac-b^). 

44.  -(r/-f5  +  r).  45.    a-\-b-i-r.  40.    («&  +  &(•  + ra) -^  "Jr. 

47.      -i(/'+c).  48.  {ab  +  c)~2a.    49.  9.     50.  2.     51.  7. 

52.4.  63.  ^V(5+v/785).        rA.  A,  (am-nb)^{n-m  +  a--b). 

55.     T-V,  i(«  +  c)--(^/2  4.,,/,_^i3).     50.  0,    -I,  +.     .  67.  10. 

Sn.      «^,^(6-r),   c{a"--ir{b-c)a-bc]+a{b^-c^-)^ 
{a"-  +b^ -c^  +ah-bG-ac).       60.   6^{rt  +  5). 

61.  vipcq-^apnq-^iapn'^ —  cqm~). 

62.  {(')?/?(a  —  c)  +  0-/(6— ff)  4-r/^j(c  -  b)]  -^ 

{m((rt-c)+/t(6-a)+7-(c- /;)}.         G3.    {u^  ■]-b^)~ab,  0,  ^J. 

dhi-o)  -q(h-d) 
^^  ^     ^  '     a{n-  g)  —  m{b  —  d}  *'    ' 

66.     100.  67.  13,  111.  68.  11,  7. 

a2  +2ac+ad+2bc  +  2ab 


69.     (a  +  ^-m-7z).  70. 


fl  — (i 


XXV 


71.     U'i+b)±i/{i(a-b)3-lc2},aorb.     72.0.      IS.  a  +  b  +  c. 


7-i. 

UW-f  "C-t-C(l 

75.     a  +  i-j-c. 

76.     «  +  6+c. 

obc 

77. 

(ab  +  hc  +  cu)  -. 

■-(«+/>  +  (•)•             78. 

i2+a3_c3. 

79. 

c  —  a  —  b. 

80.     0.              81. 

0  or  11. 

Exercise  Ivi. 
1.     .4  =  0,  or  7^"  =  0.  2.     .^  =  0,  or  i?  =  0,   or  C's*  0. 

8.  «  =  0,  ora  — Z^  =  0.  4.     x  =  0,  or?/  =  0. 

5.     In  the  first  case  either  «  — 5^  =  0,  or  x— 4^+3  =  0,  in  the 
second  case  both  conditions  hold.  6.     s;  =  0,  or  a;:=a. 

7.     x  —  O,  or  x= —b.  8.     x  =  a,  or  x^c-~h. 

9.  .f=-0,  or  a;  =  3.  10.     sj  =  0,  or;7;  =  a4-6. 
11.     j;  =  0,  orjj=+a*.                  12.     a;  =  0,  ora;  =  i^-Ha. 
13.     x^O,  oru;  =  a.                      14.     a;  =  0,  or  a. 

15.     x  =  Q,  or  x^a  +  h.  16.     a;  =  0,  or  «  +  />. 

17.      -{2ah)-^{a+b).  18.     a;  =  a,  or6. 

19.     x  =  a,  orb,  ore.  20.     5.  21.     1.  2^.     21. 

23.     x=\,  x^'S.  24.     x  =  9,  .r  =  4. 

m 

25.  a;=l,  orB.  26.     («&)  h- (ff  +  6). 

27.  .r  =  a,  or  6.  28.     :c  =  {a^ +b^)  ^(a+b),  x  =  b+a. 

''.9.  {2nh)-i-(a  +  h).  30.     a;  =  a,  orA. 

1.  ?;=1,  or  (l4-a)-^(l -«)•  32.     a;  =  ff. 

34.  .i-  =  a-6,  or  i(i4-c)-  <^5.     x  =  a  +  b,  or  ^{a  +  c). 

36.     a;  =  " ,  or  1.  37.     a-{-b-c. 

^-^b+c 

89,     a;  =  a,  or  4 (46 -a).  39.     /;s -c,  or  a  +  fe  +  e. 


XXVI  ANSWERS. 


4f\           1           m  —  n  .^  nc  —  vh 

40.    a;  =  l,  or 41.     x  = 


n  —  p  mc  —  ap 

^2_     p(a-b)-c{m-n)^  ^3      ^^i^^^^,)^  ^r  i(6- a). 

vi(c  ~b)  —  a[7i  —p) 

44.     x  =  2a  —  b,  ox  Bb — 2a.  '45.     x  =  a-\-c  —  b,  ora;  = 

46.  x  =  a-\-b,  or    .  '—      

6  8 

47.  x  =  4n+b,  or  a  +  b.  48.    x  =  ,  or  __ 

b  —  c  c 

49.  {a-b){b-c)x^-{a^-\-b^+c--ab-bc-ca)x  + 
{a-c){a.-b)  =  0. 

50.  x=±'S,  or  +2,  51.  x=±Q,  or  +2. 

a+h         a—b  a  h 

7,  or  — |— r,     54.  X  =  -r-,  or  -  — • 

a  —  b         a-{-u  0  a 

2ri  +  3b         Sa  +  26 
55.     x~b  —  2a,  or  a  — 26.  56.    x=   — ^ ,  or  — ~^ — ^ 

57.     x  =  {mb+na)-^{m-irn),  ov  {ma-nb)-i-(m+n). 

68.     x=V{{m  +  2n)-  \/{m-2n)}^]/ {{)n-\-2n)+ ^{m -2n)}. 

17(6+1)- A/(c-l)f 

60.  «{v/(36--2)  +  /(2-c)}h-{v/(3c-2)--/(2-c)}. 

61.  a{N/(2c-l)  +  l}-^{l-A/(2c-l)}. 

62.  i(a  +  26).  63.  i(a  +  6)-(a -6)  \/(m-2;i)-f-N/(m4-2n). 

64.    -^-,  or  -^.        65.    2a6~(a+6). 

8a  +  5b         36  -  5a 

66.  «  =   — g — ,  or        g — • 

67.  x  =  2a{y'(c  +  4)-v(c--4)}--{v/(c  +  4)+  v/(c-4)}. 


ANSWEKS,  XXVU 

68.  .r  =  4,  or  3. 

69.  ^{«±|/a3-4w)  where  m^a^  ±:^  ^^tc-^a"^)  ;  3  or  1. 

70.  a,  b.  71.  x  =  i[a  +  b±y/{{a  +  bp—4.{ab-\-t)}], 
where  t  =  l{a-b)^± ly' {{a-by+  ir}. 

72.  x  =  0,  or  a,  or  ^a(l±i/-8);  a;  =  4,  or  2. 

73.  x  =  0,  ov  a  +  b,  ov  :i^{{a-\-b)±^V{a~b)2  -4:ab}. 

74.  z--{a-b)x+ab  =  &c.  75.  x^  l{Sa-b).  ov  l{3b-a). 

76.  a;  =  3rt-26,  or  36-2rt. 

77.  y^  —vi-  =0,  where  y  —  vi—x  and  2m  =  a-\-b.     See  Key. 

78.  2/2-m-'=0.  79.  2/--m2=0.  80.  y^-w^^O. 
81.  2/--«i^=0.         82.  j/2_„,2=o.  83.  ;?/2-«t3=0. 

84.  (2/=^ -^2K5y2+7/.2)^0,  (where  also  A-  =  |(rt-i). 

85.  A:4-Y/4=c.  86.    A5+10^-3y2^  5^7/4  =  c(^^4_y4)  &c. 

87.  «?/±/cv/(A;-3c±:r)  =  0,  where  .s-2  =3/t  +  c,  and  r-  =  {k-^c)^ 
+(A;-c)(3A;+c-).         88.    -3±  v/(9±12/24). 

89 — 102.  Work  with  a  variable  to  such  that  Mu;  =  a;-  +1. 

89.  r(;  =  (rt±s)-4-6,  where  .s  =  rt3  + 262. 

90.  if  =  (3rt+2i±s)-^2(a-6)  where  .s=  ±5,/(a2 +2a6  +  46'-'). 

91.  w  =  (3±&-)^(l±:.s)  where  s  =  (i- 4a) -=-/>. 

92.  (u'  +  lj2=rt^(«_i).  93.  w2^2a~-[b-a). 

94.  (a;-)-l)-4-(a,--l)  =  a^(fl-8?>). 

95.  (if  +  2)--(Mj-2)=.-^(l±6')  where  s=r (16'/ +  i)-^6. 

96.  ii'2(4a-6)^(rt-6).  97.  iv^  =  {^-la-'db)~[^a-b). 
98.  w  =  (7)±s)-=-2a  where  s^  =  /;2  +  le^s. 

09.  u;  =  (fl  +  6±«)-i-2(a  — 6)  where  s  =  («-)- 6) 2  -i-8(a-6). 


XXVIU  \NSWEES. 

100.  w  —  {a-l-b±s)^2{a-b)  where  s'^~{a  —  b)»  = 

{(a  +  5)2+4(a-i)2}-f(a-&)2. 

101.  (vf+2)±(w-2)=  +s-=-(4  +  3s)  where  s^  =2a-i-{a-\-b). 

102.  {ic  +  2)~-iv  =  ±i/{5a^{a-\-U)}. 

103.  i{2a  +  b),   i(a  +  26).  104.  2a-b,  |(«+5),  &c. 
105.  1,  2,  4,  5.             lOG.  ±1,  2,  4.  107.  1,  2,  3,  4. 
108.  -i,  -1    1,  f.     109.    -1,  8,  4.              110.    -a,  5a,  5a. 
111.  15,  20.                    112.  2i«.                         113.  4,    -1. 
114.  7,    —1.     115.  |(6f^rt  +  ca-4-A  +  r/7)^c).      IIG.    ±a^m,  &c. 

117.  2s(s-ft)(s-6)(s-c)-f-|/{.s'--rt2)(s'3_  0^(5/0  _^2j^  wliere 

2s  =  a  +  ft  +  c,  2.s'i=a3+62^c. 

118.  (2a6+2ac2+26c3-rt^-6-2-c4)-f-4c2.    119.  a,  b,  ^{a  +  b). 
120.      ±rt  or  ij'jfv'S.  121.  a,  b.  ^{a+b). 

Exercise  Ivii. 

2.  x,2;  y,  1.  3.  x,  S  ;  y,  1. 

5.  X,  -lOi;  2/,  5*.  G.  a;,  -2;  ?/,  i 

8.  .r,  -2;  ?/,  -3.  9.  x,  -|;  ?/,  a, 

11.  X,  12;  ?/,  8.  12.  a;,  8;  ?/,  -9. 

14.  «,  12;  y,  15.  15.  a:,  18;  y,  13. 

17.  a:,  7 ;  y,  .9. '  18.  a;,  7 ;  ?/,  -3. 

20.  X,  2 ;  7/,  3.  21.  x,  3  ;  ?/,  4. 

23.  a;,  -3;  ?y,  |.  24.  x,  12;  v/,  15 

20.  a;,  8;  y,  9.  27.  x,  3;  ?/,  1. 

29.  a;,  11;  //,  7.  30.  a;,  17;  y,  13. 

32.  a:,  -4^;  y,  -\^.  33.  a:,  13;  y,  10. 

35.  x,  11;  ?/,  6.  36.  a;,  7*?/,  5. 


1. 

X, 

7;  ?/,  9. 

4. 

X, 

9 ;  y,  5. 

7. 

X, 

-1;  2/,  1. 

10. 

X, 

-i;  y,h 

13. 

X, 

10;  y,  12. 

16. 

X, 

•3 ;  y,  -2. 

19. 

X, 

7:  y,  3. 

22. 

X, 

25. 

X, 

7    •    1;       3 

28. 

X, 

7;  2/,  8. 

81. 

X, 

6;  y,  -4. 

S4. 

X, 

4|;  2/,3A 

ANSWERS. 

XXIX 

37. 

x,2;  y,  3. 

88. 

X,  5 ;  y,  3. 

89.     Equations 

40. 

x,  3  ;  y,  1. 

41. 

X,  7 ;  y,  5. 

not  independent. 

42. 

x  =  0  =  y  =  Q. 

43. 

0,  0.         44. 

X 

=  0  or  13  ;  y  =  Q  or  f  f- 

45. 

X,  17  ;  y,  20  ; 

z,  5. 

46. 

X 

23  ' 

',      234       -,     247 
>    2/'  T30'    ^^    rSTT- 

47. 

11,  7,  9. 

48. 

21,  22,  23. 

49. 

-15,  -6,   -8. 

50. 

3,  4,  5. 

51. 

12,  15,  10. 

52. 

5,  3,  1. 

53. 

h  H.  f  • 

54. 

3,  5,  7. 

55. 

11,  13,  17. 

56. 

5,  3,  1. 

57. 

9,  7,  3. 

58. 

7i,  8i,  9i. 

59. 

31,  2i,  14.. 

60. 

2-3,  3-4,  4-5. 

61. 

30,  20,  70. 

62. 

88 -=-59,  109^ 

}^51 

[),  1004 --59. 

63. 

30,  12,  70. 

64. 

6,  12,  20. 

65. 

5,  2,  0. 

%Q. 

1,  1,  1. 

67. 

11,  9,  7. 

68. 

5,  3,  1. 

69. 

2,  3,  1. 

70. 

3,  4,  5. 

71. 

XXX 
.S'    3'    f 

72. 

5,  4,  3. 

73. 

7,  3,  1. 

74. 

2,  3,  1. 

75. 

1,  3,  5. 

76. 

0,  1,  2. 

77. 

1755-698,   360 

^349 

,  —15705^698. 

78. 

I  h  1- 

79. 

5,  4,  1,  3. 

80. 

4|,3A.2A,U. 

81. 

31,  41,  51,  21. 

82.     7, 

41 

,4,8 

h 

83. 

20,  10,  0,  30. 

84.     11 

-^ 

24,    : 

\,  1-^24,1 

85. 

270 -- 117,   - 

52-- 

■117,  15  H- 117, 

- 

-126 

^117. 

86. 

Each  210. 

Exercise  Iviii. 

1.      {alc—ac')M^'b-<tb').  2.  h{cn- dm)~{nd -he). 

3.  h{d-c){d-a)-^d{h-c){h-a),  c{d-a){d-b)~d{c-a){o-b). 

4.  j/  =  c2-j-ff?<. -f  ^'f'  +  f'^;,  z  =  <hi-\-ew-{-ax-\-hy, 
u  =  ew+ax-{-by  +  cz,  ic  =  ax-\-by-^cz-\-d\i. 

5.  «  =  -^»!(a-^4-c),  &c.  Q.  x={p{a^-b)-m{ab-l)  + 


XXX  ANSWERi5. 

7.  x={l- arn  +  abn  —  abc2:)-}-(ibcdr)~-{l-\-abcd€),  &c. 

8.  1  =  n-^{l-^a)  +  b-T-{l  +b)  +  c-^[l  +c). 

9.  1  =  ai  +  Zw  +  ca  +  2abc. 

Exercise  lix. 

1.  (nc  —  hd)^(na  —  hm),   (vie  — ad)-^(mh  —  na). 

2.  (no-\-bd)-i-{a?i  +  bm.),   (mc  —  ad)-i-{bm-\-an). 
8.  c{n  — b)-^ {an  — 771  b),  c{m~a) -^  [bm— am). 

4.  {b  —  c)a-i-{b—a),  b{a  —  c)-7-(a  —  b).     5.  ab-i-(a-\-h),  y,  same. 

6.  ab"-i-{a^  +  b"),  a~b^{a-+b^).        7.  ac-^ia  +  b),  icH-^a  +  ft) 

8.  (a- — i-)-J-(an(  — 5m),   (/;2  _(:j3j-^^5„i_  ^^), 

9.  a  +  b  —  c,  c  +  a  —  b.  10.  a  +  c.  5-fc. 

11.  a(cn— rf»?)^-(iici  — ac),  b{cn  —  dm)-^{ad—bc). 

12.  i/={93(a2_c2)-6(6  +  2a)}-^{(«-i)''*-c34-4fcc}. 

13.  a  +  6  — c.  a-5  +  c.  14.   rt  +  6-c,  c+a— 6. 
15.  (m-rv)(w— a)-f-(Z)-c),  &c.       16.  i-r-^a-i)(a  — t),  &c. 

17.  (?»-ic)(Z-a)-^(c  — a)(a  — 6),  &c. 

18.  a;=J:?-j-(;?^+«^5+w)  +  '^  so  y  and  f. 

19.  p[l  —  (Ja-\-vib -\'nc)\-r-{}d-\-mq-{-nr)-\-n .  &c. 

20.  (m'-^  +  2a3-62-c2)-=-3<f,  &c.     21.  y  =  a-Z,-fr.  &c. 

22.  X  =  (rt.Z^+6c4-ca)(6+c  -  2a)(26-  a  -  c)-=-{(a  -  c)(6  +c  -  2a)  + 
(t_c)(26  — a  — c)|.     Corrected  equation,  a;  =  i(i+c),  &c. 

23.  ma-^(a  +  64-c),  &c.  24.  )ipi~-{aup-+bmp-Vcmg). 
25.  l---(6-c),  &c.  20.  i(i+c-a),  &c. 

27.  «M-4-(i,  &c.  28.  2=l-f-(a+o-c). 

29.  a  +  b,  &c.  SO.   1^2a,  &c. 


ANSWERS.  XXXI 

31.  (m8+»5-^2)-^-2w7^,  &c.       S2.  ^(a  +  c-b),  &c. 

33.  l{m-+n--')-^''lmn,  &c  Si.   1h-(6+c- a),  &c., 

35.  Iic^^b+c),  &o.  86.h  +  c~a,&o. 

37.  a,  h,   c.  3b.  h'^  -c^,  &c. 

39.  i(a  +  2/'-c  +  3rf),  &c.  40.  i(-ia+6  +  3c-2d  +  5e). 

Exercise  Ix. 

1.  aJfh,  a-b.  2.  K*2_|_5)^  ^(a^-S), 

4,  a^{a—h),  b^{a  +  b).  5.   l-=-(rt— 6),  l^(a-f5), 

8-  a-{-b  —  c,  a  —  b+c.  7.  a  +  i  — <;,  a  —  b+c. 

9.  {ab~l)^{a-l){b-l),   {a-b)^{a-l){b -1). 

10.  (l+a)-^-(«i-l),  (l-t-/>)-^(«6-l). 

11.  {a  +  l)(b-\-i)---{ab-l),  {a-b)^{ab-l). 

12.  fl(a+6),  b{a-b).  13.  a{i(«+C')-c(a-c)}^(a»-bc), 
a{/;(a-6)  +  c(a+c)}H-(«2_6c).         14,    ~-(a  +  b).  ab. 

15.  i(&+c),  &c.  16.  («-2/^+3c)-9-38,  &c. 

17.  2-r-(6+c),  &c.  18.  rt  +  6,  &c.     (by  symmetry). 

19.  b^-c-,&c.  20.  i3-c3,  &c. 

21.  ^abc,  {l-a)(l--h){l-c),  (2-a)(2-5)f2 -c), 

22.  2fl6c-4-(a6  +  6c-ca).  23.   1,    i,   1. 

24.  ar —  [ma  +  iib-[-pc+qd),  &,c. 

25.  «  =  0,  or(--l)-^(^-^)^ 

26.  (6+c-rt)H-(a  +  ^+cJ,  y  =  {b  -c  -aj-^ia  -  b  -  e), 

27.  ^(a  — 6  +  7?i  — n),  &o. 


XXXU  ANSWEB8. 

28.  (4a  +  2c  — <i— 36),  y-^z  by  symmetry- 

29.  —{a-b  +  c-{-d),  {ab-^bc,  &c.),  -{abd-{-&c.),  abed. 

30.  ^(a  —  b-irc  —  d+e),  others  by  symmetry. 

31.  x  =  {a  —  lb-\-  Imc  —  Imnd  +  Imnpc)  —  ( 1  +  Imnpq),  the  others  bj 
symmetry.  32.  x-=b-\-c  —  e,  &c. 

84.     ?/  =  (a+5i  +  3c-7tH9e)-j-22,  &c. 

35.     0  =  |(a-f-c),  then  symmetry.     36.  2  =  (;4-f^  +  «,  &c. 

S7.     a;  =  a— 26+3c  — 2cZ+e,  then  by  symmetry. 

Exercise  Ixi. 

1.  x={1al  +  a-Jrh-^r)^2{a-b)  where  r  =  4a(62+6-(-l)  _j- 
(3a-Z/)(36-rt). 

2.  a;  =  (ar  +  l)^(«r-l)  wliere  r^  =  (J2  -  1) -e-3(a3 -62). 

3.  a;={i/(l  +  a)(l  +  ^)-l/(l-«)(l-6)}^ 
{|/(l+rt.)(l  +  /;)+v^(l-a){l-6)}. 

4.  ^/76-a/3)H-(a6  +  a^),   (rt|3  +  6a)H-(rt/J-6a). 

6.     x={i/{a  +  h-irc){a+b-c)+->/ (b->rC—a){u-\-c-b)}  ^ 

{  ^/[a  +  b  +  c){a  +  h-c) - 1/... }. 
6.    ■{a  +  b)-^{l-ah).  7.     x=  (a/3-a6) -i- (fl;3+6a) 

la;  — 1/     "    (a—ni^ib  —  n)* 

10.  x={b-^G-a)  -4-  UV^  +  c-a)(c-f  ft-6)|^a  +  6-c).} 

11.  a;  =  (6  +  c)-5-^(a+6)(/;  +  cXc+a). 

12.  a;=  ^/(a  +  i  +  c) -j- «,  &c. 

13.  a;={62+c2-a(6+c)}  -^  i/2(a3  +  ?,s  ^c»  -  3fl6r). 

14.  (6+^-a},  &o.  16.     a  or  (a--6)-i- {l-a6). 


1 


ANSWERS.  XXXUl 

16.     x+y=s/(a-hb){a+2h)  ~  y{a-b), 

x  —  y=  y/(a  +  h){a  —  1h)  -=-  \/(a  — 6),  &c. 

18.  {x-\-y)-^{x-y)=  -/(a-f-Si)  —  |/(a.-^)  =  »i  suppose. 

19.  y^  =m-T-  (am^  —  vi+l)  where  m  = 

.1  -=-  a+6±  v/(aa  -6s  +1)  -^  (a-^b). 

20.  a,  6.  21.     «={v^(a-c)  +  |.-c}?/-=-{i,/(a-c)-iA}. 

22.  x=(a-{-c)y  -=-  (a  — c),  &c. 

23.  x-\-y  =  {ab-l)-i-ia-b),  &G. 

24.  a;i/={v/(6  +  2)-i/(&-2)}-i/{(6  +  2)--,/(ft-2))  = 
p  suppose,  x^y-  {|/«  +  2)  + |/'(a-  2)}-^ 

{N/(«  +  2)-i/(a-2)}. 

25.  a;2y2j,2  =  ^(a  +  i._c)(fc-j-c-a)(c+«-6\  (&c. 

26.  x=(a6  — fee  — ca)-j-2N/rt6c.  27.  a  — x"^  =  rfc:wi,  where  m  is 
the  value  of  v  in  the  equation  4crt  — 4(c+aj)-f-ii;''  = 

(ca  —  ah  —  be) ^  -{-^bica  —  ab  —  bc)v  -f  ih^v^ . 

28.  X  =  ii/{abc)   i-r-  +  —  1,7/  and  z  by  symmetry, 

29.  a(i3+C=)-T-(fc='+c2)^(/>2_|_c3)^    &c. 

30.  c{V{a  +  b)  +  ^{a-b)}^V{a  +  b}. 
81.  0  ora(6  +  c)-=-26c,  &c. 

32.  »= -1  or  a|/(a^  — l)-=-v'(a-*  — 6=*),  &e. 


xxxjv  answers. 

Examination  Papers, 


I. 

1.     {a+b+c){x+7j+z). 

4.    2a;2'»-ix3'",  4--  —  - -—•  5.   -!-14i/-19-=-27,  0  or  f 

•^        ^    0         c         a  • 

6.  16,  7.  199,  8,  AB  37,  CA  52,  56'  45. 

II. 

1       (a+h)^.  2.   (a+b  +  c){a^  +  hs+r^ -dahc).  8.   l-^a;^. 

4.  (a2'»4.2a'"  +  2)H-(«'"  +  2),   (a  +  />  +  c)^(a-6- c). 

7.  — -^j  4  or  6.  8.  p-q-^(r-pq). 

9.     684.         10.  (aw.+6/i)-r(a  +  />),  (by  common  rule). 

III. 

1.  -117,  a^{z-x)  +  (x-y)ah  +  {y-z)b2. 

2.  a+bx+cx^,  3  — 4a;-i-7x--^-10a;3 

3.  21 ;  6,  9,  12.  4.  160  eggs. 

5.  40,35.  6.    VU'^+a)+ym-a). 

7.  5ori;4.  9.   {a  +  b+c){a—b){b-c)[a-c. 

IV. 
1.     (^a  +  b+c)^.        2.  rt^&.       3.  (4.^3  -  9?/2)(43;3— 47/5).  1  +  Va;. 

4.  ii/3-^i/6,  17  5,  4.     ■  6.  1.       7.  Oor4;8,  1. 

8.  7t,  12.         9.  4  or  6.         10.  *(-3± -/ -39). 

V. 
1.     8x3  +  1-^125.  3.  16a'-' /^2.  4.  ft-f-8rt.  5.  15,  12. 

6.  ia,  ia;  4  or  -9,  7.  6,  7,  8,  or  -6,  -7,  -8. 


ANSWERS.  XXXV 

YI. 

1.  (,,24./,D).  2.  a-  +h^  +  c^  +ah+ac-bc. 
3.     x-+i/-'±U!/,  i7x  +  G!/-9){x-y  +  A).        4.    -20,  0. 

5.  b  or  1^1.  7.  x  =  4c  or  9,  y  =  d  or  4  ;  1,  2,  3  ; 

a;  =  M-7  +  i/33).     8.    -1,  0,  1,  or  5,  6,  7,  or  — V",  -4,  3. 
9.     x(x-+3)(x2-2a;-l)  =  0.       10.  a;-7-2/-3-=-4. 

VII. 

2.  --01.  4.  a;2+a;-  1. 

0.  (36a;2  +  18.c  +  9)H-(16j;4_81),  (^3 -a/>)^(j;2  +  a6). 

6.  d;  3;  x^i{a-b). 

VIII. 

3.  {x--xij  +  y^y^  ;  a^  +  b'i-aVj^. 

J  6  7 

5.  «^3 +ix  +  ca;~^  ;  -^a;  +^x  —  ix 

6.  a;24.^,^+^/^  ;  50(.-«-f-5)(a;-4)(a;-6). 

7.  a  =  0  =  /vorrt=l,   6  =  2. 

8.  3  or  —43-^7  satisfies  the  equation  2—  y  .  .  ,  &c. 

IX. 

4.  axy+b.     7.   +\/«/!';  V'(«+6)-f-|/(rt  — 6),  and  t/  =  reciprocal 
of  this.      8.   P(22a-216)--20a(a-6).         9.  7,   15,  48. 

X. 

1.  ax^ +hy^ +2cxy.  3.  x-2.  4.   24. 

.5.     5.c--3«u;+4rt-^;-^4-^-l/i.  7.3;^.  8.4,6. 

If  X 

XI. 

1.     2.  2.  G,  8.  3.   (4a;*-9/)3,  V^x4+/j^4. 

7.     0  or  -2J  ;  ?/=±3  or  ±^-9,  &c. 


XXXVi  ANSWERS. 

xn. 

6.  w-  —m=p,  q  =  0. 

7.  a;3  =  (a-2)-^(a+4),  -,/rc= -J/^(w- 1)  -  en} -^  |/a(w— 1) 

8.  a6-{(a-l))6-l)-l}.  9.  37. 

XIII. 
1.     2.     3.  1+r ^x  +  c„x- +&C.,  vfhere  c^,  ffg^  *^c.,  represent  the 

combiui^tions  of  a^,  a^,  .  •  ■  taken  one,  two,  &c.  at  a  times 
4.     $4000.  5.   {m{b'c  —  bc'}  —  n{ca'  —  c'a)}-i-{ab'-a'b), 

'^^J^^'c^'       7  («  +  /3)'+(a-^)'-2/  =  0.       9.65. 
XIV. 

1,  l^iihr.  4.  Smiles.  5.  rtm-=-?i ;  .'c+z/  =  ±5  or  ±1, 
x-y=±l  or  ±5.              6.  20. 

XY. 

2.  A.  3,  ^3(r3-ifn+c/2(?;2-fl'c)+ar(nr,7-ftc).  4.  2-=-(w-n). 
If  7?i— «  is  uegative  x  is  neg.  which  shows  that  they  were 
together  before  noon.  If  ?«-n  =  0,  a;  is  infinite,  i.e.,  they 
are  never  together. 

5.  a;2(a;2_„2)(a;-2«);   (a:^ -r?2)(-x2-?/3). 

6.  {a  +  b+c-{-dd)^{a  +  b  +  c  +  d) :   {x  +  y-\-z)  ^  {x-y -hz); 

XVI. 
4.     2a;4 -3.r3+4.r4-3.      '         5.  See  paper  XIX.,  prob-  4. 

IX  T 

a     |{l±i/(a"  +  4t")--\/4J".     8.   {b,c,-b,c^)-^{a,b^-a^b,) 
^Q>^c^-b.,c^)-^{a.,b^-a^b^).  9.  acH-(a-6), 


ANSWERS.  XXXvii 

XVII. 
1.      {2ah-(a  +  h)}^ab.  2.  x* -2z''{a-\-b^)  +  {a-b^)^  ] 

8a»-2a6-10rtC-3/)2+2c2  +  o6c.     6.    m^-n=4:. 
B.     (m  4-n)(^hq—iic)—-{inq—pn},  {p-\-(i)(mc— bn)-T-{niq—pn). 

xvin. 

1.     l-a;'":  2.1.  4.  x-a-fc.  6.  (a;- 7/)(a:-2). 

7.  1.  8.  9a;   ±1^  \/2  ov  zt^VG.  9.  10,11. 

XIX. 

1.        K4x-''  +  ri3.  3.    5a.2_l;    (;r2+a;.i/-|-2/2)2. 

4.     {a-c)(a-d){b-c){b-d)^(a  +  b-c-dy-. 

6.     (a  +  2)2H-4(a2+«).       7.  (T^)^ -a2)^12rt.       8.18,22,50. 

XX. 
1.     (l  +  m'^x+(l-n)ij.  3.  a;2_i. 

4.  (a;+2/--s.){//+z— a:)(2  +  ar-?y)-=-(a;  +  ?/+2)2; 
(a26-2a62  -a3+rt63  -,.^3)^(^4  _54). 

5.  1.         6.  a''{cb'-hc')  +  b''{ac'-a'c)  +  c'\a'b-ah')  =  0. 

8.  a(6-c)H-(6-a),  b{c-a)-^{b-a).         9.  8.  10.  2000. 

XXI. 
1,     8.         2.  24a7)c;  2a» +4:?j'i^x^ +  62a^.c'i'+Ua^x^ +  18x^. 

4.     0;  ^7_|_i6.     5.  16;  x+2ri  x  -2a.        7.  3?.77  oz.  of  gold, 
783  oz.  of  silver.     9.    -  (8±4i/3)H-(3zh2  \/3  ; 
a;=±2or  V  — 1,  7/==Fl  or  =p2i/-l;  8,  4. 

10-     y  =  cost  of  2nd  bale  =  60 ;i:  20  ^7. 


XXiVlll  ANSWERS. 

XXII. 

1.     '02997,  a^+ag+p^.  2.  ah''  -b^ -hc  =  0, 

{a).{a-b){8a-Sb).   {b).x{x~  l){a—b){b-c){a-c). 

3.   b^=Uc.   (a),   (d+b)^.  5.   (a)   {ax-by)^{ax-{-hij) 

7.     («)  i(a-Hi  +  6-),   {b),  I,  f,  2.^ 

(c).   b  —  c,c  —  a,a  —  h.      (d).    —  1±V''2. 

XXIII. 

1.     ^x^+^x^-x^  +  kx-  +  ^l  +  {95x^+^lx^-A0x+A2)^ 

(3x4  - 21x3 -f  9a; -6);    -882.         4.  x  =  a  +  2c,  y  =  b-\-Sc. 
5.     (2).     ^V         7.  (1).  a+i  +  c.  (2),  1,  2,  3,  4.  (3),  0  or  j\. 


12.     Coll.  to  Newmarket  63  miles. 

XXIV. 

1.     \^^(x-y){y-~z](z-x).     2.  S-m-v,  0.       8.  5,  3.  4^. 

4.  (1).   A  =  bc-^2a,   B  =  ac-i-2b,   C  =  ab^2c;   (2).  a^.^fc^^c'-'. 
11.     7y=+2,  a;=+5,  &c. ;  a;«-10x=-19or   -16, 

— ^  . — ^  4P« 

.-.  ic^B  or  3,  &c.;   {b      ±a      )p-« 
XXV. 

1.     (cr--Bx)M^^-x^);  1.       2.  — -— -/.2(.T+fl)-«2x2. 

5.  a;2-2x  =  2,  a;  =  3,  ?/  =  2;  ?/  =  53-f-24,  &c.     6.  4  miles,  3  do. 


ANSWESS.  SXXIX 

111 

10      1+  -j-+r724-^-72:o  +  &c.  =  2-71828  approximately; 

{(5x):h-3''"^^}{1-6-11  .  ..  (5r-4)}^|ji.       11.  -|  and  -^, 
XXVI. 

2.  a-v/(a6-a2);   4.  3.   2,  i,  or  ^-(-3±  V^o) ;  a;+7/+2  = 
v/(a2+2i/2),    .•.2  =  c+i/(a3+262),  &c. ;  i(-4±,/76). 

4.  3.         7,  p2^.i^r^_^27  =  0.  10.  (l+.r)--(l -x)2 - 
a:"-i{3x-l  +  2«(l-x)}--(l-x)M  vi--(15n  +  9),  ^V 

XXYII. 
1.     {hn-n7(fi.  2.  («+ /,+c)(«, -/>)(/>- c)(rt-c). 

3.  (a-6)(/j-c)  +  (6-c)(c-a)  +  (c-a)(a-&);  {a-b)^  + 

(6-c)24-(c-a)2.  4.    a;=:a^(^,2+^3_}.c2),    &c. 

5.  Vab;  18  or  -2;  i  +  3j.  7.  i{a  +  h)&c.; 
x={l+a^-b^±-l^{l-a-b){l-a  +  h){l-{-a-b){l+a+b)\ 
H-2a;    a;-f-y=:(l+rt)(l— ^>)-H(l-rtcj,   &c. 

8.  —8;  {p^-Ap^-q+8pr)-^{p^-Aq). 

9.  (m  +  «)-^-2»m,  (n  —  m) -i-2wn. 


I. 


W.  J.  OAGE  &,  €0'S, 

list  Of  EiBcatimal  PaMicate, 

FOP  SCHOOLS  AND  COLLEGES. 


MATHEMATICS. 


,r'-' 


HAMBLIN  SMITH'S  ARITHMSTIC.-An  Ad 
vanced  treatise,  on  the  Unitary  System,  by  J. 
Hamblin  Smitb,  M.A.,  of  Gotjvilio  anci  Caius  "Col- 
leges, and  late  lecturer  of  St.  Peter's  College,  Cam- 
bridge. Adapted  to  Canadian  Sobools,  by  T!'  mas 
Kirkland,  M.A.,  Science  Master,  Normsl  Sohooi, 
Toronto  "nd  V/illiam  Scott,  B.  A.,  Head  Master 
Model  School  for  Ontario,    6tb  Edition $0  75 

KEY  — A  comjilete  Key  to  the  above  Aiithmetic,  by 

the  Authors  2  CO 

"I  consider  Hamblin  Smith's  Arithmetic  by  Kirklaud  and 
Scott,  and  Hamblin  Smith's  Algebra,  with  appendix  by  Mr. 
Baker,  admirable  works,  far  better  adajited  for  use  in  our 
schools  and  for  private  study  than  anyother  sim  ilarworks  that 
I  know  of.  They  will  soon,  I  have  no  doubt,  supersede  texr- 
books  hitherto  used  in  our  schools,  as  they  have  already 
done  in  the  Gait  Collegiate  InsHtute."— ^Zex.  Murray,  M.A.. 
Mathematicul  Master  Gait  Collegiate  Institute. 

KIRKLAND  &  SCOTT'S  ELEMENTARY 
AKITHMETIC— An  Elementary  treatise,  on  tha 
Unitary  System,  intended  as  an  introductory  text- 
book to  Hamblin  Smith's  Arithmetic,  by  Thomas 
Kirkland,  M.A.,  Science  Master  Normal  School, 
Toronto,  and  William  Scott,  B.A.,  Head  Master 
Model  School,  for  Ontario ;  40th  thousand  vAthin 

first  year  of  its  issue 0  25 

"  Introducing  Fractions  immediately  after  the  '  Simple 
Rules '  will  be  hailed  by  all  practical  teachers  as  a  step  in 
the  right  direction.  I  shall  advocate  the  exclusive  use  of 
your  work  in  all  elementary  schools,  as  my  past  expei-ience 
enables  me  to  estimate  its  value  " — John  Ma.coun,  M.A., 
F.L.S.,  Rector  of  4lbert  College  Grammar  School,  Belli- 
viUe. 


VV.  J.  GAGE  &  Go's  Educational  Series. 

pmcE. 

McLELLAN     &    KIRKLAISTD'S    EXAMINA"   . 

~      IIO.N    i^\PEKS  l.N     AKIXHME    IC— A    coiiipli-te    ^' 
series  of  Problems,  designed  for  use  in  Schools  and 

^  Colleges,  and  especially  adapted  for  the  pre- 
paratiou  of  candidates  for  Teachers'  Certific'ates, 
by  J.  A.  McLellan,  M.A.,  LL.D.,  lusnector  of  High. 
Schools  and  Thomas  Kirkland,  M.  A.,  Science 
Waster  Normal  Scliool,  Toronto.     1th   Edition 1  00 

McLELLAN  &  KIEELANDS  EXAMINA- 
IIl)^  PAl'EKS.— PAKl  1.— Coiitaiumg  tue  Ex- 
amination Papers  for  admission  to  High  Schools, 
and  for  Candidates  for  Third-Class  Teachers'  Cer- 
tificates         0  50 

HINTS  AND  ANSWERS  TO  McLELLAN.  & 
KKKEANJJ'S  EXAMl.NATION  PaPEKs,  contam- 
ina  answers  to  Problems  and  Solutions  to  all  diffi- 
cult   questions.    Prepared    by   the  authors.    2nd 

Edition  1  00 

The  leadiug  American  Educational  Jouin  il  (Nntional 
Teachers'  Monthly)  says  of  McLellan  and  Kirkhmd's  Exam- 
ination Papers:— "In  our  opinion,  the  best  collection  of 
problems  on  the  American  Continent."  ■ 

SMITH    &    McMURCHY'S    ADVANCED    ^ 

AKltlEM.STIC 0  .TO 

SMITH    &    McMURCHY  S    ELEMENTARY 

ARITHMETIC 0  2i. 

McLELLAN'S  MENTAL  ARITHMETIC- 
P.-i.KT  I.  Containing  the  Fundainent-il  Kules, 
Fractions,  and  Analv^ic;.  Bv  J.  A.  McLellan,  M.A., 
LL.D.,  Inspector  of  iiigh    Schools,  Ontarior  2nd 

Edition 0  30 

McLELLAN'S  MENTAL  ARITHMETIC- 
I'.VK'l' II.  By  the  same  author,  luUy  truats  fer- 
centai^e  in  its  various  applications.  General  .Vnaly- 
sis.  Stocks  and  Shares.  Interest.  Di^count,  &c.,  Ac, 
and  gives  iTi-ctical  solutions  of  almoc^t  every  type 
of  question  likelv  to  be  met  with  in  any  treatise  on 

Arithmetic.    2nd  Edition  0  ir> 

"  His  treatnnent  of  the  subject  ha.3  been  so  practical  and 
skillful  that  teachers  have  frequently  expressed  the  wisli  . 
that  he  would  prepare  a  text-book  on  Mental  Arithmetic. 
The  volume  before  us.  Part  I.  of  the  work,  treats  systema- 
ticallv  and  comprohensivelv  of  the  fundamental  rule?;,  frac- 
tions," analysis  r»:duction,  &c.  It  contains  about  1,200  well 
graded  practical  iroblems.  We  can  recommend  the  book 
to  all  t-r-achers  of  arithmetic."— Z/Oii'7o)i      (irertiaer.  ^, 

JUVENILE   MENTAL    ARITHMETIC.  -  By  ^^ 
Joun  F.  Stodcurd,  M.A 0  15 


V/.  J  GAGE&  Go's  Educational  Series 

PRICE. 

HAMBLIN  SMITHS  ALGEBRA— An  Elemen- 
t;iry  Alsfbra.  "ov  J.  Hambiiii  Smith,  M.A.,  of  Gon- 
.ville  and  Caius' Colleges,  and  late  Lecturer  at  St. 
Peter's  College,  Cambridge,  with  Appendix  by 
Alfred  Baker,  B. A.,  Mathematical  Tutor,  University 
College,  Toronto ^ 0  90 

SEY.— A  complete  Key  to  Hamblin  Smith's  Algebra.  2  75 
"Arrangements  of  subjects  good;  explanations  and  proofs 
exhaustive,  concise  and  clear  ;  examples  lor  the  most  part 
from  University  and  College  Examination  papers  are 
numerous,  easy  and  progressive.  There  is  no  better  Al- 
gelira  in  use  in  our  Hish  Schools  and  Collegiate  Institutes." 
^■Georqe  Dickson,  S.A.,  Head  faster  Collegiate  Institute, 
Hair,  iff  on. 

HAMBLIN    SMITH'S    EXERCISE    IN    AL- 

GEBKA.    PAKT    1 0  75 

CtROSS^  ALGEBRA.-PAPvT  II.  By  E.  J.  Gross, 
M.A.,  Fellow  of  Gonville  and  Caius  Colleges,  and 
Mathematical  Lecturer  at  Genon  College,  Cam- 
bridge         2  50 

HAMBLIN  SMITHS  ELEMENTS  OF  GEO- 
METRY, containing  Books  1.  to  "VT.,  and  portions 
of  Books  XI.  and  XII.,  of  Euclid  with  Exercises 
and  Kotes,  byJ.  Hamblin  Smith,  M.A.,  Ac,  and 
Examination  Papers,  from  the  Toronto  and  McGill 
Universities,  and  Normal  School,  Toronto 0  90 

HAMBLIN   SMITHS    GEOMSTRY.-BOOKS 

I.  and  II.,  with  Exercises,   <xa 0  30 

HAMBLIN    SMITHS    GEOMETRY.-BOOKS 

il.  and  III.,  vrith    exercises,  itc 0  30 

POTTS'  EUCLID- — Containing  the  first  six  books 
with  explanatory  notes  ;  a  series  of  questions  on 
each  book ;  and  a  selection  of  Geometrical  Exer- 
cises from  the  Senate,  House,  and  College  Exami- 
nation Papers  ;  with  hints  &c.,  by  Pinbert  Potts, 
M..4..,  Trinitv  College,  with  Ajipendis  by  Thomas 
Kirkland,  M.  A.,  Science  Master  Xormal  School. 
500th  thousand 0  50 

POTTS'  EUCLID.— BOOKS  I.  and  IL,  with  Exer- 
cises      0  30 

POTTS'  EUCLID.— BOOKS  n.  and  m.  with  Ex- 
ercises      0  30 

I    shall   recommend   Pott's   Euclid  to   the   teachers  in 

training  as  a.  book  of  invaluable  use." — W.  Crockett,  A.M., 

Principal  Soi-mal  Training  School,  New  Brunswick. 


W.  J.  GAGE  &  Go's  Educational  Series. 


PRICE. 


TrrT?TrT,A"srD'S  STATICS— An  Elementary  Treat- 
^il?nu  sullies     BNXhos    Kirkland.  M.  A.,  Science 
-'  Master    iSoruml  Scbool,   Toronto,  with  numerous 
examples   and  exercises  ;  principally  designed  toi 
the  use  of  candidates  for  first  and  second  class  cer- 
tificates, and  for  the  Intermediate  Examination. 

4th  edition 

"  It  supplies  a  great  ^^ant  felt  by  those  preparing  for 
Teachers^  Certificates.  This-  did  it  possess  no  otuer  merits 
-should  make  it  a  great  success.  It  is  by  far  toe  best  text 
book  on  the  subjec  t  for  the  schools  of  Ontario  I  have  seen 
Geo  mplie^M.  A  ,  M.  D.,  Science  Master  Normal  ScJwol. 
Ottawa. 

TTAMBIilN  SMITH'S  ST  A.TICS.-Elementary 
STATiis  By  J.  Hamblm  bmith,  M.  A.  Gonyille 
aSd  Caius  College,  and  late  lecturer  f>t  St  Peter's 
College.  Cambrfdge.  with  uppeudix  by  Tnomas 
Kirklind,    M.A.,  Science    Master,  formal  School,    ^  ^^ 

Toronto 

TTVT^-Rn«^TATIGS  —  Elumentakt  Hydrostatics. 
^^WH-i^.lin  smith.  M.A.,  Gonville  and  Cams  Col- 

lege,  late  lecturer  at  St.  Peter's  College,  Cambridge.    0  75 
KEY  -A  Key  to  Hamblin  Smith's  Statics  and  Hydro-    ^  ^ 
suitics  in  one  volume    

TRIGONOMETRY -ELEMENTARY  TRIGONOMETRY. 

By  J.  HamDliu  Smith,  M.  A -^  ^ 

KEY.    A  Key  to  Hamblin  Smith's  Elementary  Trigo-    ^  ^^ 

nometry  


ENGLISH. 


■M-A<?ON'S  ADVANCED  GRAMMAR.- Includ- 

M.  i..    27th  edition 

..  T  a-ked  a  grammar  school  inspector  in  the  old  country 

5'uiii'^'-/j*^  «i--^^^-.  ^  ^■^■^■'  «'^''''"^- 


"VV.  3.  GAGE  &  Cos  Educational  Series. 

PRICE. 

MASON'S  ENGLISH  GRAMMAR.-(Comniou 
Sc!i"ol  edition)  -svitli  copious  and  carefully  graded 
exercises,  '243  pages  0  60 

MASON'S  OUTLINES  OF  ENGLISH  GRAM- 

MAB,  for  tlie  US8  ol' junior  classes 0  50 

ENGLISH  GRAMMAR  EXERCISES.-By  C. 

P.Masou.    Keprintod  from  Com.  Sell.  Edition    ....     0  30 

MILLER'S  SWINTON'S  LANGUAGE   LES- 
SONS, (revis  d  edition),  adajited  as   an  introduc- 
torv  text-book  to  Mason's  Grammar,  by  J.  A.  Mac- 
mUlnn.  B.A.,  Ott.iwa     ollegiaie  Institute.     It  con- 
tnins  the  Examination  Pajiers  for  admission  to  . 
Higli  Schools,  and  teac/ips  grammar  and  composi- 
tion simultaneoiisly.    5th  edition,  40th  thousand...    0  25 
"  In  accordance  with  a  motion  passed  at  the  last  regular 
meeting  of  the  County  of  Elgin  Teachers'  Association,  ap- 
pointing the  undersigned  a  Committee  to  consider  the  res- 
pective merits  of  different  English   Grammars,  with   a  view 
to  sutrgest  the  most  suitable  one  for  Public  Schools,  we  beg 
leave  to  report,  that,  after  ful!y  comparing  the  various  edi- 
tions that  hav-  been  recommended   we   believe  that  '  Mil- 
ler's Swiuton's  Language  Lessons'  is  the  best  adapted  to  the 
wants  of  junior  pnjDils.  and  woukl  urge  its  authorization  on 
the    Government,    and    its   introduction    into    our   Public 
Schools."  (Signed,) 

A.  F.  Butler,  Inspector. 
J.  McLean,  Town  Inspector. 

J.  MiLLAE,  M.  A.,  Head  Master  St  Thomas  High  School. 
A.  Steele,  B.A.,  Head  Master  .A,ylmer  High  School. 
N  M.  Campbell,  Head  Muster  Co  of  Elgin  Model  Scho-"!. 
It  vsas  moved  and  seconded  that  the  report  be  received 
and  adopted.    Carried  unanimously. 
NEW  ENGLISH  GRAMMAR.-In  three  parts  : 
Etvmologv,    Syntax    and    Analvsis.    Bv    William 
Sw'inton.  A.  M.     Eevised  by  J.  B.  Calkin,  M.    A., 
Principal  of  the  Is'ormal  School,  Truro,  N.  S 0  50 

DA  VIES'    INTRODUCTORY    ENGLISH 

GK.UIMAR 0  25 

DAVIES'  NOTES  ON  5TH  READER.-Liter- 
ary  Extracts  selected  from  Book  V  of  the  author- 
ized series  of  Readers,  for  "Examination  in  Eng- 
lish Literature."  of  candidates  for  third  clas-s 
certificates,  with  notes  original  and  selected.  By 
H.  W.  Davies,  D.  D.,  Principal  Kormal  School, 
Toronto.    5th  edition  0  25 

MILLER'S  ANALYTICAL  AND  PRACTICAL 

GBAMMAR 0  38 


W.  J.  GAGE  &  Go's  Educational  Series. 

PBICE. 

EPOCH  SERIES.— PAllT  1.    Containing  first  four 

of  Series  SO  50 

EPOCH  SERIES— PART  II.    Containing  last  four 

oi  the  beries  0  30 

EPOCH  SERIES.— COMPLETE,  in  one  volume  .    i  qO 

CREIGHTON'S  EPOCH  PRIMER  OP  ENG- 
LISH HISTORY.— An  introiuctory  volume  to 
"Epochs  of  English  History."  A  complete  sum- 
mary of  the  history  of  En^and,  in  140  pages. 
By   Mandell    Creighton.    M.  A.,    late    FeUow  and 

Tutor  of  Merton  C  Uege,  Oxford.'. 0  30 

"  The  work  is  admirably  done,  and  it  will  no  doubt  obtain 
a  very  considei-able  sale." — Atheneeiun. 

"  This  volume,  taken  with  the  eight  small  volumes,  con- 
taining the  accounts  of  the  different  epochs,  presents  what 
may  be  regarded  as  the  most  thorough  course  of  elementary 
English  History  ever  published. — .^berdeen  Journal. 

PINNOCKS    CATECHISM    OF     ENGLISH? 

HiSTOliY    0  10 

A  SCHOOL  MANUAL  OF  ENGLISH  COM- 

POSITIO.V.For    advanced   classes   in    Academies, 
High  and  Public  Schools.   JBy  V.^illiam  Swinton o  45 

REID'S  ENGLISH  DICTIONARY  of  the  Eng- 
lish language,  coutainiii;,'  the  Pronounciation,  Ety- 
mology, and  Explanation  of  all  words  authorized  by 
eminent  writers;  to  which  are  added  a  vocabularv 
of  the  roots  of  English  words  and  an  accented  list  ©f 
Greek,  Latin  and  Scripture  proper  names.  B5' 
Alexander  Reid,  A.M.,  Rector  of  the  Circus-place 
School.  Edinburgh ;  author  of  "  Rudiments  of  Eng- 
lish Com-;iosition,"  &c, ;  with  an  introduction  by 
Henry  Reid,  Professor  of  English  Literature  in  the 
University  of  Pennsylvania;  and  an  appendex 
showing  the  pronounciation  of  nearly  3.000  of  the 
most  important  geographical  names.  3rd  Canadian 
and  23rd  English  edition 1  CO 

"  It  is  a  very  complete  and  useful  •<f!oxk."— Montreal  Daily 
Witness. 

UNIVERSAL  PRONOUNCING  POCKET 
DICTIONARY  OF  THE  ENGLISH  LANGUAGE. 
— Founded  on  the  principles  of  Walker,  Webster, 
Worcester,  Johnston,  Goodrich  and  Porter 0  20 

NATIONAL  PRONOUNCING  DICTIONARY    0  40 


V7.  J.  GAGE  &  Co's  Educational  Series. 

PBICE. 

GOLDSMITH'S  TRAVELLER,  AND  GRAY'S 

ELK(jY. — lu  one  volume ;  edited  by  the  Ecv. 
E.  T.  Stevens,  M.  A.,  Oxon,  joint  editor  of  "The 
Grade  Lesson-books,"  "  The  Useful  Knowledge 
Series,  etc. ;  and  the  Kev.  D.  Morris,  B. A.,  London, 
author  of  "The  Clafs-Book  History  of  England," 
etc.    Interleaved  edition 0  CO 

SCOTT'S  LADY  OP  THE  LAKE.-With  in- 
troduction, notes,  and  glossarial  index ;  by  R.  W. 
Taylor,  M.A.,  Assistant  Master  at  Kugby  School,  and. 
formerly  Fellow  of  St.  John's  College,  Cambridge. 
Interleaved  edition  0  40 

MORRISONS  ENGLISH   COMPOSITION  — 

For  the  use  of  schools.  By  Thomas  Morrison,  M.A., 
Kector  of  the  Free  Church  Normal  School,  Glasgow    0  45 

CREIGHTONS      EPOCHS     OF      ENGLISH 

HISTOKY  — f.dited  by  the  Kev.  M  Creiehton,  M.A.. 
late  Feliovv'  and  Tutor  of  Jlevton  College,  Oxford. 
Eight  volumes  in  convenient  and  cheap  form,  ad- 
apted to  Public  and  High  Schools    Price  20c  each, 

THE  SEEIES  CONSIST   OF: 
I.  Early  England  up  to   the    Norman    Conquest.     By 

Frederick  York-Powell,  M.A.     With  four  maps. 
U.  England  a  Continental  Power  from  the  Conquest  to 
Magna    Charta,    10G6-i'216.      By    Louise    Creighton. 
With  u  coloured  may  of  the  Dominion  of  the  Angevin 
Kings. 
in.  The  Rise  of  the  People,  and  Growth  of  Parliament, 
ii-om  the  Great  Charter  to  the  Accession  of  Henrv 
VII,  1215  1485.    By  James  Bowley,  M.A.,  Profe.ssor  of 
Mod.  Hist,  and  Lit.,  Univ.  Coll.  Bristol.     With  four 
maps. 
IV.  The  Tudors  and  the  Reformation,  1-185-16'13.    By  the 
Rev.  Mandell  CreJRbton,  M.A.,  late  Fellow  and  Tutor 
of  Merton    College,  Oxford,  Editor   of  the  series. 
With  three  maps. 
V.  Struggle  against  Absolute  Monarchy,  from  1603  to  1C88. 

By  Bertha  M.  Cordery. 
■VI.  The  Settlement  of  the  Constitution  from  1689-1788.    By 
James  Rowley,  M.X.,  Professor  of  Modern  History 
and  Literature,  University  College,  Bristol. 
■yil.  England  during  the  American  and   European  "Wars, 
from  1789-1820.      By  O.  W.  Tancock,  M.A.,  Assistant 
Master  King's  School,  Sherborne,  Dorset. 
■VIII.  Modern  England,  from  1820-1875.    By  Oscar  Browning, 
M.A.,  Fellow  of  King's  College,  Cambridge. 
"  Amongst  naanuals  in  English  History  the  Epoch  Serjasis 
sure  to  take  high  ranis." — Daily  Glohe. 


MASON'S  GEADUATED  SERIES  OF  ENGLISH  GRAMMARS. 


M^asoii's  OiitliiKis   of*  Ii^iig:lisli 
Gri'aiiiiiifii*. 

By  C.  P.  Mason,  B.  A.,  "F.  C.  V.,  Fellosv  of  University  College  London. 
Authorized  fox  use  of  Schools  in  Ontario.     For  the  use  of  junior  classes. 

Price,  45  Cents. 

Mlason's  Shorter  £2iig-lisli  Grammar. 

With  copious  and  carefully  graded  exercises,  243  pages. 
Price,  60  Cents. 


Miason's  Advancedl  Grramimar. 

Including  the  principles  of  Grammatical  Analysis.  Enlarged  and 
thoroughly  revised,  with  Examination  Papers  added  by  W.  Houston, 
M.A. 

27th  Edition,  Price,  75  Cents, 

"I  asked  a  grammar  school  inspector  in  the  old  country  to  send  me  the 
best  grammar  published  there.  He  immeitiately  sent  Mason's.  The  chap- 
ters on  the  analysis  of  difficult  sentences  is  of  itself  sufficient  to  plact-  the 
work  far  beyond  any  English  Grammar  hitherto  before  the  Canadian  pub- 
lic."—Alex.  Sims,  M.  A.,  H.  M.  H.  S.,  Oakville. 


£)ii8:1isli  Grraiitmar  Practice. 

This  work  consists  of  the  Exercises  appended  to  the  "  Shorter  English 
Grammar,"'  published  in  a  separate  form.  They  are  arranged  in  progress- 
ive lessons  in  such  a  manner  as  to  be  availaWe  with  almost  any  text  book 
of  Englisli  Grammar,  and  take  the  learner  by  easy  stages  from  the  simp, 
lest  English  work  to  the  most  difficult  constructions  in  the  language. 

Price,  30  Cents. 

Outlines  of  Engrlish  GrraiMmai'. 

These  elementary  ideas  are  reduced  to  recular  form  by  means  of  careful 
definitions  and  plain  rules,  illustrated  by  abundant  and  var-ed  examples 
for  practice.  The  learner  is  made  acquainted,  in  modern  measure,  with 
the  most  important  of  the  older  forms  of  Kiiulisli,  witli  the  way  in  which 
words  arc  constructed,  and  with  llie  eh^nicMits  of  wliich  modern  English 
is  made  up.  Analysis  is  treated  so  f.ir  as  to  give  tlie  power,  of  dealing 
with  sentences  of  plain   constructiou   and    laoderate    dfficulty.     Id   the 

English  Grraiiiiirar, 

tlie  same  subjects  are  presented  with  much  greater  fulness,  and  carried 
to  a  more  advanced  and  difficiiU  stage.  Tlie  work  contains  ample  materi- 
als for  the  requirements  of  Competitive  Examinations  reaching  at  least 
the  standard  of  the  Matriculatiou  Examination  of  the  University  of  Lon- 
don. 

Tlie  Shorter  Eiigrliistli  Orninmar 

Ik  intendi'd  for  learners  who  have  but  a  limited  amount  of  time  at  their 
disposal  forEnglis'c  studies  ;  but  the  experience  of  schools  in  which  it  has 
been  the  only  English  Grammar  iiS' d  has  filiown  tli.'jt,  when  well  master- 
ed, this  work  also  is  sufficient  for  the  London  Matriculation  Examination. 


Examination     Piimor    in    Canadian 
IIie«toi-y. 

(History  Taught  by  Topical  Method.) 
Rv  Tames  L  Hughes,  Inspector  of  Public  Schools  Toronto. 
A  ?riJnTr  for  Schools,  and  Students  preparing  tor  Examma- 
tions.     Price  25c. 

New  and  Special  Featurbs. 
I    The  History  is  divided  into  periods  in  accordance  with  the 
great  national  changes  that  have  taken  place. 

2.  The  history  of  each  period  is  given  topically  instead  of  in 

chronological  order. 
,    Examination  Questions  are  given  at  the  end  of  each  chapter. 


4 


Examination  Papers,  selected  from  the  official  examinations 
^'fX?fereat  provinces,  are  given  in  the  Appendix. 


"of  the  different  provinces,  are 


student's  Review  Outlines,  to  enable  a  Student  to  thorough- 
^iy  tS  his  own  progress,  are  inserted  at  the  end  of  each 

6    Spec^arattention  is  paid  to  the  Educational,  Social,  and 
*     Commercial  progress  of  the  country. 
7.  ConsTitutional   Growth  is  treated  in  a  brief  but  compre- 

hensive  exercise. 
Bv  the  aid  of  this  work  Stu  ienU  can  prepare  and  re;t>iewfor  Ex- 
^       aminat.ons  in  Canadian  H,s!orv  more  quickly  than 
by   the   use   of  any  other  work. 

r3.i"-e's  Practical   Speller. 

A  MatuTaVoT  Spelling  and  Dictation.     Price  30c. 
Prominent  Features. 
The  book  is  divided  into  five  parts  as  follows: 
T3»ox  T   Contains  the  words  in  comtnon  use  in  daily  life,  to- 
Jk^rwith    A^Z,£ons,  Forms,  etc.      If  a  boy  has  to  leave 
fchol  early,  he  should  at  least  know  how  to  spell  the  words 
of  common^occurrence  in  connection  with  his  business^ 
.        Part  II  Gives  words  liable  to  be  spelled  incorrectly  because 
the  same  sounds  are  spelled  in  various  ways  m  them. 

Part  III.  Contains  words  pronounced  alike  but  spelled 
differently  with  different  meanings. 

Part  IV  Contains  a  large  collection  of  the  most  diffi-cult 
ZTin  common  use.  and  is  intended  to  supply  materiafior  a 
general  review,  and  for  spelling  matches  and  tests. 

Part  V  Contains  Literary  Selections  which  are  to  be  mem- 
orized and  recited  as  well  as  used  for  Dictation  Lessons,  and 
less  jns  in  Morals.  .    ,,     r      r^- 

Dictation  Lessons.-AU  the  lessons  are  suitable  for  Die 
lationU.essons  on  the  slate  or  in  dictation  book 

RENiEws.— These  will  be  found  throughout  the  book.